UGC NET/JRF Exam. Solved Papers Geography

Table of contents :
Cover......Page 1
Copyright......Page 3
Geography......Page 4
Backcover......Page 214

Citation preview

Editorial Board Pratiyogita Darpan

UPKAR PRAKASHAN, AGRA-2

© Publishers Publishers UPKAR PRAKASHAN (An ISO 9001 : 2000 Company)

2/11A, Swadeshi Bima Nagar, AGRA–282 002 Phone : 4053333, 2530966, 2531101 Fax : (0562) 4053330, 4031570 E-mail : [email protected], Website : www.upkar.in Branch Offices : 4845, Ansari Road, Daryaganj, New Delhi—110 002 Phone : 011–23251844/66 28, Chowdhury Lane, Shyam Bazar, Near Metro Station, Gate No. 4 Kolkata—700004 (W.B.) Phone : 033–25551510

Pirmohani Chowk, Kadamkuan, Patna—800 003 Phone : 0612–2673340

1-8-1/B, R.R. Complex (Near Sundaraiah Park, Adjacent to Manasa Enclave Gate), Bagh Lingampally, Hyderabad—500 044 (A.P.) Phone : 040–66753330

B-33, Blunt Square, Kanpur Taxi Stand Lane, Mawaiya, Lucknow—226 004 (U.P.) Phone : 0522–4109080

● The publishers have taken all possible precautions in publishing this book, yet if any mistake has crept in, the publishers shall not be responsible for the same. ● This book or any part thereof may not be reproduced in any form by Photographic, Mechanical, or any other method, for any use, without written permission from the Publishers. ● Only the courts at Agra shall have the jurisdiction for any legal dispute.

ISBN 978-93-5013-436-8

Code No. 1793

Geography UGC-NET/JRF Exam. Solved Papers

December 2006 Geography Paper-II Note—This paper contains fifty (50) objective type questions, each question carrying two (2) marks. Attempt all of them. 1. The north-pole lies with in the Arctic Ocean whereas south-pole is located within Antartic Continent. Which of the following theory uses this phenomenon as a supportive argument ? (A) Theory of Isostasy (B) Theory of Plate Tectonics (C) Convective Current Theory (D) Tetrahedral Theory 2.

Indus and Brahmaputra are examples of— (A) Subsequent drainage (B) Superimposed drainage (C) Antecedent drainage (D) Consequent drainage

3.

The average time interval between two successive high tides is— (A) 6 hrs, 13 mts. (B) 12 hrs, 26 mts. (C) 24 hrs, 50 mts. (D) 18 hrs, 39 mts. It is necessary to know both the latitude and longitude of a place in order to determine— (A) Local time (B) Altitude (C) Standard time (D) Location

4.

5.

Given below are two statements, one labelled as Assertion (A) and the other labelled as Reason (R). Select your answer from the codes given below— Assertion (A) : The troposphere is the dense lower part of the atmosphere in which metrological phenomenon such as clouds and storms occur. Reason (R) : In the troposphere temperature decreases with altitude. Codes : (A) Both (A) and (R) are true and (R) is the correct explanation of (A)

(B) Both (A) and (R) are true, but (R) is not a correct explanation of (A) (C) (A) is true, but (R) is false (D) (A) is false, but (R) is true 6. Assertion (A) : The ocean floors are much younger than continents. Reason (R) : Owing to their low density and consequent buoyancy, the ocean floors are not forced down into the mantle in subduction zone. Codes : (A) Both (A) and (R) are true and (R) is the correct explanation of (A) (B) Both (A) and (R) are true and (R) is not a correct explanation of (A) (C) (A) is true, but (R) is false (D) (A) is false, but (R) is true 7. Assertion (A) : The ultimate source of energy that makes posible the erosion of landscapes is sun. Reason (R) : Solar energy evaporates surface water, some of which subsequently falls as rain and snow on high rounds. Codes : (A) Both (A) and (R) are true and (R) is the correct explanation of (A) (B) Both (A) and (R) are true, but (R) is not a correct explanation of (A) (C) (A) is true, but (R) is false (D) (A) is false, but (R) is true 8. Assertion (A) : The magnitude of an earthquake is greatest near the epicentre and decreases with distance. Reason (R) : The energy released by an earthquake can be established from its magnitude. Codes : (A) Both (A) and (R) are true and (R) is the correct explanation of (A)

4 | UGC–NET Geography Solved Papers (B) Both (A) and (R) are true, but (R) is not a correct explanation of (A) (C) (A) is true, but (R) is false (D) (A) is false, but (R) is true 9.

Which of the following can be used for regional delineation ? (A) Gravity Potential Model (B) Rank Size Rule (C) Losch Settlement Model (D) Central Place Model

10. What does not cause Regional Imbalances ? (A) Uneven distribution of resources (B) Sustainable development (C) Lack of resources (D) Lack of technology 11. Nodal regions are delimited— (A) On the basis of spatial interaction (B) On the basis of uniformity (C) On the basis of resource base (D) On the basis flow of goods 12. Migration of people from early periods has had a beneficial impact. The important is— (A) Diffusion of ideas (B) Spread of language (C) Spread of scientific and technological innovations (D) All of the above 13. In rural areas small towns commonly grow up for providing— (A) Market facilities (B) Educational facilities (C) Health facilities (D) Administrative facilities 14. The most important criterion for classification of Indian Urban Centres is— (A) Site (B) Shape (C) Population (D) Function 15. In Rank Size Rule the population of lower Rank City is in proportion to— (A) Population of smallest city (B) Population of largest city (C) Population of middle order city (D) None of the above

16. Who is known as the father of Political Geography ? (A) Rudolf Kjellen (B) K. Haushofer (C) F. Ratzel (D) R. Hartshorne 17. Who gave the concept of Rimland Theory in political geography ? (A) N. J. Spykman (B) E. Warner (C) H. W. Weigert (D) A. T. Mahan 18. Match List-I with List-II and select the correct answer using the codes given below— List-I (Geographer) (a) Vidal de La Blache (b) Jean Brunhes (c) Carl Ritter (d) Fried Ratzel List-II (Principle) 1. Unity in Diversity 2. Social Determinism 3. Terrestrial Whole 4. Interaction Codes : (a) (b) (c) (d) (A) 3 2 4 1 (B) 4 2 1 3 (C) 3 4 1 2 (D) 2 1 3 4 19. The philosophical view advocating that man is responsible for making his own nature is known as— (A) Positivism (B) Functionalism (C) Existentialism (D) Pragmatism 20. Crop combination is calculated on the basis of— (A) Per cent area of net cultivated area (B) Per cent area of gross cultivated area (C) Ranking of crops in a region (D) None of the above 21. Jharia coal fields is located in— (A) Bihar (B) Jharkhand (C) Orissa (D) Chhattisgarh 22. The correct sequence of the advent of prominent British geographers to the development of Geography is— (A) A. J. Herbertson, S. W. Wooldrige, H. J. Mackinder, L. D. Stamp

UGC–NET Geography Solved Papers | 5 (B) S. W. Wooldrige, A. J. Herbertson, L. D. Stamp, H. J. Mackinder (C) H. J. Mackinder, A. J. Herbertson, S. W. Wooldrige, L. D. Stamp (D) L. D. Stamp, A. J. Herbertson, S. W. Wooldrige, H. J. Mackinder 23. Consider the following Seas : 1. Red Sea 2. Black Sea 3. Dead Sea 4. Baltic Sea The correct sequence of those seas in descending order of salinity is— (A) 1, 2, 3, 4 (B) 2, 1, 4, 3 (C) 4, 2, 3, 1 (D) 3, 1, 2, 4 24. When was ‘Project Tiger’, one of the most intensive conservation efforts, launched in India ? (A) 1963 (B) 1967 (C) 1973 (D) 1977 25. Who defined ‘Resource’ as ‘Resources are not they become’ ? (A) Van Thunen (B) Zimmerman (C) Hartshorne (D) Semple 26. Animal rearing on a commercial scale has developed well in— (A) Monsoon Regions (B) Savanna Regions (C) Prairie and Steppe Regions (D) Sabel Regions 27. Which State is the larget producer of wheat in India ? (A) Haryana (B) Punjab (C) Uttar Pradesh (D) Bihar 28. Which one of the following is a Footloose Industry ? (A) Cotton Textile (B) Sugar (C) Cement (D) Handloom 29. Isodopanes are associated with— (A) Line of equal height (B) Line of equal rainfall (C) Line of unequality (D) Line of equal transport cost 30. Which one of the following is not associated with a Transport System ? (A) Von-Thunen Theory

(B) Distance Decay Function (C) Principle of Least Effort (D) Central Place Theory 31. Mongoloid race is predominantly found in— (A) South Africa (B) East Asia (C) Western Europe (D) East Africa 32. Which one of the following is related to the Tribal Economy ? (A) Footloose Industries (B) Settled Cultivation (C) Shifting Cultivation (D) Industrial Economy 33. Racial traits of Man are studied in— (A) Palaeontology (B) Hydrology (C) Sociology (D) Anthropology 34. Who formulated the Heartland Theory ? (A) H. J. Mackinder (B) R. Hartshone (C) Peter Hagget (D) Herbertson 35. A plant adapted to a wet environment is called— (A) Hygrophyte (B) Xerophyte (C) Mesophyte (D) Tropophyte 36. Ecological succession normally leads to— (A) Habitat (B) Biome (C) Climatic Frontier (D) Climax 37. The petro-dollar economy has highly influenced one of the following States more than others— (A) Tamil Nadu (B) Kerala (C) Punjab (D) Gujarat 38. Population distribution in India at State level is best represented by— (A) Dot Method (B) Pie Diagrams (C) Choropleth (D) Isopleth 39. Identify the major objective of using regression model in a data set— (A) Causability between Dependent and Independent variables (B) Association between Dependent and Independent variables (C) Skewness in the data (D) Dispersion in the data

6 | UGC–NET Geography Solved Papers 40. Match List-I with List-II and select the correct answer using the codes given below— List-I (a) What measures of dispersion is defined as the difference between the lowest and highest value ? (b) It is measure of skewness. (c) If the coefficient of variation is desired, it would be found by applying which formula ? (d) Which average is used in computing the variance and standard deviation of sample ? List-II — S 1. 3( x – Median) 2. (100) X 3. Range 4. Arithmetic Mean — Σ(d – x )2 5. n Codes : (a) (b) (c) (d) (A) 3 2 5 4 (B) 3 5 2 4 (C) 4 1 2 3 (D) 3 1 2 4 41. The rank of Indian Railway Network among the countries of the World is— (A) First (B) Second (C) Third (D) Fourth 42. Which of the following is associated with the Cotton Textile Industry ? (A) Pune (B) Varanasi (C) Ahmedabad (D) Chandigarh 43. Kargil is located on the banks of— (A) Indus River (B) Jhelum River (C) Suru River (D) Chenab River 44. Imphal is the capital of— (A) Manipur (B) Nagaland (C) Meghalaya (D) Arunachal Pradesh 45. Mc Mohan Line is an International boundary between— (A) India and Bangladesh (B) India and China (C) India and Pakistan (D) France and Germany

Directions—(Q. 46–50) Read the following passage carefully and answer the given questions— Information consists of new facts, data, ideas, and routine communication. Like people, goods and services, information flows from place-toplace, moving from place of production to demand areas. When we think of demand areas we should not confuse places of need with places of effective demand. In the flows of people, movements among big and vigorous centres are much larger than migrations among the depopulated backwaters of depressed areas. In the shipment of goods, the volume of trade among the developed countries far exceeds their trade contacts with the underdeveloped world. Similarly, information flows primarily among the vigorous idea centres in a system. Islands of lethargy are bypassed because they are literally out of the system. Rich nations trade primarily with each other. Migrations often occur between clusters of mankind. Those who send the most messages get most of the return contacts. A tendency exists for flows to run from places of abundance to areas of effective demand according to the effective pulls exerted at each destination. As flows continue, a tendency toward equilibrium conditions seems common. In the physical world things move through space so that potential can equalize. Water tries to move under the force of gravity to the lowest point and if free to move to sea level it will do so. Similarly, air will move from a high pressure to a low pressure area. High pressure as a causative agent cannot move air very far in one direction. We know this from common experience when we try to blow air across a room with a fan or try to blow out a match at a distance. More air will be moved by putting the fan in one window blowing outward and opening another window across the room. Instead of emphasizing push factors, it makes more sense to think of air being pulled in a certain direction by a zone of low pressure, in the same way that water is pulled from a mountain top. Similarly, in human affairs a pull exerts a directed force with respect to excess supply somewhere else. Therefore, to explain why something moves from A to B it helps to emphasize the pull at B rather than the push at A. The push at A often is unspecified directionally, whereas a pull at B is significant not only for A, but for any other potential supply source as well.

UGC–NET Geography Solved Papers | 7 46. What causes Flow of Information ? (A) Demand (B) Use (C) Quality (D) Volume 47. What factors primarily influence exchange of goods among rich Nations ? (A) Abundance of production (B) Effective demand (C) Level of economic development (D) Generation of information 48. Generation of information depends on— (A) Nature of ideas (B) Strength of idea-centres in a system (C) Duration of ideas (D) Effectiveness of ideas 49. The flow of information from place of origin to place of destination is mainly due to— (A) Repulsion at the place of origin (B) Attraction of the place of destination (C) Distance between place of origin and place destination (D) Abundance at the place of origin 50. Which of the following is the most suitable title for the given passage ? (A) Flow of services (B) Flow of commodity (C) Migration of people (D) Flow of information

Answers with Explanations 1. 6. 11. 16. 21. 26. 27.

(D) 2. (C) 3. (B) 4. (D) 5. (B) (C) 7. (A) 8. (B) 9. (A) 10. (B) (A) 12. (D) 13. (A) 14. (C) 15. (B) (C) 17. (A) 18. (C) 19. (C) 20. (C) (B) 22. (B) 23. (D) 24. (C) 25. (B) (C) (C) The production of wheat from maximum to minimum is—Uttar Pradesh, Punjab, Haryana, Rajasthan, Bihar.

28. (A) Cotton industry is an independent trade. India has monopoly over it since ancient time. The countries importing Indian cotton cloths are—USA, UK, Eastern countries of Europe, Nepal, Singapore, Sri Lanka and African countries. 29. (D) 30. (D) 31. (B) 32. (C) 33. (D) 34. (A) 35. (A) 36. (A) 37. (D) 38. (A) 39. (B) 40. (D) 41. (D) Indian railway occupies fourth place in the World railway and first place in Asia. USA stands at first place and Russia at second. Canada has 5·2% of the total rail routes of the world. 42. (C) Ahmedabad is known for cotton cloths, Pune for IT sector, Varanasi for silk industries, Chandigarh is known as trade centre. 43. (C) 44. (A) The capitals of Northern Eastern states are—Manipur–Imphal, Meghalaya–Shillong, Nagaland–Kohima, Arunachal Pradesh– Itanagar. 45. (B) 46. (A) Information flows from one place to other like people, commodity and services. 47. (B) Flow has a tendency to move from abundance to effective demand. In affluent countries, same principle applies to the exchange of commodities. 48. (B) Production of information entirely depends on the strength of the centre of thoughts. This is the reason that the trade between the countries of same strength takes place in big quantity. 49. (B) Information freely moves from the place of its origin to the destination due to powerful attraction. 50. (D) In the given paragraph, the flow of information is shown as people, commodity and services towards demand. There the suitable title is ‘Flow of Information’.

June 2007 Geography Paper-II Note—This paper contains fifty (50) objective type questions, each question carrying two (2) marks. Attempt all of them. 1. Whaleback dunes are characteristic features of— (A) Thar desert (B) Gobi desert (C) Australian desert (D) Sahara desert 2. Given below are two statements, one labelled as Assertion (A) and the other lebelled as Reason (R). Select your answer from the codes given below— Assertion (A) : In arid and semi-arid regions, sheet flood is an important land forming process. Reason (R) : Arid and semi-arid regions experience high intensity short-duration rainfall. Codes : (A) Both (A) and (R) are true and (R) is the correct explanation of (A) (B) Both (A) and (R) are true, but (R) is not the correct explanation of (A) (C) (A) is true, but (R) is false (D) (R) is true, but (A) is false 3.

Given below are two statements, one labelled as Assertion (A) and the other lebelled as Reason (R). Select your answer from the codes given below— Assertion (A): Glacial troughs often exhibit remarkably flat floors. Reason (R) : The flat floor in a glacial trough is produced by uniform glacial erosion. Codes : (A) Both (A) and (R) are true and (R) is the correct explanation of (A) (B) Both (A) and (R) are true, but (R) is not the correct explanation of (A)

(C) (A) is false, but (R) is true (D) (R) is true, but (A) is false 4.

5.

6.

7.

Match List-I with List-II and select the correct answer from the codes given below— List-I (Authors) (a) Thornbury (b) Dury (c) Steers (d) Strahler List-II (Books) 1. Rivers and River terraces 2. Principles of Geomorphology 3. Physical Geography 4. The Unstable Earth Codes : (a) (b) (c) (d) (A) 2 1 4 3 (B) 2 4 1 3 (C) 2 1 3 4 (D) 2 3 1 4 Who propounded the concept of dynamic equilibrium in geomorphology ? (A) Davis (B) King (C) Hack (D) Crickmay Global pattern of surface temperature is generally controlled by— (A) Latitude, Longitude and Altitude (B) Latitude, Longitude and ContinentOcean contrast (C) Continent-Ocean contrast, Longitude and Altitude (D) Continent-Ocean contrast, Latitude and Altitude The criteria used by Thomthwaite in his classification of World Climates are— 1. Precipitation effectiveness 2. Thermal efficiency 3. Potential evapotranspiration 4. Radiation efficiency

UGC–NET Geography Solved Papers | 9 Select your answer from the codes given below— Codes : (A) 1, 2 and 4 (B) 1, 2 and 3 (C) 1, 3 and 4 (D) 2, 3 and 4 8.

The earth’s average albedo is estimated as— (A) 32 per cent (B) 40 per cent (C) 21 per cent (D) 50 per cent

9.

Obsequent streams generally flow— (A) Along the consequent stream (B) Opposite to the consequent stream (C) At right angle to the consequent stream (D) Diagonal to the consequent stream

(B) It started with increased wheat production in Punjab, Haryana and Western U.P. (C) It has resulted in increase in disparity in rural income (D) It has caused substantial increase in the production of millets 17. Coffee as a plantation crop is likely to be adversely affected by many causes of which the most important one is— (A) Decline in international prices (B) Attack by pests and diseases (C) Tariff barriers for trade (D) Development of substitutes

10. The principle of uniformitarianism in Geomorphology was first advocated by— (A) Davis (B) Playfair (C) Gilbert (D) Hutton

18. Taaffe’s transportation model has a number of development phases. The number of phases are— (A) Four (B) Two (C) Six (D) Eight

11. Which of the following steel plants of India has the least cost location in the ideal location triangle conceived by Weber ? (A) Bokaro (B) Bhilai (C) Rourkela (D) Durgapur

19. Which of the following arrangements of biomes in order of availability of soil, water and heat is correct ? (A) Savanna, grassland, forest, tundra and desert (B) Savanna, forest, grassland, tundra and desert (C) Tundra, forest, grassland, savanna and desert (D) Forest, savanna, grassland, desert and tundra 20. Mangrove forests are mainly found along the sea faces of the delta of which one of the following rivers ? (A) Mahanadi (B) Krishna (C) Cauvery (D) Ganga

12. Gonds have their major concentration in— (A) Koraput region (B) Aravali region (C) Chota Nagpur region (D) Bastar region 13. Topographical maps are produced in India by— (A) N.A.T.M.O. (B) Survey of India (C) N.R.S.A. (D) D.R.D.O. 14. Which one of the following is the commonly used measure of central tendency ? (A) Mean (B) Median (C) Mode (D) Harmonic mean 15. Identify the author who wrote several volumes entitled ‘GEOGRAPHICA’— (A) Eratosthenes (B) Ptolemy (C) Strabo (D) Herodotus 16. Which of the following statements about the Green Revolution in India is not true ? (A) It was characterised by the use of high yielding variety of seeds and intensive irrigation and application of chemical fertilizers

21. Which one of the following is known as the ‘Carbon Sink’ of the globe ? (A) Pacific Ocean (B) Equatorial rain forests (C) Antartica (D) Savanna 22. Deep sea floor comprises which of the following deposits ? (A) Muds and Sands (B) Sands and Gravels (C) Muds and Gravels (D) Muds and Red clay

10 | UGC–NET Geography Solved Papers 23. Which one of the following is the most important air pollutants ? (A) Carbon dioxides (B) Sulphur dioxides (C) Hydro carbons (D) SPM 24. Mechanisation of agriculture is best suited for countries with— (A) High population density (B) Low population density (C) Vast agricultural land (D) High standard of living 25. Which one of the following crops enters international trade in large quantities ? (A) Rice (B) Wheat (C) Corn (D) Barley 26. The book entitled ‘Socio-Economic Models’ was edited by— (A) Spate and Learmonth (B) Herbert and Smith (C) Johnston and Herbert (D) Chorley and Haggett 27. Who among the following Arab Geographers spent several years in India and wrote a book on India ? (A) Ibn Batuta (B) Al Idrisi (C) Ibn Khaldun (D) Al-Biruni 28. Who propounded the organic theory of the State ? (A) Humboldt (B) Richthofen (C) Ratzel (D) Ritter 29. Name the scholar who used the term geopolitik for the first time before the Second World War— (A) Mackinder (B) Kjellan (C) Houshofer (D) Spykman 30. Arunachal Pradesh was formerly known as— (A) NWEP (B) NEFA (C) Buffer zone (D) Frontier 31. Which of the following tribal combination is confined to a single State of North-East India ? (A) Khasi — Naga (B) Garo — Khasi (C) Naga — Mizo (D) Khasi — Mizo 32. Which one of the following indicators is not relevant in the context of social well being ? (A) Provision of housing (B) Improved accessibility to health facilities

(C) Provision of educational facilities (D) Entry of multinational companies in developing countries 33. The concept of ‘Cultural health’ was first used by— (A) Berkeley School of Geography (B) French School of Geography (C) German School of Geography (D) Chicago School of Geography 34. Winter rains in the North Indian Plains are caused by depressions coming from the— (A) Arabian Sea (B) Bay of Bengal (C) Mediterranean Sea (D) Pacific Ocean 35. Match List-I with List-II and select the correct answer from the codes given below— List-I (a) Petroleum (b) Paper Mill (c) Mica (d) Hiracud Project List-II 1. Amlai 2. Jharkhand 3. Orissa 4. Cambay basin Codes : (a) (b) (c) (d) (A) 4 1 2 3 (B) 3 2 1 4 (C) 2 3 4 1 (D) 1 4 3 2 36. The leading saffron producing State of India is— (A) Himachal Pradesh (B) Arunachal Pradesh (C) Jammu and Kashmir (D) Nagaland 37. The theory of ‘demographic transition’ is developed by— (A) Warren Thompson (B) C. P. Blacker (C) Frank Notestein (D) A. J. Coale and E. M. Hoover 38. Which of the following is the correct sequence in terms of density of population ? (A) Africa, Asia, America, Europe (B) America, Asia, Africa, Europe

UGC–NET Geography Solved Papers | 11 (C) Europe, Asia, America, Africa (D) Asia, Europe, Africa, America 39. Which one of the following indicators is a direct measure of regional imbalance in development ? (A) Percentage of cultivated area to total geographical area (B) Per capita consumer expenditure (C) Accessibility to roads (D) Literacy 40. Which one of the following attributes is not spatial data base ? (A) Location (B) Lines (C) Polygon (D) Triangle 41. Which of the following pairs does not match correctly ? (A) Malajkhand — Coal (B) Bailadila — Iron ore (C) Kudremukh — Iron ore (D) Neyveli — Lignite 42. The predominent soil type developed on the Archaean and Cuddapah geological systems in India as— (A) Black soils (B) Alluvial soils (C) Red soils (D) Intrazonal soils 43. Siachin glacier is located in— (A) Greater Himalayas (B) Trans Himalayas (C) Lesser Himalayas (D) Sub-Himalayas 44. The leading green tea producing country in the World is— (A) China (B) Sri Lanka (C) Kenya (D) India 45. Who tried to resolve the problem of dualism in geography by modifying the ‘environment’ concept with major focus on man ? (A) Vidal de la Blache (B) H. H. Barrows (C) E. Huntington (D) E. C. Semple 46. Which of the following pairs does not match correctly ? (A) Satpura Mountain — Mahadeo hills

(B) Chhota Nagpur — Rajmahal hills Plateau (C) Himalaya Mount — Khasi hills ains (D) Vindhyan Range — Bhander Range Directions—(Q. 47–50) Read the following passage carefully and answer the given questions— In order to formulate a regional development policy in national planning with special reference to the federating States, an inventory of the spatial and locational patterns of the key sectors of the national economy is needed. On the basis of the analysis of the inventory, the country may be divided into 5 or 6 macro-regions comprising groups of federating States. Contiguity, inter-State and intra-State relationship and the resource potential for developing distinct regional economies could be the considerations in regional delineation. These would be the regions for formulating a regional development startegy in National Planninng. For this purpose an attempt should then be made to analyse the existing spatial and locational components of the national development plans and to indicate alternative locations and areas within each of the macro-region to which the additional targets of production could be allocated. This analysis involves a study of the spatial dimension of the individual sector as well as regional and inter-regional analysis. The acceptance of any one inter-regional sectoral plan and the corresponding location framework would be subject to political considerations too. It is in this context that the regional development policy has to be formulated in the national plan. The impact of such a development policy on the goals of economic and social development should further be spelt out on the basis of federating States which are the parts of the macro-regions. For a lack of the inventory on locational and spatial aspects and inadequate regional economic analysis of the development plans, the national plan leans heavily on the aggregated sectoral plans of States and inter-State comparison of selected sectors of development. The spatial and the locational framework arrived at in the manner mentioned above would serve to pin-point areas of growth and corresponding locations that take care of the key sectors of the national plan. Political factors that enter into actual policy formulation would then have an objective basis for reconciliation. More intensive analysis is needed at the State level for integrating

12 | UGC–NET Geography Solved Papers the growth areas and growth-points evolved out of the regional development policy consideration into the plans of different sub-regions within the States. 47. The central theme of the passage is concerned with— (A) The importance of regional development policy (B) The need for inventory of the spatial and locational patterns (C) The goals of economic and social development (D) The sectoral plans of States 48. The crux of the problem for regional development planning is— (A) Leaning heavily on the sectoral plans of State (B) Political factors entering into actual policy formulation (C) Integration of growth areas and growth points (D) Lack of inventory of the spatial and locational aspects 49. Dividing the country into macro-regions comprising groups of federating States could be done on the basis of— (A) The inter-State and intra-State plans (B) The analysis of inventory (C) The individual sector analysis (D) The inter-regional analysis 50. Political considerations are very much essential in— (A) National planning (B) Regional development policy

(C) Inter-sectoral development (D) All of the above

Answers with Explanations 1. 6. 11. 12.

13. 18. 23. 28. 31. 32. 37. 42. 47. 48.

49. 50.

(B) 2. (C) 3. (C) 4. (A) 5. (C) (D) 7. (B) 8. (A) 9. (B) 10. (D) (A) (D) Following is the concentration of Gonds (from higher to lower)—Bastar, Chandrapur, Durg, Andhra Pradesh (Southern–Western parts); and Southern Western parts of Odisha. (B) 14. (B) 15. (C) 16. (D) 17. (D) (A) 19. (D) 20. (D) 21. (B) 22. (D) (A) 24. (C) 25. (B) 26. (D) 27. (D) (C) 29. (C) 30. (B) (B) Garo and Khasi are mainly confined to the eastern part of Meghalaya. It has maximum population of both the tribes. (D) 33. (A) 34. (C) 35. (A) 36. (C) (C) 38. (C) 39. (A) 40. (D) 41. (A) (C) 43. (B) 44. (A) 45. (A) 46. (C) (A) The main subject of the text is to study National economy for State Development Policy under National Planning. (A) The main subject of the problem of State Development Planning is excessive dependence on divisional projects of the state, whereas it should be solved by developing the existing structure. (B) It could be done by dividing the country into Macro-regions comprising of groups of federating states and study of their inventory. (D) For political plans, state development policy and inter divisional development, political determination is required. For this political thoughts must be amalgamated with deep analysis to form the policy thereby.

December 2007 Geography Paper-II Note—This paper contains fifty (50) objective type questions, each question carrying two (2) marks. Attempt all of them. 61. Which one of the following forces is responsible for syncline formation ? (A) Crustal faulting (B) Crustal rifting (C) Crustal compression (D) Crustal sinking 62. Pediplains and inselbergs are features of the old stage of which cycle of erosion ? (A) Glacial (B) Karst (C) Arid (D) Marine 63. Which of the following terms does not indicate similar processes ? (A) Diastrophism (B) Folding (C) Exfoliation (D) Warping 64. In which of the following the direction of flow of streams is controlled by rock structure ? (A) Consequent (B) Subsequent (C) Insequent (D) Obsequent 65. Match List-I with List-II and mark the correct answer in the codes given below— List-I (a) Wind erosion (b) Groundwater depletion (c) Glacial Deposition (d) River erosion List-II 1. Drumlins 2. Stalagmites 3. Barkhans 4. Peneplains Codes : (a) (b) (c) (d) (A) 3 2 1 4 (B) 2 1 4 3

(C) (D)

2 4

4 2

3 3

1 1

66. What is recorded with the help of a hydrograph ? (A) Wind velocity (B) Vapour retention (C) Relative humidity (D) Water discharge 07. The increase in the chloroflurocarbon in the atmosphere is associated with— (A) Ozone depletion (B) Sea level rise (C) Aquatic life depletion (D) Intense sonic pollution 68. What is the point of origin of an earthquake called ? (A) Quake centre (B) Epicentre (C) Seismic focus (D) Tectonic point 69. Surface inversion of temperature is a phenomenon related to— (A) Increase in temperature with increase in elevation (B) Decrease in temperature with increasing elevation (C) Decrease in temperature with decreasing elevation (D) Increase in temperature with decreasing elevation 10. Who had used the concept of evapo transpiration in climate classification ? (A) Thornthwaite (B) Koppen (C) Kendrew (D) Austin Miller

14 | UGC–NET Geography Solved Papers 11. Eustatic changes in the sea levels are caused by— (A) Ice ages (B) Epirogenic movements (C) Orogenic movements (D) Isostatic changes 12. Which of the following is a coral island ? (A) Tasmania (B) Lakshadweep (C) Maldives (D) Rameswaram 13. In which of the following States the CHIPCO movement was started ? (A) Madhya Pradesh (B) Jharkhand (C) Uttaranchal (D) Rajasthan 14. Which one of the following is Taiga biome ? (A) Sub-Arctic biome (B) Sub-Saharan biome (C) Savanna grass (D) None of the above 15. Match List-I with List-II and select the correct answer from the codes given below— List-I (a) Coral Reef (b) Shoal (c) Abyssal Plain (d) Continental Slope List-II 1. Organic ocean deposit 2. Deep, relatively flat surface of the ocean floor 3. Steep descent from the shelf break 4. Detatched elevation with shallow depths Codes : (a) (b) (c) (d) (A) 2 3 1 4 (B) 1 4 2 3 (C) 3 2 4 1 (D) 4 1 3 2 16. Given below are two statements, one labelled as Assertion (A) and the other lebelled as Reason (R). Select your answer from the codes given below— Assertion (A): Ritter did not state the relationship between history and geography as clearly as Humboldt.

Reason (R)

: Immanual Kant understood the relationship between history and geography much more clearly than Ritter.

Code : (A) Both (A) and (R) are correct and (R) explains (A) (B) Both (A) and (R) are correct, but (R) does not explain (A) (C) (A) is true, but (R) is false (D) (A) is false, but (R) is true 17. The book ‘COSMOS’ of Alexander Von Humboldt was based on his travel to one of the following continents— (A) Asia (B) North America (C) Europe (D) South America 18. Given below are two statements, one labelled as Assertion (A) and the other labelled as Reason (R). Select your answer from the codes given below— Assertion (A) : Humboldt had no immediate followers in academic circle in Germany, since he never occupied any academic position. Reason (R) : The real followers of Humboldt scientific travellers. Codes : (A) Both (A) and (R) are correct and (R) explains (A) (B) Both (A) and (R) are correct, but (R) does not explain (A) (C) (A) is true, but (R) is false (D) (A) is false, but (R) is true 19. “Nature does not drive man along a particular road, but it offers a number of opportunities from among which man is free to select”. This statement implies— (A) Deterministic approach (B) Possibilistic approach (C) Deterministic and possibilistic both approach (D) Probabilistic approach

UGC–NET Geography Solved Papers | 15 20. Match the following List-I with List-II and select the correct answer from the codes given below— List-I (Name of Geographers) (a) Alexander Von Humboldt (b) Ellen Churchil Semple (c) Peter Haggett (d) Jean Brunhes List-II (Name of Country) 1. France 2. U.K. 3. U.S.A. 4. Germany Codes : (a) (b) (c) (d) (A) 4 2 3 1 (B) 1 2 3 4 (C) 4 3 2 1 (D) 2 3 1 4 21. Urban environmental degradation is caused by various factors : 1. Suspended Particular Matter (SPM) 2. Polluted Water 3. Carbondioxide 4. Vehicular emission Choose the correct answer— (A) 1, 2, 3 and 4 are correct (B) 2 and 4 are correct (C) 2 and 3 are correct (D) 1 and 4 are correct 22. An increase in urban population takes place at a rate much faster than rural populations because— (A) Urban population has high birth rate (B) Rural population has high death rate (C) Large number of people migrate to urban centres (D) Urban centres offer better scope for employment 23. The most important criterion for classification of towns is— (A) Population (B) Site (C) Shape (D) Functions

24. A high and gradually declining fertility of over 30 per thousand and a sharply reduced mortality rate of over 15 per thousand is characteristic of demographic transition of— (A) First stage (B) Second stage (C) Third stage (D) Last stage 25. When the population density of an area is obtained through division of the total population by the total agricultural area it is termed— (A) Arithmetic density (B) Agricultural density (C) Economic density (D) Physiological density 26. North Atlantic Oceanic route is the World’s most important route because— (A) There is Labrodor cold-water current (B) There are developed nations on both sides (C) There is U.S.A. on the west coast (D) It is a safe navigation route 27. Which one of the following is not related to Plantation Agriculture ? (A) Tea cultivation (B) Coffee cultivation (C) Rubber latex production (D) Soyabean cultivation 28. Given below are two statements, one labelled as Assertion (A) and the other lebelled as Reason (R). Select your answer from the codes given below— Assertion (A) : Weber’s theory hinges on transportation costs but ignores the fact that freight rates may not always increase in direct proportion to the distance. Reason (R) : The freight rates, however, may not be similar for raw material and finished goods. Codes : (A) Both (A) and (R) are correct and (R) explains (A) (B) Both (A) and (R) are correct, but (R) does not explain (A) (C) (A) is correct, but (R) is wrong (D) (A) is wrong, but (R) is correct

16 | UGC–NET Geography Solved Papers 29. Which one of the following is a large scale map ? (A) 1 : 2,50,000 (B) 1 : 10,00,000 (C) 1 : 25,000 (D) 1 : 50,000 30. The best-suited soil for cotton cultivation is— (A) Laterite soil (B) Regur soil (C) Sandy soil (D) Loamy soil 31. Given below are two statements, one labelled as Assertion (A) and the other labelled as Reason (R). Select your answer from the codes given below— Assertion (A) : Virgin soils newly cleared off forests account for highest yield of coffee. Reason (R) : Coffee is a soil exhaustive crop. Codes : (A) Both (A) and (R) are correct and (R) explains (A) (B) Both (A) and (R) are correct, but (R) does not explain (A) (C) (A) is right, but (R) is wrong (D) (A) is wrong, but (R) is right 32. Cultural landscape shows— (A) Human impact on environment (B) Dominance of nature (C) Impact of ecological factors (D) Association of fauna and flora

36. Which of the Indian states has the maximum number of common borders with other Indian States ? (A) West Bengal (B) Madhya Pradesh (C) Chhattisgarh (D) Karnataka 37. Which one of the following attributes is not a measure of Human Development ? (A) Literacy (B) Life expectancy (C) Per capita income (D) Consumer expenditure 38. Given below are two statements, one labelled as Assertion (A) and the other labelled as Reason (R). Select your answer from the codes given below— Assertion (A) : The levels of urbanization in the Advanced countries followed an elongated ‘S’ shaped curve. Reason (R) : The levels of urbanization in India has followed almost a straight line. Codes : (A) Both (A) and (R) are correct and (R) explains (A) (B) Both (A) and (R) are correct, but (R) does not explain (A) (C) (A) is correct, but (R) is wrong (D) (A) is wrong, but (R) is correct

33. Which one of the following districts is dominated by tribal population ? (A) Kanyakumari (B) Bastar (C) Rohtak (D) Karnal

39. Which of the following is the most suitable term to define a metropolitan region ? (A) Formal (B) Functional (C) Economic (D) Generic

34. The primitive people who live in Kalahari desert are known as— (A) Pigmies (B) Bantus (C) Bushman (D) Todas

40. Who among the following suggested the triad of mutual relationship among “Place, work and folk” ? (A) Le play (B) Clarence Stein (C) Le Corbusier (D) Ebenezer Howard

35. Which one of the following has no caste in the scheduled category ? (A) Nagaland (B) Manipur (C) Dadra and Nagar Haveli (D) Puducherry

41. Factors that contribute to minimisation of regional imbalances in development are many. Which one of the following is the most relevant ? (A) Increasing accessibility (B) Strengthening forward and backward linkages of regional economies

UGC–NET Geography Solved Papers | 17 (C) Investment in less developed regions (D) Planning the development of metropolitan centres 42. Which one of the following states is the leading one in terms of total fish catch ? (A) Kerala (B) Tamil Nadu (C) West Bengal (D) Gujarat 43. Which river out of the following is older than the Himalayas ? (A) Sutlej (B) Ganga (C) Beas (D) Ravi 44. Which one of the following states experienced heavy snowfall during February 2005 ? (A) Assam (B) J&K (C) Mizoram (D) Sikkim 45. Which one of the following is correct ? (A) Arithmetic mean is a numerical value (B) Arithmetic mean is not affected by the variability in the data set (C) Arithmetic mean is always a positive number (D) Arithmetic mean is not useful for any further statistical analysis of the data 46. Which of the following is the appropriate method for study of dispersion ? (A) Mean (B) Median (C) Mode (D) Standard Deviation 47. Isohrone Map shows lines connecting— (A) Equal travel time (B) Equal transport costs (C) Equal travel distance (D) Areas with similar accessibility 48. Dasymetric map is an improved version of choropleth map as it effectively shows the following— (A) The homogeneity and rapid change (B) The differences and uniqueness (C) The gradual areal changes (D) The random distribution Directions—(Q. 49 and 50) Read the following passage carefully and answer the given questions— Human geography has used absolute, relative and relational concepts of space. In the absolute conception, space is considered a distinct physical, empirical and real entity. In the relative

conception space is a relation between events and thus related to time and process. In the relational conception space is not viewed in term of its geometry, but in terms of how spatial relations are carved out through human practice, that is through spatial organization. 49. Morphological studies adopt which concept of space ? (A) Absolute (B) Relational (C) Relative (D) Both (A) and (C) 50. Spatial organization is primarily based on— (A) Spatial geometry (B) Spatial process (C) Spatial relations (D) Spatial-Social dialectic

Answers with Explanations 1. (C) Syncline formation is attached with foldings, which is due to crustal compression. 2. (C) Pediplains & Inselbergs are the features associated with Aid region. 3. (C) Diastrophism, folding and warping are the same features which exfoliation is associated with, breaking, splitting and peeling of outer rock layers caused by the expansion of salt crystals with in the interstices of the rock surface. 4. (A) 5. (A) 6. (C) Relative humidity is recorded with the help of hydrograph. 7. (A) Increase in CFC in the atmosphere is associatic with ozone depletion. 8. (C) The point of origin of an earthquake is known as seismic focus or focus. 9. (A) In normal lapse rate there is a decrease in temperature with increasing altitude, but in surface inversion there is a increase in temperature with increase in elevation. 10. (A) It was Thornthwaite who used the concept of evapo-transpiration in his climatic classification of world. 11. (A) Eustatic changes in sea level is caused by decay of development of ice-sheet. 12. (B) Lakshadweep island is a coral island. 13. (C) CHIPCO Movement was Started at Uttranchal/Uttarakhand. 14. (A) 15. (B)

18 | UGC–NET Geography Solved Papers 16. (D) Ritter, like Vidal-de-la Blache was of the opinion that earth and its inhabitants stand in the closest reciprocal relations and are cannot be truly presented in all its relationship without the other. Hence, History and Geography must always remains in separable. 17. (D) 18. (C) 19. (D) 20. (C) 21. (A) 22. (C) In order to get jobs, and education alongwith other facilities, large number of people migrate to urban areas, cause increase in urban population. 23. (A) Indian towns has been classified according to size of population. 24. (B) 25. (B) 26. (B) North Atlantic Oceanic route is not only the most important sea route, but also one of the busiest route. It is due to the developed nations like USA, Canada, England and French, on its both side (i.e. in East & West). 27. (D) Tea, Coffee and Rubber can be grown in plantation agriculture, but Soyabean cannot be grown as a plantation agriculture. 28. (A) 29. (C) 30. (B) 31. (A) 32. (A)

33. (B) Bastar district of Chhattisgarh is dominated by the tribal people. 34. (C) Bushman primitive people live in Kalahari desert of South Africa. 35. (A) As per census of 2001, Nagaland does not has any Scheduled caste population. 36. (C) Chhattisgarh is surrounded by 6 states (i.e. Uttar Pradesh, Jharkhand, Orissa, Andhra Pradesh, Maharashtra and Madhya Pradesh) While West Bengal, Madhya Pradesh and Karnataka are surrounded by 5 states each. 37. (D) Measurement of Human development India is 1. Literacy, 2. Life expectancy, 3. Percapita income. But the consumer expenditure is not the measurement of Human Development. 38. (C) 39. (B) 40. (A) Le-play has suggest the trio idea of relationship of place, work and folk. 41. (C) 42. (C) 43. (A) 44. (B) 45. (A) 46. (B) Median is used for the study of dispersion. 47. (D) 48. (A) 49. (B) 50. (D)

June 2008 Geography Paper-II Note—This paper contains fifty (50) objective type questions, each question carrying two (2) marks. Attempt all of them. 1. V-shaped valley is formed during the— (A) Mature stage of glacial erosion (B) Old age of fluvial cycle of erosion (C) Youthful stage of fulvial cycle of erosion (D) Old age of arid cycle 2.

The term ‘Pangea’ was coined by– (A) Alfred Wegener (B) A. N. Strahler (C) Sharp (D) W. M. Smith

3.

The ocean current Benguela is noticed in— (A) Pacific Ocean (B) Indian Ocean (C) Atlantic Ocen (D) Arctic Ocean

4.

The concept ‘‘landscape is a function of structure, process and stage’’ was evolved by— (A) Griffith Taylor (B) Davis (C) Penck (D) Thornbury

5.

Acid rain is caused by— (A) Excess carbon dioxide in the atmosphere (B) Washout of sulphuric acid by precipitation (C) Excess in greenhouse gases emission from tropical forests (D) Abnormal variations in composition of gases in the atmosphere Given below are two statements, one labelled as Assertion (A) and the other labelled as Reason (R). Select your answer from the codes given ahead— Assertion (A) : The river valleys are usually narrow shaped in the youthful stage. Reason (R) : Pothole drilling is a dominant erosional process responsible for down cutting of river valleys.

6.

Codes : (A) Both (A) and (R) are true and (R) is the correct explanation of (A) (B) Both (A) and (R) are true, but (R) is not the correct explanation of (A) (C) (A) is true, but (R) is false (D) (A) is false, but (R) is true 7.

8.

Which of the following types of clouds occupy the lowest layer ? (A) Stratus (B) Cumulus (C) Cumulonimbus (D) Cirrus

Pollution plume is the result of— (A) A regional wind caused by a pressure gradient (B) Unequal distribution of temperature (C) Abnormalities caused by latent heat of condensation (D) Interaction among different types of air masses 9. Polar frontal theory is associated with— (A) Origin of trade wind tract (B) Origin of tropical cyclones (C) Origin of Tornadoes (D) Origin of Jet Stream 10. Atoll is one of the stages of— (A) A natural reservoir (B) A pressure system (C) A coral development (D) Development of the carbon sink in the ocean 11. Match List-I with List-II and select the correct answer from the codes given ahead— List-I (Physiographic Region) (a) Aravallis (b) Karnataka Plateau (c) Satpura Mountains (d) Himalayas

20 | UGC–NET Geography Solved Papers List-II (Hills/Range) 1. Mishmi hills 3. Baba Budan hills Codes : (a) (b) (c) (A) 1 2 3 (B) 4 3 2 (C) 3 2 1 (D) 2 3 4

2. Delhi ridge 4. Maikal range (d) 4 1 4 1

12. The following are the four land use zones assumed to exist around the central town in an isolated state according to the Von Thunen’s Model : 1. Firewood and lumber. 2. Intensive crop farming (grazing and root crops). 3. Crop farming (with fallow and pasture). 4. Market gardening and fresh milk. Select the correct sequence of the land use zones from centre to outward using the codes given below— Codes : (A) 1, 2, 3 and 4 (B) 4, 2, 3 and 1 (C) 4, 1, 2 and 3 (D) 3, 2, 4 and 1 13. Who propounded the concept of areal differentiation ? (A) Jean Brunhes (B) Richard Hartshorne (C) David M. Smith (D) H. S. Barrows 14. Match List-I with List-II and select the correct answer from the codes given below— List-I (a) Geographic Humane (b) Influence of Geographic Environment (c) Erdkunde (d) Almagest List-II 1. Ptolemy 2. Carl Ritter 3. Jean Brunhes 4. E. C. Semple Codes : (a) (b) (c) (d) (A) 3 4 2 1 (B) 4 3 1 2 (C) 1 2 3 4 (D) 2 1 4 3

15. Quantitative Revolution in Britain was mainly popularised by— (A) David M. Smith (B) T. W. Freeman (C) Peter Haggett (D) L. Dudley Stamp 16. Which one of the following sequences related to the development of geographical thought is correct ? (A) Kant-Varenius-Vidal-de-la Blach-Humboldt (B) Varenius-Kant-Humboldt-Vidal-de-la Blach (C) Vidal-de-la Blach-Humboldt-VareniusKant (D) Humboldt-Varenius-Kant-Vidal-de-la Blach 17. Settlement pattern in a region is affected in India by— (A) Social infrastructure (B) Physical Linkages (C) Population growth-rate (D) Agricultural Practices 18. A city region is marked with— (A) Under development (B) Backwardness (C) Availability of mineral resources (D) Concentrated development 19. Decadal population growth-rate in India during 1991-2001 was— (A) 20·11% (B) 26·20% (C) 23·60% (D) 21·34% 20. Centrality of a place is determined on the basis of its— (A) Transport modes (B) Administrative status (C) Functional strength (D) Number of education institutions 21. Name the propounder of Rimland theory— (A) Mackinder (B) Spykman (C) Monkhouse (D) Hartshorne 22. Which one of the following Transport models is suitable for rural development in India ? (A) Truck–Railway–Shipment–Bullockcart (B) Bullockcart–Truck–Railway–Ships (C) Railway–Ship–Air Transport–Bullockcart (D) Bullockcart–Railway–Air transport–Ship

UGC–NET Geography Solved Papers | 21 23. Who among the following was first to formulate the method of crop-combination ? (A) Nelosn (B) Doi (C) Rafiullah (D) Weaver 24. Population migration streams are determined by— (A) Political decision (B) Push and pull factor (C) Historical events (D) Religious factor 25. Hoyt propounded— (A) Sector model of city structure (B) Rank-size rule (C) Concept of Primate City (D) Multi-nuclei model of city structure 26. Match the List-I with List-II and select the correct answer from the codes given below— List-I (State) (a) Orissa (b) Jharkhand (c) Madhya Pradesh (d) Rajasthan List-II (Iron ore reserves %) 1. 0·7 2. 18 3. 26 4. 22 Codes : (a) (b) (c) (d) (A) 4 3 2 1 (B) 1 2 3 4 (C) 3 1 2 4 (D) 2 3 1 4 27. Demographic transition model is based on data from the demographic experience of— (A) North Western Europe (B) USA (C) South-East Asia (D) South America 28. Given below are two statements, one labelled as Assertion (A) and the other lebelled as Reason (R). Select your answer from the codes given ahead— Assertion (A) : Japan is a developed country. Reason (R) : It is necessarily critical to have natural resources within the country for its development.

Codes : (A) Both (A) and (R) are true and (R) is the correct explanation of (A) (B) Both (A) and (R) are true, but (R) is not the correct explanation of (A) (C) (A) is true, but(R) is false (D) (R) is true, but (A) is false 29. Name the theory of Industrialization proposed by Weber— (A) Transport–cost theory (B) Resource–based theory (C) Footloose industires theory (D) Labour–based theory 30. Modern concept of agricultural diversification is related to— (A) Large number of crops (B) Value added crops (C) Monoculture crops (D) Ecologically most suitable crops 31. In which year did Von-Thunen develop the model of agricultural land use Zonation ? (A) 1817 (B) 1826 (C) 1850 (D) 1847 32. Gond’s concentration is largely located in— (A) Indian Thar desert (B) Baghel Khand Region (C) Bastar Region (D) Bundel Khand Region 33. Which one of the following crop is used for large-scale supply and demand in international market ? (A) Millets (B) Rice (C) Wheat (D) Gram 34. The origin of Monsoon in the Indian subcontinent is largely linked with— (A) EI Nino and ENSO Events (B) Localised weather perturbations (C) Shape of the Indian Sub-continent (D) Sea surface temperature variation in the Indian Ocean 35. Which one of the following sequences of States is in the decreasing order of sex ratio ? (A) Andhra Pradesh–Tamil Nadu–Chhattisgarh–Kerala (B) Tamil Nadu–Kerala–Andhra Pradesh– Chhattisgarh

22 | UGC–NET Geography Solved Papers (C) Kerala–Chhattisgarh–Tamil Nadu– Andhra Pradesh (D) Chhattisgarh–Andhra Pradesh–Kerala– Tamil Nadu 36. Meghalaya State is known for the following Tribal combination— (A) Khasi–Mizo (B) Garo–Khasi (C) Naga–Mizo (D) Khasi–Naga 37. Given below are two statements, one labelled as Assertion (A) and the other labelled as Reason (R). Select your answer from the codes given below— Assertaion (A) : The new industrial towns are numerically dominated by make adult population. Reason (R) : Migration is age and sex selective. Codes : (A) Both (A) and (R) are true and (R) is the correct explanation of (A) (B) Both (A) and (R) are true, but (R) is not the correct explanation of (A) (C) (A) is true, but (R) is false (D) (A) is false, but (R) is true 38. The Western Ghats in Maharashtra is known is— (A) Nilgiris (B) Cardmom Hills (C) Annamalai (D) Sahyadris 39. Map showing the depths of Oceans and Seas is termed as— (A) Ethnographic map (B) Orographic map (C) Bathymetric map (D) Geological map 40. Which one of the following is largest scale of map ? (A) 1 : 500,000 (B) 1 : 100,000 (C) 1 : 50,000 (D) 1 : 10,000 41. River basin is a— (A) Micro-region (B) Macro-region (C) Hilly region (D) Backward region 42. Which one of the following is a Nodal Region ? (A) Industrial Region (B) Tribal Region (C) Metropolitan Region (D) Central district in a State

43. Forward linkages facilitate— (A) Diversification (B) Unbalanced development (C) Excessive pressure on resource (D) Social development 44. National Remote Sensing Agency (NRSA) is located at— (A) Dehradun (B) Hyderabad (C) Bangalore (D) Thumba 45. Which one of the following is centrographic measure ? (A) Standard distance (B) Median (C) Arithmetic Mean (D) Geometric Mean 46. Match the List-I with List-II and select the correct answer from the codes given below— List-I (a) Mathematical Tradition (b) Exploration (c) Inductive Method (d) Literary Tradition List-II 1. Phoenician 2. Thales 3. Homer 4. Aristotle Codes : (a) (b) (c) (d) (A) 3 1 2 4 (B) 1 2 3 4 (C) 4 3 2 1 (D) 2 1 4 3 Directions—(Q. 47–50) Read the following passage and answer the given questions— One hazard of the green revolution lies in the requirement that the genetic strains bred for high yields must be used to the exclusion of a variety of native strains. Should the high-yield strain prove vulnerable to an epidemic plant disease, the entire crop of a whole nation could be wiped out in one season. Current research in crop breeding is now stressing the development of more individualized varieties that respond better to local conditions. These varieties are less subject to epidemic diseases and can still produce improved yields with less fertilizer input. A second hazard lies in the need to increase the size of fields by merging many small plots into large ones to allow mechanized agriculture to work most efficiently. In so doing, a variety of food crops is no longer grown, and dependence for survival comes to rest

UGC–NET Geography Solved Papers | 23 on the single crop. Under traditional practices, the Asiatic farmer planted several food crops to ensure that if some failed, others would yield enough food to prevent starvation. The small farmer is therefore reluctant to move to more efficient, single-crop agriculture. It seems obvious now that future increases in yields will be achieved only by modifying the techniques of the green revolution to become less dependent on industrial midlatitude technology. New crop strains and methods of cultivation and fertilization will have to be more compatible with ancient agricultural systems and local culture patterns. Soil scientists have pointed out that large areas of Vertisols remain to be placed under cultivation. The Vertisols are rich in nutrient bases, but will require machine cultivation to overcome difficult tillage. Primitive plows and hoes cannot work these soils. Most of the arable land of the savanna environment in South-East Asia has already been intensively developed by existing standards. To hope for major expansion of agriculture into poor Oxisols and to semidesert zones marginal to the tropical deserts is, at best, unrealistic. 47. What is the focal theme of the passage given above ? (A) Need for emphasis on traditional agriculture (B) Need for emphasis on the use of industrial midlatitude technology (C) Need for modification in the techniques of Green Revolution (D) Need for development of unproductive lands 48. One of the hazards of the Green Revolution is— (A) Monoculture (B) Multicrop Agriculture (C) Use of Machines (D) Small size of fields 49. Current research in agriculture is being oriented to— (A) The development of new fertilizers and pesticides (B) Development of the varieties of seeds suitable for local conditions (C) Development of areas of Oxisols (D) Development of new machines 50. Traditional Asiatic Cultivation emphasizes— (A) Mechanised agriculture

(B) Use of fertilizers in large quantity (C) Monoculture (D) Multicrop agriculture

Answers with Explanations 1. (C) V-shaped valley is formed due to youthful stage of fulvial cycle of erosion. In this stage the river appears in the plains with constantly reducing speed of the flow to make the erosion confined and the river with its Vshape valley moves forward. 2. (A) 3. (C) 4. (B) 5. (B) 6. (B) 7. (A) 8. (C) 9. (D) 10. (C) 11. (D) 12. (C) 13. (B) 14. (A) 15. (C) 16. (B) 17. (D) 18. (D) 19. (D) 20. (C) 21. (B) 22. (B) 23. (D) 24. (B) 25. (A) 26. (A) The iron ore reserves in ascending order is Jharkhand (Singhbhoom), Odisha (Mayurbhanj, Kyojhar), Madhya Pradesh, West Bengal, Karnataka and Rajasthan. 27. (A) 28. (C) 29. (A) 30. (D) 31. (B) 32. (C) Gond is the biggest among all Indian tribes. They are mainly concentrated in Bastar, Chandrapur and Durg. Besides, Andhra Pradesh and Southern-Western parts of Odisha are also inhabited by them. 33. (C) 34. (A) 35. (C) 36. (B) The tribes in Northern-Eastern hill state Meghalaya are more than 50%, in which Garo and Khasi take the lead. Besides, Naga, Chakma, Dafla, Gurung, Limbu, Mikir, Bhutia, Kuki and Gaddi also live there, but their population is comparatively low. 37. (A) 38. (D) 39. (C) 40. (D) 41. (A) 42. (D) 43. (A) 44. (B) 45. (B) 46. (D) 47. (C) In the given passage, the central idea is probable dangers of Green Revolution and necessary unprovements in the technique. 48. (A) Monoculture is one of the severe hazards of the Green Revolution. Should the high yield strain prove vulnerable to an epidemic plant desease, the entire crop of a whole nation could be wiped out in one season. 49. (B) Looking into hazards of Green Revolution the current research in agriculture is being oriented to the development of varieties of seeds suitable for local conditions. 50. (D) Traditional Asiatic cultivation emphasizes on multicrop agriculture because in the wake of destruction of single crop, another crop could be had to satisfy hunger.

December 2008 Geography Paper-II Note—This paper contains fifty (50) objective type questions, each question carrying two (2) marks. Attempt all of them. 1. The process of weathering refers to— (A) Lateral expansion of valleys (B) Spread of Continental-shelf (C) Breakdown of rock materials (D) Building of rock cut platform 2. Stalagtites and stalagmites are depositional landscape of— (A) Glaciated region (B) Limestone region (C) Coastal topography (D) Fluvial topography 3.

Heat budget refers to— (A) Solar radiation (B) Balance between incoming and outgoing radiation (C) Temperature distribution over the continents (D) Heat distribution over land and sea

4.

Match List-I with List-II and select the correct answer from the codes given below— List-I (Airmass type) (a) Arctic Airmass (b) Antarctic Airmass (c) Equatorial Airmass (d) Tropical Airmass List-II (Symbol) 1. T 3. AA Codes : (a) (b) (A) 1 2 (B) 2 3 (C) 3 4 (D) 4 1

2. A 4. E (c) 3 4 1 2

(d) 4 1 2 3

5.

6.

The correct sequence of landforms evolved during the cycle of fluvial erosion is— (A) Rapids – waterfalls – alluvial fans – peneplain (B) Waterfalls – rapids – peneplain – alluvial fans (C) Peneplain – alluvial fans – rapids – waterfalls (D) Waterfalls – peneplain – alluvial fans – rapids

Given below are two statements, one labelled as Assertion (A) and the other labelled as Reason (R). Select your answwer from the codes given below— Assertion (A) : The water droplets are found to be high in the troposphere. Reasons (R) : The temperature variation in the troposphere is not alarming compared to stratosphere. Codes : (A) Both (A) and (R) are true and (R) is correct explanation of (A) (B) Both (A) and (R) are true, but (R) is not correct explanation of (A) (C) (A) is true, but (R) is false (D) (A) is false, but (R) is true 7. The soil-water budget was developed by— (A) Thornthwaite (B) Kendrew (C) Koppen (D) Austin Miller 8. The correct sequence of the Oceans in term of area is— (A) Indian – Pacific – Arctic – Atlantic (B) Pacific – Atlantic – Indian – Arctic (C) Indian – Pacific – Atlantic – Arctic (D) Arctic – Indian – Atlantic – Pacific 9. Bay of Bengal is a— (A) Large marine ecosystem (B) Small marine ecosystem

UGC–NET Geography Solved Papers | 25 (C) A subset of small marine ecosystem (D) A subset of large coastal ecosystem 10. Match List-I with List-II and select the correct answer from the codes given below— List-I (Fields of Study) (a) Micro climatology (b) Geomorphology (c) Meteorology (d) Hydrology List-II (Scholars) 1. Petterson 2. Griffith Taylor 3. Ward 4. Worchester Codes : (a) (b) (c) (d) (A) 1 2 3 4 (B) 2 4 1 3 (C) 4 3 2 1 (D) 3 1 4 2 11. Given below are two statements, one labelled as Assertion (A) and the other labelled as Reason (R). Select your answer from the codes given below— Assertion (A) : In arid and semi-arid regions sheet flood is an important landforming process. Reason (R) : Arid and semi-arid regions experience high intensity short duration rainfall. Codes : (A) Both (A) and (R) are true and (R) is correct explanation of (A) (B) Both (A) and (R) are true, but (R) is not correct explanation of (A) (C) (A) is true, but (R) is false (D) (R) is true, but (A) is false 12. Which one of the following is an example of footloose industry ? (A) Paper industry (B) Manufacturing of electronic components (C) Cement industry (D) Aluminium industry 13. The winter position of subtropical jet stream fluctuates between— (A) 20° and 35° N (B) 35° and 65° N (C) 5° and 20° N (D) 30° and 50° N

14. If the population of the largest city of a region is 8 lacs, what will be the populatin of fourth rank city according to Ziff’s Rank-size Rule ? (A) 6 lacs (B) 4 lacs (C) 2 lacs (D) 0·5 lacs 15. Which of the following characteristics is not true about frontier ? (A) Frontier is a zone (B) Frontier is outer-oriented (C) Frontier is inner-orinted (D) Frontier between early tribal societies was generally called a ‘no man’s land’ 16. Pattern of population distribution is determined by— (A) Availability of Minerals (B) Agricultural practices (C) Cultural practices (D) Physiographic characteristics 17. Which of the following groups of religions originated in the Middle east ? (A) Judaism – Christianity – Confucianism (B) Judaism – Islam – Christianity (C) Judaism – Islam – Taosim (D) Islam – Christianity – Confucianism 18. Ozone depletion may lead to— (A) Water shortage (B) Forest fire (C) Occurrence of skin cancer (D) Resurgence of waterborne diseases 19. Who wrote the book ‘Anthropogeographie’ ? (A) F. Ratzel (B) E. C. Semple (C) Vidal-de-la-Blache (D) Charles Darwin 20. Humboldt wrote his famous book ‘Cosmos’ after he conducted fieldwork in— (A) Europe (B) West Africa (C) South America (D) North America 21. Philosophical approach in geographical writings was first adopted by— (A) Carl Ritter (B) Richard Hartshorne (C) Bernhard Varenius (D) Immanuel Kant

26 | UGC–NET Geography Solved Papers 22. Who edited the book ‘Geography of the Twentieth Century’ ? (A) H. J. Mackinder (B) Vidal-de-la-Blache (C) T. W. Freeman (D) Griffith Taylor 23. Match List-I with List-II and select the correct answer from the codes given below— List-I (Scholar) (a) Sauer (b) Taaffe (c) L. C. King (d) Baker List-II (Field of contribution) 1. Agricultural Geography 2. Geomorphology 3. Transport Geography 4. Cultural Geography Codes : (a) (b) (c) (d) (A) 4 3 2 1 (B) 2 3 4 1 (C) 1 2 3 4 (D) 3 1 2 4 24. Match List-I with List-II and select the correct answer from the codes given below— List-I (Scholars) (a) Christaller (b) Ravenstein (c) Tansley (d) Lucien Febvre List-II (Model/Concept) 1. Possibilism 2. Ecosystem 3. Laws of Migration 4. Central Place Theory Codes : (a) (b) (c) (d) (A) 1 2 3 4 (B) 4 3 2 1 (C) 3 2 1 4 (D) 2 1 4 3 25. Match List-I with List-II and select the correct answer from the codes given ahead— List-I (Tribe) (a) Mikir (b) Korwa (c) Tharu (d) Aong

List-II (State) 1. Uttarakhand 2. Andaman and Nicobar Islands 3. Meghalaya 4. Chhattisgarh Codes : (a) (b) (c) (d) (A) 2 4 1 3 (B) 3 4 1 2 (C) 1 2 3 4 (D) 4 3 2 1 26. Which sector of the economy is least necessary for sustainable economic development ? (A) Primary (B) Secondary (C) Tertiary (D) Quaternary 27. Which one fo the following is considered non-renewable resource ? (A) Water Resource (B) Animal Resource (C) Coal Resource (D) Forest Resource 28. Who evolved the method of measurement of agricultural efficiency ? (A) S. S. Bhatia (B) R. L. Singh (C) S. P. Chatterjee (D) Enayat Ahmad 29. Which one of the following is an ideal combination for the establishment of heavy industries ? (A) Coal-Iron-Water (B) Coal-Electricity-Market (C) Coal-Electricity-Bank (D) Coal-Cotton-Bank 30. Which one of the following philosophies was advanced by August Comte ? (A) Empiricism (B) Positivism (C) Marxism (D) Humanism 31. The concept of sequent occupance was advocated by— (A) H. H. Barrows (B) C. D. Harris (C) D. S. Whittlesey (D) Richard Hartshorne 32. Regional Planning aims at— (A) Development of agriculture (B) Balanced development

UGC–NET Geography Solved Papers | 27

33.

34.

35.

36.

37.

38.

(C) Development of heavy industries (D) Development of small scale industries The concept of counter-magnet is associated with— (A) Western Ghats plan (B) South-East resource region (C) Rayalseema plan (D) National Capital Region Plan Given below are two statements, one labelled as Assertion (A) and the other labelled as Reasons (R). Select your answer from the codes given below— Assertion (A) : Ratzel’s concept of ‘Lebensraum’ was based on the idea that ‘State’ is similar to the ‘Organism’. Reason (R) : Ratzel was a determinist and was under the influence of Darwin’s theory of ‘Survival of the fittest’. Codes : (A) Both (A) and (R) are true and (R) is the correct explanation of (A) (B) Both (A) and (R) are true, but (R) is not the correct explanation of (A) (C) (A) is true, but (R) is false (D) (R) is true, but (A) is false The soils which originate in the humid region due to the accumulation of large amounts of organic matter are known as— (A) Desert soils (B) Peat soils (C) Red soils (D) Laterite soils The areas lying to the east of the Western Ghats covering Karnataka, interior Tamil Nadu, Western Andhra Pradesh and Central Maharashtra are known as— (A) Sub-tropical Steppe Region (B) Tropical Steppe Region (C) Tropical Savana (D) Tropical Rainy Climate Which State of India is the largest producer of Soyabean ? (A) Rajasthan (B) Gujarat (C) Uttar Pradesh (D) Madhya Pradesh Which State of India is the largest producer of wheat ? (A) Punjab (B) Haryana (C) Uttar Pradesh (D) Madhya Pradesh

39. Which State of India has the highest productivity of wheat per hectare ? (A) Bihar (B) Uttar Pradesh (C) Punjab (D) Haryana 40. Which one of the following places of India was worst affected by Tsunami in 2004 ? (A) Nagapattinam (B) Kanyakumari (C) Vishakhapattanam (D) Chennai 41. Which one of the following is small scale map ? (A) Cadastral map (B) SOI Topographical map (C) World Atlas map (D) Tamil Nadu State map 42. In remote sensing, there are four different types of resolution characters. Identify the correct order in which they are used— (A) Radiometric–Spatial–Spectral–Temporal (B) Spatial–Spectral–Radiometric–Temporal (C) Temporal–Spectral–Radiometric–Spatial (D) Spectral–Temporal–Spatial–Radiometric 43. Which one of the following is a measure of dispersion ? (A) Median (B) Geometric Mean (C) Standard Deviation (D) Arithmetic Mean 44. Who was the originator of the regression analysis ? (A) Clark and Evans (B) Karl Pearson (C) Spearman (D) Francis Galton 45. Arrange the following river systems of India in sequence from North to South— (A) Godavari – Mahanadi – Kaveri – Krishna (B) Mahanadi – Godavari – Krishna – Kaveri (C) Krishna – Kaveri – Godavari – Mahanadi (D) Kaveri – Godavari – Mahanadi – Krishna 46. Road accessibility can best be represented by— (A) Bar diagram (B) Pie diagram (C) Graded symbol map (D) Flow line diagram

28 | UGC–NET Geography Solved Papers Directions—(Q. 47–50) Read the following passage and answer the given questions— The seventeenth century witnessed the beginning of the scientific revolution that led to the development of more useful ways of generalizing, explaining and communicating. The effort to provide more exact descriptions of specific things was replaced by the effort to formulate general theory in relation to which specific things could be made significant. In the formulation and testing of theory and in communicating the findings, a step of major importance was the independent development of calculus by Newton and Leibniz. The use of mathematical procedures made the process of reasoning more precise and provided a universal language for the communication of the results. Most of the present fields of science had their roots in the eighteenth century, during which time the acceptable methods of study were being formualted and reliable procedures for verifying hypotheses were being established. By the end of the eighteenth century, the ideas expressed by Kant had become generally accepted. As bodies of theory developed and proved useful, special fields of study appeared. These new fields of study became what Kant called the logical division of knowledge-in contrast to the physial classification of knowledge in terms of time and space. The last great figure who could claim universal scholarship was Humboldt. No student of the earth before or since has enjoyed such acclaim by his contemporaries. The world of scholarship underwent a basic change during the nineteenth and twentieth centuries. Not only have the fields of learning, the academic disciplines has been greatly elaborated, but also the total number of scholars has reached unprecedented size. Moreover, the number of recorded facts about the earth has increased astronomically. A Humboldt could once master a very large part of the available knowledge concerning the earth, but this is no longer possible. 47. The central theme of the passage is concerned with— (A) History of scientific revolution (B) Scientific research done by Newton and Leibniz

(C) Procedure of verifying hypotheses (D) Changing methodological and conceptual perspective in Geographic thought 48. Large number of various branches of science has their roots in— (A) Seventeenth Century (B) Sixteenth Century (C) Eighteenth Century (D) Nineteenth Century 49. Why, in eighteenth century, were the concepts and methods of Kant acceptable ? (A) Because Kant was a physical geographer (B) He provided method of classification of knowledge (C) He separated geographical research from historical research (D) Development of theory and its verification was possible 50. Who was the ‘last of the great polymaths’ ? (A) Humboldt (B) Leibniz (C) Kant (D) Newton

Answers with Explanations 1. 6. 11. 16. 21. 26. 31. 36. 37.

38.

39.

40. 41.

(C) 2. (B) 3. (B) 4. (B) 5. (A) (B) 7. (A) 8. (D) 9. (B) 10. (B) (C) 12. (B) 13. (A) 14. (C) 15. (A) (D) 17. (B) 18. (C) 19. (A) 20. (C) (D) 22. (D) 23. (A) 24. (B) 25. (B) (D) 27. (C) 28. (A) 29. (A) 30. (B) (C) 32. (B) 33. (D) 34. (B) 35. (B) (B) (D) The order of states with maximum soyabean production is—Madhya Pradesh, Uttar Pradesh, Rajasthan and Gujarat. (C) The order of states producing maximum wheat is—Uttar Pradesh, Punjab, Haryana, Rajasthan and Bihar. (C) Productivity of wheat per hectare in India (from higher to lower) is—Punjab, Haryana, Uttar Pradesh, Rajasthan and Bihar. (A) In the year 2004 Nagapattinam was badly affected by Tsunami, which cost it dearly. (C) 42. (B)

UGC–NET Geography Solved Papers | 29 43. (C) The measure of dispersion is standard deviation (σ) D irect Method—SD(σ ) ∑d ∑(x – y - ) 2 or = N N Short Method—SD = i

∑fx2 – N

44. (D)

46. (D)

45. (B)

(∑fxN )

2

47. (A) The central theme of the passage is concerned with scientific revolution from 17th century to 19th century. 48. (C) The root of different branches of science lies in 18th century. According to Kant–‘‘As

bodies of theory developed and proved useful, special fields of study appeared. These new fields of study became what Kant called the logical division of knowledge in contrast to the physical classification of knowledge in terms of time and place.’’ 49. (D) The concept and methods of Kant were acceptable till the end of 18th century because, the development and verification of those principles was possible. 50. (A) The last great figure who could claim universal scholarship was Humboldt. No other person of the earth before or since has enjoyed such acclaim by his contemporaries.

June 2009 Geography Paper-II Note—This paper contains fifty (50) objective type questions, each question carrying two (2) marks. Attempt all of them. 1. The slope replacement model was first proposed by— (A) A. Young (B) A. N. Strahler (C) W. Penck (D) L. C. King 2.

3.

4.

5.

List-I (Theories) (a) Planetesimal hypothesis (b) Thermal contraction theory (c) Geosynclinal Orogen theory (d) Hypothesis of sliding continents

Obsequent streams generally flow— (A) Along the consequent streams (B) Opposite to the consequent streams (C) At right angle of the consequent streams (D) Diagonal to the consequent streams Given below are two statements, one labelled as Assertion (A) and the other labelled as Reason (R). Select your answer from the codes given below— Assertion (A) : Magma is generated of the divergent plate boundaries. Reason (R) : Upcoming magma reaches the surface of the constructive plate margins at the top of the divergent plates. Codes : (A) Both (A) and (R) are true and (R) is the correct explanation of (A) (B) Both (A) and (R) are true, but (R) is not the correct explanation of (A) (C) (A) is true, but (R) is false (D) (A) is false, but (R) is true Arrange the following inter-glacial periods in a chronological order and select the correct answer using the codes given below— 1. Wurm 2. Riss 3. Mindal 4. Gunj Codes : (A) 4, 3, 2, 1 (B) 3, 1, 4, 2 (C) 1, 2, 3, 4 (D) 1, 2, 4, 3

Match List-I with List-II and select the correct answer from the codes given below—

List-II (Authors) 1. Kober 3. Daly Codes : (a) (A) 2 (B) 3 (C) 1 (D) 4

(b) 4 4 2 3

2. Chamberlin 4. Jefferys (c) 1 2 3 2

(d) 3 1 4 1

6.

The greenhouse effect is due to— (A) Absorption of ultra-violet radiation by sea water (B) Absorption of infrared radiation by carbon dioxide and water vapour in the atmosphere (C) Infrared reflection by oceanic surfaces (D) Absorption of infrared radiation by plants

7.

Which of the following conditions would provide for the least amount of insolation ? (A) The sun is near the horizon (B) The earth is at its closest point in its orbit to the sun (C) The length of the day is maximum (D) The length of the night is at its minimum

8.

Given below are two statements, one labelled as Assertion (A) and the other labelled as Reason (R). Select your answer from the codes given ahead— Assertion (A) : Every planet in our solar system has an atmosphere.

UGC–NET Geography Solved Papers | 31 Reason (R)

: Atmosphere is the basis of all elements of climate.

Codes : (A) Both (A) and (R) are true and (R) is the correct explanation of (A) (B) Both (A) and (R) are true and (R) is not correct explanation of (A) (C) (A) is true, but (R) is false (D) (A) is false, but (R) is true 9.

Match List-I with List-II and select the correct answer from the codes given below— List-I (a) Ultra-violet (b) Atmosphere window (c) Ionised radiation (d) Near infrared List-II 1. > 0·8 μm 2. < 0·3 μm 3. 8 μm to 13 μm 4. X-rays and gamma rays Codes : (a) (b) (c) (d) (A) 4 3 2 1 (B) 3 4 1 2 (C) 2 1 3 4 (D) 2 3 4 1 10. Arrange the following layers of atmosphere vertically outward from the earth. Select the correct answer from the codes given below— 1. Ozonosphere 2. Troposphere 3. Stratosphere 4. Ionosphere Codes : (A) 1, 2, 3, 4 (B) 2, 1, 3, 4 (C) 2, 3, 1, 4 (D) 1, 3, 2, 4 11. Water mass heats move slowly than land mass because— (A) Specific heat of water is small (B) Evaporation warms the water surfaces (C) Radiation cannot penetrate the surface water layer (D) The surface water layer is easily mixed 12. Which of the following is not a characteristic of Phanerophytes ? (A) They can sustain effects of extreme cold or hot temperatures, drought and high speed winds

(B) They are commonly found in temperate and moist tropical regions (C) They are tall perenial trees, shrubs, bushes (D) The renewal buds of these plants always remain buried 13. In which of the following sequences, the increasing order of salinity concentration is correct ? (A) Red Sea – Gulf of California – Arctic Ocean – Baltic Sea (B) Gulf of California – Baltic Sea – Red Sea – Arctic Ocean (C) Baltic Sea – Arctic Ocean – Gulf of California – Red Sea (D) Arctic Ocean – Gulf of California – Baltic Sea – Red Sea 14. Which of the following best describes, how the second law of thermodynamics is applied to the concept of trophic levels ? (A) All the organic matter in one trophic level is converted during the transfer to the next higher level (B) None of the usable energy in one trophic level is lost as heat or during transfer to the next higher level (C) Much of the usable energy in one trophic level is lost as heat or during transfer to the next higher level (D) Energy flows at 100% efficiency from one trophic level to another 15. Given below are two statements, one labelled as Assertion (A) and other labelled as Reason (R). Select your answer from the codes given below— Assertion (A) : Spring tides occur on fullmoon and no moon days. Reason (R) : On fullmoon and no moon days, the sun and moon are almost at right angles to the earth. Codes : (A) Both (A) and (R) are true and (R) is the correct explanation of (A) (B) Both (A) and (R) are true, but (R) is not the correct explanation of (A) (C) (A) is true, but (R) is false (D) (A) is false, but (R) is true

32 | UGC–NET Geography Solved Papers 16. Given below are two statements, one labelled as Assertion (A) and other labelled as Reason (R). Select your answer from the codes given below— Assertion (A) : Recognizing regions required investigation of similarities as well as differences over space. Reason (R) : Areal differentiation is about establishing degrees of sameness as well as differences between regions. Codes : (A) Both (A) and (R) are true and (R) is the correct explanation of (A) (B) Both (A) and (R) are true, but (R) is not correct explanation of (A) (C) (A) is true, but (R) is false (D) (A) is false, but (R) is true 17. Welfare Geography emphasizes the analysis of— (A) Regional balanced development (B) Public–private partnership in development (C) Urban–industrial development (D) Aspects of Quality of life 18. David Harvey is best known for his contributions to— (A) Urban Geography (B) Radical Geography (C) Political Geography (D) New Economic Geography 19. Given below are two statements, one labelled as Assertion (A) and other labelled as Reason (R). Select your answer from the codes given below— Assertion (A): The distribution of the world population is highly uneven. Reason (R) : Political environments are responsible for uneven distribution of population. Codes : (A) Both (A) and (R) are true and (R) is the correct explanation of (A) (B) Both (A) and (R) are true, but (R) is not correct explanation of (A) (C) (A) is false, but (R) is true (D) (A) is true, but (R) is false

20. The growth of metropolitan cities is best explained by which combination of factors— 1. Extension of spatial boundaries 2. Immigration 3. High Birth-rate 4. Annexation of peripheral settlement Codes : (A) 1, 2, 3 (B) 2, 3, 4 (C) 3, 4, 1 (D) 1, 2, 4 21. Settlements are considered to have an even distribution of R value is close to— (A) 0·5 (B) 1·0 (C) 1·5 (D) 2·0 22. An urban settlement to be classified as a city in the Indian Census should have a minimum population of— (A) 1 lakh (B) 3 lakhs (C) 5 lakhs (D) 10 lakhs 23. Given below are two statements, one labelled as Assertion (A) and other labelled as Reason (R). Select your answer from the codes given below— Assertion (A) : Cities with very rapid rates of growth have more male adult population. Reason (R) : City-ward migration is sex age selective. Codes : (A) Both (A) and (R) are true and (R) is the correct explanation of (A) (B) Both (A) and (R) are true, but (R) is not correct explanation of (A) (C) (A) is true, but (R) is false (D) (A) is false, but (R) is true 24. Coolgardie and Kalgoorlie are famous as— (A) Resort towns (B) Mining towns (C) Industrial towns (D) Institutional towns 25. Which of the following is a biotic resource ? (A) Iron (B) Coal (C) Copper (D) Mica 26. Which one of the following is based on the principle of least cost ? (A) Weber’s theory (B) Losch’s theory

UGC–NET Geography Solved Papers | 33 (C) Smiths spatial profitability theory (D) Von-thunen theory 27. Match List-I with List-II and select the correct answer from the codes given below— List-I (a) France (b) Ukraine (c) Germany (d) USA List-II 1. Salgilter Region 2. Mesabi Region 3. Lorraine Region 4. Krivoy Rog Region Codes : (a) (A) 4 (B) 3 (C) 1 (D) 4

(b) 3 4 2 1

(c) 1 1 4 3

(d) 2 2 3 2

28. Arrange the following with respect to bulk shipping trade (2001) in India in descending order— (A) Kandla, Chennai, Mumbai, Vishakapatnam (B) Mumbai, Kandla, Chennai, Vishakapatnam (C) Vishakapatnam, Chennai, Kandla, Mumbai (D) Mumbai, Chennai, Vishakapatnam, Kandla 29. In McKinder’s Heartland theory, the Indian Ocean was included in— (A) Outer Crescent (B) Inner Crescent (C) Secondary Heartland (D) World Island 30. Who made the statement that whosoever rules Eurasia controls the destiny of the world ? (A) Spykman (B) Kjellan (C) Mackinder (D) Haushofer 31. Which combination of the following tribes is confined to a single State of North-East India ? (A) Khasi – Nagas (B) Garo – Khasi (C) Naga – Mizo (D) Khasi – Mizo

32. Which one of the following sets belongs to Dravidian group of languages ? (A) Kannada, Telugu (B) Marathi, Gujarati (C) Malayalam, Oriya (D) Tamil, Marathi 33. The concept of place–folk–work was given by— (A) Clarence Stein (B) Le Play (C) Vidal-de-la-Blache (D) Jean Brunhes 34. Given below are two statements, one labelled as Assertion (A) and other labelled as Reason (R). Select your answer from the codes given below— Assertion (A): Regional disparities have increased both at the regional and interpersonal levels in the post-liberalization period in India. Reason (R) : New-liberal policies are based on the principles of comparative advantage and adhere to the efficiency criterion. Codes : (A) Both (A) and (R) are true and (R) explains (A) (B) Both (A) and (R) are true, but (R) does not explain (A) (C) (A) is true, but (R) is false (D) (A) is false, but (R) is true 35. The concept of the ‘neighbourhood unit’ in urban planning was given by— (A) Le Corbusier (B) Clarence Perry (C) Lewis Mumford (D) Frand LLoyd Wright 36. One of the most comprehensive attempts to delineate India into planning regions was made by— (A) The Planning Commission of India (B) Town and Country Planning Organisation (C) National Council for Applied Economic Research (D) Indian Institute of Public Administration

34 | UGC–NET Geography Solved Papers 37. Lakshadweep Islands are of— (A) Volcanic origin (B) Coral origin (C) Tectonic origin (D) Alluvial origin 38. Domestication of plants and animals was started in the— (A) Eolithic period (B) Mesolithic period (C) Neolithic period (D) Paleolithic period 39. Which of these historic monuments is not associated with Delhi ? (A) Gateway of India (B) Appughar (C) Qutubminar (D) Red Fort 40. The upliftment of the Himalayas was first initiated the period of— (A) Middle Pliocene (B) Middle Oligocene (C) Middle Eocene (D) Middle Miocene 41. Match List-I with List-II and select the correct answer from the codes given below— List-I (a) Meghalaya (b) Manipur (c) Nagaland (d) Mizoram List-II 1. Kohima 2. Aizwal 3. Shillong 4. Imphal Codes : (a) (b) (c) (d) (A) 3 2 4 1 (B) 3 4 1 2 (C) 2 3 1 4 (D) 3 2 1 4 42. Cadastral map is meant for— (A) Showing Urban areas (B) Relief and drainage (C) Distribution of crops (D) Showing plot/field boundaries 43. International projection is a modified form of— (A) Polyconic (B) Sinusoidal (C) Bonne’s (D) Cylindrical equal Area 44. Which of the following is the component of GIS ? (A) Data input and verification sub-system (B) Data reporting sub-system (C) Data storage and retrieval sub-system (D) Data manipulation and analysis subsystem 45. Given below are two statements, one labelled as Assertion (A) and other labelled as Reason

(R). Select your answer from the codes given below— Assertion (A) : Median is not affected by the values of extreme items. Reason (R) : It needs an arrangement of the data in ascending or descending order. Codes : (A) Both (A) and (R) are true and (R) is the correct explanation of (A) (B) Both (A) and (R) are true, but (R) is not correct explanation of (A) (C) (A) is true, but (R) is false (D) (A) is false, but (R) is true 46. Match List-I with List-II and select the correct answer from the codes given below— List-I (a) Homolographic (b) Orthomorphic (c) Equidistant (d) Azimuthal List-II 1. True bearing 2. Equal area 3. Correct shape 4. Equal distance Codes : (a) (b) (c) (d) (A) 3 2 1 4 (B) 2 3 4 1 (C) 2 3 1 4 (D) 4 2 1 3 Directions—(Q. 47–50) Read the following passage and answer the given questions— The great underlying principle of modern Geomorphology is known as the principle of uniformitarianism. It was first enunciated by Hutton in 1785, beautiful restarted by Playfair in 1802 and popularized by Lyell in the numerous editions of his ‘Principles of Geology’. Hutton taught that ‘‘the present is the key to the past’’, but he applied this principle some what too rigidly and argued that geologic processes operated throughout geologic time with the same intensity as now. We know now that this is not true. Glaciers were much more significant during the Pleistocene and during other periods of geologic time than now; world climates have not always been distributed as they now are, and, thus, regions that are how humid have been desert and areas now desert have been humid; periods of crustal instability seem to have separated periods of relative crustal stability, although there are

UGC–NET Geography Solved Papers | 35 some who doubt this and there were times when vulcanism was more important than now. Numerous other examples could be cited to show that the intensity of various geologic processes has varied through geologic time but these is no reason to believe that streams did not cut valleys in the past as they do now; that the more numerous and more extensive valley glaciers of the pleistocent behaved any differently from existing glaciers; that the winds which deposited the Navajo sandstone during Jurassic times obeyed the different laws from those which control wind movements today, groundwater opened up solutional passageways in limestones and other soluble rocks and formed surface depressions which are now called sinkholes during the permian and Pennsylvanian periods as it does today in many parts of the world. Without the principle of Uniformitarianism there could hardly be a science of Geomorphology that was more than pure description. 47. What is the main theme of uniformitarianism ? (A) Climate has been uniform in geological times (B) The present is the key to the past (C) Glaciers were always significant in geological past (D) The geomorphic processes operated throughout geological times, operated with the same intensity as now 48. Who first enunciated the principle of uniformitarianism ? (A) Playfair (B) Dutton (C) Hutton (D) Kober 49. Glaciers were more significant in the geological period— (A) Tertiary period (B) Pleistocene period (C) Eocene period (D) Permian period 50. A characteristics feature of limestone topography is— (A) Yardang (B) Sink hole (C) Bajada (D) Pediment

Answers with Explanations 1. (C) 2. (B) 3. (A) 4. (A) 5. (A) 6. (B) 7. (A) 8. (B) 9. (D) 10. (C) The layer at the bottom of atmosphere is Troposphere which spreads at the height of 10

to 15 km. Above this is Stratosphere whose average height is considered as 30 km. After this, there is Ozone layer which spreads 30 to 60 km on equator and 15 to 35 km on poles. Above this is Ionosphere at the height of 80 to 640 km. 11. (C) 12. (D) 13. (C) 14. (D) 15. (C) 16. (C) 17. (D) 18. (B) David Harvey is know for his contribution to Radical Geography. Radical Geography came into being in the first issue of Geography Research Magazine, named ‘Antipode’. In its development William Bungi and David Harvey emphasised to bring it into the ambit of social values rather than traditional capitalist system. 19. (A) 20. (C) 21. (D) 22. (D) 23. (A) 24. (B) 25. (B) Coal is a biotic resource which is made from the fossils of plants and trees. Iron, copper and mica are non-biotic natural resources. 26. (A) 27. (B) 28. (D) 29. (A) 30. (A) 31. (B) 32. (A) 33. (B) 34. (B) 35. (B) 36. (B) 37. (B) 38. (C) 39. (A) Gateway of India is in Mumbai, whereas Appughar, Qutabminar and Red Fort are in Delhi. 40. (B) 41. (B) Capital of Meghalaya is Shillong Capital of Manipur is Imphal Capital of Nagaland is Kohima, and Capital of Mizoram is Aizwal. 42. (D) 43. (A) 44. (C) 45. (B) 46. (B) 47. (B) The principle of uniformitarianism says– ‘‘the present is the key to the past’’. The principle was propounded by Hutton in 1785. 48. (C) Initially Hutton propounded the principle of uniformitarianism in 1785, which was beautifully restarted by Playfair in 1802. 49. (B) Glaciers were more significant in the geological period of Pleistocene period. 50. (B) Limestone topography is an important shape of sink hole. It is formed due to soluble action of underground water and friction. It needs the following stages—(i) Thick layer of lime rock, (ii) Below this unpregnable rock and (iii) Position of this area above water line.

December 2009 Geography (Paper-II) Note—This paper contains fifty (50) objective type questions, each question carrying two (2) marks. Attempt all the questions. 1. When endogenetic forces act horizontally from the opposite directions causing compression and tension, the resultant land form is a large dome shaped structure, the process is called— (A) Folding (B) Faulting (C) Fracturing (D) Warping 2. Which one of the following explanations of the origin of pediments was first propounded by A. C. Lawson (1915) ? (A) Sheet flood deposition (B) Horizontal erosion (C) Recession of mountain front (D) Lateral erosion 3. Given below are two statements, one labelled as Assertion (A) and the other labelled as Reason (R). Select your answer from the codes given below— Assertion (A) : By far the most common and widespread topographic form in a Karst terrain is the sinkhole. Reason (R) : Topographically, a sinkhole is depression that varies in depth from less than a metre to few hundred metres. Codes : (A) Both (A) and (R) are true and (R) is the correct explanation of (A) (B) Both (A) and (R) are true, but (R) is not the correct explanation of (A) (C) (A) is true, but (R) is false (D) (A) is false, but (R) is true

4. Match List-I with List-II and select the correct answer from the codes given below— List-I (Books) (a) Morphology of the Earth (b) Techniques in Geomorphology (c) The Earth (d) The Unstable Earth List-II (Authors) 1. C. A. King 2. Jeffreys 3. L. C. King 4. J. A. Steers Codes : (a) (b) (c) (d) (A) 3 1 2 4 (B) 2 4 3 1 (C) 3 2 1 4 (D) 4 1 2 3 5. Arrange the following concepts in the order in which they appeared. Use the codes given below— 1 Principle of Uniformitarianism 2. Theory of Continental Drift 3. Dynamic Equilibrium Theory 4. Convection Current Theory Codes : (A) 1 3 4 2 (B) 2 1 4 3 (C) 4 1 2 3 (D) 1 2 4 3 6. Given below are two statements, one labelled as Assertion (A) and the other labelled as Reason (R). Select your answer from the codes given ahead— Assertion (A) : Stable atmospheric condition prevails when environmental lapse rate is less than dry and wet adiabatic lapse rates.

UGC–NET Geography Solved Papers | 37 Reason (R)

: The parcel of air is warmer than the surrounding environment at all the levels as it ascends.

Codes : (A) Both (A) and (R) are true and (R) is the correct explanation of (A) (B) Both (A) and (R) are true, but (R) is not the correct explanation of (A) (C) (A) is true, but (R) is false (D) (A) is false, but (R) is true 7. The mature stage of thunderstorm is characterized by— (A) Strong updrafts and downdrafts (B) Lack of electrical charges (C) End of precipitation (D) Lack of ice crystals 8. Match List-I with List-II and select the correct answer using codes given below— List-I (a) Katabatic Wind (b) Chinooks (c) Santa Ana (d) Foehn List-II 1. Warm dry winds moving down the east slopes of Rockies. 2. Warm dry winds in Southern California. 3. Cold air situated over highland area set in motion. 4. Warm dry winds moving down the east slopes of Alps. Codes : (a) (b) (c) (d) (A) 2 1 3 4 (B) 3 1 2 4 (C) 1 3 4 2 (D) 4 2 3 1 9. The idealised global pattern of surface wind from the equator of pole is— (A) doldrums – trade winds – westerlies – easterlies (B) doldrums – westerlies – trade winds – easterlies (C) doldrums – easterlies – trade winds – westerlies (D) doldrums – trade winds – easterlies – westerlies

10. The marine influence upon climate results into— (A) Increase in annual rainfall (B) Increase in aridity (C) Increase in annual temperature range (D) Decrease in annual temperature range 11. Plants that grow under high intensity of disturbance and low intensity of stress are called as— (A) Competitors (B) Stress tolerators (C) Coniferous forests (D) Ruderals 12. Match List-I with List-II and select the correct answer by using the codes given below— List-I List-II (a) Producer 1. Precipitation (b) Biota 2. Microbes (c) Abiotic factor 3. Trees (d) Decomposer 4. All organisms Codes : (a) (A) 1 (B) 3 (C) 2 (D) 2

(b) 3 4 3 4

(c) 2 1 1 3

(d) 4 2 4 1

13. Given below are two statements, one labelled as Assertion (A) and the other labelled as Reason (R). Select your answer from the codes given below— Assertion (A) : Water temperature over the ocean decreases with depth. Reason (R) : The decrease in temperature is well marked from 370 m to 730 m. Codes : (A) Both (A) and (R) are true and (R) is the correct explanation of (A) (B) Both (A) and (R) are true, but (R) is not the correct explanation of (A) (C) (A) is true, but (R) is false (D) (A) is false, but (R) is true

38 | UGC–NET Geography Solved Papers 14. Given below are two statements, one labelled as Assertion (A) and the other labelled as Reason (R). Select your answer from the codes given below— Assertion (A) : The spatial analysis of regions advocated by Haggett considers them as closed system. Reason (R) : The concepts that spatial analysis uses are : focality, localization, interconnection and discontinuity. Codes : (A) Both (A) and (R) are correct and (R) explains (A) (B) Both (A) and (R) are correct, but (R) does not explain (A) (C) (A) is correct, but (R) is wrong (D) (A) is false, but (R) is correct 15. Theories of spatial organization draw mainly from— (A) Positivism (B) Functionalism (C) Structuralism (D) Behaviouralism 16. The concept of the ‘natural region’ was given by— (A) Jean Brunhes (B) Le Play (C) Carl Sauer (D) Vidal-de-la Blache 17. Match List-I with List-II and select the correct answer from the codes given below— List-I (a) Walter Isard (b) David Harvey (c) Richard Hartshorne (d) Richard Peet List-II 1. Social Justice and the City 2. Methods of Regional Science 3. Radical Geography 4. Perspectives on the Nature of Geography Codes : (a) (b) (c) (d) (A) 1 2 4 3 (B) 2 1 4 3 (C) 2 3 1 4 (D) 4 2 3 1

18. Natural population growth is a function of— (A) Births (B) Deaths (C) Fertility and mortality (D) Migration 19. Migration in India, as per 2001 Census, is maximum in which of the following streams ? (A) Rural to Urban (B) Urban to Urban (C) Urban to Rural (D) Rural to Rural 20. The sequence of events in Demographic Transition Theory is— 1. High birth-rate and high death-rate 2. High birth-rate and low death-rate 3. Low birth-rate and low death-rate 4. Low birth-rate and high death-rate Codes : (A) 1 2 3 4 (B) 2 3 1 4 (C) 4 3 2 1 (D) 3 2 1 4 21. The spatial distribution pattern of rural settlements can best be observed from— (A) Wall maps (B) Cadastral maps (C) Geological maps (D) Topographical maps 22. Match List-I with List-II and select correct answer from the codes given below— List-I (Principle) (a) Fixed K (b) Non-Fixed K (c) Population Threshold and Range of a good (d) P 1 P 2 /D Hypothesis List-II (Contributor) 1. W. Christaller 2. A. Losch 3. BJL Berry 4. Zipf 5. Reed and Muench Codes : (a) (b) (c) (d) (A) 1 4 2 5 (B) 2 5 4 1 (C) 4 1 2 3 (D) 1 2 3 4

UGC–NET Geography Solved Papers | 39 23. How many agricultural systems in the world were recognised by Whittlesey ? (A) Five (B) Six (C) Nine (D) Thirteen 24. Match List-I with List-II and select the correct answer from the codes given below— List-I List-II (a) Venezuela 1. Petroleum (b) China 2. Solid fuel (c) Netherlands 3. Wind power (d) France 4. Nuclear power Codes : (a) (b) (c) (d) (A) 1 2 3 4 (B) 1 4 3 2 (C) 3 2 1 4 (D) 3 4 2 1 25. Match List-I with List-II and select the correct answer from the codes given below— List-I List-II (a) Tungsten 1. Zambia (b) Cobalt 2. China (c) Chromium 3. Ukraine (d) Manganese 4. South Africa Codes : (a) (A) 1 (B) 1 (C) 2 (D) 2

(b) 2 2 1 1

(c) 3 4 3 4

(d) 4 3 4 3

26. Given below are two statements, one labelled as Assertion (A) and the other labelled as Reason (R). Select your answer from the codes given below— Assertion (A) : In temperate regions the grasslands have been converted into fertile agricultural lands at places. Reason (R) : Low temperatures in these lands make the rainfall more effective. Codes : (A) Both (A) and (R) are correct and (R) explains (A) (B) Both (A) and (R) are correct, but (R) does not explain (A)

(C) (A) is true, but (R) is false (D) (A) is false, but (R) is true 27. The Heartland Theory explains— (A) Inter-province relationship (B) Centre-State relationship (C) Struggle between land power and sea power (D) Classification of States 28. Match List-I with List-II and select the correct answer from the codes given below— List-I List-II (a) Yakuts 1. North Africa (b) Berbers 2. Russia (c) Khirgiz 3. North America (d) Red Indians 4. Central Asia Codes : (a)

(b)

(c)

(d)

(A) (B) (C) (D)

4 1 4 1

1 4 2 4

3 3 3 2

2 2 1 3

29. Which one of the following is most spoken Indo-Austric language in India ? (A) Khasi (B) Santhali (C) Munda (D) Kharia 30. Match List-I with List-II and select the correct answer from the codes given below— List-I (a) Heartland (b) Rimland (c) Classification of States (d) Law of Territorial Growth of States List-II 1. Van Valkenburg 2. Ratzel 3. Spykman 4. Mackinder Codes : (a) (b) (c) (d) (A) 4 3 1 2 (B) 3 4 2 1 (C) 1 2 3 4 (D) 2 1 3 4

40 | UGC–NET Geography Solved Papers 31. Which of the following is not a social indicator of regional development ? (A) Number of primary schools in the region (B) Number of primary health centres in the region (C) Number of medical professionals per thousand population (D) Number of tertiary workers in the population 32. Regional diversity is essentially caused by— (A) Per capita income (B) Natural resource base (C) Industrial development (D) Levels of urbanization 33. Given below are two statements, one labelled as Assertion (A) and the other is labelled as Reason (R). Select your answer from the codes given below— Assertion (A) : The growth pole strategy failed to bring balanced regional development in India. Reason (R) : Diffusion of development under conditions of general regional backwardness is weak. Codes : (A) Both (A) and (R) are correct and (R) explains (A) (B) Both (A) and (R) are correct, but (R) does not explain (A) (C) (A) is correct, but (R) is wrong (D) (A) is wrong, but (R) is correct 34. Patrick Geddes applied which of the following perspectives in Regional Planning ? (A) Technological (B) Socio-economic (C) Environmental (D) Historical-ecological 35. Concern with regional disparities in development was explicitly mentioned in which Five Year Plan of India ? (A) Second (B) Fourth (C) Seventh (D) Tenth

36. Which city of Gujarat is associated with salt production ? (A) Surat (B) Kandla (C) Vadodara (D) Gandhidham 37. Match List-I with List-II and select the correct answer from the codes given below— List-I List-II (a) Iron Ore 1. Assam (b) Petroleum 2. Chhattisgarh (c) Mica 3. Rajasthan (d) Rock Phosphate 4. Jharkhand Codes : (a) (A) 4 (B) 2 (C) 2 (D) 2

(b) 3 1 1 3

(c) 1 4 3 1

(d) 2 3 4 4

38. Which of the following cities is situated on the mouth of river Tapi (Tapti) ? (A) Ankleshwar (B) Vadodara (C) Ahmedabad (D) Surat 39. Which one of the following is in the rain shadow of the Western Ghats ? (A) Mahabaleshwar (B) Baramati (C) Panchghani (D) Ratnagiri 40. Arrange the following Indian States in descending order of their geographical area— 1. Maharashtra 2. Karnataka 3. Rajasthan 4. Tamil Nadu Codes : (A) 3 1 4 2 (B) 1 3 4 2 (C) 4 3 1 2 (D) 3 1 2 4 41. Circle and sector diagram is also known as— (A) Ring diagram (B) Pile diagram (C) Pie diagram (D) Pictorial diagram 42. The projection in which Loxodromes are shown as straight lines is— (A) Gnomonic (B) Mercator’s

UGC–NET Geography Solved Papers | 41 (C) Gall’s Stereographic (D) Cylindrical Equal Area 43. A system which consists of data acquisition, data processing and data analysis is called— (A) Digital Image (B) Geographic Information System (C) Remote Sensing System (D) Global Positioning System 44. Match List-I with List-II and select the correct answer by using the codes given below— List-I (a) Dispersion (b) Tally mark (c) Slope of regression line (d) Correlation List-II 1. b 2. SD 3. r 4. frequency Codes : (a) (b) (c) (d) (A) 4 2 3 1 (B) 2 4 3 1 (C) 2 4 1 3 (D) 4 2 1 3 45. The method which shows by the use of light and shade is called— (A) Hill shading (B) Spot heights (C) Hachure (D) Benchmarks Directions—(Q. 46–50) Read the following passage and answer the given questions— The main problem of theory-development in geography is not a failure to formalise theory. It is, rather, a weak understanding of the role of theory in explanation and a failure to make verbal statements in ways which are ‘explanatory’ in some logically consistent way. This is not to say that these verbal statements are inherently uninteresting. Indeed they are not. Consider the following statement by Sauer (1963, 359)— The whole task of human geography, therefore, is nothing less than comparative study of areally localized cultures…… But culture is the learned and conventionalized activity of a group that occupies an area. A culture trait or complex originates at a certain time in a particular locality. It gains acceptance – that is, is learned by a group

and is communicated outward, or diffuses, until it encounters sufficient resistance, as from unsuitable physical conditions, from alternative traits, or from disparity of culture level. This statement is part a ‘directive’ for geographic study, it is part an explanation schemes for understanding areally localised cultures, it is part a description of what happens. But all three notions are intertwined and it is not clear whether Sauer is proposing a theory or whether he is merely describing an average process. Yet the statement is surely a stimulating one and one that manages to sum up intuitively in very short space much of the activity of the human geographer. We could well pay more attention to statements of this type and attempt to show how they may function in an explanatory way. 46. Through this statement, Sauer is proposing a— (A) Theory (B) Process (C) Directive (D) Not very clear 47. The task of Human Geography is— (A) Intuitive (B) Stimulating (C) Intertwined (D) All of these 48. Culture is— (A) Conventionalized Activity (B) Directive for the society (C) An explanatory way of life (D) Stimulation for future generation 49. Human Geography is the study of— (A) Human-beings in Society (B) Human Adaptation of Nature (C) Areal Differentiation in Culture (D) Areal Differentiation in Human Resources 50. What is the focal theme of the passage given above ? (A) Culture and areal differentiations in cultures (B) Human Geography (C) Laws and theories in Geography (D) The role of Geography in Scientific Disciplines

Answers with Explanations 1. (D) Endogenetic forces that make changes over the surface are of two types : 1. Horizontal movement, 2. Vertical movements. Due to

42 | UGC–NET Geography Solved Papers

2. 3.

4. 5.

6.

7. 8. 9.

10. 11. 15. 16. 17. 18. 19. 21.

crogenetic forces tension and compression take place. In to compression due wraping a dome shaped structural landform in resulted. (C) As per A. C. Lawson’s view pediments is propounded due to recession of mountain front. (C) In Karst Region, when the water enters in the cracles of rocks and dissolve the soluble material, numerous sink holes are formed but its depth is mere a few metres to 20 metres (normally from 3 metres to 10 metres) but not few hundred metres. (A) (D) Correct sequence of the different theories given in Q. is as follows— 1. Principle of uniformitarianism—1785 2. Theory of continental drift—1858 3. Dynamic equilibrium theory—1924–53 4. Convection current theory—1928-29 (B) When environmental lapse rate is less than the dry and wet adiabatic lapse rate, the atmospheric conditions are stable in the same way parcel of air is warmer than the surrounding environment at all the levels as it ascends. (A) Strong updrafts and downdrafts are the characteristics of mature stage of thunderstorm. (B) (D) Global pattern of surface wind from the equator to poleward is as follows— 1. Doldrums 5° Lat. to both the hemisphere from equator trade winds 5–10° to 30–35° Lat. in both the hemisphere Easterly upper part of atmosphere (though they are the part of trade winds) Westerlies 30–35° to 60° Lat. (D) Due to the marine influence the annual range of temperature is decreased. (D) 12. (B) 13. (C) 14. (A) (A) Spatial analysis is closely associated with the philosophy of positivism. (D) The concept of natural region was given by Vidal-de-Le Blache, to which he said pays. (B) (A) Birth rate governs the natural population growth. (B) 20. (A) (D) The spatial distribution of Rural Settlements can best be observed from topogra-

22. 23.

24. 27.

28. 33. 38. 39.

40.

41.

42. 47.

phical maps rather than wall cadastral and topographical maps. (D) (D) Whittlesey has divided world into following thirteen agricultural regions : 1. Nomadic herding region, 2. Livestock ranching region, 3. Shifting cultivation region, 4. Rudimental sedentary tillage region, 5. Intensive subsistence tillage with rice dominant, 6. Intensive subsistence tillage without paddy rice, 7. Commercial plantation, 8. Mediterranean agricultural region, 9. Commercial grain farming region, 10. Commercial livestock and crop farming region, 11. Subsistence crop and livestock farming region, 12. Commercial dairy farming region, 13. Specialized horticultural region. (A) 25. (D) 26. (A) (C) F. J. Maickender (England) was the Prof. of Geography in London University, was also the member of British Parliament as well as director of Political Science. He in his article, “The Geographical Pivot of history” geographically analyse the political aspect of the world, which was based on struggle of land and sea power. (B) 29. (B) 30. (A) 31. (C) 32. (C) (B) 34. (C) 35. (B) 36. (A) 37. (C) (D) (B) Baramati, situated in Eastern part of Western Ghat which is a leeward side while Mahabaleshwar, Panchgani and Ratnagiri are situated in the Western part of Western Ghat is a windward side. (D) Area of the given states in the question is as follows— 1. Rajasthan 3,42,239 km 2 , 2. Maharashtra 3,07,713 km2 , 3. Karnataka 1,91,791 km2 , 4. Tamil Nadu 1,30,158 km2. (C) Pie diagram is also known as circle and sectoral diagram. In this diagram the circle is associated with the area of a pie and different sectors shown in circle are associated with different aspect of that item which are shown on the bases of angles. (B) 43. (B) 44. (C) 45. (A) 46. (A) (B) 48. (A) 49. (C) 50. (A)

June 2010 Geography (Paper-II) Note—This paper contains fifty (50) objective type questions, each question carrying two (2) marks. Attempt all the questions. 61. The statement–“The present is the key to the past” was given by— (A) G.K. Gilbert (B) James Hutton (C) W.H. Davis (D) Walter Penk 62. Which process of chemical weathering causes rusting of iron ? (A) Carbonation (B) Oxidation (C) Hydration (D) Dissilication 63. Sea floor spreading theory was propounded by— (A) Harry Hess (B) Tuzo Wilson (C) A. Hobbes (D) D. L. Holms 64. Match List-I with List-II and select the correct answer from the codes given below— List-I (Theory) (a) Sea floor spreading theory (b) Continental drift theory (c) Elastic rebound theory (d) Radioactivity theory List-II (Names of Scientist) 1. Wegner 2. Harry Hess 3. Joly 4. Reed Codes : (a) (b) (c) (d) (A) 2 1 4 3 (B) 3 4 2 1 (C) 1 2 3 4 (D) 4 3 2 1 65. Red Sea is an example of— (A) Synclinal valley (B) Volcanic structure

(C) Rift valley (D) Eroded valley 66. Which of the following factors is responsible for the planetary wind belts ? (A) The tilt of earth’s axis (B) The rotation of earth (C) High and low pressure belts (D) Differential rate of heating of land and water 67. Given below are two statements, one is labelled as Assertion (A) and the other is labelled as Reason (R). Select your answer from the codes given below— Assertion (A) : Remains of older mountain belts lie within the shields in few places. Reason (R) : Older mountain belts in shield areas are composed of soft sedimentary rocks. Codes : (A) (A) is correct, but (R) is wrong (B) Both (A) and (R) are correct (C) (A) is wrong, but (R) is correct (D) Both (A) and (R) are wrong 68. The correct sequence of the following features towards sea from the coast is— (A) Berm – Lowtide terrace – Longshore trough – Longshore bar (B) Longshore trough – Longshore bar – Berm – Lowtide terrace (C) Lowtide terrace – Berm – Longshore bar – Longshore trough (D) Longshore bar – Lowtide terrace – Berm – Longshore trough

44 | UGC–NET Geography Solved Papers 69. Given below are two statements, one is labelled as Assertion (A) and the other is labelled as Reason (R). Select your answer from the codes given below— Assertion (A) : When sand arrives at a particular section of the beach more rapidly, than it is carried away, the beach is widened and built ocean ward. Reason (R) : The progradation process is everywhere common in the coastal environment. Codes : (A) (A) is correct, but (R) is wrong (B) (A) is wrong, but (R) is correct (C) Both (A) and (R) are correct (D) Both (A) and (R) are wrong 10. Who propounded the concept of ‘Poly Climax’ ? (A) Clement (B) Tanslay (C) Whittaker (D) Haeckel 11. The ‘Agenda 21’ was adopted in which of the following conventions ? (A) Stockholm convention (B) Rio-Earth summit (C) Rotterdam convention (D) Ramsar convention 12. The correct sequence of Koppen’s climatic type from Nile delta towards Congo basin is— (A) BWh – BSh – Aw – Af (B) Aw – Af – Bsh – BWh (C) BWh – Aw – BWh – Af (D) Af – Aw – BSh – BWh 13. Given below are two statements, one is labelled as Assertion (A) and the other is labelled as Reason (R). Select your answer from the codes given below— Assertion (A) : There is a little mixing of air between the troposphere and stratosphere. Reason (R) : The stratosphere holds large volume of water vapour and dust.

Codes : (A) Both (A) and (R) are true and (R) is the correct explanation of (A) (B) Both (A) and (R) are true, but (R) is not the correct explanation of (A) (C) (A) is true, but (R) is false (D) (A) is false, but (R) is true 14. “The special purpose of Geography is the comparison of unity in multiplicity” is the statement of— (A) Carl Ritter (B) Ratzel (C) Hartshorne (D) Humboldt 15. Which of the following area's observation motivated Davis to develop his concept on ‘Cycle of Erosion’ ? (A) Missouri (B) Grinoco (C) Montana (D) Red River 16. Kant viewed Geography as a— (A) Spatial science (B) Chorological science (C) Regional science (D) Systematic science 17. Match List-I with List-II and select the correct answer from the codes given below— List-I (a) Mark Jefferson (b) Robert Dichinson (c) Arthur Robinson (d) Alexander Von Humboldt List-II 1. Urban Geography 2. Settlement Geography 3. Physical Geography 4. Cartography Codes : (a) (b) (c) (d) (A) 2 4 1 3 (B) 2 1 4 3 (C) 1 3 4 2 (D) 3 1 2 4 18. Match List-I with List-II and select the correct answer from the codes given ahead— List-I (a) Strabo (b) Firamanus

UGC–NET Geography Solved Papers | 45 (c) Solinus (d) Elisee Reclus List-II 1. Christian Topography 2. Historical Memoir 3. La Terre 4. Rerum Memorabilum Codes : (a) (A) 2 (B) 2 (C) 4 (D) 2

(b) 3 1 2 1

(c) 1 4 3 3

(d) 4 3 1 4

19. City-size distribution is best depicted by— (A) Urban hierarchy (B) Central place theory (C) Rank-size rule (D) Losch’s Hexagonal Model 20. According to UN projections, which one of the following regions will contribute maximum to the global urban population by 2030 ? (A) South Asia (B) East Asia (C) South East Asia (D) Latin America 21. Given below are two statements, one is labelled as Assertion (A) and the other is labelled as Reason (R). Select your answer from the codes given below— Assertion (A) : Social landscape is a region in which one or several groups live and share a common set of ideas about their immediate environment. Reason (R) : It is a region where human needs, identity, security and stimulation are satisfied. Codes : (A) Both (A) and (R) are true and (R) is the correct explanation of (A) (B) Both (A) and (R) are true, but (R) is not the correct explanation of (A) (C) (A) is true, but (R) is false (D) (A) is false, but (R) is true

22. Given below are two statements, one is labelled as Assertion (A) and the other is labelled as Reason (R). Select your answer from the codes given below— Assertion (A) : Logistic population growth is allowed to develop in an optimal environment of unlimited size. Reason (R) : The growth follows an exponential curve. Codes : (A) Both (A) and (R) are true and (R) is the correct explanation of (A) (B) Both (A) and (R) are true, but (R) is not the correct explanation of (A) (C) (A) is true, but (R) is false (D) (A) is false, but (R) is true 23. Carl Sauer is best identified for his classic work related to— (A) cultural landscape (B) economic landscape (C) social landscape (D) physical lanscape 24. pH value of moderately alkaline soils varies between— (A) 4·5 to 5·0 (B) 5·0 to 5·5 (C) 5·8 to 6·4 (D) 7·8 to 8·4 25. Which one of the following agricultural systems is described as ‘Child of Industrial Revolution’ ? (A) Intensive subsistence tillage (B) Livestock ranching (C) Subsistence tillage (D) Collective farming 26. Match List-I with List-II and select the correct answer from the codes given ahead— List-I (Scholar) (a) Von Thunen (b) Julian Wolpart (c) W.W. Leontieff (d) Toresten Hagerstrand List-II (Model) 1. Input output model 2. Diffusion model

46 | UGC–NET Geography Solved Papers 3. Concentric rings of landuse 4. Decision making model Codes : (a) (b) (c) (d) (A) 1 3 2 4 (B) 4 2 1 3 (C) 3 1 2 4 (D) 3 4 1 2 27. Which one of them is a footloose industry ? (A) Iron and steel industry (B) Automobile industry (C) Cement industry (D) Cotton textile industry 28. Transport route development model by Taffe is based on his studies on— (A) Ethiopia and Egypt (B) Sudan and Nigeria (C) Kenya and Nigeria (D) Ghana and Nigeria 29. Which of the following ports has an outer harbour for export of iron ore ? (A) Kolkata (B) Mumbai (C) Vishakapatanam (D) Cochin 30. Match the association of the following ethnic groups in List-I with List-II and find the correct answer from the codes given below— List-I List-II (a) Masai 1. African grassland (b) Bhils 2. North-East India (c) Bushman 3. Kalahari (d) Kukis 4. Rajasthan Codes : (a) (b) (c) (d) (A) 2 1 3 4 (B) 1 2 3 4 (C) 3 1 2 4 (D) 1 4 3 2 31. Which of the following set of languages are spoken in descending order ? (A) English, Chinese, French, Russian (B) Chinese, French, English, Russian (C) Russian, English, French, Chinese (D) Chinese, English, French, Russian

32. Regions deleanated on the basis of relationship are called— (A) Natural regions (B) Nodal regions (C) Planning regions (D) Developing regions 33. Improving human welfare without causing damage to the environment is called— (A) Regional development (B) Regional planning (C) Sustainable development (D) Resource analysis 34. Which one of the following is an ideal combination for setting up iron and steel industry ? (A) Coal – Electricity – Market (B) Coal – Iron ore – Water (C) Electricity – Water – Iron ore (D) Coal – Water – Cotton 35. In which of the following States of India Uranium is embedded in the igneous and metamorphic rocks ? (A) Tamil Nadu (B) Maharashtra (C) Rajasthan (D) Kerala 36. Which is the largest physiographic division of India ? (A) Great Northern Plains (B) Great Indian Plateau (C) Greater Himalayas (D) The Coastal Plains 37. Which of the following earthquakes was the severely devastating and disastrous that occurred in India ? (A) Kangra Earthquake of 1905 (B) Bihar earthquake of 1934 (C) Gujarat earthquake of 2001 (D) Koyna earthquake of 1967 38. Match List-I with List-II and select the correct answer from the codes given ahead— List-I List-II (Waterfalls) (Rivers) (a) Chulia 1. Narmada (b) Sivasamudram 2. Chambal (c) Bheraghat 3. Cauvery (d) Mundru 4. Subernrekha

UGC–NET Geography Solved Papers | 47 Codes : (a) (A) 2 (B) 1 (C) 2 (D) 4

(b) 3 2 4 3

(c) 1 4 1 1

(d) 4 3 3 2

39. Economic reforms in India are leading to— 1. Concentration of economic activities. 2. Widening gaps between the rich and poor people. 3. Higher growth rate of metropolitan areas. 4. Low cost access to urban amenities and services. (A) 1 and 2 are correct (B) 1, 2 and 3 are correct (C) 2 and 4 are correct (D) 1, 3 and 4 are correct 40. According to nearest-neighbour index, what would be the maximum value for a complete dispersed settlement pattern ? (A) 0·00 (B) 1·55 (C) 2·15 (D) 2·88 41. Which of the following is a geo-synchronous satellite ? (A) LANDSAT (B) METEOSAT (C) IRS (D) KITSAT 42. Match List-I with List-II and select the correct answer from the codes given below— List-I (Characteristics) (a) Depiction of spatial interaction (b) Lines of distribution of equal value (c) Distinguish different areas by use of colours (d) A real unit of mapping List-II (Map type) 1. Isopleth 2. Flow map 3. Chorochromatic 4. Choropleth Codes : (a) (b) (c) (d) (A) 2 1 3 4 (B) 2 1 4 3 (C) 1 2 4 3 (D) 3 2 1 4

43. Relative shift of mean centre of population distribution in any region over a period of time can be examined applying— (A) Nearest neighbour analysis (B) Harmonic mean (C) Mean deviation from mean (D) Weighted arithmetic mean 44. Temperature measured on Kelvin scale is an example of— (A) Comparative scale (B) Vernier scale (C) Interval scale (D) Ratio scale 45. The suitable method to measure the change in spatial concentration of scheduled tribes population in India during the last three decades is— (A) Coefficient of variation (B) Gini coefficient (C) Index of diversification (D) Index of dissimilarity 46. Diversified cropping system in subsistence agricultural system is governed by : 1. Local demands 2. Local resource base 3. Market forces 4. Technological know–how Seclect the correct codes— Codes : (A) 1, 2 and 3 are correct (B) 1, 2 and 4 are correct (C) 2, 3 and 4 are correct (D) 1, 3 and 4 are correct Directions—(Q. 47–50) Read the following passage and answer the given questions— The chorological concept of geography focused on the study of the earth’s surface in terms of variations between areas (later named areal differentiation), was originally developed by Richthofen (1883) and subsequently advanced by Hettner in Germany. It was popularized and raised to the level of the dominant concept of geography in the English-speaking world through Hartshorne’s monumental Nature of Geography (1939). A strong tradition of regional geography had already developed in France under the leadership of Paul Vidal de la Blache. Indeed

48 | UGC–NET Geography Solved Papers regional geography reigned supreme in the world of geographical scholarship until the early 1950’s after which the regional paradigm for comprehending the earth as the world of man began, starting with Schaefer (1953), to be attacked as one that propagated ‘exceptionalism’ and ‘ideography’ in the discipline and thereby led the geographer away from the fundamental goal of science, i.e., the search for general principles of behaviour. A few years earlier, Kimble (1951) had condemned the concept of region as an eighteenth century concept. He maintained that “it is links in the landscape …… rather than the breaks that impress” the scientific mind. Bunge (1966) was convinced that the regionalists error lay in their “nation of the uniqueness of location”, owing to which they emphasized exceptions rather than similarities between places and locations. Coupled with this was a general impression that the French tradition of viewing the region as an ecological unit for the analysis of human groups was no longer relevant to the study of places and regions in the post-Industrial Revolution societies like those in Europe. All this contributed to a widespread retreat from regional geography. This meant the “eclipse of a traditional component of geography” which was until recently considered a sine quanon of the discipline. 47. The central theme of the passage is concerned with— (A) Contributions made by Richard Hartshorne (B) Research done by Ferdinand von Richthofen and Alfred Hettner (C) Contributions made by Fred K. Schaefer (D) History of regional concept 48. Who developed the concept of areal differentiation in geography ? (A) Richard Hartshorne (B) Paul Vidal de la Blache (C) Alfred Hettner (D) Ferdinand Von Richthofen 49. Who stated that the region is an eighteenth century concept and it links the land slope …… rather than the breaks ? (A) Fred K. Schaefer (B) Richard Hartshorne (C) G.H.T. Kimble (D) Paul Vidal-de-la Blache

50. ‘Pays Concept’ is associated with— (A) German school (B) French school (C) American school (D) Russian school

Answers with Explanations 1. (B) “The same physical processes and laws that operate today, operated throughout geologic time although not necessarily always with the same intensity as now”, This concept of modern geology known as a uniformitarianism, was propounded by James Huttan a Scotish Geomorphologist, which was later on known as Present is the key to past. 2. (B) Oxidation is one of the varieties of chemical weathering in which oxygen dissolved in water reacts with certain rock mineral especially iron to form oxides and hydroxides. The menifests itself in a brownish or yellowish staining of the rock surface which ultimately disintegrates. 3. (A) During the decade of 1970’s (Herris 1960) has proved through its researches that oceans and continents were never stationary. 4. (A) 5. (C) Red sea is an example of Rift valley related with valley of river Jordan, extended upto Zembeji river in a length of 4800 km. 6. (C) Origin of Planetary winds is associated with the Law and High pressure area, resulted with the rotation and temperature of the earth surface. 7. (A) Remains of old mountain beets are never composed of soft sedimentary rocks. 8. (A) 9. (C) 10. (B) The concept of poly-climax was propounded by Tanslay. 11. (B) Agenda 21 was adopted in Rio-Earth summit in June 1992. 12. (A) 13. (C) There is no dust and wsatervapour in the stratosphere. It is found in troposphere. 14. (A) The fundamental principle evolved by Ritter was unity in diversity or in multiplicity. 15. (C) W.M. Davis in 1877 while doing observations in Montana developed the theory of cycle of erosion, which he defined as geomorphological cycle.

UGC–NET Geography Solved Papers | 49 16. (B) According to Kant-History studies the phenomena which follow one after the other in time (Chronological Science) while Geography studies phenomena which lie side by side in space (Chorological Science). 17. (B) 18. (B) 19. (C) Difference in the rank size of cities in any region is the reference of the characteristics of urbanization in that region. It is supposed that rank size rule is the best among all the methods which produce correct picture of urban things as per their size. F. Ayerbuck, G.K. Giff, H.R. Smith are the urban Geographers who supported rank size rule. 20. (A) 21. (C) In a region it is not necessary that human needs, identity, security and stimulation are satisfied. 22. (C)

23. (A) Carl O, Sauer focused that human Geographers should make cultural processes the base of their thinking and observation. 24. (D) pH value of moderality alkaline soil varies between, 7·8 to 8·4. 25. (D) 26. (D) 27. (B) 28. (A) 29. (C) Vishakapatnam port has an outer harbour for export of coal. 30. (D) 31. (B) 32. (B) 33. (C) 34. (B) 35. (C) 36. (B) 37. (C) In Gujarat Earthquake of 2001, occurred at Bhuj. Nearly one lac people were killed and thousands were injured, its intensity on rictor scale was 7·9. 38. (A) 39. (D) 40. (C) 41. (B) 42. (B) 43. (D) 44. (A) 45. (A) 46. (A) 47. (D) 48. (D) 49. (C) 50. (B)

December 2010 Geography Paper-II Codes : (A) 1 (B) 2 (C) 4 (D) 3

Note—This paper contains fifty (50) objective type questions, each question carrying two (2) marks. Attempt all of them. 1.

2.

Delta Kames are the outcome of— (A) Glacial erosion (B) Wind deposition (C) River deposition (D) Glacial deposition Match List-I with List-II and select the correct answer using the codes given below— List-I (Name of the book) (a) Essays in Geomorphology (b) Morphology of the Earth (c) Unstable Earth (d) Techinques in Geomorphology List-II (Author) 1. L. C. King 2. G. H. Dury 3. C. A. M. King 4. J. A. Steers Codes : (a) (b) (c) (d) (A) 2 1 4 3 (B) 1 3 4 2 (C) 4 2 3 1 (D) 4 3 1 2

3.

4.

5.

A ramp of sand deposited by streams along foothills in the arid land is called— (A) Hamada (B) Bajada (C) Desert Wash (D) Pediment Arrange the following mountain building theories in order in which they published and select the correct answer using the codes given ahead— 1. Continental Drift Theory 2. Convection Current Theory 3. Sea Floor Spreading Theory 4. Plate Tectonic Theory

6.

7.

2 1 2 1

3 3 1 2

4 4 3 4

Given below are two statements, one labelled as Assertion (A) and the other labelled as Reason (R). Select your answer from the codes given below— Assertion (A) : Flowing as a sheet across a land surface, running water picks up particles and moves them downslope into a stream channel. Reason (R) : Stream flow always transport all kinds of sediment load downstream. Codes : (A) Both (A) and (R) are true (B) Both (A) and (R) are true, but (R) is not the correct explanation (C) (A) is true, but (R) is false (D) (A) is false, but (R) is true Flat topped steep sided upland capped by a resistant rock layer normally formed in arid lands is called— (A) Erg (B) Cuesta (C) Mesa (D) Escorponant Given below are two statements, one labelled as Assertion (A) and the other labelled as Reason (R). Select your answer from the codes given below— Assertion (A) : A characteristic feature of the tropical cyclone is its central eye. Reason (R) : Tropical cyclone is not associated with cloudfree vortex produced by the intense spiralling of the storm.

UGC–NET Geography Solved Papers | 51 Codes : (A) Both (A) and (R) are true and (R) is the correct explanation of (A) (B) Both (A) and (R) are true, but (R) is not the correct explanation of (A) (C) (A) is false, but (R) is true (D) (A) is true, but (R) is false 8.

9.

Rating curve is useful for estimation of— (A) River discharge (B) Stream velocity (C) Dissolved load (D) Hydraulic radius Given below are two statements, one labelled as Assertion (A) and the other labelled as Reason (R). Select your answer from the codes given below— Assertion (A) : Coral reefs grow in shallow and warm water. Reason (R) : Near vertical and direct incidence of solar radiation is essentially required for the growth of coral reefs. Codes : (A) Both (A) and (R) are true (B) Both (A) and (R) are false (C) (A) is true, but (R) is false (D) (A) is false, but (R) is true

10. Sublimation describes the direct transition from— (A) Solid to vapour (B) Vapour to solid (C) Liquid to vapour (D) Vapour to liquid 11. The savanna biome is usually associated with— (A) Tropical wet-dry climate (B) Equatorial wet clmate (C) Tropical dry climate (D) Monsoon climate 12. Who was the first Geographer to ascertain the length of the equator ? (A) Eratosthenes (B) Herodotus (C) Anaximander (D) Thales 13. Who was the first person to plot the Bay of Bengal ? (A) Strabo (B) Ptolemy (C) Al-Masudi (D) Humboldt 14. Who is the author of the book Modern Geographical Thought ? (A) David Harvey

(B) R. J. Johnston (C) Brian J. L. Berry (D) Richard Peet 15. Match List-I with List-II and select the correct answer from the codes given below— List-I (a) Humboldt (b) Oscar Peschell (c) Richtofen (d) Ratzel List-II 1. Anthropogeographie 2. Erdteile 3. Kosmos 4. Das Ausland Codes : (a) (b) (c) (d) (A) 1 2 3 4 (B) 2 4 3 1 (C) 3 2 4 1 (D) 3 4 2 1 16. Match List-I with List-II and select the correct answer from the codes given below— List-I (a) Isochrone Map (b) Isoneph Map (c) Isotim Map (d) Isophene Map List-II 1. Equal biological event 2. Equal time 3. Equal cloudiness 4. Equal transport cost Codes : (a) (b) (c) (d) (A) 2 3 4 1 (B) 2 4 1 3 (C) 4 3 2 1 (D) 3 2 4 1 17. ‘Principle of Least Effort’ related to human migration is propounded by— (A) Stouffer (B) Revenstein (C) Weber (D) Jiff

52 | UGC–NET Geography Solved Papers 18. Arrange the following countries in accordance with their population size— (A) Pakistan, Bangladesh, Sri Lanka, Nepal (B) Pakistan, Sri Lanka, Nepal, Bangladesh (C) Bangladesh, Pakistan, Nepal, Sri Lanka (D) Sri Lanka, Bangladesh, Nepal, Pakistan 19. Among the following South-Asian countries, which one is the most urbanized ? (A) Nepal (B) Bangladesh (C) Maldives (D) Sri Lanka 20. Given below are two statements, one labelled as Assertion (A) and the other labelled as Reason (R). Select your answer from the codes given below— Assertion (A) : At present urban natural increase is not an important contributor to high rates of urban growth as the rural urban migration. Reason (R) : Because of lower mortality, natural rate of increase is higher in the modern era than in the historical past. Codes : (A) Both (A) and (R) are true and (R) is the correct explanation of (A) (B) Both (A) and (R) are true, but (R) is not the correct explanation of (A) (C) (A) is true, but (R) is false (D) (A) is false, but (R) is true 21. Which one of the following is a renewable resource ? (A) Coal (B) Wind energy (C) Iron ore (D) Mica 22. ‘‘All resources are renewable on some time scale …… what matters for the sustainability of future supplies is the relative rates of replenishment and use …’’ In view of the above statement identify the correct order of the resource continuum in descending order of renewability— (A) Air, tidal power, bauxite, forest (B) Air, bauxite, forest, tidal power (C) Tidal power, air, bauxite, forest (D) Forest, air, tidal power, bauxite 23. The office of Registrar General and Census Commissioner of India is responsible for— I. Conducting decennial census and publication of its related data.

II. Publication of annual land-use data. III. Publication of annual health and education data. IV. Publication of annual vital statistics based on sample registration system. (A) I and II are correct (B) I and IV are correct (C) I and III are correct (D) All are correct 24. Given below are two statements, one labelled as Assertion (A) and the other labelled as Reason (R). Assertion (A) : Biomass energy sources are renewable resources lying in the critical limits. Reason (R) : Biomass can be replenished over a time period. In the context of the two statements, which one of the following is correct ? Codes : (A) Both (A) and (R) are true and (R) is the correct explanation of (A) (B) Both (A) and (R) are true, but (R) is not the correct explanation of (A) (C) (A) is true, but (R) is false (D) (A) is false, but (R) is true 25. Which of the steel plants is port based ? (A) Bhilai (B) Durgapur (C) Vizagapattanam (D) Rourkela 26. Which one of the following is not associated with political geography ? (A) F. Ratzel (B) I. Bowman (C) R. J. Johnston (D) H. J. Mackinder 27. Match List-I with List-II and select the correct answer from the codes given ahead— List-I (Geographers) (a) C. Sauer (b) D. Harvey (c) F. Ratzel (d) H. J. Johnston List-II (Associated with) 1. Radical geography 2. Cultural geography

UGC–NET Geography Solved Papers | 53 3. 4.

Social geography Political geography

Codes : (a)

(b)

(c)

(d)

(A) (B) (C) (D)

3 1 3 1

4 4 2 3

2 3 1 4

1 2 4 2

28. In India the State is used as a national planning unit because— (A) It is an administrative unit (B) It is a homogeneous unit (C) Of political considerations (D) Unitform physical features 29. One of the important determinants of urban land value is— (A) Climate (B) Historical factors (C) Topography (D) Accessibiity 30. The control or direction of resource development is called— (A) Resource utilization (B) Resource allocation (C) Resource management (D) Resource availability 31. The principle of homogeneity and interrelationship can be used to delineate units for— (A) Development planning (B) Resources analysis (C) Allocation of resources (D) Location specific facilities 32. During which of the following year National Water Policy was formulated in India ? (A) 2002 (B) 1987 (C) 2007 (D) 1985 33. Which of the following is not an Agro-climatic region of India ? (A) East Coast Plains and Hills (B) The Eastern Plateau and Hills (C) The Western Ghats (D) The Trans Ganga Plains

34. Which one of the following is wrongly matched ? (Peaks) (Height) (A) Anaimudi — 2965 m (B) Nanga Parbat — 7756 m (C) Mt. Everest — 8848 m (D) K2 — 8611 m 35. Given below are two statements, one labelled as Assertion (A) and the other labelled as Reason (R). Select your answer from the codes given below— Assertion (A) : The Northern plains and Eastern coastal areas possess a dense network of railways in India. Reason (R) : These regions have well developed industries. Codes : (A) Both (A) and (R) are true and (R) is the correct explanation of (A) (B) Both (A) and (R) are true, but (R) is not the correct explanation of (A) (C) (A) is true, but (R) is false (D) (A) is false, but (R) is true 36. Match List-I with List-II and select the correct answer from the codes given below— List-I List-II (Rivers) (Dams) (a) Damodar 1. Pong (b) Mahanadi 2. Konar (c) Chambal 3. Naraj (d) Satlej 4. Gandhi Sagar Codes : (a) (A) 2 (B) 2 (C) 4 (D) 1

(b) 1 3 3 2

(c) 3 4 2 3

(d) 4 1 1 4

37. Which of the following statement is correct on the share of India’s population to world population as per 2001 Census ? (A) 14·7% (B) 15·7% (C) 16·7% (D) 17·7%

54 | UGC–NET Geography Solved Papers 38. In remote sensing systems which sensors, measures naturally available energy ? (A) Active Sensors (B) Passive Sensors (C) Synthetic Aperture Radar (D) Microwave Remote Sensing 39. In which of the following diagrams the relationship of relative humidity and temperature is depicted ? (A) Hythergraph (B) Climograph (C) Ergograph (D) Band graph 40. Variable class interval for mapping will prove more meaningful when the variable is— (A) Normally distributed (B) Highly skewed (C) Highly consistent (D) Measured on ordinal scale 41. Harmonic mean as a method of central tendency is suitable for data pertaining to— (A) Index numbers (B) Growth-rates (C) Wind speed (D) Population density 42. Match List-I with List-II and select the correct answer from the codes given below— List-I (Mapping technique) (a) Chorochromatic (b) Choroschematic (c) Isoplething (d) Choroplething List-II (Characteristics) 1. Density 2. Sign and symbol 3. Tint layering 4. Interpolation Codes : (a)

(b)

(c)

(d)

(A) (B) (C) (D)

3 4 2 2

2 1 4 3

4 3 1 4

1 2 3 1

43. Given below are two statements, one labelled as Assertion (A) and the other labelled as

Reason (R). Select your answer from the codes given ahead— Assertion (A): Great Lakes region of U.S.A. is well developed with industries. Reason (R) : This region was the region of earlier settlement in U.S.A. by migrants from Britain. Codes : (A) Both (A) and (R) are true and (R) is the correct explanation of (A) (B) Both (A) and (R) are true, but (R) is not the correct explanation of (A) (C) (A) is true, but (R) is false (D) (A) is false, but (R) is true 44. ‘‘A Handbook of Commercial Geography’’ is a translated work published in 1889. Who of the following is the translator ? (A) Dudley Stamp (B) George Chisholm (C) Von Thunen (D) Karl Marx 45. Given below are two statements, one labelled as Assertion (A) and the other labelled as Reason (R). Select your answer from the codes given below— Assertion (A) : In modern time artifacts are changing faster than the mentifacts of cutlure. Reason (R) : Economic liberalization and modernization forces tend to bring critical economy and cultural change. Codes : (A) Both (A) and (R) are true, but (R) is not the correct explanation of (A) (B) Both (A) and (R) are true and (R) is the correct explanation of (A) (C) (A) is true, but (R) is false (D) (A) is false, but (R) is true 46. Match List-I with List-II and select the correct answer from the codes given below— List-I (Geographer) (a) Losch (b) Weber (c) Thombson (d) Reily

UGC–NET Geography Solved Papers | 55 List-II (Theory) 1. Agglomeration Theory 2. Market Area Location Theory 3. Least Coast Theory 4. Gravitation Theory Codes : (a)

(b)

(c)

(d)

(A) (B) (C) (D)

2 3 4 1

3 1 1 4

4 4 3 2

1 2 2 3

Directions—(Q. 47–50) Read the following passage and answer the given questions— Yeung and Lo (1996) and Tasaka (1998) have shown that the dynamics of urbanization in East Asia and South-East Asia started to change repidly in the 1980s. Tasaka (1998) characterizes urbanization in developing countries as ‘urban involution’, describing the characteristics of urbanization in developing countries by three terms : ‘concentrated urbanization’ (urbanization occurs while the rural population level is higher than that in developed countries); ‘primate cities’ (overconcentration of economic, political and cultural functions in primary cities); and ‘over-urbanization’ (inflow of population beyond a level the city can productively absorb). Among the factors associated with over-urbanization are high levels of unemployment and employment insecurity; largescale inadequancies of infrastructure such as mass transportation systems, water supply and sewerage systems, and waste treatment facilities; environmental pollution; severe stress on urban governance; and overall poverty levels that hamper the formation of financial mechanisms that could facilitate urban environmental infrastructure improvements (Kidokoro 1998). It is important to bear in mind that besides the mechanisms for urban growth described here, there are other driving forces for the urbanization process in Asia, and they vary in different countries. Urbanization in Asia is not a uniform process. The changes in the mechanisms of urban growth described above can be seen as imparting a huge impact on urban environmental issues. As the pace of urbanization accelerates, environmental problems may worsen due to inadequate urban environmental infrastructure. On the other

hand, the promotion of economic growth could speed up the solutions to environmental problems. 47. The tone of the above passage can best be described as— (A) Analytical and scientific (B) Factual but descriptive (C) Behavioural (D) Tentative and inconclusive 48. The author is primarily concerned with— (A) Criticizing the urbanization process (B) The process of urbanization and overurbanization in the developing countries (C) Factors of sub-urbanization in the developing countries (D) Urban infrastructural growth in Asia 49. Environmental problems according to the author can be improved with— (A) Social inclusion (B) Economic growth (C) Political will (D) Strong legislation 50. ‘Urban Involution’ in the developing countries is not characterized by— (A) Rural population level is higher than that in developed countries (B) A uniform process of rural urban change (C) Over-concentration of economic, political and cultural functions in primate cities (D) Inflow of population beyond a level the city can productively absorb

Answers with Explanations 1. (D) Delta Kame is a deposition at the mouth of Esker by Glacial deposition. It is a kind of ridge. It is formed by small sands in morain. 2. (A) 3. (B) 4. (A) 5. (A) 6. (C) 7. (D) 8. (B) 9. (A) 10. (A) 11. (A) 12. (A) 13. (B) 14. (D) 15. (D) 16. (A) 17. (D) 18. (C) 19. (C) 20. (B) 21. (B) 22. (D) 23. (B) 24. (A) 25. (C) India’s only steel plant Dockyard is Vizagapattanam. 26. (C) 27. (B) 28. (A) 29. (D) 30. (C)

56 | UGC–NET Geography Solved Papers 31. 36. 41. 46. 47.

(B) 32. (A) 33. (D) 34. (B) 35. (A) (B) 37. (C) 38. (B) 39. (B) 40. (C) (B) 42. (C) 43. (A) 44. (B) 45. (B) (B) (B) Yeung and Lo and Tasaka have shown the dynamics of urbanisation and over-urbanisation in East Asia and South-East Asia. 48. (B) The author in this para is primarily concerned with the process of urbanisation and over urbanisation in the developing countries.

49. (B) According to the author as the pace of urbanisation accelerates, environmental problem may worsen due to inadequate urban environmental infrastructure which could be solved through economic growth. 50. (B) Since, the process of rural-urban changes are almost same and are not easily visible in developing countries. It can be seen only when rural population outcast urban population in developing countries.

June 2011 Geography (Paper-II) Note—This paper contains fifty (50) objective type questions, each question carrying two (2) marks. Attempt all the questions. 1. The ‘firn’ is associated with— (A) Fluvial deposition (B) Glacial compaction and recrystallization (C) Wind accumulation (D) Coastal accumulation 2. The ‘Moho’ discontinuity is the surface of demarcation between— (A) Crust and Mantle (B) Mantle and Core (C) Mantle and Outer Core (D) Outer Core and Inner Core 3. Match List-I with List-II and select the correct answer from the codes given below— List-I (Theories) (a) Continental Drift theory (b) Sea floor spreading theory (c) Convection Current Theory (d) Elastic Rebound Theory List-II (Propounders) 1. Harry Hess 2. Reid 3. Wegener 4. Holmes Codes : (a) (b) (c) (d) (A) 3 1 4 2 (B) 1 2 4 3 (C) 4 1 3 2 (D) 1 3 4 2

4. Given below are two statements, one labelled as Assertion (A) and the other labelled as Reason (R). Select your answer from the codes given below— Assertion (A): The equatorial area at which the trade winds converge is known as Inter Tropical Convergence Zone (ITCZ). Reason (R) : The ITCZ usually lies at or near the meteorological equator (Doldrums). Codes : (A) Both (A) and (R) are true and (R) is correct explanation of (A) (B) Both (A) and (R) are true, but (R) is not correct explanation of (A) (C) (A) is true, but (R) is false (D) (A) is false, but (R) is true 5. Cirrus clouds are at about the height of— (A) 0 – 1·5 km (B) 1 – 3 km (C) 4 – 6 km (D) 8 – 14 km 6. The standard air pressure at the sea level is— (A) 1010·25 mb (B) 1013·25 mb (C) 1015·25 mb (D) 1017·25 mb 7. Match List-I with List-II and select the correct answer using the codes given ahead— List-I (Type of climate based on Koppen’s classification) (a) As (b) E (c) Bw (d) Cwg List-II (Region where found) 1. North Himalayan Area 2. Desert 3. Indo–Ganga Plain 4. Coromandel coast

58 | UGC–NET Geography Solved Papers Codes : (a) (A) 1 (B) 4 (C) 4 (D) 3

(b) 2 1 3 4

(c) 3 2 2 1

(d) 4 3 1 2

8. Match List-I with List-II and select the correct answer using the codes given below— List-I (Theory of origin of tides/coral reefs) (a) Progressive Wave Theory (b) Equilibrium Theory (c) Subsidence Theory (d) Glacial Control Theory List-II (Propounders) 1. Darwin 2. W Whewell 3. R. P. Daly 4. Issac Newton Codes : (a) (b) (c) (d) (A) 1 4 2 3 (B) 2 1 3 4 (C) 2 4 1 3 (D) 3 4 1 2 9. Match List-I with List-II and select the correct answers from the codes given below— List-I (Continents) (a) North America (b) South America (c) Africa (d) Australia List-II (Names of Temperate Grassland) 1. Veldt 2. Downsland 3. Prairie 4. Pampas Codes : (a) (b) (c) (d) (A) 1 2 3 4 (B) 2 4 3 1 (C) 3 1 4 2 (D) 3 4 1 2

10. Find out the sequence of vegetation zones from equator to pole— (A) Taiga, Tundra, Selva, Savanna (B) Selva, Savanna, Taiga, Tundra (C) Savanna, Taiga, Selva, Tundra (D) Tundra, Taiga, Savanna, Selva 11. Given below are two statements, one labelled as Assertion (A) and the other labelled as Reason (R). Select your answer from the codes given below— Assertion (A): The sea water is more saline in tropical areas. Reason (R) : Salinity of sea-water is dependant on temperature and fresh-water mixing. Codes : (A) (A) is true, (R) is false (B) (A) is false, (R) is true (C) Both (A) and (R) are true (D) Both (A) and (R) are false 12. Which of the following is different from the other three on the basis of their locations ? (A) Canaries Current (B) Bengula Current (C) Guinea Current (D) Labrador Current 13. Match List-I (Concept/Book) with List-II (Author) and select the correct answer using the codes given below— List-I (Concept/Book) (a) Erdkunde (b) Kosmos (c) Chorology (d) Areal differentiation List-II (Propounder/Author) 1. Humboldt 2. Ritter 3. Hettner 4. Ratzel 5. Richthofen Codes : (a) (b) (c) (d) (A) 1 2 5 4 (B) 2 1 3 4 (C) 1 2 4 5 (D) 2 1 5 3

UGC–NET Geography Solved Papers | 59 14. Match List-I with List-II and select the correct answer from the codes given below— List-I (Books) (a) Explanation in Geography (b) Principles of Geomorphology (c) Geography and Geographers (d) Where the grass is greener List-II (Authors) 1. R. J. Johnston 2. W. D. Thornbury 3. D. M. Smith 4. David Harvey Codes : (a) (b) (c) (d) (A) 4 2 1 3 (B) 2 4 3 1 (C) 3 4 2 1 (D) 1 2 3 4 15. Given below are two statements, one labelled as Assertion (A) and the other labelled as Reason (R). Select your answer from the codes given below— Assertion (A) : The work on Geography of Strabo was largely an encyclopaedic description of the world known to the Greeks. Reason (R) : Strabo’s book had laid down a clear foundation for chorological writing in geography. (A) Both (A) and (R) are correct and (R) explains (A) (B) Both (A) and (R) are correct but (R) does not explain (A) (C) (A) is true, but (R) is false (D) (A) is false, but (R) is true

(B) Communist – Socialist Theory (C) Demographic Transition Model (D) Neo-Malthusian Model 18. The regularity between the sizes of cities and their rank was first noted by— (A) Jefferson (B) Zipf (C) Christaller (D) Auerbach 19. According to Burgess, which of the following sequences of the concentric zones of cities from the centre to the outer limit is correct ? (A) CBD, Zone of transition, Residential zone, Zone of worker’s home, Commuters zone (B) Residential zone, CBD, Zone of transition, Commuters zone, Zone of worker’s home (C) Residential zone, Zone of transition, CBD, Zone of Worker’s home, Commuters zone (D) CBD, Zone of transition, Zone of worker’s home, Residential zone, Commuters zone

16. The correct sequence of the advent of the following geographers— (A) Ritter – Humboldt – Richthofen – Ratzel (B) Humboldt – Ritter – Ratzel – Richthofen (C) Humboldt – Ritter – Richthofen – Ratzel (D) Ritter – Humboldt – Ratzel – Richthofen

20. Match List-I with List-II and select the correct answer from the codes given below— List-I (a) Migration theory (b) Optimum population theory (c) Demographic transition theory (d) Intervening opportunities theory List-II 1. Edwin Cannon 2. S. Stouffer 3. E. G. Ravenstein 4. W. Thompson Codes : (a) (b) (c) (d) (A) 3 1 2 4 (B) 3 1 4 2 (C) 3 4 2 1 (D) 4 1 3 2

17. Which one of the population theories / models postulates a necessary causal link between modernisation on the one hand and fertility and mortality reduction on the other ? (A) Malthusian Model

21. Given below are two statements, one labelled as Assertion (A) and other labelled as Reason (R). Select your answer from the codes given ahead— Assertion (A) : Japan is a developed nation.

60 | UGC–NET Geography Solved Papers Reason (R) : A ample supply and diversity of natural resources is a major advantage to a country for economic growth. Codes : (A) Both (A) and (R) are true and (R) is the correct explanation of (A) (B) Both (A) and (R) are true, but (R) is not the correct explanation of (A) (C) (A) is true, but (R) is false (D) (A) is false, but (R) is true 22. Given below are two statements, one labelled as Assertion (A) and the other labelled as Reason (R). Select your answer from the codes given below— Assertion (A) : A footloose can be defined as an industry with material index = 1. Reason (R) : A footloose industry can be located at any location without any special economic advantage or disadvantage. Code : (A) Both (A) and (R) are true and (R) is correct explanation of (A) (B) Both (A) and (R) are true, but (R) is not the correct explanation of (A) (C) (A) is true, but (R) is false (D) (A) is false, but (R) is true 23. Which indicator from among the following could be the best indicator of socialwellbeing ? (A) Per Capita Income (B) Population Growth Rate (C) Crude Literacy Rate (D) Life–Expectancy 24. The transport model which suggests a sequential expansion of the transport network was propounded by— (A) Alan Gilbert (B) Taaffe, Morrill and Gould (C) de Souza and Porter (D) Edward Soja 25. Match the following economic activities with sectors and indicate the correct matches— (Sectors) (a) Primary sector (b) Secondary sector (c) Tertiary sector (d) Quarternary sector

(Economic activities) 1. BPO 2. Pisciculture 3. Handloom textiles 4. City bus service Codes : (a) (b) (c) (A) 1 2 3 (B) 2 3 4 (C) 2 3 1 (D) 4 3 2

(d) 4 1 4 1

26. The zones and strata theory pertaining to the origin and diffusion of human races was propounded by— (A) Ripley (B) Taylor (C) Huntington (D) Baker 27. Boundaries that are allocated and delimited before significant settlement in an area, are called— (A) Antecedent boundaries (B) Subsequent boundaries (C) Superimposed boundaries (D) Relic boundaries 28. Given below are two statements, one labelled as Assertion (A) and the other labelled as Reason (R). Select your answer from the codes given below— Assertion (A) : Mackinder’s theory of Heartland put forward a key formula – ‘‘Who rules East Europe commands the Heartland; who rules the Heartland commands the World island; who rules the World Island commands the World.’’ Reason (R) : Mackinder’s formula offered Poland the strategy for territorial expansion and international supremacy. Codes : (A) Both (A) and (R) are true and (R) is the correct explanation of (A) (B) Both (A) and (R) are true, but (R) is not the correct explanation of (A) (C) (A) is true, but (R) is true (D) (A) is false, but (R) is correct

UGC–NET Geography Solved Papers | 61 29. Given below are two statements, one is labelled as Assertion (A) and the other labelled as Reason (R). Select your answer from the codes given below— Assertion (A) : Denmark is a nation state in true sense of the term. Reason (R) : A nation–state has a territorial base, but does not have necessarily a social or cultural base. Codes : (A) Both (A) and (R) are true and (R) is the correct explanation of (A) (B) Both (A) and (R) are true, but (R) does not explain (A) (C) (A) is true, but (R) is false (D) (A) is false, but (R) is true 30. Match List-I with List-II and select the correct answer from the codes given below— List-I List-II (Language) (Family) (a) Tamil 1. Indo–Aryan (b) Sinhalese 2. Dravidian (c) Burmese 3. Astronesian (d) Javanese 4. Sind–Tibetan Codes : (a) (b) (c) (d) (A) 1 2 3 4 (B) 3 1 4 2 (C) 2 1 4 3 (D) 4 3 2 1 31. A functional region is delineated on the basis of— (A) Politico-administrative boundaries (B) Isolines (C) Physical division (D) Fields of Interactions 32. Who developed a methodology for the study of human communities through focused attention on the ‘Place-Work-Folk’ progression ? (A) T. H. Huxley (B) Patrick Geddes (C) H. R. Mill (D) Vidal-de-la Blache 33. After economic liberalisation started in India in 1991, the per capita State incomes in the country have— (A) Converged (B) Remained unchanged

(C) Diverged (D) Spread 34. Given below are two statements, one labelled as Assertion (A) and the other labelled as Reason (R). Select your answer from the codes given below— Assertion (A) : Dandakaranya was the first large inter-state effort to reclaim land for the settlement of refugees. Reason (R) : Dandakaranya region lies partly in Bihar and partly in West Bengal. Code : (A) Both (A) and (R) are correct and (R) explains (A) (B) Both (A) and (R) are correct, but (R) does not explain (A) (C) (A) is true, but (R) is false (D) (A) is false, but (R) is true 35. The correct sequence of planners in the Anglo-American tradition is— (A) Unwin, Perry, Howard, Geddes (B) Geddes, Howard, Perry, Unwin (C) Howard, Geddes, Unwin, Perry (D) Perry, Geddes, Unwin, Howard 36. Which State of India is the largest producer of wheat ? (A) Uttar Pradesh (B) Haryana (C) Punjab (D) Rajasthan 37. Electricity production in India through coal is (%)— (A) 60 (B) 30 (C) 90 (D) 10 38. Which of the following statements are correct ? (i) The soils of the Indian plateau are transported ones. (ii) Rivers have cut alluvial terraces in the Himalayas. (iii) Red soils occur in Tamil Nadu, South Karnataka plateau and Kerala. (iv) Petroleum is the major source of commercial energy in India. Select the answer from the codes given below— Codes : (A) (i) and (iii) (B) (i) to (iv) (C) (ii) and (iii) (D) (ii) and (iv)

62 | UGC–NET Geography Solved Papers 39. Match List-I with List-II and select the correct answers from the codes given below— List-I (Names of the Tribes) (a) Santhals (b) Mikirs (c) Jarawas (d) Bhils List-II (States) 1. Rajasthan 2. Andaman & Nicobar 3. Jharkhand 4. Assam Codes : (a) (b) (c) (d) (A) 4 3 2 1 (B) 3 4 1 2 (C) 1 2 3 4 (D) 2 3 1 4 40. Which of the following is the correct sequence in increasing order of growth rate of population in South Indian States during 1991–2001 ? (A) Tamil Nadu, Kerala, Andhra Pradesh, Karnataka (B) Kerala, Tamil Nadu, Andhra Pradesh, Karnataka (C) Kerala, Andhra Pradesh, Karnataka, Tamil Nadu (D) Karnataka, Kerala, Andhra Pradesh, Tamil Nadu 41. Vector and Raster data are used in— (A) Global Positioning System (B) Remote Sensing System (C) Geographical Information System (D) All of these 42. Consider the following statements, which of these statements are correct ? (i) Median is positional average in nature (ii) Rn. Value of 2·15 indicates dispersed pattern (iii) Standard Deviation will alter by change of origin (iv) Gini coefficient is a measure o f inequality in distribution (A) (i), (ii) and (iv) (B) (i), (ii), (iii) and (iv)

(C) (i), (iii) and (iv) (D) (i) and (iv) 43. Sector diagram is also known as— (A) Pictorial diagram (B) Ring diagram (C) Block Pile diagram (D) Pie diagram 44. Match List-I with List-II and select the correct answer from the codes below— List-I (Lands use features) (a) Cultivated area (b) Water bodies (c) Built up (d) Uncultivable waste List-II (Conventional colour code) 1. Red 2. Brown 3. Blue 4. Green 5. Yellow Codes : (a) (b) (c) (d) (A) 2 1 5 4 (B) 4 5 2 3 (C) 5 3 1 2 (D) 4 2 3 5 45. Find out the correct sequence of the following passes from West to East— (A) Lipu Lekh, Nathu La, Shipki La, Zojila (B) Zojila, Shipki La, Nathu La, Lipu Lekh (C) Zojila, Shipki La, Lipu Lekh, Nathu La (D) Shipki La, Zojila, Nathu La, Lipu Lekh 46. Which of the following is the correct sequence in terms of east flowing rivers from North to South in India ? (A) Mahanadi, Godavari, Krishna, Pennar, Kaveri (B) Mahanadi, Krishna, Godavari, Pennar, Kaveri (C) Mahanadi, Godavari, Krishna, Kaveri, Pennar (D) Mahanadi, Krishna, Godavari, Pennar, Kaveri

UGC–NET Geography Solved Papers | 63 Directions—(Q. 47–50) Read the following passage and answer the question given below— Richthofen’s answer to the question : What is geography ? As soon to be acclaimed as the pioneer statement on the scope and method of modern geography – one that, in the words of Hartshorne “set the direction of geographic thought for the future.” According to Richthofen, it was the distinctive purpose of geography to focus attention on the diverse phenomena that occur in interrelation on the face of the earth. He emphasized that in order to reach useful and reliable conclusions, geographical study of any part of the earth surface must start with a careful description of its physical features, and from there the student should move on to examine the interrelationships of other features of the earth’s surface to the physical geographic framework described at the outset. He underlined that the highest goal of geography was the exploration of the relationship of man to the physical earth and to the biotic features associated with it. 47. Richthofen’s ultimate goal of geography was to explore— (A) Development of regional geography (B) Areal differentiation (C) Man-environment relationship (D) Development of physical geography 48. Hartshorne’s opinion that Richthofen’s views on geography was ‘thought for the future’, was due to— (A) Its scientific content (B) Its idea of interrelationship of phenomena (C) Its emphasis on method of study (D) Its emphasis on purpose of geography 49. Richthofen’s method may be categorised in modern terminology as— (A) Ideographic (B) Chorographic (C) Chorological (D) Nomothetic 50. Richthofen’s aim in redefining geography was to develop geography as— (A) Human geography (B) Physical geography (C) Science (D) Humanities

Answers with Explanations 1. (B) Firn is a German term means old granular snow (density ranges 0·4 to 0·89 kg m.) at least in its second accumulation season in the process of being transformed into glacier ice. 2. (A) Moho-discontinuity – It is a seismic discontinuity occurring between the crust of the earth and underline mantel across which the velocities of Pauos Waves are significantly modified. 3. (A)

4. (A)

5. (C) Cirrus clouds (symbol Ci) white in colour, occurring as silky, fibrous or wisp, detached from each other. Its base can occur between 5000 metre to 13,700 m (i.e., 5 km to 13·7 km). 6. (B) The average pressure at sea level is 1013·25 mb. 7. (B)

8. (C)

9. (D) Following terms are used for temperate grass land in given continents— 1. North America – Prairie 2. South America – Pampas 3. Africa – Veldt 4. Australia – Downsland Hence, the correct sequence will be (3), (4), (1), (2) rather than (3), (4), (2), (1). 10. (B)

11. (A)

12. (D) Canaries, Bengula and Couimea currents are the currents of Western Coast of Africa while Labrador Current is the current of Western Coast of Greenland. 13. (B)

14. (A)

15. (D) Strabo work was not based on the work of Greece but Strabo laid the foundation of chorological writtings in Geography and was the first who codified the term chorology most elegantly. 16. (A) The correct sequence of the advent of given thinkers is as follows— 1. Ritter (1779–1859) 2. Humboldt (1790–1859) 3. Richthofen (1833–1905 4. Ratzel (1844–1904)

64 | UGC–NET Geography Solved Papers 17. (C) The demograhic Transition theory given by Thompson and W. Notestein is based on the trends in fertility and mortality. 18. (D) The Regularity between the size of city and their rank was first noted by F. Auerbach in 1913. 19. (D) 24. (B)

20. (B) 25. (B)

21. (C) 26. (B)

22. (D)

23. (A)

folk as a basic concepts in the study of cities and regions. 33. (D) Dandakaranya region lies in Chhattisgarh state, rather than Bihar and West Bengal. 34. (C)

35. (B)

36. (A) In 2008-09 As per Eco-Survey, Uttar Pradesh produces 35·39% of country’s wheat.

27. (A) Those boundaries which are demarcated or allocated before the significance of cultural senerio (settlement) are called Antecedent boundaries.

37. (C) As per data provided by Britannica Year Book 2011, in India 84% of the electricity in India is produced through coal.

28. (C)

40. (B) The sequence in increasing order of growth of rate (1991–2001) among the South Indian states was as follows— 1. Kerala 9·43% 2. Tamil Nadu 11·72% 3. Andhra Pradesh 14·59% 4. Karnataka 17·51%

29. (C)

30. (C)

31. (A)

32. (B) Sir Patric Geddes — the Scottish Geographer was the follower of F. Le Ploy; who Carried on research on family life style and family budget. He recognized that family life is dependent on family life style and family budgets. Le Ploy recognized that family life is dependent on means of obtaining subsistence that is work, while the character of the letter is largely determined by the nature of environment that is place. This led to the famous Le-Ploy formula which is basic to this ideas : place–work–family which Geddes transformed into the slogan : place – work –

38. (A)

41. (D)

39. (B)

42. (D)

43. (D) Pie diagram is also known as sector diagram. 44. (C) 45. (C) 46. (A) 47. (C) 48. (B) 49. (A) 50. (A)

December 2011 Geography Paper-II Note : This paper contains fifty (50) objective type questions, each question carrying two (2) marks. Attempt all the questions. 1. The slope declining model was proposed by— (A) W. Penck (B) W. M. Davis (C) L.C. King (D) Wood 2.

Nazca Plate lies— (A) West of South America (B) East of South America (C) East of North America (D) West of North America

3.

A narrow block elevated between two normal faults is called— (A) Graben (B) Horst (C) Monocline (D) Anticline

4.

Match List-I with List-II and select the correct answer from the codes given below— List-I (Landforms) (a) Arete (b) Polje (c) Yardang (d) Gorge List-II (Agents) 1. Running water 2. Glaciers 3. Underground water 4. Wind Codes : (a) (b) (c) (d) (A) 2 3 4 1 (B) 1 2 4 3 (C) 4 3 2 1 (D) 3 2 1 4

5.

Given below are two statements, one labelled as Assertion (A) and the other labelled as Reason (R). Select your answer from the codes given below— Assertion (A) : Spheroidal weathering is the result of chemical weathering.

Reason (R) : Spheroidal weathering occurs when water penetrates joints and dissolves the rock cementing material. Codes : (A) Both (A) and (R) are true and (R) is correct explanation of (A) (B) Both (A) and (R) are true, but (R) is not correct explanation of (A) (C) (A) is true, but (R) is false (D) (A) is false, but (R) is true 6.

Given below are two statements, one labelled as Assertion (A) and the other labelled as Reason (R). Select your answer from the codes given below— Assertion (A) : The islands along the Mid Atlantic Ridge are highly prone to volcanic activity. Reason (R) : The sea floor spreading is the main cause of earthquakes in the Atlantic Ocean. Codes : (A) Both (A) and (R) are true and (R) is correct explanation of (A) (B) Both (A) and (R) are true, but (R) is not correct explanation of (A) (C) (A) is true, but (R) is false (D) (A) is false, but (R) is true

7.

The position of Thermal equator is normally at— (A) 0° (B) 5°N (C) 5°S (D) 10°N

8.

Which of the following is not relevant to Ice Age Hypothesis ? (A) Thermal Contraction Theory (B) Carbon dioxide Hypothesis (C) Ozone Depletion Hypothesis (D) Volcanic Dust Hypothesis

66 | UGC-NET Geography Solved Paper 9.

Given below are two statements, one labelled as Assertion (A) and other labelled as Reason (R). Select your answer from the codes given below— Assertion (A) : Pressure in the troposphere falls with increasing height. Reason (R) : The upper atmosphere has relatively low temperature. Codes : (A) Both (A) and (R) are correct and (R) is correct explanation of (A) (B) Both (A) and (R) are true, but (R) is not correct explanation of (A) (C) (A) is true, but (R) is false (D) (A) is false, but (R) is true

10. Given below are two statements, one labelled as Assertion (A) and the other labelled as Reason (R). Select your answer from the codes given below— Assertion (A) : The Indian monsoon is influenced by El Nino. Reason (R) : At the occurrence of El Nino there develops a high pressure area along the coast of Peru. Codes : (A) Both (A) and (R) are true and (R) is correct explanation (B) Both (A) and (R) are true, but (R) is not correct explanation of (A) (C) (A) is true, but (R) is false (D) (A) is false, but (R) is true 11. Which of the following is not a calcareous ooze ? (A) Pteropod ooze (B) Diatom ooze (C) Globigerina ooze (D) None of the above 12. Which of the following is the most important element of ecosystem ? (A) Energy flow (B) Ecological succession (C) Food chain (D) Food web 13. Which of the following is considered to be the global carbon sink ? (A) Antarctic ice sheet (B) Sahara Desert

(C) Equatorial rainforest (D) Arctic Ocean 14. Which of the following is not correct about the desert biomes vegetation ? (A) Short tape roots (B) Reflective surface of leaf (C) Ephemeral plants (D) Waxy coating 15. Match List-I with List-II and select the correct answer from the codes given below— List-I (Names of Geographers) (a) Roger Minshull (b) Richard Hartshorne (c) Patrick Geddes (d) Norbert Wiener List-II (Contributions) 1. Cities in Evolution 2. The Human Use of Human Being 3. Regional Geography Theory and Practice 4. Nature of Geography Codes : (a) (b) (c) (d) (A) 3 4 1 2 (B) 2 1 3 4 (C) 4 3 2 1 (D) 1 2 3 4 16. Critical isodapane refers to— (A) The point of the least transport cost (B) The point of the maximum transport cost (C) The point of the minimum raw material cost (D) The point of the maximum raw material cost 17. Given below are two statements, one labelled as Assertion (A) and the other labelled as Reason (R). Select your answer from the codes given below— Assertion (A) : Environmental determinism in geography mainly dealt with racial distribution and the climate. Reason (R) : Environmental determinism was a philosophy of the coloniser. Codes : (A) Both (A) and (R) are correct and (R) is the correct explanation of (A)

UGC-NET Geography Solved Paper | 67 (B) Both (A) and (R) are correct, but (R) is not the correct explanation of (A) (C) (A) is correct, but (R) is false (D) (A) is false, but (R) is correct 18. Given below are two statements, one labelled as Assertion (A) and the other labelled as Reason (R). Select your answer from the codes given below— Assertion (A) : The locational-climatic factors put Mackinder’s Heartland at a relative disadvantage as compared to certain other larger and well-endowed areas of the world. Reason (R) : Heartland was a region of permanent difficulties because of its interior location and resultant extremes of climates. Codes : (A) Both (A) and (R) are correct and (R) is the correct explanation of (A) (B) Both (A) and (R) correct, but (R) is not the correct explanation of (A) (C) (A) is correct, but (R) is false (D) (A) is false, but (R) is correct 19. Given below are two statements, one labelled as Assertion (A) and the other labelled as Reason (R). Select your answer from the codes given below— Assertion (A) : Between the two World Wars the whole process of sub-urban growth and decentralisation started to speed up. Reason (R) : The forces behind these are partly economic due to great depressions, partly social due to rise of middle class and partly technological due to rapid development in transport system. Codes : (A) Both (A) and (R) are correct and (R) explains (A) (B) Both (A) and (R) are correct, but (R) does not explain (A) (C) (A) is correct, but (R) is wrong (D) (A) is wrong, but (R) is correct 20. Given below are two statements, one labelled as Assertion (A) and the other labelled as Reason (R). Select your answer from the codes given below— Assertion (A) : Malthus stated population has a tendency to increase in geometric ratio, while food production tends to increase in an arithmetic ratio.

Reason (R) : Malthus was optimistic that man could control his own growth in numbers. Codes : (A) Both (A) and (R) are correct and (R) is the correct explanation of (A) (B) Both (A) and (R) are correct, but (R) is not the correct explanation of (A) (C) (A) is correct, but (R) is false (D) (A) is false, but (R) is correct 21. Given below are two statements, one labelled as Assertion (A) and the other labelled as Reason (R). Select your answer from the codes given below— Assertion (A) : Population concentration is largely found in the regions like river basins and coastal plains. Reason (R) : Concentration of population is mainly due to availability of basic resources like soil, minerals and water. Codes : (A) Both (A) and (R) are true and (R) is the correct explanation (B) Both (A) and (R) are true, but (R) is not the correct explanation (C) (A) is true, but (R) is false (D) (A) is false, but (R) is true 22. Given below are two statements, one labelled as Assertion (A) and the other labelled as Reason (R). Select your answer from the codes given below— Assertion (A) : The Christian population is relatively numerous in the tribal-hill areas of North-Eastern India. Reason (R) : Many tribals were converted to Christianity during the British rule. Codes : (A) Both (A) and (R) are correct and (R) is the correct explanation of (A) (B) Both (A) and (R) are correct, but (R) is not the correct explanation of (A) (C) (A) is correct, but (R) is false (D) (A) is false, but (R) is correct 23. For the establishment of cotton textile industry which one of the following factor is more ideal ? (A) Nearness to market (B) Nearness to raw material

68 | UGC-NET Geography Solved Paper (C) Nearness to water resource (D) Transport network 24. Who among the followings was the first to develop the method of crop combinations ? (A) Weber (B) Wegener (C) Weaver (D) Wittfogel 25. Arrange the following authors in chronological order. Use the code given below— I. Strabo II. Ptolemy III. Herodotus IV. Erastothenes Codes : (A) Herodotus, Erastothenes, Strabo, Ptolemy (B) Erastothenes, Strabo, Ptolemy, Herodotus (C) Herodotus, Ptolemy, Strabo, Erastothenes (D) Erastothenes, Herodotus, Ptolemy, Strabo 26. Given below are two statements, one labelled as Assertion (A) and the other labelled as Reason (R). Select your answer from the codes given below— Assertion (A) : Many jute mills in India were closed down after the Independence. Reason (R) : There is acute shortage of raw materials and also there is shortfall in foreign demand for jute goods. Codes : (A) Both (A) and (R) are true and (R) is correct explanation of (A) (B) Both (A) and (R) are true, but (R) is not the correct explanation of (A) (C) (A) is true, but (R) is false (D) (A) is false, but (R) is true 27. Which sector of economy is most important for economic development ? (A) Primary (B) Secondary (C) Tertiary (D) None of these 28. Given below are two statements, one labelled as Assertion (A) and the other labelled as Reason (R). Select your answer from the codes given below— Assertion (A) : Agricultural productivity refers to the sum total of land productivity, labour productivity and capital productivity. Reason (R) : All of the three components are factors of production. Codes : (A) Both (A) and (R) are true and (R) is the correct explanation of (A)

(B) Both (A) and (R) are true and (R) is not the correct explanation of (A) (C) (A) is true, but (R) is false (D) (A) is false, but (R) is true 29. The boundary between India and Pakistan is an example of— (A) Subsequent Boundary (B) Superimposed Boundary (C) Relict Boundary (D) Antecedent Boundary 30. Which of the following Scheduled Tribe has a dispersed core ? (A) AO (B) Garo (C) Santhal (D) Bhil 31. Arrange the following religions in chronological order of their emergence— (A) Judaism-Christianity-Islam-Sikhism (B) Sikhism-Christianity-Judaism-Islam (C) Islam-Sikhism-Judaism-Christianity (D) Judaism-Islam-Christianity-Sikhism 32. Match List-I with List-II and select the correct answer from the codes given below— List-I (Tribes) List-II (States) (a) Bodo 1. Chhattisgarh (b) Garo 2. Jharkhand (c) Gond 3. Assam (d) Santhal 4. Meghalaya Codes : (a) (A) 1 (B) 2 (C) 3 (D) 4

(b) 2 3 4 3

(c) 3 4 1 2

(d) 4 1 2 1

33. The concept of the ‘Radiant City’ was propounded by— (A) Le Corbusier (B) Ernst May (C) Soria Y. Mata (D) Clarence Perry 34. The sector theory of city morphology— (A) incorporates the importance of transportation systems into the evolution of the social structure of the city (B) is particularly concerned with the concept of ecology

UGC-NET Geography Solved Paper | 69 (C) postulates that multiple centres or nuclei are responsible for urban growth (D) is concerned with directional bias 35. Harris and Ullman are best known for their contributions to— (A) Political Geography (B) Urban Geography (C) Economic Geography (D) Agricultural Geography 36. Territorial production complex is associated with— (A) Indian model of development (B) Brazilian model of development (C) Korean model of development (D) Russian model of development 37. It has a core area and a periphery— (A) Formal region (B) Perceptual region (C) Functional region (D) Vernacular region 38. Who laid the philosophical basis of regional planning ? (A) Benton MacKaye (B) Fredric le Play (C) F.S. Chapin (D) Carol Aronovinci 39. Match List-I with List-II and select the correct answer from the codes given below— List-I (Concept) (a) Garden Cities (b) New Towns (c) Neighbourhood (d) Traffic Segregation List-II (Propounder) 1. Clarence Perry 2. Sir Colin Buchanan 3. Ebenezer Howard 4. Sir Patrick Abercrombie Codes : (a) (b) (c) (d) (A) 4 3 1 2 (B) 3 4 1 2 (C) 1 2 4 3 (D) 4 1 3 2

40. Find out the correct bottom-up hierarchy of growth foci in India— (A) Growth centre, Growth point, Growth pole, Service centre (B) Growth pole, Growth point, Growth centre, Service centre (C) Service centre, Growth point, Growth centre, Growth pole (D) Growth pole, Growth centre, Growth point, Service centre 41. What is the correct sequence of area under soil cover in India in descending order ? (A) Alluvial soils, Black soils, Red soils, Laterite soils (B) Alluvial soils, Red soils, Black soils, Laterite soils (C) Red soils, Laterite soils, Black soils, Alluvial soils (D) Red soils, Black soils, Laterite soils, Alluvial soils 42. Which of the following is the correct sequence of States in decreasing order of density of population in India according to 2001 Census ? (A) West Bengal, Uttar Pradesh, Bihar, Kerala (B) Uttar Pradesh, West Bengal, Kerala, Bihar (C) West Bengal, Bihar, Kerala, Uttar Pradesh (D) Bihar, West Bengal, Kerala, Uttar Pradesh 43. Which of the following cartographic method is generally used to show population density ? (A) Contours (B) Choropleth (C) Dot method (D) Isopleth 44. Lorenz curve measures— (A) Curvature (B) Inequality (C) Similarity (D) Diversity 45. Arrange the following maps in order in which they are prepared with small to large scale. Use the codes given below— I. World Map II. Toposheet III. Cadastral Map IV. State Map Codes : (A) II, I, IV, III (B) III, II, I, IV (C) I, IV, II, III (D) IV, III, I, II

70 | UGC-NET Geography Solved Paper 46. Match List-I with List-II and select the correct answer from the codes given below— List-I (Satellites) (a) LANDSAT (b) SPOT (c) IRS (d) ERS Codes : (a) (A) 3 (B) 2 (C) 2 (D) 3

(b) 2 1 3 1

List-II (Launched by) 1. France 2. NASA 3. Europe 4. India 5. Russia (c) 5 4 1 5

(d) 1 3 4 2

Directions—Read the following paragraph and answer the following questions— In the United States many people consider economic growth a necessity, whereas some still support an attitude of ‘growth at any cost’, increasing numbers are becoming aware of the environmental costs of sustained economic growth. During the 1970s public dis-satisfaction with growthmania appeared in a variety of ways. Oregonians, for example, were saying that visitors were welcome but new residents were not. In Colorado bumper stickers decried the Californication of Colorado. In California voters passed proposition 20, which placed tight control on all coastal zone developments. Communities in many places became more selective about the types of industries which they were willing to accept; clean industries were considered desirable but dirty industries were not. 47. In the United States, many people consider economic growth a necessity, because of— (A) additional employment potential (B) additional tax resources for the government (C) potential for higher GDP growth (D) potential for higher personal incomes 48. Many Americans in recent times are becoming environmentally conscious because of— (A) spread of environmental awareness (B) environment friendly technologies add jobs in the economy

(C) the understanding of trade-offs between economic growth and environmental costs (D) economic growth is desirable not necessity 49. Growthmania refers to— (A) Aversion to growth (B) Growth only attitude (C) Growth of employment (D) Growth with environmental concerns 50. Many American States approve of tourists but not immigrants because— (A) the tourists create employment (B) the immigrants are non-English speaking (C) the immigrants take away jobs of the locals (D) the tourists bring in money

Answers with Explanations 1. (A) 2. (A) 3. (B) 4. (A) 5. (C) 6. (A) 7. (B) 8. (C) 9. (B) Troposphere is the uppermost layer in the atmosphere. On ascending the temperature decreases and pressure falls with increasing height. 10. (C) With the advent of EL Nino, variation in air pressures can be felt over oceans, consequently where there is high pressure over Indian Ocean, the climate is conclusive for Indian Monsoon. During last few years EL Nino has brought normal to heavy downpour because of dense water vapours in the atmosphere. 11. (B) Diatom ooze is a silica based ooze. It has only 3 – 30% Calcium content. 12. (C) 13. (C) 14. (A) Desert biomes vegetation has extreme capability to bear the dryness. They are ephemeral plants with waxy coating leaves, reflective surface of the leaves, sharp thorns, long tape root. ‘Nagfani’ and ‘Babool’ are the examples. 15. (A) 16. (A) 17. (D) Environmental determinism in geography is considered to be a philosophy of coloniser and deals with racial distribution and climate. 18. (A) 19. (B) 20. (C) 21. (A) 22. (A) 23. (A) 24. (C) 25. (A) 26. (A) 27. (B) 28. (A) 29. (D) 30. (D) 31. (A)

UGC-NET Geography Solved Paper | 71 32. (C) Following are the tribes and state they are found in— Tribes States Naga Nagaland, Manipur, Mizoram, Asom, Arunachal Pradesh Tharu Uttar Pradesh, Bihar, Uttarakhand and Rajasthan Bhil Rajasthan, Madhya Pradesh, Maharashtra, Gujarat Todo Kerala, Karnataka and Tamilnadu Santhal Jharkhand, West Bengal, Odisha, Asom and Bihar Gond Uttar Pradesh, Andhra Pradesh and Odisha Meena Rajasthan Khasi Rajasthan Bhotia Uttarakhand Jaunsari Uttar Pradesh and Uttarakhand Buksa Uttarakhand Ragi and Uttarakhand Bamrot Khakhar Uttar Pradesh and Jharkhand Bodo Asom Garo Meghalaya

33. (A) 34. (A) 35. (B) 36. (D) 37. (B) 38. (A) 39. (B) 40. (B) 41. (A) According to Indian Council of Agriculture Research the order of soil cover in India is— Alluvial soils > Black soils > Red soils > Laterite soils > Desert soils > Alkaline soils > Bio-soils > Forest soils. 42. (C) 43. (B) 44. (B) 45. (B) Cadastral Map—6" to 25" per mile Toposheet—1" per 1, 2, or 4 miles World Map—1" per 32 or 64 miles State Map—To depict bigger area with small scale. 46. (B) 47. (C) Economic development of a country depends on the growth GDP. 48. (C) Economic growth is important and it directly depends on environmental costs. Therefore to have the economic growth, it is important to understand economic growth and environmental cost. 49. (B) 50. (A)

June 2012 Geography (Paper-II) Note—This paper contains fifty (50) objective type questions, each question carrying two (2) marks. Attempt all the questions. 1. The normal cycle of erosion is associated with— (A) Marine Erosion (B) Wind Erosion (C) River Erosion (D) Glacial Erosion 2. Which among the following is one of the forces responsible for continental drift according to Wegner ? (A) Tidal force (B) Convection currents (C) Tensional force (D) Compressional force 3. Match List-I with List-II and select the correct answer using the codes given below— List-I (a) Kettle (b) Tarn (c) Esker (d) Drumlin List-II 1. Small mountain lake occupying a cirque. 2. Low winding glacio — Alluvial ridge. 3. Oval or elliptical hill of glacial tilt. 4. Deep depression usually situated along the moraine belt. Codes : (a) (b) (c) (d) (A) 4 3 2 1 (B) 2 4 1 3 (C) 4 1 2 3 (D) 3 4 2 1 4. Most of the details of world’s topography belongs to the— (A) Pre-cambrian era (B) Paleozoic era (C) Mesozoic era (D) Cenozoic era

5. Match List-I with List-II and select the correct answer using the codes given below— List-I (Rivers) (a) Krishna (b) Brahmaputra (c) Godavari (d) Yamuna List-II (Tributaries) 1. Chambal 2. Indravati 3. Tista 4. Bhima Codes : (a) (b) (c) (d) (A) 4 3 2 1 (B) 3 4 1 2 (C) 4 3 1 2 (D) 3 4 2 1 6. Air movement in the troposphere is caused by— (A) Solar wind (B) Convective current (C) Gravitational force (D) Air pressure 7. The unit of air pressure measurement is— (A) Degree (B) Isobars (C) Metres per second (D) Hecta Pascal 8. Match List-I with List-II and select the correct answer using the codes given ahead— List-I (a) Ferrel cell (b) Hadley cell (c) Polar cell (d) Intertropical convergence zone List-II 1. Doldrums 2. Easterlies 3. Westerlies 4. Trade winds

UGC-NET Geography Solved Paper | 73 Codes : (a) (A) 3 (B) 1 (C) 3 (D) 2

(b) 2 4 4 3

(c) 1 3 2 4

(d) 4 2 1 1

9. Which one of the following is not a mechanism of energy transfer ? (A) Conduction (B) Advection (C) Radiation (D) Ablation 10. Given below are two statements, one labelled as Assertion (A) and other labelled as Reason (R). Assertion (A) : Stable air resists vertical movement and unstable air ascends freely because of its own buoyancy. Reason (R) : When stable air is forced aloft the clouds that form are widespread and have little vertical thickness. In the context of the above two statements, which one of the following is correct ? (A) (A) is correct, but (R) is wrong (B) (A) is wrong, but (R) is correct (C) Both (A) and (R) are correct (D) Both (A) and (R) are wrong 11. The cold current flowing along the coast of Chile and Peru is known as— (A) Agulhas (B) EL-Nino (C) Humboldt (D) Canary 12. The average difference in the water level between high tide and low tide at a place is referred to as— (A) Tidal bore (B) Tidal period (C) Tidal range (D) Tidal wave 13. Domestication of plants and animals was started in— (A) Paleolithic period (B) Mesolithic period (C) Neolithic period (D) Pre-paleolithic period 14. Which one of the following pairs is not correctly matched ? (A) Autotrophs — Take energy from inorganic sources and fix it into energy rich organic compound

(B) Heterotrophs — Obtain their energy from living organism (C) Herbivores — Organisms that consume plants and animals (D) Detrivores — Obtain their energy from dead organism 15. Flat topped sea mounts are known as— (A) Shoals (B) Reefs (C) Guyots (D) Banks 16. Which one of the following pairs does not match correctly ? (A) P. Haggett — Geography : A modern synthesis (B) P. E. James — All possible worlds (C) G. P. Marsh — Explanation in Geography (D) Abler, Adams & Gould — Spatial organisation 17. Which one of the following statements is not correct ? (A) The concept of areal differentiation is related with ideographic approach (B) The concept of areal differentiation is related with nomothetic approach (C) The exponent of the concept of areal differentiation was Richard Hartshorne (D) The concept of areal differentiation is related with chorology 18. “The same environment carries different meanings to people with different ways of living and culture.” This statement is related with the concept of— (A) Determinism (B) Stop-and-go determinism (C) Probabilism (D) Possibilism 19. Who among the following emphasized on the behavioural environment in Geography ? (A) Burton (B) Gregory (C) Kirk (D) Johnston 20. Who defined Geography as human ecology ? (A) Schaefer (B) Hartshorne (C) Richthofen (D) Barrows

74 | UGC-NET Geography Solved Paper 21. Which one of the following metropolitan cities has registered the largest population in 2011 census ? (A) Chennai (B) Delhi (C) Kolkata (D) Mumbai 22. Which of the following States in India is passing through the stage of Demographic Transition which entails low fertility and low mortality ? (A) Assam (B) Haryana (C) Punjab (D) Tamilnadu

28. The theory of least cost location was proposed by— (A) Losch (B) Isard (C) Dicey (D) Weber 29. The intensive cultivation of vegetables, fruits and flowers in the periphery of cities in USA is called— (A) Truck farming (B) Factory farming (C) Market gardening (D) Agri-farming

23. Which one of the following is the important factor in Rural out — migration in India ? (A) Agricultural inefficiency (B) Population growth (C) Lack of Basic services in rural areas (D) Unemployment

30. Why are steel mills within Japan drawn to coastal locations ? (A) Nearness to iron ore reserves (B) Nearness to local markets (C) Availability of cheap labour (D) To minimise transportation cost for imported raw materials and steel exports

24. The ‘Range of a Good’ refers to— (A) The maximum distance which people have to travel to purchase a good or service (B) The variety of goods available at a market place (C) The maximum demand to make the service or good viable (D) The maximum population necessary for survival of a function

31. Which one of the following criteria is not used for the classification of human races ? (A) Eye (B) Nose (C) Ear (D) Hair

25. The ‘Administrative Principle’ in Christaller’s ‘Central Place Theory’ refers to— (A) K = 3 (B) K = 4 (C) K = 7 (D) K = 9 26. Which one of the following may be categorised as footloose industry ? (A) Cement (B) Ship building (C) Electronic components (D) Cotton textile 27. Gross Cultivated Area divided by Net Sown Area is known as— (A) Agricultural efficiency (B) Crop diversification (C) Crop concentration (D) Crop intensity

32. Match the following tribes and their habitats— (a) Khirgiz 1. Central Asia (b) Yakuts 2. Siberia (c) Lapps 3. Scandinavia (d) Eskimos 4. Canada Codes : (a) (b) (c) (d) (A) 1 3 2 4 (B) 4 2 3 1 (C) 2 1 3 4 (D) 1 2 3 4 33. Which of the following statements is not true about a State ? (A) It has a territorial base (B) It always has a social or cultural base (C) It claims internal sovereignty (D) It claims sovereignty in external relations 34. Which of the following languages does not belong to the Dravidian family ? (A) Sinhalese (B) Brahui (C) Kannada (D) Malayalam

UGC-NET Geography Solved Paper | 75 35. The habitat of the Toda tribe is— (A) Aravalli range (B) Siwalik range (C) Kaimur range (D) Nilgiri hills 36. Which one of the following indicators is most commonly used to measure the economic development of a country ? (A) Infant mortality rate (B) Availability of potable water (C) Female literacy (D) Gross National Product per capita 37. Which of the following matches is not correct ? (A) Myrdal — Cumulative causation model (B) Friedmann — Coreperiphery model (C) Boudville — Transport model (D) Frank — Dependency model 38. Who among the following formalized the concept that there exists relationship between the spatial organisation and national development ? (A) Luten (B) Dickens (C) Kuklinski (D) Friedmann 39. Which of the following sequences of the stages of economic growth according to Rostow’s model is correct ? (A) Traditional society, drive to maturity, take off, high mass consumption (B) Traditional society, take off, drive to maturity, high mass consumption (C) Traditional society, take off, high mass consumption, drive to maturity (D) Traditional society, drive to maturity, high mass consumption, take off 40. Which one of the following is a ‘Formal’ Region ? (A) Delhi metropolitan region (B) Hyderabad — Secunderabad region (C) Delhi — Jaipur — Agra Triangle (D) Assam — Valley 41. Between 2001 and 2011, the sex-ratio (Females per thousand males) has— (A) Increased by 5 to 10 (B) Decreased by 5 to 10 (C) Remained constant (D) Increased by less than 5

42. Match List-I with List-II and select the correct answer using the codes given below— List-I (Mineral) List-II (Mines) (a) Copper 1. Kudremukh (b) Iron ore 2. Balaghat (c) Manganese 3. Kodarma (d) Mica 4. Khetri Codes : (a) (b) (c) (d) (A) 1 2 4 3 (B) 3 2 1 4 (C) 4 1 3 2 (D) 4 1 2 3 43. In India, mangrove is the most important tree of— (A) Tropical moist deciduous forests (B) Tidal forests (C) Tropical dry deciduous forests (D) Sub-tropical moist forests 44. Which one of the following does not match correctly ? (A) Vishakhapatnam — Ship building (B) Titagarh — Railway equipment (C) Bhadravati — Iron & Steel (D) Pinjore — Machine Tools 45. In which of the following Decades, India experienced highest percentage of decadal growth ? (A) 1951–61 (B) 1961–71 (C) 1971–81 (D) 1981–91 46. Which one of the following is most appropriate for showing frequency distribution ? (A) Bar graph (B) Histogram (C) Pie diagram (D) Line graph 47. Which one of the following is commonly used tor computing the average value of river discharge ? (A) Arithmetic Mean (B) Geometric Mean (C) Harmonic Mean (D) Weighted Mean

76 | UGC-NET Geography Solved Paper 48. Which one of the following Rn values shows the random pattern of point distribution ? (A) 0 (B) 1·0 (C) 1·35 (D) 2·14 49. Which one of the following sampling techniques is most suitable for selection of representative villages of an area with diversified geographical features ? (A) Systematic (B) Stratified-random (C) Random (D) Clustered 50. In which one of the following methods interpolation is used ? (A) Choroschematic (B) Chorochromatic (C) Isopleth (D) Choropleth

Answers with Explanations 1. (C) Davis Normal Cycle of Erosion is associated with River Erosion. 2. (A) Wegner in his continental drift theory had presumed that the drifting of continents look place towards two sides one towards equator which was related with Gravitational force and buoyancy, the other towards West which was due to Tidal force. 3. (C) 4. (D) 5. (A) 6. (D) 7. (B) 8. (C) 9. (D) Ablation is related with erosion (Breaking of Rocks) rather than energy transfer. 10. (A) 11. (C) 12. (C) 13. (C) Neolithic period the last period of stone stage (after the mesolithic period) that period in Europe and West Asia was supposed to be 6000 to 8000 years ago. During this period beside pointed instruments and arms, the domestication of plants and animals also started. 14. (C) 15. (C) Guyot is a flat-topped variety of sea mount occuring mainly in pacific ocean. 16. (C) Explanation in Geography is written by D. Harvey not by G. P. Marsh.

17. (B) The term areal differentiation was coined and used by Hartshorne in his classic work of Nature of Geography. It is also known as chorology or chorography which is also ideographic. 18. (B) 19. (C) Kirk (1952–63) supplied are of the first behavioural model. 20. (D) 21. (D) As per provisional data of population released by census deptt. Govt. of India. The 2011 population of the given cities is as follows— 1. Mumbai — 1,24,78,447 2. Delhi — 1,10,07,835 3. Chennai — 46,81,087 4. Kolkata — 44,86,679 22. (C) 23. (D) 24. (B) 25. (C) In Christaller’s Theory Administrative Law is denoted by K = 7, Transport Law by K = 4 and Market Law by K = 3. 26. (C) 27. (C) 28. (D) Weber, a German Economists, in 1909 profounded the least cost location for Industry. In this theory he discussed two most important factors—(i) Transport cost of raw material as well as finished goods, (ii) Labour Cost & agglomeration. 29. (A) Truck farming is a specific type of cultivation in which vegetable, fruits & flowers are grown in periphery of cities in U.S.A. This type of farming is practiced in coastal areas of Mexico Gulf, East Atlantic Coast (Florida) and California Valley. 30. (D) 31. (C) 32. (D) 33. (B) 34. (B) 35. (D) Toda are the habitat of Nilgiri Hills (Tamilnadu). 36. (D) Economic development of a country can be known from Gross National Product per Capita (per capita income) rather than Infant mortality rate, availability of potable water and female literacy. 37. (B) 38. (C) 39. (B) 40. (D) Formal Region is a region which has uniform relief with uniform climate, uniform soil distribution in a uniform landuse, settlement and mode of life.

UGC-NET Geography Solved Paper | 77 41. (A) 42. (D) 43. (B) Mangrove tree is the most important tree of Tidal forests. 44. (B) 45. (B) Decadal growth of population (Percentage) in the given question has been as follows— 1951–61—21·64%, 1961–71—24·80%, 1971–81—24·66%, 1981–91—23·87%. 46. (B) 47. (A)

48. (B) In Nearest Neighbour analysis Rn value may be from 0 to 2·15. 0 means clustered distribution, 2·145 means uniform distribution and 1·0 means random distribution. 49. (C) For the selection of representative villages of an area with diversified Geographical features, Random sampling is the best suited sampling. 50. (C) Isopleth is a line drawn on a map connecting the point assumed to be of equal value i.e. interpolation of equal values points.

June 2012 Geography (Paper-III) Note—This paper contains seventy five (75) objective type questions of two (2) marks each. All questions are compulsory. 1. The valleys which drain in the same direction as the original consequent drainage but at the lower topographic levels and have developed with respect to new base levels are known as— (A) Resequent (B) Obsequent (C) Insequent (D) Subsequent 2. The concept of ‘base level erosion’ was proposed by— (A) Davis (B) Malott (C) Johnson (D) Powell 3. Given below are two statements, one labelled as Assertion (A) and the other labelled as Reason (R). Assertion (A) : The velocity of moving ice increases with steepness of slope of the area and thickness of glacial ice. Reason (R) : The velocity decreases to the sides owing to lesser depth of ice and friction against the valley walls and the bottom floor. In the context of the above two statements, which one of the following is correct ? (A) Both (A) and (R) are true and (R) is the correct explanation of (A) (B) Both (A) and (R) are true, but (R) is not the correct explanation of (A) (C) (A) is true, but (R) is false (D) (A) is false, but (R) is true 4. The normal cycle of erosion is associated with— (A) Marine erosion (B) Wind erosion (C) River erosion (D) Glacial erosion

5. Which of the following groups of scholars stressed the role of lateral erosion by streams in the formation of pediments ? (A) Mc. Gee, Paige, Blackwelder (B) Mc Gee, Blackwelder, Johnson (C) Paige, Blackwelder, Johnson (D) Paige, Johnson, Lawson 6. Which of the following is not a topographic evidence of rejuvenation in landscape ? (A) Incised meanders (B) Structural benches (C) Paired valley terraces (D) Multi-cyclic valley 7. Match List-I with List-II and select the correct answer from the codes given below— List-I List-II (a) Loess 1. river deposit (b) Moraines 2. glacial deposit (c) Gravels 3. wind deposit (d) Silt 4. marine deposit Codes : (a) (A) 1 (B) 3 (C) 4 (D) 3

(b) 2 2 1 4

(c) 4 4 3 1

(d) 3 1 2 2

8. Given below are two statements, one labelled as Assertion (A) and the other labelled as Reason (R). Assertion (A) : Ozone hole is observed only over Antartica. Reason (R) : Ozone depleting gases are present throughout the stratosphere. In the context of the above two statements, which one of the following is correct ? (A) Both (A) and (R) are true (B) (A) is true and (R) is wrong

UGC-NET Geography Solved Paper | 79 (C) Both (A) and (R) are wrong (D) (A) is wrong, but (R) is true 9. Which of the following types of clouds appears at the highest level in the sky ? (A) Alto-cumulus (B) Cirro-cumulus (C) Cumulo-nimbus (D) Strato-cumulus 10. Atmospheric temperature decreases with height in the troposphere because— (A) Air at higher altitude is less dense (B) Solar radiation is less at higher level (C) There are more atmospheric gases at higher altitudes (D) The atmosphere is heated by radiation from the surface 11. Which one of the following pairs is not correctly matched ? (A) Foehn — Warm dry winds moving down the Alps (B) Mistral — Cold wind blowing from Alps over France (C) Santa Ana — Warm dry wind moving down Appalachian (D) Bora — Cold winds experienced along the eastern coast of the Adriatic sea 12. Which one of the following gases is not a natural element of the atmosphere ? (A) Krepton (B) Argon (C) Chloroflurocarbon (D) Water vapour 13. In Koppen’s classification of climate, the symbol Aw refers to— (A) Tropical Savanna climate (B) Monsoon climate (C) Tropical Rainforest climate (D) Steppe climate 14. The strongest winds in a hurricane are found in— (A) the eye of the hurricane (B) the spiral bands of thunderstorm (C) the eye wall of the hurricane (D) the easterly waves 15. Which one of the following characterizes paternoster lake ? (A) A lake of volcanic origin

(B) A shallow stretch of water separated from the sea (C) A crescent shaped lake formed due to cut-off of a river meander (D) Lake formed on glacial stairways 16. Thornthwaite’s modified climatic classification is based on the concept of— (A) Effective temperature (B) Precipitation index (C) Potential evapotranspiration (D) Potential precipitation 17. Selva forests are— (A) Broad leaf evergreen forests (B) Broad leaf deciduous forests (C) Coniferous evergreen forests (D) Coniferous deciduous forests 18. The specific place of an organism in an ecosystem is called as— (A) Niche (B) Autotroph (C) Trophic level (D) Food Pyramid 19. Which of the following is referred to as producer in marine ecosystem ? (A) Small fish (B) Fungi (C) Zooplanktons (D) Phytoplankton 20. Saline soils are the results of— (A) very high precipitation and leaching of the soil (B) high rate of evaporation and very little leaching of the soil (C) high precipitation and very little leaching of the soil (D) All of the above 21. “There is no holiday for vegetation—growth is rapid, uninterrupted and continuous.” This statement applies to— (A) Taiga region (B) Monsoon region (C) Mediterranean region (D) Rainy tropics 22. The largest variety of plants and animals is found in the— (A) Temperate forests (B) Monsoon forests (C) Tropical forests (D) Tropical grassland

80 |

UGC-NET Geography Solved Paper

23. Which one of the following statements about salinity is not correct ? (A) Salinity is directly related to precipitation (B) There exists a direct relationship between the rate of evaporation and salinity (C) Low salinity is found near the mouth of a river (D) The salinity is highest at the tropics and decreases towards poles and equator 24. The origin of the ocean currents is related to— (A) Gravitational force (B) Winds (C) Salinity and density (D) All of the above 25. Schaefer was in favour of— (A) Exceptionalism (B) Idiography (C) Areal differentiation (D) Scientific generalization 26. Which of the following principles does not belong to logical positivism ? (A) Principle of causation (B) Structuralism (C) Behaviourism (D) Functionalism 27. Which of the following matches is not correct ? Author Book (A) William Bunge — Theoretical Geography (B) David Harvey — Social Well being : A Spatial Perspective (C) David Smith — Human Geography : A Welfare Approach (D) R. Peet — Modern Geographical Thought 28. The statement that “Egypt is the gift of the river Nile” is attributed to— (A) Aristotle (B) Herodotus (C) Strabo (D) Seneca 29. Who among the following pleaded that history be treated geographically and geography be treated historically ? (A) Homer (B) Thales (C) Hecataeus (D) Herodotus

30. Who among the following Arab scholars made corrections to Ptolemy’s book ? (A) Al-Masudi (B) Al-Idrisi (C) Ibn-Khaldun (D) Ibn-Batuta 31. Which one of the following is the philosophical basis of quantitative revolution ? (A) Existentialism (B) Idealism (C) Phenomenology (D) Positivism 32. The concept of paradigm was propounded by— (A) Kant (B) Peet (C) Kuhn (D) Haggett 33. Who among the following wanted to develop ‘Universal Science’ encompassing all aspects of knowledge ? (A) Humboldt (B) Ritter (C) Recluse (D) Guyot 34. Who among the following geographers, focussed on man centered geography ? (A) W. H. Davis (B) Jefferson (C) Semple (D) Huntington 35. Who, among the following, developed the concept of mental map ? (A) Downs and Stea (B) Gould and White (C) Saarinen (D) Boulding and Hagerstrand 36. Who among the following first initiated the concept of Second Demographic Transition ? (A) Van de Kaa (B) Lesthaeghe (C) Coleman (D) Fitzgerald 37. The Age-Specific Fertility Rate is maximum in the age-group of— (A) 20–24 years (B) 25–29 years (C) 30–34 years (D) 35–39 years 38. Which one of the following reasons dominates is Rural to Urban migration in India ? (A) Marriage (B) Education (C) Family movement (D) Employment

UGC-NET Geography Solved Paper | 81 39. Who argued that multiple centres of nuclei were responsible for urban growth ? (A) Harris and Ullman (B) Hoyt (C) Park and Burgess (D) Nelson

44. The is— (A) (B) (C) (D)

40. The largest and most dominating city in a State with respect to commercial, industrial, educational and political activities refers to— (A) City Region (B) Urbanfield (C) Primate city (D) Neckropolis

45. The footloose industries are those for whom— (A) Transportation costs are relatively unimportant (B) Transport costs are very important (C) Presence of raw material in the neighbourhood is necessary (D) All of the above are necessary

41. Which one of the following sequences of land use in Burgess model is correct ? (A) Central Business District, zone of workers’ home, commuters’ zone, zone of better residences (B) Central Business District, zone of better residences, zone of workers’ home, commuters’ zone (C) Central Business District, zone of workers’ home, zone of better residences, commuters’ zone (D) Central Business District, zone of better residences, commuters’ zone, zone of workers’ home

46. Match List-I with List-II and select the correct answer using codes given below— List-I List-II (a) Malaysia 1. Coffee (b) Brazil 2. Sugar (c) Cuba 3. Pineapple (d) Hawai 4. Rubber

42. If a circle with 1 cm radius represents one lac population, what will be the radius of the circle representing 4 lac population ? (A) 1 cm (B) 2 cm (C) 4 cm (D) 16 cm 43. Which one of the following market locations is optimal in the given diagram of industrial landscape ? Cost line Production line

a

(A) b (C) a

b

c

d

(B) c (D) d

essential feature of shifting cultivation Intensity of crops Rotation of fields Single cropping Double cropping

Codes : (a) (A) 2 (B) 1 (C) 4 (D) 4

(b) 3 2 1 3

(c) 4 3 3 2

(d) 1 4 2 1

47. The three iron and steel plants at Kulti, Burnpur and Hirapur have been merged together as— (A) Tata Iron and Steel Company (B) Indian Iron and Steel Company (C) Bengal Iron Works (D) Mysore Iron Works 48. The rice producing areas are associated with— (A) Low density of population (B) Moderate density of population (C) High density of population (D) None of the above 49. Match List-I with List-II and select the correct answer using the codes given ahead— List-I (Stage of Urbanisation) (a) Urbanisation

82 |

UGC-NET Geography Solved Paper

(b) Suburbanisation (c) Disurbanisation (d) Re-urbanisation List-II (Explanation) 1. Certain settlements grow at the cost of surrounding countryside. 2. The commuter belt grows at the cost the urban core. 3. The rate of population loss of the core tapers off or the core starts regaining population. 4. The population loss of the urban core exceeds the population gain of the commuter belt. Codes : (a) (b) (c) (d) (A) 1 2 3 4 (B) 1 2 4 3 (C) 2 3 4 1 (D) 3 2 1 4 50. Which of the following statements about the consequences of migration is correct ? (A) The population — resource relationship of the two areas involved in the process of migration gets modified significantly as a result of migration (B) The people who migrate from the rural areas to the urban industrial areas suffer from the lack of open space, fresh air and good housing (C) The dietary habits of emigrants also changes in new areas (D) All of the above 51. The informal sector of economy refers to— (A) Capitalist mode of economy (B) Bazaar type of economy (C) Dynamic and stable income (D) Permanent and productive establishment 52. Which one of the following is not related with informal sector of economy ? (A) Market type of economy (B) Unskilled/semiskilled labour force (C) Temporary and unstable income (D) Capitalist mode of economy

53. The law of primacy in the context of urban development is most relevant to the countries which have— (A) A relatively simple economy and spatial structure (B) Complex economy (C) Integrated spatial structure (D) Matured economy 54. The main occupation of the Gonds is— (A) Food gathering (B) Forestry (C) Hunting (D) Cultivation 55. Which one of the following is largest in areal extent ? (A) Cultural realm (B) Cultural region (C) Cultural landscape (D) Cultural point 56. What among the following is the correct sequence of the three stages through which a new boundary under most conditions is determined ? (A) Delimitation, demarcation, allocation (B) Allocation, delimitation, demarcation (C) Demarcation, delimitation, allocation (D) Allocation, demarcation, delimitation 57. Which one of the following is not usually considered an indicator of social well being ? (A) Infant mortality (B) Female literacy (C) Availability of potable water (D) Crime 58. Who among the following was an advocate of export based model ? (A) Alonso (B) Kuklinski (C) Perroux (D) North 59. The Vale of Kashmir is the only level strip of land in the Himalayas. Which river has laid its deposits to form this level plain ? (A) Ravi (B) Sutlej (C) Beas (D) Jhelum 60. In which of the following States of India women outnumber men ? (A) Uttar Pradesh (B) Karnataka (C) Haryana (D) Kerala

UGC-NET Geography Solved Paper | 83 61. Which one of the following States of India records the lowest density of population according to 2011 Census ? (A) Arunachal Pradesh (B) Meghalaya (C) Tripura (D) Manipur 62. About 85 per cent of the total iron ore production of India is from— (A) Jharkhand and Orissa (B) Karnataka and Orissa (C) Bihar and Madhya Pradesh (D) Karnataka and Andhra Pradesh 63. Name the Five Year Plan (FYP) in which the ‘Voluntary Sterilization Population Policy’ was introduced— (A) First Five Year Plan (B) Second Five Year Plan (C) Third Five Year Plan (D) Vth Five Year Plan 64. In the sub-continent of India, the region frequently affected by tropical cyclones is the— (A) Gujarat coast (B) Coromandel coast (C) Konkan coast (D) Malabar coast 65. Which of the following regions/areas is not affected by the monsoon of the Arabian Sea branch ? (A) Western Ghat (B) Deccan Plateau (C) Madhya Pradesh (D) Punjab Plain 66. The heights of individual points on topographical maps are indicated by— (A) Spot height only (B) Benchmark only (C) Triangulation point only (D) All of the above 67. The broadest part of the Himalayas lies in— (A) Himachal Pradesh (B) Arunachal Pradesh

(C) Jammu & Kashmir (D) Nagaland 68. Which of the following regions of India is most developed economically ? (A) North-eastern hill region (B) Eastern region (C) North-western region (D) Central region 69. If point A is at 230 m and point B at 570 m elevations from mean sea level with a horizontal equivalent of 2·0 km, which one of the following gradients is correct between these two points ? (A) 0·7 per cent (B) 7·0 per cent (C) 17·0 per cent (D) 27·0 per cent 70. Match List-I with List-II and select the correct answer using the codes given below— List-I (Satellite) List-II (Country) (a) SPOT 1. India (b) GOES 2. France (c) Meteor-3 3. Russia (d) INSAT 4. USA Codes : (a) (A) 2 (B) 2 (C) 4 (D) 4

(b) 4 4 2 2

(c) 3 1 3 1

(d) 1 3 1 3

71. Which one of the following techniques is not suitable for showing urban population on the map ? (A) Circles (B) Divided circles (C) Spheres (D) Hachures 72. Which one of the following statements is not correct ? (A) Contour maps are isopleth maps (B) Isopleths are lines of equal value (C) Isopleth technique always takes administrative boundaries into account (D) Isopleth technique is suitable for continuous data

84 |

UGC-NET Geography Solved Paper

73. Consider the following statements and select the correct answer using the codes given below— 1. Deserts, marshes and hilly tracts are known as the negative areas of population. 2. Dot method is the best technique to show the distribution of population. 3. Dot method can be used to show age and sex composition. 4. The dot map cannot be transferred into isopleth. Codes : (A) Only 1 and 3 are correct (B) 1, 2 and 3 are correct (C) 1, 2 and 4 are correct (D) 1, 2, 3 and 4 are correct 74. Which of the following is most suitable to show the unequality in the distribution ? (A) Lorenz curve (B) Bar graph (C) Circle diagram (D) Isopleth 75. Which one of the following is the correct geometric mean of the data set : 0, 50, 100, 100, 150, 250, 450, 500 ? (A) 125 (B) 100 (C) 200 (D) 0

Answers with Explanations 1. (A) Resequent drainage—A stream pattern which flows in the same direction as that of initial consequent stream but later on which develops in response to a new base level is called resequent drainage. 2. (D) The concept of Base level of erosion was profounded by J. W. Powell. 3. (B) The surface velocity of Glacier is greater in the central part then its side because the resistance is offered by the sides, by the rock Walls and Morain. Similarly, the ice of the surface moves faster than ice with in the Glacier. The surface velocity being the sum of slipping a shear deformation with in the ice. 4. (C)

5. (C) Paige (1912), Blackwelder (1931) and Johnson (1932). Gave the theory of formation of Pediments by later erosion. 6. (A) Incised meanders are formed prior to rejuvenation stage of river. 7. (B) 8. (B) There is no doubt the ozone hole was observed over Antartica, but O3 deplet gases are not present throughout the stratosphere. 9. (C) The height of the clouds given in the question are as follows— 1. Strato-cumulus—3000 metre 2. Alto-cumulus—3000–7500 metre 3. Cirro-nimbus—7000 metre 4. Cumulo-nimbus—More than 7500 metre 10. (A) At higher altitudes, the air is varified hence its density decreases. 11. (C) 12. (C) cfc is not a natural gas it is due to aeroplane, air conditioner and refrigerators. While Krepton, Argon and Water vapour are present in the atmosphere in their natural ways. 13. (A) 14. (A) Hurricane, a spanish name which refers primarily to the revolving tropical storms of the Carriben and Gulf of Mexico, Characterize the central areas of calm i.e. ‘eye’ around which the winds of very high velocity > 160 km/hr revolve and is accompained by torrential rain and by thunder and lighting. 15. (D) Paternoster lakes-a series of lakes in a glacial valley impounded behind moraines or in rock basins. Closed at their lower level by rock bars. 16. (C) Thornthwaits in his second classification (1948) use rainfall as-well-as potential Evapotranspiration. 17. (A) Selva—A Portuguese term for the equatorial rainforest of Amazon basin, now also applied to similiar types of vegetation else where in the world. 18. (A) Niche-a term used in ecology to describe the optimum habitat in an environment for its specific organism. 19. (D) 20. (B) 21. (D) 22. (C) 23. (A) 24. (D) 25. (C) 26. (A) 27. (B) 28. (B)

UGC-NET Geography Solved Paper | 85 29. (D) 30. (A) Al-Masudi tried to solve the proble concerning the exact source of river Nile. He rejected the idea of Ptolemy that the indus was connected with the Nile river. 31. (A) 32. (C) Kuhn, the American historian of science postulated a very important theory about the growth and development of science. For the development of science Kuhn prepared a model which he termed as ‘Paradigm of Science’. 33. (B) 34. (B) 35. (D) 36. (A) 37. (B) 38. (D)

39. (A) Multiple Nuclei theory for urban growth was profounded by C. D. Harris and Ullman in 1945. 40. (C) 41. (A) 42. (B) 43. (B) 44. (C) 45. (A) Footloose industries are those industries which can be established where the row material is even not available and the transport cost is unimportant e.g. (electrical goods). 46. (D) 47. (B) 48. (C) 49. (A) 50. (D) 51. (A) 52. (C) 53. (C) 54. (D) 55. (A) 56. (B) 57. (A) 58. (B) 59. (D) 60. (D) 61. (A) 62. (A) 63. (D) 64. (B) 65. (D) 66. (A) 67. (C) 68. (D) 69. (C) 70. (A) 71. (D) 72. (C) 73. (C) 74. (D) 75. (D)

December 2012 Geography (Paper-II) Note—This paper contains fifty (50) objective type questions of two (2) marks each. All questions are compulsory. 1. Landslides involve— (A) Downslope movement of individual rock blocks which is usually not perceptible (B) Rapid perceptible movement of relatively dry mass of earth debris or bed rock (C) Slow to very rapid movement of water saturated rock debris (D) Slow-flowing downslope masses of rock debris saturated with water. 2. Which of the following pairs is not a correct match of desert and country/continent ? (A) Atacama desert — South America (B) Gobi desert — Central Asia (C) Sonoran desert — West Asia (D) Great Victorian — Australia desert 3. One of the proponents of the turbidity current theory of the origin of submarine canyons was— (A) Daly (B) Dana (C) Salis (D) Gregory 4. The term panplane refers to— (A) A level surface formed in the old age of fluvial cycle (B) A level surface formed by wind erosion (C) A plain formed by marine action (D) A plain formed by joining of flood-plains 5. In the process of erosion, the removal of material by solution is called— (A) Attrition (B) Corrasion (C) Corrosion (D) Sapping

6. Arrange the solar spectrum in ascending order of its wavelength— 1. Ultraviolet 2. Microwaves 3. X-rays 4. Television waves Codes : (A) 4, 3, 2, 1 (B) 1, 3, 4, 2 (C) 3, 1, 2, 4 (D) 2, 3, 1, 4 7. Energy that is reflected by the atmosphere— (A) does not heat the atmosphere (B) has a different wavelength than the incoming energy (C) is usually short energy like visible light which does not penetrate the atmosphere (D) heats the atmosphere by absorption 8. Which one of the following statements about the polar jet stream is not true ? (A) It mvoes northwards during summer (B) It supplies energy to the circulation of surface storms (C) Its location roughly coincides with that of the polar front (D) Its velocity is greater during summer 9. The percentage of water vapour in the air in the tropical regions may be as high as— (A) 4 per cent (B) 10 per cent (C) 30 per cent (D) 50 per cent 10. The vertical air flow in an anticyclone results in— (A) diversion aloft (B) conversion aloft (C) diversion both aloft and at the surface (D) convergence at the surface and divergence aloft 11. The most common salt in the ocean is— (A) Calcium chloride

UGC-NET Geography Solved Paper | 87 (B) Sodium nitrate (C) Sodium chloride (D) Calcium nitrate 12. In the hydrological cycle evaporation is preceded by— (A) Clouds (B) Precipitation (C) Heat (D) Condensation 13. Which of the following occupies Trophic Level I in food chain ? (A) Bacteria (B) Carnivores (C) Primary consumers (D) Plants 14. Which of the following regions is known as the ‘carbon sink’ of the globe ? (A) Antarctica (B) Savanna (C) N.W. Europe (D) Tropical rainforest 15. Ecological Niche refers to— (A) Interaction between man and environment (B) The functional role of an organism in the ecosystem (C) Interdependence of man and plant (D) None of the above 16. Who among the following expounded the concept that “the present is the key to the past” ? (A) Buffon (B) Lyell (C) Hutton (D) Playfair 17. The approach that some of the geographical facts according to Ritter, cannot be explained scientifically, is termed as— (A) Ecological (B) Regional (C) Teleological (D) Locational 18. Whose pioneer statement on the scope and method of modern geography, according to Hartshorne, set the direction of geographic thought for the future ? (A) Blache (B) Schluter (C) Ratzel (D) Richthofen 19. Who among the following geographers emphasized the predominant role of climate in human life ? (A) Geddes (B) Jefferson (C) Huntington (D) Brunhes

20. Who is usually given the credit for establishing the validity of the idea that there was an ice age during which ice sheets covered much of northern Europe ? (A) Agassiz (B) Ramsay (C) Jukes (D) Penck 21. Given below are two statements, one labelled as Assertion (A) and the other labelled as Reason (R). Select your answer from the codes given below— Assertion (A) : The metropolitan cities are numerically dominated by male adult population. Reason (R) : Migration is age and sex selective. Codes : (A) Both (A) and (R) are true and (R) is the correct explanation of (A) (B) Both (A) and (R) are true, but (R) is not the correct explanation of (A) (C) (A) is true, but (R) is false (D) (A) is false, but (R) is true 22. The percentage of Aged population (60+) to the total population in India lies between— (A) 2–5 (B) 5–10 (C) 10–12 (D) 12–15 23. Which of the following migration streams experiences the maximum migration in India ? (A) Rural to Rural (B) Rural to Urban (C) Urban to Urban (D) Urban to Rural 24. Which one of the following describes a projected urbanised world or universal city by the end of 21st century ? (A) Conurbation (B) City-Region (C) Megalopolis (D) Ecumenopolis 25. Which one of the following indicates ‘Random’ distribution of settlements ? (A) 0·49 (B) 1·00 (C) 1·59 (D) 2·15 26. The stretchability of resource refers to— (A) The ability to find adequate substitution (B) Susceptibility to destruction (C) Ability to produce greater yields (D) Non-exhaustibility

88 | UGC-NET Geography Solved Paper 27. Which one of the following sequences of land use according to Von Thunen’s model is correct ? (A) Market gardening and fresh milk, firewood and lumber, intensive crop farming (B) Firewood and lumber, intensive crop farming, market gardening and fresh milk (C) Intensive crop farming, market gardening and fresh milk, firewood and lumber (D) Market gardening and fresh milk, intensive crop farming, firewood and lumber 28. Resource creation is the result of— (A) interaction of natural and cultural processes (B) interaction of different natural processes (C) interaction of different social processes (D) interaction of different economic processes 29. The final stage in the process of energy resource utilization is— (A) Transport (B) Storage (C) Disposal of waste material (D) Conversion 30. Who among the following developed the sequential expansion transport network model ? (A) De Souza and Porter (B) Brown and Bracey (C) Myrdal, Seers and Rostow (D) Taaffe, Morrill and Gould 31. The concept of the Geographical Pivot of History was proposed by— (A) Mickinder (B) Martin (C) Kropotkin (D) Ratzel 32. Which of the following categories does Chile belong to, according to shape ? (A) Fragmented (B) Fragmented cum elongated (C) Elongated (D) Compact cum elongated 33. The number of children dying in the first year of their birth per thousand live birth in a year is called— (A) Death rate

(B) Standardised mortality rate (C) Infant mortality rate (D) Life expectancy rate 34. Which one of the following arrangements of States in descending order of literacy rates according to 2011 Census is correct ? (A) Kerala, Mizoram, Tripura, Goa (B) Kerala, Goa, Mizoram, Tripura (C) Goa, Kerala, Tripura, Mizoram (D) Mizoram, Tripura, Kerala, Goa 35. Which one of the following is related to Tribal Economy ? (A) Intensive cultivation (B) Footloose Industry (C) Mining and Quarrying (D) Shifting cultivation 36. What are the bases of agro-climatic regionalization proposed by the Planning Commission of India ? (A) Relief, soil and drainage (B) Rainfall, temperature and cropped area (C) Vegetation, soil and land use (D) Relief, land use and Net sown Area 37. Which category of regions does metropolitan region belongs to ? (A) Formal region (B) Functional region (C) Single purpose region (D) Natural region 38. Who among the following attempted division of India into planning regions ? (A) O.H.K. Spate (B) L.D. Stamp (C) R.L. Singh (D) R.P. Misra 39. In which of the Five Year Plans regional imbalance was given prominence for the first time ? (A) First (B) Second (C) Third (D) Fourth 40. Which one of the following is not the ingredient of sustainable development ? (A) Intergeneration transferability (B) Social Justice (C) Inter-regional Transferability (D) Non-optimal resource utilization

UGC-NET Geography Solved Paper | 89 41. The main species of the coniferous forests of the Himalayas between the altitudes 1600 metres and 3300 metres are— (A) Cedar, Pine, Sine, Silverfir, Spruce (B) Sagon and Sakhu (C) Sandalwood, Rosewood (D) Oak, Chestnut, Chir, Chil 42. The Bundelkhand plateau covers parts of which two of the following States ? (A) Chhattisgarh and Jharkhand (B) Uttar Pradesh and Madhya Pradesh (C) Chhattisgarh and Uttar Pradesh (D) Madhya Pradesh and Chhattisgarh 43. Which of the following places in India ranks second among the coldest inhabited places in the world ? (A) Dras (B) Itanagar (C) Manali (D) Mana 44. Which of the following States has the highest percentage of its agricultural area under canal irrigation ? (A) Madhya Pradesh (B) Jammu & Kashmir (C) Uttarakhand (D) Uttar Pradesh 45. Which of the following ranges separates the Kashmir Valley from the Indus River Valley ? (A) Pir Panjal Range (B) Dhauladhar Range (C) Great Himalayan Range (D) Siwalik Range 46. Which one of the following methods is suitable to represent the proportions of different land uses in a region ? (A) Line graph (B) Pie diagram (C) Hythergraph (D) Triangular diagram 47. The dot method of distribution refers to— (A) Iconic Model (B) Analogue Model (C) Symbolic Model (D) Graphical Model 48. Which one of the following is a relative measure of dispersion ? (A) Standard deviation

(B) Quartile deviation (C) Coefficient of variation (D) Range 49. If a map is drawn on RF 1/5500, which one of the following representative fractions will be correct if the map is reduced to half ? (A) 1/2750 (B) 1/11,000 (C) 1/5,000 (D) 1/33,000 50. Consider the following statements and select the correct answer using the codes given below— 1. NASA is the national acquisition and distribution agency for all statellite data in India. 2. NASA sends the de-coded data to the NNRMS whose activities are guided by various Standing Committees. Codes : (A) Only 1 is correct (B) Only 2 is correct (C) Both 1 and 2 are correct (D) Neither 1 nor 2 is correct

Answers with Explanations 1. (C) A landslide or landslip is a geological phenomenon which includes a wide range of ground movement, such as rockfalls, deep failure of slopes and shallow debris flows, which can occur in offshore, coastal and onshore environments. 2. (C) The Sonoran Desert is a North American desert which covers large parts of the Southwestern United States in Arizona, California, Northwest Mexico in Sonora, Baja California, and Baja California Sur. It is one of the largest and hottest deserts in North America, with an area of 311,000 square kilometers (120,000 sq mi). 3. (A) 4. (A) 5. (C) Corrosion is the gradual destruction of material, usually metals, by chemical reaction with its environment. In the most common use of the word, this means electrochemical oxidation of metals in reaction with an oxidant such as oxygen. Rusting, the formation of ironoxides, is a well-known example of electrochemical corrosion. 6. (C) 7. (A)

90 | UGC-NET Geography Solved Paper 8. (D) Jet streams are fast flowing, narrow air currents found in the atmospheres of some planets, including Earth. The main jet streams are located near the tropopause, the transition between the troposphere (where temperature decreases with altitude) and the stratosphere (where temperature increases with altitude). 9. (A) 10. (B) An anticyclone is a weather phenomenon defined by the United States National Weather Service’s glossary as large-scale circulation of winds around a central region of high atmospheric pressure, clockwise in the Northern Hemisphere, counterclockwise in the Southern Hemisphere. (Wind Convergence in upper level and divergence in lower level). 11. (C) Sodium chloride, also known as salt, common salt, table salt or halite, is an ionic compound with the formula NaCl, representing equal proportions of sodium and chloride. Sodium chloride is the salt most responsible for the salinity of the ocean and of the extracellular fluid of many multicellular organisms. As the major ingredient in edible salt, it is commonly used as a condiment and food preservative. 12. (C) 13. (D) A food chain is a linear sequence of links in a food web starting from a trophic species that eats no other species in the web and ends at a trophic species that is eaten by no other species in the web. A food chain differs from a food web, because the complex polyphagous network of feeding relations are aggregated into trophic species and the chain only follows linear monophagous pathways. 14. (D) A carbon sink is a natural or artificial reservoir that accumulates and stores some carbon-containing chemical compound for an indefinite period. The process by which carbon sinks remove carbon dioxide (CO2 ) from the atmosphere is known as carbon sequestration. Public awareness of the significance of CO2 sinks has grown since passage of the Kyoto Protocol, which promotes their use as a form of carbon offset. 15. (B) In ecology, a niche is a term describing the way of life of a species. Each species is thought to have a separate, unique niche. The

16.

17.

18. 19.

20. 25. 30. 31.

32. 33.

ecological niche describes how an organism or population responds to the distribution of resources and competitors (e.g., by growing when resources are abundant, and when predators, parasites and pathogens are scarce) and how it in turn alters those same factors. (C) James Hutton was a Scottish physician, geologist, naturalist, chemical manufacturer and experimental agriculturalist. His work helped to establish the basis of modern geology. His theories of geology and geologic time, also called deep time, came to be included in theories which were called plutonism and uniformitarianism. He is also credited as the first scientist to publicly express the Earth was alive and should be considered a superorganism. (C) A teleology is any philosophical account that holds that final causes exist in nature, meaning that design and purpose analogous to that found in human actions are inherent also in the rest of nature. The adjective ‘teleological’ has a broader usage, for example in discussions where particular ethical theories or types of computer programs are sometimes described as teleological because they involve aiming at goals. (D) (C) Ellsworth Huntington was a professor of geography at Yale University during the early 20th century, known for his studies on climatic determinism, economic growth and economic geography. (A) 21. (A) 22. (B) 23. (A) 24. (C) (B) 26. (C) 27. (A) 28. (A) 29. (D) (D) (A) Sir Halford John Mackinder was an English geographer and is considered one of the founding fathers of both geopolitics and geostrategy. (B) (C) Infant mortality occurs when a child dies after birth. Childhood mortality is the death of a child before their fifth birthday. National statistics tend to group these two mortality rates together. Ten million infants and children die each year before their fifth birthday. Ninety-nine per cent of these infant and childhood deaths occur in developing nations. Infant mortality takes away society’s potential physical, social and human capital.

UGC-NET Geography Solved Paper | 91 34. (A) Rank 1 2 3 4

State Kerala Mizoram Tripura Goa

Literacy Rate (%) 100 75·9 74·2 83·3

Literacy Rate (%) 2001 Census 92·19 88·8 73·19 87·4

35. (D) Shifting cultivation is an agricultural system in which plots of land are cultivated temporarily, then abandoned. This system often involves clearing of a piece of land followed by several years of wood harvesting or farming, until the soil loses fertility. Once the land becomes inadequate for crop production. It is left to be reclaimed by natural vegetation, or sometimes converted to a different long-term cyclical farming practice. 36. (B) 37. (B) A functional region or Nodal region, is a region that has a defined core that retains a specific characteristic that diminishes outwards. To be considered a Functional region, atleast one form of spatial interaction must occur between the centre and all other parts of the region. A functional region is organized around a node or focal point with the surrounding areas linked to that node by transportation systems, communication systems, or other economic association involving such activities as manufacturing and retial trading. 38. (D) 39. (C) 40. (D) Sustainable development refers to a mode of human development in which resource use aims to meet human needs while preserving the environment so that these needs can be

41. 43.

44. 46.

47.

Literacy Rate (%) 2011 Census 93·9 91·6 87·8 87·4

% Increase 3·04% 2·80% 14·61% 0·00%

met not only in the present, but also for generations to come. The term ‘sustainable development’ was used by the Brundtland Commission which coined what has become the most often-quoted definition of sustainable development : “development that meets the needs of the present without compromising the ability of future generations to meet their own needs.” (A) 42. (B) (A) Dras is a town in the Kargil District of Jammu and Kashmir, India. It is often called ‘The Gateway to Ladakh’. (D) 45. (C) (B) A pie chart is a circular chart divided into sectors, illustrating proportion. In a pie chart, the arc length of each sector (and consequently its central angle and area), is proportional to the quantity it represents. When angles are measured with 1 turn as unit then a number of per cent is identified with the same number of centiturns. Together, the sectors create a full disk. It is named for its resemblance to a pie which has been sliced. The size of the sectors are calculated by converting between percentage and degrees or by the use of a percentage protractor. (B) 48. (C) 49. (B) 50. (C)

December 2012 Geography (Paper-III) Note—This paper contains seventy five (75) objective type questions of two (2) marks each. All questions are compulsory. 01. Most of the details of our present topography belong to the— (A) Pre-cambrian era (B) Paleozoic era (C) Mesozoic era (D) Cenozoic era 02. If you are sitting on the beach facing the sea on a sunny afternoon, the local wind will be— (A) striking your back and continuing to the sea because of sea breeze effect (B) towards your from the sea because of the sea breeze effect (C) towards your from the sea because of the lapse rate effect (D) striking your back and continuing to the sea because of the land breeze effect 03. The temperature at which water vapours condense, is the— (A) Relative humidity (B) Condensation point (C) Dew point (D) Evaporation point 04. Examine the following statements and select the correct answer using the codes given ahead— 1. The atmospheric air is never completely dry. 2. Water vapour can occupy as much as 4 per cent of the atmospheric volume. 3. Water vapour in the atmosphere is always invisible.

Codes : (A) 1 and 2 are correct (B) 2 and 3 are correct (C) 1 and 3 are correct (D) 1, 2 and 3 are correct 05. Chemical weathering is more effective than mechanical weathering in— (A) Semi arid region (B) Arid region (C) Coastal region (D) Cool temperate region 06. A drainage system which is not related to the structure of the region where it occurs is called a— (A) Radial drainage pattern (B) Trellis drainage pattern (C) Superimposed drainage pattern (D) Dendritic drainage pattern 07. Who calculated the slope based on the following formula ? tan α =

number of contours per mile × contour interval 3361

(A) Wentworth (C) Henry

(B) Raiz (D) Davis

08. Given below are the two statements one labelled as Assertion (A) and other Labelled as Reason (R). Assertion (A) : The work of wind as an agent of gradation is not as widespread as that of water. Reason (R) : It is effective only in the desert regions of the world where rainfall is scanty and soil particles are loose.

UGC-NET Geography Solved Paper | 93 In the context of the given two statements which one of the following is correct ? (A) Both (A) and (R) are true and (R) is the correct explanation of (A) (B) Both (A) and (R) are true, but (R) is not the correct explanation of (A) (C) (A) is true, but (R) is false (D) (A) is false, but (R) is true

In the context of the given two statements which one of the following is correct ? (A) Both (A) and (R) are true, but (R) is not correct explanation (B) Both (A) and (R) are true and (R) is the correct explanation (C) (A) is true, but (R) is false (D) (A) is false, but (R) is true

09. ‘The Two-Cycle Theory’ of the origin of limestone caverns was proposed by— (A) Davis (B) Swinerton (C) Gardner (D) Malott

15. When sun is shining vertically over Tropic of Capricorn ? (A) Days and nights are equal over the entire globe (B) Nights are longer than days in the Southern hemisphere (C) Nights are equal to 24 hours over the Arctic region (D) Days and nights are of 12 hour duration over Antarctica

10. All those processes which tend to bring the surface of the lithosphere to a common level are collectively known as— (A) Gradation (B) Degradation (C) Aggradation (D) Mass wasting 11. Which one of the following atmospheric layers reflects radio waves that are transmitted from the earth again back to the earth ? (A) Mesosphere (B) Ionosphere (C) Troposphere (D) Stratosphere 12. According to ‘Single cell circulation’ model proposed by Hadley, the most important factor causing global atmospheric circulation is— (A) subsidence at the equator (B) temperature contrast between equator and pole (C) the jet stream (D) rotation of the earth 13. Which of the following would likely to have the lowest albedo ? (A) Mirror (B) Fresh snow in the Arctic (C) Moist ploughed soil (D) Thick cloud 14. Given below are the two statements one labelled as Assertion (A) and the other labelled as Reason (R). Assertion (A) : Hurricane cannot develop in the vicinity of equator. Reason (R) : Coriolis force is maximum over the equator.

16. Which of the following statements is not true ? (A) Evaporation is directly proportional to temperature (B) Evaporation is more over land than ocean (C) Evaporation is higher over warm surface than on cold surface (D) Evaporation reduces the amount of heat available in the atmosphere thereby reducing the temperature 17. Match List-I with List-II and select the correct answer by using codes given below— List-I (a) Sublimation (b) Freezing (c) Evaporation (d) Condensation List-II 1. The energy absorbed is used to give the motion needed the escape the surface. 2. Energy is released. 3. Releases 80 cal/gm. 4. Absorb 680 cal/gm to change the state. Codes : (a) (b) (c) (d) (A) 2 3 4 1 (B) 1 3 2 4 (C) 4 3 1 2 (D) 3 2 4 1

94 | UGC-NET Geography Solved Paper 18. Pelagic deposits consist of matter derived from algae and are mostly in the form of liquid mud known as— (A) Blue mud (B) Ooze (C) Red mud (D) Coral mud

26. Organic deposits made by living or dead organisms that form rock elevation like a ridge are known as— (A) Corals (B) Conyons (C) Reefs (D) Guyots

19. The average temperature of the surface water of the oceans is— (A) 22°C (B) 15°C (C) 26·7°C (D) 18·6°C

27. Which one of the schools of thought first developed possibilism ? (A) German School of Geography (B) Russian School of Geography (C) French School of Geography (D) British School of Geography

20. Ecosystem has two components which are— (A) Animals and plants (B) Trees and weeds (C) Frogs and men (D) Abiotic and biotic 21. All the ecosystems taken together in a geographical area form a bigger unit called— (A) Community (B) Territory (C) Biome (D) Biosphere 22. Which one of the following ocean currents belongs to Indian Ocean ? (A) Canary Current (B) Antilles Current (C) Labrador Current (D) Agulhas Current 23. The Western Coast of Australia records higher salinity due to— (A) Ocean currents (B) Wind direction (C) Dry weather condition (D) All of the above 24. The highest salinity is found in the— (A) Baltic Sea (B) Okhotsk Sea (C) Mediterranean Sea (D) Red Sea 25. Which of the following statements about synecology is correct ? (A) It is a study of individual species in relation to its environment (B) It is a study of complex inter-relationship of biotic communities (C) It is essentially habitat ecology (D) It is mainly social ecology

28. Which one of the following is not a supporter of the theory of determinism ? (A) Wolfgang Hartake (B) R. Hartshorne (C) O.H.K. Spate (D) All of the above 29. Who among the following noted first the regularity between the sizes of cities and their rank ? (A) Zipf (B) Jefferson (C) Auerbach (D) Christaller 30. Who among the following is an advocate of idealism in geography ? (A) Guelke (B) Gilbert (C) Samuels (D) Pocock 31. The statement that ‘a method by which one can rethink the thought of those whose actions one seeks to explain’ refers in human geography to the philosophy of— (A) Phenomenology (B) Idealism (C) Existentialism (D) Positivism 32. Which among the following was responsible for the radical transformation of spirit and purpose of geography ? (A) Post Modernism (B) Humanism (C) Structuralism (D) Quantification 33. Which among the following was one of the forces responsible for continental drift according to Wegner ? (A) Tidal Force (B) Convection Currents (C) Tensional Force (D) Compressional force

UGC-NET Geography Solved Paper | 95 34. Which one of the following States/Union Territories recorded the highest density of population as per census 2011 ? (A) Chandigarh (B) West Bengal (C) Kerala (D) Maharashtra 35. If economic landscape of a region is dependent on ‘transport principle’ of central place system, with the population size of 4·0 lakh of its highest order town, which one of the following population size of next lower order town is correct ? (A) 3·0 lakh (B) 2·0 lakh (C) 1·0 lakh (D) 0·5 lakh 36. Who defined settlement geography first time incorporating the concept of settlement pattern, house types and field system ? (A) Meitzen (B) Stone (C) Huntington (D) Trewartha 37. The sparsely populated countries have slow growth of industries because— (A) of shortage of skilled labour (B) small population does not provide a good market (C) of both (A) and (B) (D) agriculture is more developed than industries 38. Which one of the following countries has the highest density of population ? (A) Bangladesh (B) Pakistan (C) Sri Lanka (D) India 39. The size of population in lower age group is large in countries where— (A) Birth-rate is high (B) Birth-rate is low (C) Death-rate is high (D) Death-rate is low 40. The base of the age-sex pyramid is broad and apex is narrow in— (A) Developed countries (B) Developing countries (C) Underdeveloped countries (D) None of the above

41. Which one of the following market locations is optimal in given diagram of industrial landscape ? Cost line Production line a b

(A) c (C) d

c

d

(B) b (D) a

42. Which one of the following matches in case of shifting cultivation is not correct ? (A) Ladang — Indonesia (B) Ray — Vietnam (C) Milpa — Mexico (D) Tamrai — Brazil 43. The polar regions are sparsely populated because— (A) The growing season is short owing to low temperature (B) There are no mineral resources (C) The regions are forested (D) Both (B) and (C) 44. Which one of the following countries is the major producer of mica ? (A) India (B) U.S.A. (C) Brazil (D) China 45. Match List-I with List-II and select the correct answer using the codes given below— List-I (Sectors) (a) Primary sector (b) Secondary sector (c) Tertiary sector (d) Quarternary List-II (Economic Activities) 1. Business organisation 2. Cultivation 3. Handloom textiles 4. Transport services Codes : (a) (b) (c) (d) (A) 1 2 3 4 (B) 2 3 4 1 (C) 2 3 1 4 (D) 4 3 2 1

96 | UGC-NET Geography Solved Paper 46. Match List-I with List-II and select the correct answer from the codes given below— List-I (Industrial activities) (a) Processing activities (b) Fabricating activities (c) Integrative activities (d) Administrative activities List-II (Explanations) 1. Inputs are processed goods. 2. Major inputs are raw material. 3. Involve neither inputs nor outputs. 4. Inputs are processed goods undergoing little change. Codes : (a) (b) (c) (d) (A) 1 2 3 4 (B) 2 1 4 3 (C) 1 4 3 2 (D) 3 2 4 1 47. The number of persons expressed in terms of unit area of agricultural land is known as— (A) Economic Density (B) Physiological Density (C) Arithmetical Density (D) Agricultural Density 48. Who among the following proposed a grand scheme of natural regions on a global scale by combining physical determinants of plants and animals life ? (A) Unstead (B) Herbertson (C) Whittlesey (D) Dickinson 49. Which of the following was not mentioned by Brunhes as essential fact of human geography ? (A) Facts of unproductive occupation of soil (B) Facts of plants and animal conquest (C) Facts of destructive exploitation (D) Facts of manufacturing 50. Which one of the following factors is not affecting the voting behaviour ? (A) Religious (B) Castes (C) High level of literacy (D) Size of landholdings

51. Which indicator from among the following could be the best indicator of social-well being ? (A) Population growth rate (B) Literacy rate (C) Life-expectancy (D) Per capita income 52. Who among the followings gave the concept of ‘Rimland’ theory ? (A) A Ratzel (B) O.H.K. Spate (C) Spykman (D) Griffith Taylor 53. Who propounded the theory of ‘Heartland’ in political geography ? (A) Mackinder (B) Spykman (C) Smith (D) Raiz 54. A functional region is delineated on the basis of— (A) Administrative boundaries (B) Field of interactions (C) Isolines (D) Physical divisions 55. Who in census of India applied the ‘ternary diagram’ for the functional classification of towns in India ? (A) P. Padmanabha (B) A.R. Nanda (C) Ashok Mitra (D) Chandrasekhar 56. Match List-I with List-II and select the correct answer from the codes given below— List-I (Author) (a) W. Christaller (b) V.L.S. Prakasa Rao (c) Manzoor Alam & W. Khan (d) M. Chisholm List-II (Name of the Book) 1. Rural settlement and land use. 2. Metropolitan Hyderabad and its region. 3. Regional Planning. 4. Central places in Southern Germany. Codes : (a) (b) (c) (d) (A) 1 3 4 2 (B) 2 1 3 4 (C) 4 3 2 1 (D) 1 2 3 4

UGC-NET Geography Solved Paper | 97 57. Who authored the book entitled ‘Growth Pole and Growth Centres for Regional Economic Development in India’ ? (A) Sen and Wanmali (B) Sadasyuk and Sengupta (C) Misra, Rao and Sundaram (D) Rao and Sundaram 58. Which one of the following states recorded the highest decadal growth rate of population between 2001 and 2011 as per census record ? (A) Bihar (B) Uttar Pradesh (C) Rajasthan (D) Arunanchal Pradesh 59. Match List-I with List-II and select the correct answer from the codes given below— List-I (Type of climate) (a) Aw (b) Amw (c) BShw (d) BWhw List-II (Region) 1. Rain shadow zone of Karnataka 2. Thar Desert 3. W. Bengal and Bihar 4. Malabar Coast Codes : (a) (b) (c) (d) (A) 3 4 1 2 (B) 3 1 4 2 (C) 2 1 4 3 (D) 2 4 1 3 60. Which one of the following rivers does not make a delta ? (A) Mahanadi (B) Godavari (C) Tapti (D) Ganga 61. Which one of the following rivers does not flow through Himachal Pradesh ? (A) Jhelum (B) Beas (C) Chenab (D) Ravi 62. Match List-I with List-II and select the correct answer from the codes given ahead— List-I (Rivers) (a) Krishna (b) Brahamaputra (c) Godavari (d) Yamuna List-II (Their Tributaries) 1. Chambal 2. Indravati 3. Tista 4. Bhima

Codes : (a) (A) 4 (B) 3 (C) 4 (D) 1

(b) 3 4 3 4

(c) 2 2 1 3

(d) 1 1 2 2

63. Match List-I with List-II and select the correct answer from the codes given below— List-I (Soil) (a) Alluvial (b) Black soil (Regur) (c) Desert (d) Red List-II (State) 1. Rajasthan 2. Uttar Pradesh 3. Maharashtra 4. Meghalaya Codes : (a) (b) (c) (d) (A) 1 2 3 4 (B) 2 3 4 1 (C) 2 3 1 4 (D) 4 2 3 1 64. Which one records the lowest normal temperature in the month of January ? (A) Bikaner (B) Bengaluru (C) Jaisalmer (D) Firozpur 65. Match List-I with List-II and select the correct answer from the codes given below— List-I (Pass) (a) Shipki La (b) Niti pass (c) Nathula (d) Bomdi La List-II (State) 1. Arunachal Pradesh 2. Himachal Pradesh 3. Uttarakhand 4. Sikkim Codes : (a) (b) (c) (d) (A) 2 3 4 1 (B) 2 3 1 4 (C) 4 2 1 3 (D) 4 2 3 1 66. Which one of the following states has the lowest percentage of scheduled tribes ? (A) Arunachal Pradesh (B) Manipur (C) Mizoram (D) Nagaland

98 | UGC-NET Geography Solved Paper 67. Which of the following hills forms the North Eastern edge of the Chhotanagpur Plateau ? (A) Rajmahal Hills (B) Mikir Hills (C) Parasnath Hills (D) Javadi Hills 68. Consider the following statements and select the correct answer using codes given below— 1. In a Representative Fraction (R.F.), the numerator is always one. 2. In an R.F. Scale, the unit of denominator is always as of the unit of numerator. 3. R.F. is not suitable technique of showing scale on a map. 4. The statement scale, e.g., 1 cm to 1 km is suitable scale on a map. Codes : (A) 1, 2 and 3 is correct (B) 1, 3 and 4 is correct (C) 2, 3 and 4 is correct (D) 1, 2 and 4 is correct 69. If Representative Fraction (RF) of a map is 1/5000 which is reduced three times, which of the following RF is correct for reduced map ? (A) 1/2,500 (B) 1/1,000 (C) 1/15,000 (D) 1/25,000 70. Consider the following statements and select the correct answer using the codes given below— 1. IRS is the mainstay of National Natural Resources Management System (NNRMS) in India. 2. Development and operational responsibilities of earth observation in India is overseen by the National Remote Sensing Agency (NRSA). Codes : (A) Only 1 is correct (B) Only 2 is correct (C) Both 1 and 2 are correct (D) Neither 1 nor 2 is correct 71. Which one of the following statement is correct in contest of Sten-de-Geer’s method used for showing population distribution ? (A) Rural population is shown by sphere method (B) Urban population is shown by dot method

(C) Scale of Cube-roots are used for showing the spheres (D) Spheres occupy bigger space on map 72. Which one of the following measurements is used for inequality distribution of farmers income subject to their landholding size of an area ? (A) Standard Deviation (B) Coefficient of Variation (C) Gini Index (D) Correlation Coefficient 73. Consider the following statements and select the correct answer using the codes given below— 1. Random sampling is called as probability sampling. 2. Random sampling is done with help of random table. 3. Landscape elements are generally scattered at random. Codes : (A) 1 and 2 are correct (B) 2 and 3 are correct (C) 1 and 3 are correct (D) 1, 2 and 3 are correct 74. Match List-I with List-II and select the correct answer from the codes given ahead— List-I (Cartographic method) (a) Chorochromatic (b) Choroschematic (c) Isopleth (d) Choropleth List-II (Map) 1. Contour 2. Administrative map 3. Use of letters like R = Rice, W = Wheat 4. Density of population Codes : (a) (A) 2 (B) 2 (C) 3 (D) 3

(b) 3 3 2 2

(c) 4 1 4 1

(d) 1 4 1 4

UGC-NET Geography Solved Paper | 99 75. According to Nearest Neighbour Index, what would be the maximum value for the perfect uniform settlement distribution ? (A) 0·00 (B) 2·15 (C) 1·55 (D) 2·89

Answers with Explanations 1. (D) Cenozoic, a greek term means ‘recent life’ adopted to describe the third of the era of Geological time. Originally the term was regarded as being synonymous with Tertiory in which the Alpine movements took place and present topography look shape. 2. (B) 3. (C) The dew point is the temperature below which the water vapor in a volume of humid air at a constant barometric pressure will condense into liquid water at the same rate at which it evaporates. Condensed water is called dew when it forms on a solid surface. 4. (A) 5. (A) Chemical weathering is a processes that lead to the decomposition or breakdown of solid rocks by means of chemical reactions. These comprise—1. Carbonation, 2. Hydrolysis 3. Oxidation and 4. Reduction and Solution. These processes are most effective in semi-arid region. 6. (D) Dendritic drainage is a type of drainage pattern which develops as an entirely random network because of the absence of structural control. Thus a dendritic pattern will be characteristics of terrain which is of uniform lithology. 7. (A) 8. (A) 9. (A) 10. (C) Aggradation means building up of the land surface by the deposition of fluvial or marine deposits and making the surface to a level area. 11. (B) The ionosphere is a part of the upper atmosphere, from about 85 km (53 mi) to 600 km (370 mi) altitude, comprising portions of the mesosphere, thermosphere and exosphere, distinguished because it is ionized by solar radiation. It plays an important part in atmospheric electricity and forms the inner edge of the magnetosphere. It has practical importance because, among other functions, it influences radio propagation to distant places on the Earth. 12. (B)

13. (C) Albedo means reflection coefficient or reflectivity of an object. It varies according to colour as well as texture of the surface snow 85%, Dark soil 3%. 14. (C) 15. (C) 16. (B) 17. (C) 18. (B) 19. (D) 20. (D) An ecosystem is a community of living organisms (plants, animals and microbes) in conjunction with the non-living components of their environment (things like air, water and mineral soil), interacting as a system. These biotic and abiotic components are regarded as linked together through nutrient cycles and energy. 21. (C) Biomes are climatically and geographically defined as similar climatic conditions on the Earth, such as communities of plants, animals, and soil organisms, and are often referred to as ecosystems. Some parts of the earth have more or less the same kind of abiotic and biotic factors spread over a large area, creating a typical ecosystem over that area. Such major ecosystems are termed as biomes. Biomes are defined by factors such as plant structures (such as trees, shrubs, and grasses), leaf types (such as broadleaf and needleleaf), plant spacing (forest, woodland, savanna), and climate. 22. (D) The Agulhas Current is the Western Boundary Current of the south-west Indian Ocean. It flows down the east coast of Africa from 27°S to 40°S. It is narrow, swift and strong. 23. (C) 24. (D) Red Sea, being the inlet of Indian ocean, lying between the Africa and Asia has salinity ranging between 37–41‰, while Baltic Sea with salinity 3–13‰, Okhotsk Sea 30–32‰, and Mediterranean Sea with 37 to 39‰. 25. (B) 26. (C) In nautical terminology, a reef is a rock, sandbar, or other feature lying beneath the surface of the water or less beneath low water. Many reefs result from abiotic processes—deposition of sand, wave erosion planning down rock outcrops, and other natural processes—but the best-known reefs are the coral reefs of tropical waters developed through biotic processes dominated by corals and calcareous algae.

100 | UGC-NET Geography Solved Paper 27. (C) Possibilism in Geography developed as a reaction to extreme generalization of environment determinists that led to a counter thesis of Possibilism. Possibilism attempts to explain man-environment relationship in a different way taking man as an active agent in environment. This is a belief which asserts that natural environment provides options, the number of which increases as the knowledge technology of a cultural group develop led by French geographers followers of historian Lucian Febure. 28. (C) After the Second World War, the philosophy of environmentalism was attacked. Many Geographers in U.S.A., Canada, U.K. and other countries drew attention to the one sided approach adopted by the environmentalists in their interpretation Spate-criticized the fanatic approach of environmental determinists. For example States environment taken by itself is a meaningless phase without man environment does not exist. 29. (A) G.K. Zipf in 1949 explained the Rank size rule since then it got the popularity. He described the city rank size on the baises of General theory of human behaviour. He used this formula (P r = P1 /r). Pr means population of rank size city, P1 means population of primate city and r result in the rank of city. He used this formula over 100 cities (big) of America. 30. (A) 31. (B) 32. (D) 33. (A) Wegner in his continental drift theory was of the view that the movement of continents took places at two side— 1. Toward the North from equator—it was due to gravitational force and force of buoyancy. 2. Towards the west—it was due to Tidal force.

34. (A) Chandigarh — 9252 West Bengal — 1030 Kerala — 859 Maharashtra — 365 35. (C) 36. (A) 37. (C) 38. (A) As per Britanica year book, 2011 the density of population in the given countries is as follows— 1. Bangladesh (2010) 1071·1 persons 2. Pakistan (2010) 209·1 persons 3. Sri Lanka (2010) 311·1 persons 4. India (2011) 382 persons 39. (A) 40. (D) 41. (B) 42. (D) 43. (A) 44. (A) 45. (B) 46. (B) 47. (D) 48. (B) 49. (D) 50. (D) 51. (C) 52. (C) 53. (A) Sir Halford John Mackinder was an English geographer and is considered one of the founding fathers of both geopolitics and geostrategy. 54. (B) 55. (C) Ashok Mitra in 1971 using the census data of 1961 divided them into three groups— A, B and C to know the function of a city. He used triangular co-ordinates for the first order and last order (Higher level to low level function). According to him functional characteristics of a city can be known by intersecting points in triangle. 56. (C) 57. (C) 58. (D) 59. (A) 60. (C) 61. (A) Jhelum River is a river that flows in India and Pakistan. It is the largest and most western of the five rivers of Punjab, and passes through Jhelum District. It is not through Himachal Pradesh state. 62. (A) 63. (C) 64. (D) 65. (A) 66. (B) 67. (A) 68. (D) 69. (C) 70. (A) 71. (C) 72. (D) 73. (A) 74. (B) 75. (B)

Geography (Paper–II)

UGC-NET/JRF Exam., Solved Paper

June 2013

Geography (Paper-II) Note—This paper contains fifty (50) objective type questions of two (2) marks each. All questions are compulsory. 1. Which one of the following is not a glaciofluvial depositional landform ? (A) Esker (B) Drumlin (C) Kame (D) Horn 2. Which one of the following arrangements of glacial periods of Pleistocene Ice Age is in a correct chronological order ? (A) Würm – Mindel – Gunz – Riss (B) Riss – Mindel – Würm – Gunz (C) Mindel – Würm – Gunz – Riss (D) Gunz – Mindel – Riss – Würm 3. The rigid masses surrounding the geosyncline were termed by Kober as— (A) Orogen (B) Kratogen (C) Primarumph (D) Zwischengebirge 4. Given below are two statements, one labelled as Assertion (A) and the other labelled as Reason (R). Select your answer from the codes given below— Assertion (A) : On Hawai Islands volcanic eruption are effusive type. Reason (R) : Less viscous lavas erupt quitly. Codes : (A) Both (A) and (R) are true and (R) is correct explanation of (A) (B) Both (A) and (R) are true, but (R) is not correct explanation of (A) (C) (A) is true, but (R) is false (D) (R) is true, but (A) is false 5. Match List-I with List-II and select the correct answer from the codes given ahead— List-I (River) (a) Mississippi (b) Ganga (c) Narmada (d) Old course of yellow

List-II (Types of Delta) 1. Arcuate 2. Birdfoot 3. Abandoned 4. Estuarine Codes : (a) (b) (c) (A) 1 2 4 (B) 2 1 3 (C) 2 1 4 (D) 4 2 1

(d) 3 4 3 3

6. Which of the following is not true about the Taiga Forests ? (A) The forest cover is dominated by coniferous trees (B) The forest floors are quite clear (C) Tree growth is very fast (D) The diameters of trees mostly range between 2 cm and 60 cm 7. Surface ocean currents derive their energy from— (A) Salinity (B) Tectonics (C) Wind (D) Tsunamis 8. Especially high and low tides that occur near the times of new and full moons are called— (A) Spring tide (B) Neap tide (C) Super tide (D) Fall tide 9. Given below are two statements, one labelled as Assertion (A) and the other labelled as Reason (R). Select your answer from the codes given ahead— Assertion (A) : In the Arctic ecosystem, if lichen production were either impaired or destroyed, the whole system would collapse because the herbivorous and carnivorous directly and indirectly depend upon lichens.

4 | UGC-NET Geography-II (J-13)

10.

11.

12.

13.

14.

Reason (R) : The Arctic ecosystem has a very complex trophic structure. Codes : (A) Both (A) and (R) are true and (R) is correct explanation of (A) (B) Both (A) and (R) are true, but (R) is not correct explanation of (A) (C) (A) is true, but (R) is false (D) (R) is true, but (A) is false Which one of the following is not true about the native animal life of the rainy tropics ? (A) An immense number and variety of insects (B) Few arboreal fauna (C) Few grazing and carnivorous animals (D) Abundant aquatic life Which atmospheric gas filters out most of the ultraviolet radiation in sunlight ? (A) Argon (B) Carbon dioxide (C) Ozone (D) Helium Which one of the following best describes surfaces air circulation in a Northern Hemisphere anticyclone ? (A) Divergent – Clockwise (B) Convergent – Counter clockwise (C) Stationary (D) Inward – Clockwise Which type of front has the steepest slope ? (A) Warm front (B) Cold front (C) Stationary front (D) Occluded front Which one of the following pairs is not correctly matched ? (A) Evaporation : The process of converting a liquid to a gas. (B) Condensation : The change of state from a gas to a liquid. (C) Element : A substance that cannot be decomposed into simpler substances by ordinary chemical or physical means. (D) Conduction : Tiny bits of particulate matter that serve as surfaces on which water vapour condenses.

15. Which one of the following used ‘Precipitation effectiveness’ and ‘Thermal efficiency’ as the basis of world’s climatic classification ? (A) Trewartha (B) Thornthwaite (C) Koppen (D) Kendrew 16. Which one of the following pairs is not correctly matched ? (A) Strabo : Published 17 volumes work entitled as geography (B) Ptolemy : Published a famous text called Almagest or Geographike Syntaxis (C) Hipparchus : Designed orthographic and stereographic projections (D) Hecataeus : Invented an instrument known as ‘Astrolabe’ for determination of latitudes and longitudes 17. Who among the following geographers laid down the foundation of dichotomy of General versus Special Geography ? (A) Bernhard Varenius (B) Immanuel Kant (C) Peter Apian (D) Sebastian Munster 18. The concept of Teleology was propounded by— (A) Alexander Von Humboldt (B) Carl Ritter (C) Oscar Peschel (D) Vidal de la Blasche 19. Who among the following geographers specifically emphasized on climatic determinism ? (A) Griffith Taylor (B) Friedrich Ratzel (C) Ellsworth Huntington (D) Ellen Churchill Semple 20. The concept of ‘Areal Differentiation’ is also termed as— (A) Exceptionalism (B) Chorological Study (C) Systematic Geography (D) Regionalism

UGC-NET Geography-II (J-13) | 5 21. Which one of the following is correct sequence of contributors in the development of Greek Geography ? (A) Anaximander, Herodotus, Hipparchus, Eratosthenes (B) Herodotus, Eratosthenes, Hipparchus, Anaximander (C) Anaximander, Eratosthenes, Herodotus, Hipparchus (D) Anaximander, Herodotus, Eratosthenes, Hipparchus 22. Match the following List-I with List-II and select the correct answer from the codes given below— List-I (Scholar) (a) Brian Berry (b) Carl Sauer (c) Dorreen Massey (d) O.H.K. Spate List-II (Area of Specialization) 1. Regional Geography 2. Radical Geography 3. Social Geography 4. Urban Geography Codes : (a) (b) (c) (d) (A) 4 3 2 1 (B) 1 2 4 3 (C) 3 4 1 2 (D) 2 1 3 4 23. Which one of the following pairs is not correctly matched ? Theory related Proponent with urban structure (A) Homer Hoyt : Sector theory (B) Sinclair : Concentric theory (C) Christaller : Central place theory (D) Harris and : Multiple Nuclei Ullman theory 24. Given below are two statement, one labelled as Assertion (A) and the other labelled as Reason (R). Select your answer from the codes given ahead— Assertion (A) : In the coreperiphery model, the core, region dominates whilst the periphery is dependent.

Reason (R) : This dependence is structured mainly through relations of exchange and production between core and periphery. Codes : (A) Both (A) and (R) are true and (R) is correct explanation of (A) (B) Both (A) and (R) are true, but (R) is not correct explanation of (A) (C) (A) is true, but (R) is false (D) (A) is false, but (R) is true 25. In the acceleration stage of urbanisation, the urban population constitutes. (A) more than 70% of the total population of the State (B) 25% to 70% (C) 10% to 25% (D) less than 10% 26. In developing countries, the population pressure increases in some parts due to immigration. Such parts are characterized by— (A) Low cost of living (B) More suitable climate (C) Large employment opportunities (D) Accessibility 27. Followings are the names given to estate farming in different parts of the world. Match the correct pairs— List-I List-II (a) Latifundia 1. Europe (b) Ranches 2. Anglo America (c) Fazendas 3. Latin America (d) Plantation 4. Asia Codes : (a) (b) (c) (d) (A) 1 3 4 2 (B) 2 3 4 1 (C) 1 2 3 4 (D) 4 3 1 2 28. The soils characterised by the dominant influence of climate are known as— (A) Zonal soils (B) Intrazonal soils (C) Interzonal soils (D) Azonal soils 29. Which one of the following is the most important bases of Whittlesey’s classification of agriculture of the world ? (A) Functional forms of agriculture (B) Distribution of climatic elements

6 | UGC-NET Geography-II (J-13) (C) Forms of livestock grazing activities (D) Regional distribution of the principal crops and animals of commercial significance 30. Who among the followings has used for the first time the ranking co-efficient method of agricultural productivity ? (A) L. D. Stamp (B) Mohammad Shafi (C) M. G. Kendall (D) G. Eneydi 31. Gibbs-Martin Index is used for the measurement of— (A) Crop specialisation (B) Crop diversification (C) Crop concentration (D) Crop combination 32. The most dominant blood group among the Mongoloid races is— (A) A (B) B (C) AB (D) O 33. Given below are two statements, one labelled as Assertion (A) and the other labelled as Reason (R). Assertion (A) : ‘Social indicator movement’ was quickly developed in 1960s in the United States of America (U.S.A.). Reason (R) : It grew in response to rise of a wide range of social problems in U.S. Society. In the context of the above two statements, which one of the following is correct ? (A) Both (A) and (R) are true, but (R) is not correct explanation of (A) (B) Both (A) and (R) are true and (R) is the correct explanation of (A) (C) (A) is true, but (R) is false (D) (A) is false, but (R) is true 34. Which one of the following sub races does not belong to the Caucasoid race ? (A) Eskimo (B) Celtic (C) Nordic (D) East Baltic 35. “Regions are good servants but bad masters” was stated by— (A) C.C. Carter (B) A.J. Herbertson (C) O.H.K. Spate (D) Vidal de la Blasche

36. The regions developed along transportation lines or irrigation channels are called as— (A) Transport regions (B) Axial regions (C) River-Valley regions (D) Transitional regions 37. Which one of the following statements is not correct ? (A) Regions are mental concepts formulated to arrange earth features in some abstract order (B) Regions could either be finite or infinite/ boundless three-dimensional extending much beyond the limits of terrestrial sphere (C) The regional concept is a device to comprehend likenesses and differences on the earth surface (D) A region has some kind of internal homogeneity which distinguishes it from the neighbouring areas 38. Which one of the following objectives of Regional Planning is most important ? (A) Cultivating habitability and community building (B) Conservation of forests (C) Rationalisation of population distribution (D) Perfect regional mobility system 39. Regionalism is the expression of— (A) Ignorance of other regions (B) Parochialism (C) Sectionalism (D) Local individuality, local loyalties and cultural traits 40. Which one of the following sequences of the steel-plants from East to West is correct in India ? (A) Jamshedpur, Rourkela, Durgapur, Bhilai (B) Bhilai, Rourkela, Durgapur, Jamshedpur (C) Durgapur, Jamshedpur, Rourkela, Bhilai (D) Durgapur, Rourkela, Bhilai, Jamshedpur 41. Match List-I with List-II and select the correct answer using the codes given ahead— List-I (Cities and Towns) (a) Madurai-Rameswaram (b) Varanasi-Kanyakumari

UGC-NET Geography-II (J-13) | 7 (c) Delhi-Kolkata (d) New Delhi-Amritsar List-II (NH Roads) 1. NH-1 2. 3. NH-7 4. Codes : (a) (b) (c) (A) 4 2 3 (B) 2 1 4 (C) 4 3 2 (D) 1 2 3

NH-2 NH-49 (d) 1 3 1 4

42. Which one of the following States recorded the lowest population density in India at 2011 Census ? (A) Sikkim (B) Arunachal Pradesh (C) Nagaland (D) Mizoram 43. The ‘Vale of Kashmir’ lies between— (A) Pir Panjal and Karakoram range (B) Pir Panjal and Zaskar range (C) Zaskar and Ladakh range (D) Shiwalik and Ladakh range 44. Which one of the following migration types contributes most to population movement in India ? (A) Rural to Rural (B) Urban to Rural (C) Rural to Urban (D) Urban to Urban 45. Which of the following tribes is known as true ‘jungle nomads’ ? (A) Pygmies (B) Semang (C) Masai (D) Kirghiz 46. The oldest religion of the Middle East is— (A) Islam (B) Judaism (C) Christianity (D) Zoroastrianism 47. Which one of the following methods is best suited to show the distribution of population with rural component and urban centre of verying sizes in a region ? (A) Uniform dot method (B) Multiple dot method (C) Uniform dot and circle method (D) Dot and sphere method

48. Match the following satellites with their altitudes and select the correct answer— List-I List-II [Satellites] [Altitudes (km)] (a) Land Sat V 1. 832 (b) IRS 2. 705 (c) SPOT 3. 904 (d) IKONOS 4. 681 Codes : (a) (b) (c) (d) (A) 2 4 1 3 (B) 4 1 2 3 (C) 4 2 3 1 (D) 2 3 1 4 49. Nearest Neighbour analysis was evolved by— (A) Henry and Raize (B) Clark and Evans (C) Zipf (D) Carl Pearson 50. Spatial resolution of LISS IV Sensor is— (A) 5·8 m (B) 2·0 m (C) 23·5 m (D) 10·0 m

Answers with Hints 1. (D) Horn, a German term is used for Pyramidal Peak a erosional feature formed by Glacier, while all the three are depositional features. 2. (D) Gunz are of the earliest of the Pleistocence glacials in Central Europe, Distinguished as the earliest phase of glacio-fluvial outwash to the North of Alps by A. Penck and E. Bruckner in 1909. It proceed to Mindel which is older the Riss and Wurm. 3. (B) Kober in his Geosynclinal Orogen theory termed the Geosyncline as ‘Orogen’ and the rigid land masses surrounding the Geosyncline as ‘Kratogen’ 4. (A) 5. (C) 6. (C) 7. (C) It is due to wind that surface ocean current gets its energy from the wind. 8. (A) Spring tide is a tide with a range considerable increased from that of the mean tidal level (low tides are lower and high tides are higher. It occurs twice each month at the time of new moon-(conjunction) and the full moon (opposition).

8 | UGC-NET Geography-II (J-13) 9. (C) 10. (C) 11. (C) It is the layer of O 3 in the stratosphere that filters out ultra-violet rays of the sunlight. 12. (A) In an anticyclonic area in Northern Hemisphere, the winds are clockwise. 13. (A) The warm front represents a marked thermal gradient in the atmosphere. 14. (D) Conduction is a process in which heat is transferred directly through matter from a point of high temperature to a point of low temperature by molecular impact. 15. (B) Thornthwaite’s method of world classification in climatic regions was based on P.E. (Potential Evapotranspiration and T.E. (Thermal efficiency). 16. (D) Hipparchus invented a instrument called astrolabe for measurement of latitude at sea by observing the angle of polar star not Hecataeus. 17. (A) The issue of General Geography Vs Regional geography was most probably raised by Bernhard Varen known as Varenius in 17th Century, this period is known as classical period of Modern Geographical thought. Varenius recognised the two main division geography General or Universal and Special or particular. 18. (B) Carl Ritter, the Contemporaries of Alexander Von Humboldt was a scholar of diversified interest. He was a dedicated field worker and believed in empirical research. He was a teleologist and had a strong belief in God. 19. (C) Ellsworth Huntington, a pupil of Davis was an environmental determinist who tried to explain the styles of life of human groups and nations in the light of Weather and Climatic Conditions. 20. (B) The term areal differentiation was coined and used by Hartshorne in his classic work the ‘Nature of Geography’ the areal differentiation is also known Chorology or chorrography which is the study of areal differentiation of the earth surface. 21. (D) Correct Sequence of contribution in the development of Greek Geography is

22. 23. 24. 28.

29.

30.

31. 32. 33. 34. 35. 40. 43.

44. 45.

46. 47.

48. 49. 50.

Anaximander—(611 to 547 BC) Herodotus (485-425 BC) Eratosthenes (276-194 BC) Hipparchus (140 BC). (A) (B) Concentric zone theory was given by E. Burgers in 1925 not by Sinclair. (A) 25. (B) 26. (C) 27. (C) (A) Zonal soils are those soils whose characteristics are dominated by the influence of climate and vegetation. (A) D. Whittlesey in 1936 had classified the world into agricultural regions as the bases of agricultural conditions, their pattern and regional disparity in the form of agriculture. (C) M.G. Kendall in 1940 used the ranking co-efficient method of agricultural productivity. For this he took into consideration productivity cost of 10 crops in 48 counties. (B) Gibbs-Martin Index is the index of diversification (B) Most important blood group among the Mongoloid is high ‘B’. (C) (A) Eskimos belong to Mongoloid race, while rest three belong to Caucasoid race. (A) 36. (B) 37. (B) 38. (D) 39. (D) (C) 41. (C) 42. (B) (B) Vale of Kashmir is in between Pir Panjal Range in the south and an offshoot of Zaskar Range in the North. (C) (D) Pygmies (Hunters), Semang (Forest product gather), Masai (Pastoral of Tropical grassland), Kirghiz Nanadic life leader of temp grassland (Khirgistan). (D) (C) In ‘Areal or Stilgenbauer’s method the rural population is shown by Uniform Dot and Urban population is shown by circle method. (D) (B) N.N.A. (Near Ness Neighbourhood Analysis) was developed by Clark and Evans. (A)

Geography (Paper–III)

UGC-NET/JRF Exam., Solved Paper

June 2013 Geography (Paper-III) Note : This paper contains seventy five (75) objective type questions of two (2) marks each. All questions are compulsory. 1. Which of the following is not considered to be characteristics feature of the youthful stage of an ideal normal cycle of erosion ? (A) Natural levees (B) Pot holes (C) Gorges (D) River capture 2. Sand dunes formed as long ridges oriented at right angles to the wind direction are called— (A) Linear dunes (B) Barchan dunes (C) Transverse dunes (D) Star dunes 3. Match List-I with List-II and select the correct answer from the codes given below— List-I (Scholar) (a) Harry Hass (b) Hall and Dava (c) Jeffreys (d) Hutton List-II (Theory) 1. Doctrine of Uniformitarianism 2. Thermal Contraction Theory 3. Theory of Geosyncline 4. Sea Floor Spreading Theory Codes : (a) (b) (c) (d) (A) 4 3 2 1 (B) 3 1 2 4 (C) 1 3 2 4 (D) 2 4 1 3 4. Who stated that the slope profiles are convex, plane or concave according to the circumstances of the uplifting action ? (A) Davis (B) Penck (C) Johnson (D) Wood 5. Given ahead are two statements, one labelled as Assertion (A) and the other labelled as

Reason (R). Select your answer from the codes given below— Assertion (A) : Hawaii Island is a site of volcanic eruptions. Reason (R) : Convergent plate margins are sites of volcanic eruptions. Codes : (A) Both (A) and (R) are correct, and (R) is the correct explanation of (A) (B) Both (A) and (R) correct, but (R) is not the correct explanation of (A) (C) (A) is correct, but (R) is false (D) (A) is false, but (R) is correct 6. Match the following List-I with List-II and select the correct answer from the codes given below— List-I (a) Karst cycle (b) Sheet flood theory of pediment formation (c) Concept of pediplain (d) Origin of natural bridges in Karst regions List-II 1. McGee 2. Cvijic 3. Woodword 4. Maxon and Anderson Codes : (a) (b) (c) (d) (A) 1 3 4 2 (B) 2 1 4 3 (C) 3 2 1 4 (D) 1 2 3 4 7. The statement ‘‘The present is the sky to the past’’ is related to— (A) Catastrophism (B) Uniformitarianism (C) Isostasy (D) Diastrophism

UGC-NET Geography-III (J-13) | 3 8. The theory of plate tectonics does not help to explain the origin and location of which one of the following ? (A) Earthquakes (B) Mountains (C) Ocean currents (D) Major sea floor features 9. Given below are two statements, one labelled as Assertion (A) and the other labelled as Reason (R). Select your answer from the codes given below— Assertion (A) : The boundary between two air masses of different density is known as a front. Reason (R) : The difference in density is caused by differences in temperature and humidity. Codes : (A) Both (A) and (R) are true and (R) is the correct explanation of (A) (B) Both (A) and (R) are true and (R) is not the correct explanation of (A) (C) (A) is fasle, but (R) is true (D) (A) is true, but (R) is false 10. Which of the following winds fall in the Zone of Hadley Cell ? (A) Monsoon winds (B) Trade winds (C) Westerlies (D) Polar winds 11. Which one of the following is correct, when the combined effect of the pressure gradient force and coriolis force on air current produces ? (A) Deflection force (B) Geostrophic force (C) Frictional force (D) Gravitational force 12. The term adiabatic change of temperature stands for— (A) Fall of temperature after precipitation (B) Heating and cooling of an ascending or descending air through compression of expansion (C) Heating of ascending air (D) Cooling of descending air 13. What are the effects of the low level temperature inversion occurring in cool nights over the urban areas ?

(1) Entrapment of pollutants causing heavily smog. (2) Pleasant weather conditions. (3) Unusually unstable air. (4) Unusually stable air. Select your answer using the codes given below— Codes : (B) (2) and (3) (A) (1) and (3) (C) (1) and (4) (D) (2) and (4) 14. The ‘Ice Crystal Theory’ related to precipitation was propounded by— (A) Boven (B) Muir (C) Peterson (D) Bergeron 15. Which one of the following parts of the Indian coast is a compound coast showing evidence of both submergence and emergence ? (A) Orissa (Odisha) coast (B) Malabar coast (C) Konkan coast (D) Coromandel coast 16. Darwin’s subsidence theory is related to— (A) Marine terraces (B) Coral reefs (C) Cycle of erosion (D) Tides 17. Fiord shorelines are formed due to— (A) Submergence of subaerially eroded coast (B) Submergence of glaciated coast (C) Fluvial deposition (D) Marine erosion 18. Tsunamis are produced by— (A) Tides (B) Cyclones (C) Sub-marine earthquakes (D) Shrinking of the earth’s crust 19. Which one of the following sediment deposits cover the largest percentage of the ocean floor ? (A) Terrigenous (B) Cosmogenous (C) Biogenous (D) Hydrogenous 20. The term Ecology was first used by— (A) Tinsley (B) Taylor (C) De Martonne (D) Jeans Brunhes 21. Match List-I with List-II and select the correct answer from the given codes—

4 | UGC-NET Geography-III (J-13) List-I (a) Bhuj Earthquake (b) RIO-World Summit (c) World Environment Day (d) Sandy Storm List-II 1. 2002 2. 5th June 3. 2012 4. 2001 Codes : (a) (b) (c) (d) (A) 4 1 2 3 (B) 1 4 3 2 (C) 2 1 4 3 (D) 3 2 1 4 22. In which of the following years the Kyoto Protocol was signed ? (A) 1987 (B) 1997 (C) 1990 (D) 1985

List-II (Contributions) 1. Laws of primate city 2. Cycle of erosion 3. Geography as human ecology 4. Morphology of landscape Codes : (a) (b) (c) (d) (A) 2 3 4 1 (B) 4 3 2 1 (C) 4 3 1 2 (D) 3 4 1 2 25. ‘An Introduction to the Application of Geography to History’ and ‘The Geographical Distribution of Mankind’ are the works of— (A) Vidal de la Blache (B) Carl Ritter (C) Alfred Hettner (D) Friedrich Ratzel

23. Given below are two statements, one labelled as Assertion (A) and the other labelled as Reason (R). Assertion (A) : Behavioural Geography is now becoming much closer to a constructivists or transactionalist epistemology, as propounded by neo-Kantian philosophers and psychologists. Reason (R) : The Geographers like White, Kirk, Gold and Kates have made signal contributions for behavioural movement in Geography. Select your answer from the codes given below— Codes : (A) (A) is correct, but (R) is wrong (B) (A) is wrong, but (R) is correct (C) Both (A) and (R) are correct and (R) is the correct explanation of (A) (D) Both (A) and (R) are correct, but (R) is not the correct explanation of (A)

26. First quarter of 20th century was to be considered as the golden age of German geography. Among the following, who were the great leaders of this period ? (1) A. Penck (2) H. Lautensach (3) C. Ritter (4) A. Hettner

24. Match List-I with List-II and select the correct answer from the codes given ahead— List-I (American Geographers) (a) Carl Sauer (b) H. H. Barrows (c) W. M. Davis (d) M. Jafferson

28. Which of the following statements are correct relating to Ratzel’s concept of ‘Lebensraum’? (1) It is the geographical area within which living organisms develop. (2) It is the economic and cultural activity of peoples outside their enclosed settled area.

(A) (B) (C) (D)

(1) and (3) are correct (3) and (4)) are correct (1) and (4)) are correct (1), (2) and (3) are correct

27. Who one of the following determined the place of Geography in the classification of sciences ? (A) Beruhard Varenius (B) Philip Cluverius (C) Immanuel Kant (D) Sebastian Munster

UGC-NET Geography-III (J-13) | 5 (3) It deals with the relations between human society as a spatial organization and its physical setting. (4) It speaks ‘geopolitik’ and studies space from the view point of State. (A) (1), (2) and (3) are correct (B) (2) and (3) are correct (C) (2), (3) and (4) are correct (D) (1) and (3) are correct 29. Given below are two statements, one labelled as Assertion (A) and the other labelled as Reason (R). Assertion (A) : Humanistic Geography rejects positivism and quantification in geography. Reason (R) : Humanistic Geography emphasizes the role of the human agency and its creativity in the construction of place, space and landscape. Select your answer from the codes given below— Codes : (A) Both (A) and (R) are true and (R) is the correct explanation of (A) (B) Both (A) and (R) true, but (R) is not the correct explanation of (A) (C) (A) is true, but (R) is false (D) (A) is false, but (R) is true 30. Which one of the following is correct chronological sequence of Arab geographers in the development of geography ? (A) Al-Biruni, Al-Idrisi, Ibn Battuta, Ibn Khaldun (B) Al-Idrisi, Ibn-Khaldun, Al-Masudi, AlBiruni (C) Ibn-Khaldun, Al-Masudi, Al-Biruni, AlIdrisi (D) Al-Masudi, Ibn-Battuta, Ibn-Khaldun, Al-Biruni 31. In India which of the following criteria applied in defining a town has varied from census to census in recent decades ? (A) Density of population (B) Total population (C) Administrative status (D) Working population

32. Match List-I with List-II and select the correct answer from the codes given below— List-I (Theory) (a) Theory of Spatial Integration (b) Theory of Innovation Diffusion (c) Dependency Theory (d) Theory of Circular and Cumulative Causation List-II (Authors) 1. G. Myrdal 2. A. G. Frank 3. John Friedman 4. T. Hagerstrand Codes : (a) (b) (c) (d) (A) 2 1 3 4 (B) 1 2 4 3 (C) 3 4 2 1 (D) 4 3 1 2 33. ‘Informational city’ is a term coined by— (A) David Harvey (B) Richard Peet (C) Edward Soja (D) Manuel Castells 34. Which one of the following models is based on the study of human ecology ? (A) Hagerstrand’s Innovation Diffusion Model (B) Weber’s Location Model (C) Zelinsky’s Mobility Transition Model (D) Burgess and Park’s Concentric Zone Model 35. ‘Time-space compression’ is a spatial concept given by— (A) Dorreen Massey (B) David Harvey (C) Daniel Bell (D) Edward Soja 36. Given below are two statements, one labelled as Assertion (A) and the other labelled as Reason (R). Select your answer from the codes given ahead— Assertion (A) : Regional disparities tend to reduce over time in a competitive free market economy. Reason (R) : The factors of production move from one region to another to maximize their rate of returns.

6 | UGC-NET Geography-III (J-13)

38. Factorial ecology is a method generally used to analyse— (A) Urban socio-spatial structures (B) Physiographic regions (C) Agro-climatic regions (D) Combination of (B) and (C) 39. To which of the following, ‘nomadism’ is the response of man ? (A) Large resources spread over large areas (B) Large resources localised in a small area (C) Limited resources spread over a large area (D) Limited resources concentrated in a small area 40. According to Von Thunen model, land rent and distance relationship of 5 crops i.e. ‘a’, ‘b’, ‘c’, ‘d’ and ‘e’ is shown in the diagram given below. Which one of the following pairs of crops substitute crop ‘c’ ? 60

Locational Rent

50 40 30

a b

20

c d

10 0

1

2 3 4 Distance in km

5

e 6

(B) d-e (D) b-d

41. On the basis of the given map of an area, the best suitable place for locating a big iron and steel plant would be the site marked as—

Ri

v

C er

nn

Soft Coal b ad

ha

el

ad

Ro

Iron ore c

Navigable river

37. When higher income groups re-occupy and revive older housing in attractive inner city areas, the process is called— (A) Filtering (B) Gentrification (C) Housing re-development (D) Residential upgradation

(A) a-d (C) a-b

Ro

Codes : (A) Both (A) and (R) are true and (R) is the correct explanation of (A) (B) Both (A) and (R) true, but (R) is not the correct explanation (C) (A) is true, but (R) is false (D) (A) is false, but (R) is true

Coking Coal d

a Limestone Sea Port

(A) a (B) d (C) b (D) c 42. Consider the following statements and select the correct answer using the codes given below— The ships on the North Atlantic sea route are forced to the South of the true great circle route by— (1) Fog associated with the contact of the cold Labrador current and the warm Gulf stream current. (2) Icebergs brought southward by the Labrador current at certain seasons of the year. (3) The strong polar icy winds from the North. (4) The projecting landmass of Northeastern North America and the presence of warm Gulf stream current. Codes : (A) (1) and (2) are correct (B) (1), (2) and (3) are correct (C) (1) and (3) are correct (D) All are correct 43. Which one is not the advantage of pipeline transport ?

UGC-NET Geography-III (J-13) | 7 (A) Pipelines can be laid through difficult terrain as well as under water (B) Pipelines ensure steady supply (C) Pipeline operation involves very low consumption of energy (D) It is very easy to detect leakages and in the event of damage, pipelines can be repaired easily and quickly 44. Industries that manufacture individual components for the final assembly of product are known as— (A) Subsidiary industries (B) Small scale industries (C) Ancillary industries (D) Cottage industries 45. In the transport network shown in the figure given below, which one of the locations has the maximum accessibility ? d

Codes : (A) Both (A) and (R) are true and (R) is the correct explanation of (A) (B) Both (A) and (R) true, but (R) is not the correct explanation of (A) (C) (A) is true, but (R) is false (D) (A) is false, but (R) is true 47. Given below are two statements, one labelled as Assertion (A) and the other labelled as Reason (R). Assertion (A) : There are wide variations in evolution of languages and species in the world. Reason (R) : Geographic isolation plays an important role in developing several forms of languages and species. Codes : (A) Both (A) and (R) are true and (R) is the correct explanation of (A) (B) Both (A) and (R) are true, but (R) is not the correct explanation of (A) (C) (A) is true, but (R) is false (D) (A) is false, but (R) is true

Transport Network (B) b (D) d

48. Given below are two statements, one labelled as Assertion (A) and the other labelled as Reason (R). Assertion (A) : In spatio-temporal diffusion of the culture, the language plays a major role. Reason (R) : Geographers often use language as an identification mark for different cultures. Codes : (A) Both (A) and (R) are true and (R) is the correct explanation of (A) (B) Both (A) and (R) are true, but (R) is not the correct explanation of (A) (C) (A) is true, but (R) is false (D) (A) is false, but (R) is true

46. Given below are two statements, one labelled as Assertion (A) and the other labelled as Reason (R). Assertion (A) : Tribal religion, where natural objects are deemed as spirits and demons, is often characterised by animinism. Reason (R) : Simplistic beliefs and practices often find a close association with ecological niche.

49. Given below are two statements. One labelled as Assertion (A) and the other labelled as Reason (R). Select your answer from the codes given ahead— Assertion (A) : Biologically human races do not exist, yet races do exist as social constructs. Reason (R) : Racial hatred and prejudices are all too common in the world today, resulting

a

b c

(A) a (C) c

8 | UGC-NET Geography-III (J-13)

50.

51.

52.

53.

from an irrational dislike of whatever is different. Codes : (A) Both (A) and (R) are true and (R) is the correct explanation of (A) (B) Both (A) and (R) are true, but (R) is not the correct explanation of (A) (C) (A) is true, but (R) is false (D) (A) is false, but (R) is true Given below are two statements, one labelled as Assertion (A) and the other labelled as Reason (R). Assertion (A) : India is a multireligious and multiethnic society. Reason (R) : The Constitution of India offers to all citizens, individually and collectively, fundamental rights without discrimination on grounds of religion, race, caste, sex or place of birth. Codes : (A) Both (A) and (R) are true and (R) is the correct explanation of (A) (B) Both (A) and (R) are true, but (R) is not the correct explanation of (A) (C) (A) is true, but (R) is false (D) (A) is false, but (R) is true The statement that “A race is a valid biological concept ……… not a valid socio-cultural concept” was made by— (A) A. L. Kroeber (B) J. B. Birdsell (C) Vidal de la Blache (D) A. C. Haddon Geography of public finance can simply be defined as “Who gets what, were, at what cost ?” Name the geographer who made this statement— (A) R. J. Chorley (B) Richard Peet (C) Alaistar Bonnett (D) R. J. Bennett International aid helps poor countries due to resource transfers from rich countries but— 1. Divides world into donor and recipient countries. 2. Makes donor countries more powerful in International trade and power relations. 3. Forces recipient countries to carry out such economic reforms that might leave people worse-off. 4. Compels recipient countries to buy donor nation goods.

(A) (B) (C) (D)

1 and 4 are correct 1, 2 and 3 are correct 3 and 4 are correct All are correct

54. Match List-I with List-II and select the correct answer using the codes given below— List-I (a) Long-term planning (b) Public sector planning (c) Private sector planning (d) Town planning List-II 1. Indicative planning 2. Spatial planning 3. Imperative planning 4. Perspective planning Codes : (a) (b) (c) (d) (A) 3 4 2 1 (B) 2 4 1 3 (C) 2 4 3 1 (D) 4 3 2 1 55. Given below are two statements, one labelled as Assertion (A) and the other labelled as Reason (R). Assertion (A) : Regions, in fact, do change over time; they change in response to changing interrelationship of the spatial elements. Reason (R) : Planned intervention certainly proves to be of rapid momentum within the frame of socio-economic activities. Select your answer from the codes given below— Codes : (A) Both (A) and (R) are correct and (R) is the correct explanation of (A) (B) Both (A) and (R) are correct, but (R) is not the correct explanation of (A) (C) (A) is correct, but (R) is wrong (D) (A) is wrong, but (R) is true 56. Which technique was adopted by B. J. L. Berry in his classic study on functional regionalization of Indian economy ? (A) Composite Index Method (B) Cluster Analysis

UGC-NET Geography-III (J-13) | 9 (C) Regression Analysis (D) Multiple Factor Analysis 57. Which of the following are not the major objectives of the Hill Area Development Programme ? (1) Agro-forestry (2) Large scale industries (3) Eco-preservation (4) Major irrigation projects (A) (1) and (2) are correct (B) (2) and (3) are correct (C) (2) and (4) are correct (D) (3) and (4) are correct 58. Who among the following applied the growth pole concept to geographic space, whereby growth poles came to be known as growth centres ? (A) Friedmann (B) Perroux (C) Frank (D) Boudeville 59. In which Five Year Plan, balanced regional development was emphasized as one of the major objectives of the planning exercise and was one full chapter in the plan document ? (A) Second Plan (B) Third Plan (C) Fourth Plan (D) Fifth Plan 60. In ancient geography, the concept of region was first postulated by whom ? (A) Homer (B) Herodotus (C) Strabo (D) Ptolemy 61. The city region is an example of— (A) Formal region (B) Functional region (C) Compage region (D) Adhoc region 62. Given below are two statements, one labelled as Assertion (A) and the other labelled as Reason (R). Assertion (A) : The task of preparing a reasonably sound district plan has not made much headway in the States. Reason (R) : There is lack of effective coordination at the district level between various agencies involved in planning exercises. In the context of the above two statements, which one of the following is correct ? Codes : (A) (A) is correct, but (R) is wrong (B) (A) is wrong, but (R) is correct

(C) Both (A) and (R) are correct and (R) is the correct explanation of (A) (D) Both (A) and (R) are correct, but (R) is not the correct explanation of (A) 63. Which one of the following islands of India has volcanic origin ? (A) Car-Nicobar (B) Barren (C) North-Andaman (D) Little-Nicobar 64. Which of the following hills does not join the Anaimudi knot ? (A) Elamalai hills (B) Palni hills (C) Anaimalai hills (D) Kudremukh hills 65. Among the following which one is related to Blue Revolution in India ? (A) Horticulture (B) Floriculture (C) Pisciculture (D) Sericulture 66. Which one of the following groups of iron and steel plants is located outside the Chhota Nagpur region ? (A) Bhilai, Bokaro, Rourkela (B) Jamshedpur, Bokaro, Durgapur (C) Bhilai, Salem, Bhadravati (D) Salem, Bhadravati, Jamshedpur 1 is enlarged by two-and2‚500 a-half times, what will be the scale of the enlarged map ? 1 1 (A) (B) 6‚250 1‚000 1 1 (C) (D) 10‚000 12‚500

67. A map of R.F.

68. Population density is usually shown by— (A) Isopleth method (B) Choropleth method (C) Chorochromatic method (D) Decimetric method 69. What is the pattern of settlement distribution of its Rn = 2·14 ? (A) Perfect cluster (B) Partially cluster (C) Random (D) perfect uniform 70. Ajai ranks seventeenth in a class of thirty one. What is his rank from the last ? (A) 13 (B) 14 (C) 15 (D) 16

10 | UGC-NET Geography-III (J-13) 71. Given below are two statements, one labelled as Assertion (A) and the other labelled as Reason (R). Assertion (A) : Passive remote sensing systems have their own source of energy. Reason (R) : Active remote sensing systems depend on solar energy. In the context of the above two statements, which one of the following is correct ? Codes : (A) (A) is correct, but (R) is wrong (B) Both (A) and (R) are correct, but (R) is not the correct explanation of (A) (C) Both (A) and (R) are corect, and (R) is the correct explanation of (A) (D) Both (A) and (R) are wrong 72. The satellite image shown in televison clearly depicts— (A) Temperature and rainfall (B) Pressure and wind (C) Clouds and snow cover (D) Humidity and fog 73. Match List-I with List-II and select the correct answer using the codes given below— List-I (Satellite) (a) Landsat V (b) IRS (c) SPOT (d) IKONOS List-II (Altitude in km) 1. 832 2. 705 3. 904 4. 681 Codes : (a) (b) (c) (d) (A) 2 4 1 3 (B) 2 3 1 4 (C) 4 2 3 1 (D) 4 1 3 2 74. Which one of the following characteristics separates GIS from the other systems ? (A) Map making and generalized output (B) Linking spatial and non-spatial data (C) Map designing and lay-out (D) All of the above 75. Cephalic index, which is to classify human races, is calculated as—

(A) Ratio between the length and breadth of the nose (B) Ratio between the length of head and weight of human body (C) Ratio between flatness and length of the head (D) Ratio between breadth and length of the head

Answers with Explanations 1. (A) Natural levee is an elevated bank flanking the channel of a river and standing above the level of the flood plain. It is naturally formed by the river in times of flood when the over bank flow causes a decease in streams velocity. It is naturally formed hence is called Natural levee. Rest the three land forms are associated with youthful stage (Eroding capacity) of the river. 2. (C) A asymmetrical sand dune which stands at right angle to the direction of wind blow. 3. (A) 4. (C) 5. (C) Hawaii island is a site of Extinct volcano. But convergent plate margins are the margins of Trenches or Rift valleys rather than Valcano. 6. (B) 7. (B) The concept of uniformitarianism was propounded by James Huttan in 1785. On the bases of this he propounded that present is the key to past. The Geomorphic forces which are active in present were also active in past. 8. (C) Earthquakes, Mid oceanic ridges and sea floor spreading can be studied by platetectonic theory, but ocean currents can not be studied with plate tectonic. 9. (A) Front is a term introduced by Bjerkness school in Norway (1918) to describe a sloping boundary plane or surface, separating two airmasses that exhibits different meterosgical properties i.e. and temperature. Density is determined by the temperature. High density low temperature low density high temperature. 10. (B) Hadley Cell is a part of the Global atmosphere circulation pattern based on an idea suggested in 1686 and later modified and developed by G. Hadley in his explanation of trade winds in 1735.

UGC-NET Geography-III (J-13) | 11 11. (B) Geostrophic is a term used to describe the horizontal flow of air (wind) in its theoretical movement parallel to the Isobars, thus creating a Geostrophic wind. The wind is a resultant of two opposing forces the → Pressure Gradient or pressure force in one direction and the deflecting force → coriolis force in the other direction. 12. (B) Adiabatic change refers to the change in temperature which occurs in a mass of Gas (air) when it is compressed (heated) and expanded (cooled), without the aid of any external source of cooling and heating. The rate at which this occars is called the lapse rate. 13. (C) 14. (D) 15. (D) Beside the emergence coastal area of the continental sheef is submerged in the sea and the deposition takes place. Hence a coastal plain is developed. The coromandel coast is developed due to this type of submergence and emergence. 16. (B) Darwin’s theory of subsidence is related with coral reef. 17. (B) Fiord/Fyord is a long, narrow inlet of the sea bounded by steep mountain slope which are of great height and extend to considerable depth below the sea level. It is formed by the submergence of glacially overdeepened valley due to raising level after the melting of Pleistocenoc iceage. 18. (C) 19. (A) 20. (*) The term ecology was first coined by German scientist Ernst Haeckel in 1866. However, the term ecosystem was coined in 1935 by the British ecologist Sir Arthur George Tinsley. 21. (A) 22. (B) A convection was held in Kyoto (Equador) in 1997 in which a pact was signed by the developed as well as developing countries to restrict the emission of Green-house gases. 23. (D) 24. (B) 25. (B) Ritters monumental work is entitled as Erdkunde in a comprehensive german word which stands for science of the Earth in relation to nature and history. History and Geography must always remain inseparable.

26.

27.

28. 29.

30.

31. 33. 34.

35. 36.

Through his writings, Ritter tried to prove that Earth is made for man, the body is made for soul. So, the physical globe made for mankind. (C) A. Penck was a peading German geographer of the early part of 20th century, who formulated the concept of geomorphology. While Alfred Hettner (1859–1941) was essentially a physical and regional Geographer. (C) Kant was of the view that there are two different ways of grouping or classifying empirical phenomena. The phenomena may be classified either in accordance with nature or in accordance with there position in time and space. The former is a logical and later is a physical ore. The physical classification gives the scientific basis for History and Geography. Geography studies phenomena which lie side by side in space (chronological science) History and Geography both are essential sciences. (C) (A) Humanism in Geography developed as a criticism against positivism and quantitative revolution in Geography. The basic object of humanistic against quantative revolution is that it tools and assumptions do not adequately explain human world and human issues. But the followers of humanistic view consider Geography as the study of the earth as the home of man.’’ (A) The correct chronological sequence of Arab Geographer is Al-Biruni (973–1039 AD), Al-Idrisi (1099–1180 AD), Ibn-Battuta (1304–1368 AD), Ibn Khaldun (1332–1406 AD). (D) 32. (C) (D) The term informational city was coined by Manuel Castells for Urban Reality. (D) Concentric zone model also known as the Burgess Model is based on human ecology. During 1990s Robert E. Park and Ernest W. Burgess developed distinct programme of urban research. (B) It was David Harvey in 1989, who coined the term Time-Space compression. (A) 37. (C)

12 | UGC-NET Geography-III (J-13) 38. (A) Most studies in factorial ecology used units on the lowest system level that study of Urban socio-spatial structure. 39. (D) Nomadic people are wander periodically according to season, usually live in a small band in a vicinity moving with in a define territory. 40. (D) 41. (D) Limestone, Coking coal, Iron and Water are needed for Iron-Steel industry. Except Iron-ore all the three things are available at Point ‘d’. Iron ore can be transported through navigable river and finished goods will be exported from part (a). Hence, ‘d’ is the best suited place for Iron-Steel industry. 42. (B) 43. (C) 44. (A) 45. (A) ‘a’ Location has the maximum accessibility in the network figure. Because in this figure from all the direction roads meet at ‘a’ point instead of ‘b’ ‘c’ and ‘d’. 46. (A) 47. (C) 48. (C) 49. (D) 50. (B) 51. (A) A. Kroeber has defined human race as a valid biological concept. It is a group united by heredity, a breed or genetic strain or sub species. It is not a valid Socio-cultural concept. 52. (D) 53. (D) 54. (*) None of the answer is correct. The correct code is— (a) Long-term Planning (b) Public Sector Planning (c) Private Sector Planning (d) Town Planning

55. 56.

57. 58.

(4) Perspective Planning (3) Imperative Planning (1) Indicative Planning (2) Spatial Planning (B) (D) B. J. L. Berry in his classic study of functional regionalization of Indian economy adopted the Multiple factor analysis technique. (C) (B) Growth-Pole concept in geographic space was adopted by Perroux.

59. (A) In second five year plan, regional development was emphasized. In third five year plan the main emphasized was on self. Sustaining growth in fourth for the development of reducing fluctuations in agricultural production which in fifth. The back drop of severe inflationary pressures, to attain Self reliance. 60. (C) Strabo gave little attention to topographic features, mountains, rivers, their courses while giving geographical account of regions. While Ptolemy (AD 90–168), came later. Homer was a great poet, while Herodotus was a great historian. 61. (B) 62. (C) 63. (B) Barren island in Andaman and Nicobar island is a island of volcanic origin. 64. (D) 65. (C) 66. (C) Bhilai (Chhattisgarh), Salem (Tamilnadu) and Bhadravati (Karnataka) are not the located steel plant at Chotanagpur Region. 67. (A) 68. (B) Choropleth method is the best suitable method for showing the population diversity of a region. 69. (D) 2·00 and above perfect uniform. 0·91 to 1·0—Randim. 0·31 to 0·60—Partially cluster 0·01 to 0·30—Perfect cluster. 70. (C) 71. (D) Passive remote system does not have their own source of energy and absorb the incoming radiation. While in Active remote system, reflects the specific radiation towards the object and the return absorb it but not absorb directly. Hence, both (A) and (R) are wrong. 72. (C) The satellite image shown in Weather Map, depicted by Television clearly indicates clouds and snow covers not temperature and rainfall, pressure and wind humidity and fog. 73. (B) 74. (D) 75. (D) Cephalic index Breadth of the head × 100 = Length of the head

Geography UGC-NET/JRF Re-Exam. Solved Paper

September 2013

Geography (Paper-II) Note : This paper contains fifty (50) objective type questions of two (2) marks each. All questions are compulsory. 1. The premise that present-day processes have been operating throughout geological time is the principle of— (A) Continental drift (B) Uniformitarianism (C) Earth Systems (D) Plate Tectonics 2. The concept of Panplain is associated with— (A) W. M. Davis (B) W. Penk (C) A. Strahler (D) E. Huntington 3. Match the List-I with List-II and select the correct answer by using the codes given below— List-I List-II (a) River 1. Dolines (b) Glacier 2. Inselburg (c) Wind 3. Moraaines (d) Underground water 4. Leaves Codes : (a) (b) (c) (d) (A) 4 3 2 1 (B) 1 2 4 3 (C) 2 3 1 4 (D) 4 1 2 3 4. Identify the order of the stream at the section marked ‘X’—

(A) 2 (C) 4

(B) 3 (D) 5

5. Match List-I with List-II and select the correct answer from the codes given below— List-I (Land form) (a) Monadnocks (b) Blind valley (c) Drumlin (d) Dreikanter List-II (Agenta of Denudation) 1. Wind 2. Clacier 3. River 4. Ground Water Codes : (a) (b) (c) (d) (A) 2 4 1 3 (B) 4 1 2 3 (C) 1 3 2 4 (D) 3 4 2 1 6. Which of the following pairs of countries receive maximum insolation ? (A) Indonesia and Sri Lanka (B) Indonesia and Libya (C) Yemen and Somalia (D) Canada and Australia 7. Solid material passes directly into vapour state by means of— (A) Condensation (B) Sublimation (C) Volatalization (D) Convention 8. Which of the following pairs is correctly matched ? 1. Cwa and hot summers, dry winters. 2. Cwb and hot dry summers.

4 | UGC-NET Geography-II (S-13) 3. Csb and warm dry summers. 4. Cfb and warm summers Select the correct answer— (A) 1, 2, 3 and 4 (B) 1, 3, and 4 (C) 1 and 4 (D) 1 and 2 9. Match the List-I with List-II and select the correct answer using the codes given below— List-I (Local Winds) (a) Loo (b) Samum (c) Harmattan (d) Bora List-II (Countries) 1. West Africa 2. Former Yugoslavia 3. Pakistan 4. Iran Codes : (a) (b) (c) (d) (A) 2 3 4 1 (B) 3 4 1 2 (C) 4 1 2 3 (D) 1 2 3 4 10. Radiolarian Ooze are— (A) Terrigenous Deposits (B) Neritic Deposits (C) Pelagic Deposits (D) Littoral Deposits 11. Certain substances are classified as water pollutants, others as air pollutants and still others as land pollutants. Which one of the following pollutes are three parts of our environment ? (A) Flouracarbons (B) Smog (C) Acid rain (D) Ozone 12. Ria is an example of— (A) Emerged upland shore (B) Submerged upland shore (C) Neutral shore (D) Compound shore 13. Flat-topped sub-marine mountains are called— (A) Sea mounts (B) Abyssal mounts (C) Guyots (D) Sea Volcanoes

14. Anacondas are largely found in— (A) Irrawady Basin (B) Congo Basin (C) Orinoco Basin (D) Amazon Basin 15. Match the List-I with List-II and select the correct answer from the codes given below— List-I (Natural Vegetation) (a) Conifers (b) Evergreen (c) Xerophytes (d) Deciduous List-II (Indian States/UTs) 1. Jharkhand 2. Rajasthan 3. Andaman and Nicobar 4. Jammu and Kashmir Codes : (a) (b) (c) (d) (A) 3 4 1 2 (B) 4 3 2 1 (C) 2 1 3 4 (D) 1 2 4 3 16. Who among the following did not support the concept of environmental determinism ? (A) Ratzel (B) Davis (C) Huntington (D) None of these 17. Match the following and find the correct answer from the codes given below— List-I (a) Probabilism (b) Stop-and-go determinism (c) Systematic Geography (d) Regional Geography List-II 1. Von Humboldt 2. Carl Ritter 3. Griffith Taylor 4. O.H.K. Spate Codes : (a) (b) (c) (d) (A) 4 3 1 2 (B) 1 2 3 4 (C) 3 2 4 1 (D) 4 3 2 1

UGC-NET Geography-II (S-13) | 5 18. Who wrote the book ‘Airs, Water and Places’ ? (A) Anaximander (B) Hippocrates (C) Eratosthones (D) Hecateus 19. The Humanistic Geography aims at ‘Verstehn’, that is— (A) Life style of man (B) Inter personal relations of men (C) Understanding of man in his environment (D) Objective experiences of man 20. Geographia in 17 volumes was written by— (A) Ptolemy (B) Miletus (C) Strabo (D) Ritter 21. Given below are two statements, one labelled as Assertion (A) and the other labelled as Reason (R). Select your answer from the codes given below— Assertion (A) : The newly emerged industrial cities in India are numerically dominated by both male and female adult population. Reason (R) : Human migration is age and sex selective. Codes : (A) Both (A) and (R) are correct, and (R) is correct explanation of (A) (B) Both (A) and (R) are correct, but (R) is not the correct explanation of (A) (C) (A) is correct, but (R) is false (D) (A) is false, but (R) is correct 22. Which one of the following is not correctly matched ? (A) Exurbanisation — Commuter belt grows at the expense of core (B) Counterurbanisation — Population loss of ring is more than the urban core (C) Reurbanisation — Core starts regaining population (D) Exurbanisation — Labour intensive and export oriented industrialization is the base 23. International migration is no longer a factor in population distribution mainly because— (A) People do not wish to migrate (B) People are afraid of racial prejudices

(C) Nations have placed restrictions on migration (D) Employment opportunites are not bright 24. Match the following— List-I (Name of Scholar) (a) Charles Darwin (b) Wilbur Zelinsky (c) Thomas Malthus (d) George Ravenstein List-II (Idea/Concept) 1. Migration laws 2. Population and resources 3. Theory of evolution 4. Hypothesis of the mobility Codes : (a) (b) (c) (d) (A) 1 2 3 4 (B) 2 3 4 1 (C) 3 4 2 1 (D) 4 3 2 1 25. Who among the following used the term Ecumenopolis to describe a projected urbanised world or universal city by the end of the twenty first century ? (A) Geddes (B) Gottmann (C) Doxiades (D) Dickinson 26. Match the items in the List-I with that in List-II. The items in the two lists relate to sector theory proposed by Hoyt— List-I (Sector Number) (a) 1 (b) 2 (c) 3 (d) 4 List-II (Urban Land use Patten) 1. Wholesale & Light Manufacturing 2. CBD 3. Medium-class Residential 4. Low-class Residential Codes : (a) (b) (c) (d) (A) 1 2 3 4 (B) 2 3 4 1 (C) 2 1 4 3 (D) 4 3 2 1

6 | UGC-NET Geography-II (S-13) 27. What is generally not true of shifting cultivation ? (A) Rotation of field (B) Absence of draught animals (C) More often than not the homestead are not shifted (D) It is a great catalytic force for community life 28. Which one of the following States is the largest producer of Tin ? (A) Rajasthan (B) Odisha (C) Jharkhand (D) Chhattisgarh

35. Which one of the following rock systems is the main sources of coal in India ? (A) Gondawana (B) Cuddappa (C) Vindhyan (D) Dharwar

29. The classical mode of Industrial location theory in which least cost approach is primary consideration is recognised as the industrial location theory of— (A) Weber (B) Smith (C) Isard (D) Hoover

37. Which of the following States in India has the largest number of sugar mills ? (A) Gujarat (B) Maharashtra (C) Bihar (D) Uttar Pradesh

30. Kiel Canal connects— (A) Berring Sea and the Pacific Ocean (B) Red Sea and the Mediterranean Sea (C) Baltic Sea and North Sea (D) Caribbean Sea and the Pacific Ocean 31. Special Economic Zone (SEZ) Policy of India was introduced in— (A) 1999 (B) 2000 (C) 2001 (D) 2002 32. In the contemporary context, which one of the following countries is most significant from geo-strategic view point ? (A) Maldives (B) Sri Lanka (C) Afghanistan (D) Bangladesh 33. ‘Carrot and Stick’ technique is used as a regional development policy in— (A) Sweden (B) Netherlands (C) Germany (D) Great Britain 34. The core of the Great Himalayn is made up of— (A) Dharwar Rocks (B) Archaean Rocks (C) Quaternary Rocks (D) Gondwana Rocks

36. Which one of the following sets of commodities is imported by India from South West Asian Countries ? (A) Raw Wool and Carpets (B) Dates and Olives (C) Precious Stones and Pearls (D) Perfumes and Coffee

38. Match the following— List-I (Dialect) (a) Awadhi (b) Bundeli (c) Maithili (d) Bagri Codes : (a) (b) (c) (A) 1 2 3 (B) 2 3 4 (C) 3 4 1 (D) 4 3 2

List-II (State) 1. Haryana 2. Bihar 3. Madhya Pradesh 4. Uttar Pradesh (d) 4 1 2 1

39. The concept of Lebensraum became popular in the discipline of political geography after the publication of— (A) Politische geographic (B) Geographical pivot of history (C) Political geography : world economy, nation-state and locality (D) Making political geography 40. Pred criticized location theory on following grounds— (A) Logical inconsistency, motives and capacity (B) Motives and capacity (C) Logical inconsistency and capacity (D) Logical inconsistency and motives

UGC-NET Geography-II (S-13) | 7 41. Match the following lists— List-I (a) Cumulative Causation Model (b) Spatial Diffusion (c) Behavioural Matrix (d) Stages of Economic growth List-II 1. Rostow 2. Myrdal 3. Hagerstrand 4 Pred Codes : (a) (b) (c) (d) (A) 2 3 4 1 (B) 1 2 4 3 (C) 3 1 2 4 (D) 4 3 2 1 42. Who coined the term Geopolitics ? (A) Kjellen (B) Haushoffer (C) Ratzel (D) Hess 43. Who among the following is credited with incorporation of concept of geographical space in the Growth Pole Theory ? (A) Bouldville (B) Friedmann (C) Myrdal (D) Haggerstand 44. Arrange the following maps in decreasing order of scale— 1. Atlas map 2. Wall map 3. Cadastral map 4. Topographical map Select the answer from code— (A) 2, 3, 4, 1 (B) 1, 2, 3, 4 (C) 4, 3, 1, 2 (D) 3, 4, 2, 1 45. Area of a drainage basin on a map with R. F. 1 : 50,000 is measured as 200 sq. cm. What will be the basin area of map when it is reduced at the R. F. 1 : 100, 000 ? (A) 1000 sq. cm (B) 100 sq. cm (C) 50 sq. cm (D) 500 sq. cm 46. On a topographic map, lines that indicate water-depth are termed as— (A) Isogonic line (B) Isobath (C) Isopachs (D) Isoneph

47. In order to identify Qutub Minar on an arial photograph, which of the following elements of image interpretation would be more expressive ? (A) Shape (B) Shadow (C) Tone (D) Texture 48. Match List-I with List-II and select the correct answer by using the codes the given below— List-I (Standard Colour) (a) Red (b) Dark Green (c) Brown (d) Yellow List-II (Landuse feature) 1. Cultivated area 2. Built-up land 3. Forest area 4. Uncultivable land Codes : (a) (b) (c) (d) (A) 2 1 4 3 (B) 4 3 2 1 (C) 2 1 3 4 (D) 4 1 2 3 49. Whose portrayals epitomized the relationship of political geography to foreign policy ? (A) Isaiah Bowman (B) Friedrich Ratzel (C) Karl Haushofer (D) Adolf Hitler 50. Who for the first time emphasized the importance of mental map of environment in decision-making process ? (A) Woolridge (B) Downs (C) Gould (D) Lynch

Answers with Explanations 1. (B) The same physical processes and laws that operate through out geologic time although not necessary always with the same intensity as now is the basic concept of modern Geology, known as uniformitarianism, was profounded by Scottish Geomorphologist James Huttan in 1782. 2. (A) W. M. Davis in his cycle of erosion has said that during the 3rd stage of erosion A Panplain (An almost flat land surface created

8 | UGC-NET Geography-II (S-13)

3. 4. 5. 6.

7.

8. 10.

11.

12. 13.

14.

15.

by the coalescence of flood plains owing to the lateral erosion by river) is developed (A) (A) It is a dendritic type of drainage in which X stands for second order stream. (D) (C) Yemen and Somalia (Arab and Africa) being a deserted area get more insolation (Solar heat) that the other countries either surrounded by sea (Indonesia + Sri Lanka) and Canada and Australia (Countries situated away from the equator in Northern and Southern Hemisphere). (B) Sublimation is process where by a solid is converted into a vapour or vice-versa. While in condensation vapour is changed into liquid. In convection—the transference of the constituent particles into a gas or liquid owing to difference in temp. (B) 9. (B) (C) Radiolarian Ooze is a type of deep Sea Siliceous — Pelagic deposits are Deep Sea deposits unaffected by land derived material and derived mainly from the marine pelagic organism. While Terrigenous deposits are organic deposits that have either been formed and laid down on a land surface. Littoral and Neritic zone are in between shallow and deep water zone. (C) Acid rain a colloquial terms used to describe precipitation in a polluted environment. Where rain drops become contaminated by either Sulphur Oxides—Caustic gases, or Sulphur dioxide and Nitrozen oxide is a polluted material in all the three farms, i.e., Land, Atmosphere, Water. (B) Ria is a Submerged coastal valley or estuary resulting from a rise of Sea level. (C) Guyot a flat topped variety of — Sea mount occurring mainly in Pacific ocean, conical shape of Guyot is known as volcanic origin. A Sea mount is a isolated mount an Abyssal mount is a plain mount. (D) Anaconda — a semi-aquatic constructing serpent found in tropical South America cost of Andes, it is also known as the great water boa, is the largest of the snakes in the Western Hemisphere. (B)

16. (D) Ratzel, Huntington and Davis all three are the supporter of environmental determinism. 17. (D) The Concept of Neo-determinism was put forward by Griffith Taylor — He coined the term stop and go determinism to deserve his views requesting the Neo-determinism. The world probabilism was used by O.H.K. Spate, Carl Ritter was the founder of in Regional Geography while Von-Humboldt laid the foundation of systematic Geography. 18. (B) 19. (C) Humanism does not treat humans as machines. It is a subjective approach which aims at ‘Verstehn’ as to understand of man in his environment. 20. (C) Strabo wrote as many as 43 volumes under the title Historical memoir, more-over he wrote 17 volumes of Geographical treatise. 21. (D) 22. (D) 23. (C) 24. (C) 25. (C) 26. (C) 27. (C) In shifting cultivation, the primitive people after growing crops at a place, migrate to other place which is suitable for cultivation alongwith their bag or baggages. 28. (D) As per Annual Report (2012-13) ministry of mines Govt. of India Chhattisgarh is the sole, producer of Tin in India. The state accounts for about 36% of tin ore. 29. (A) Weber, a German economists in 1909 published his theory of Industrial Location, in which he emphasized Least Cost Location approach for raw material as well as finished good. 30. (C) 31. (B) SEZ policy in India was announced in 2000, but Parliament passed the Bill in 2005. 32. (C) Afghanistan is a buffar State (Geostrategic importance). 33. (D) 34. (B) Core of Great Himalayan is composed of Granite Gneiss and Schists (Igneous Rocks). 35. (A) 98% of the coal resources of India are found in the rocks of Gondawana period (i.e. Damodar Valley area, Godavari area, Mahanadi area and Wardha area etc.). 36. (B) India imports Dates (Arab countries) and Olive (from Mediterranean countries) 37. (D) 38. (D)

UGC-NET Geography-II (S-13) | 9 39. (A) Influenced by Darwin, a very important concept namely Lebensraum (living space) was coined by Ratzel in his book on Political Geography. In which Ratzel equated a nation with a living organism. 40. (A)

41. (A)

42. (A) Johan Rudolf Kjellen (1864–1922) was a Swedish Political Scientist and Politician who first coined the term ‘geopolitics’. 43. (A) 44. (D) Correct sequence will be Cadastral map (large scale map), Topographical maps from 1/50,000 to 1/1000000—than Wall maps and in the end Atlas Map (small scale map). 45. (B)

46. (B) Isobath → Line joining the point on sea bed, situated at an equal vertical distance beneath the surface. Isoneph → A line on a map connecting places of equal cloudness. Isogonic → Line joining points of equal magnetic declination. Isopachs—Line joining the points of equal thickness of Geological Stratum. 47. (B) 48. (A) 49. (B) Ratzel’s Philosophy of Lebensraum ‘living space’ which created the controversy of superior and interior races by claiming that superior people (nations) have a right to expand their territory (Kingdom)—loving space at the expense of interior nations. 50. (B)

September 2013

Geography (Paper-III) Directions—This paper contains seventy five (75) objective type questions of two (2) marks each. All questions are compulsory. 1. The Drift Theory of Wegener was postulated mainly to explain— (A) Ice Age (B) Ditribution of landforms (C) Major climatic changes (D) Geological similarities of coastal regions 2. Which of the following groups is produced by erosion ? (A) Drumlins, Fjords, Arete (B) Eskern, Out-wash plains, Moraines (C) Cirque, V-shaped valley, Lavees (D) Playa, Pincles, Swallow holes 3. As per the theory of platetectonics which of the following is a super continent ? (A) Eurasia (B) Africa (C) Antarctica (D) Australia 4. The direction of a horizontal line on an inclined rock strata is a— (A) Dip (B) Strike (C) Syncline (D) Anticline 5. Sunda peninsula was submerged in the Pacific Ocean due to— (A) Constructive activities of the plates (B) Destructive activities of the plates (C) Subduction of the plates into ocean (D) Sea floor spreading 6. Match the List–I with List–II and select the correct answer by using codes given below— List–I (a) Endogenetic force (b) Geosyncline

(c) Weathering (d) Continental Drift List–II 1. Exfoliation 2. Mediterranean sea 3. Laurasia 4. Orogenesis Codes : (a) (b) (c) (A) 3 1 2 (B) 2 4 3 (C) 4 2 1 (D) 1 3 4

(d) 4 1 3 2

7. Match the List–I with List–II and select the correct answer by using codes given below— List–I (a) Palaenteological Evidence (b) Biological Evidence (c) Jig-saw-Fit Evidence (d) Land Bridges List–II 1. Coast lines of S. America and Africa 2. Glossopteris 3. Lystrosaurs fossils 4. Marsupials Codes : (a) (b) (c) (d) (A) 1 2 3 4 (B) 4 3 2 1 (C) 3 2 1 4 (D) 2 1 4 3 8. The tectonic plates on which India and the adjacent portion of Asia ride are colliding at a rate of— (A) 0·1–1 millimetre per year

UGC-NET Geography-III (S-13) | 11 (B) 1–10 millimetre per year (C) 0·1–1 centimetre per year (D) 1–10 centimetre per year 9. Who propounded the theory of Rural Settlement ? (A) Ritter (B) Ratzel (C) Meitzen (D) Schluter 10. Who postulated the concept of plate tectonics? (A) Le Pichon (B) W. J. Morgan (C) Harry Hen (D) Tuzo Wilson 11. The following Question consists of two statements. One is labelled as Assertion (A) and the other labelled as (R). Examine the two statements carefully and select the correct answer using the codes given— Assertion (A) : There is close relationship between Volcanoes and plate margins. Reason (R) : About 80 per cent of the World’s Volcanoes are associated with the convergent plate boundaries. Codes : (A) Both (A) and (R) are true and (R) is the correct explanation of (A) (B) Both (A) and (R) are true and (R) is not the correct explanation of (A) (C) (A) is true, but (R) is false (D) (A) is false, but (R) is true 12. Knick points are associated with— (A) River Rejuvenation (B) Glacial Erosion (C) Aeolian Deposits (D) Karst topography 13. When the Kaveri river as soon as it enters Tamil Nadu, which waterfall does it create ? (A) Hoggenakka fall (B) Jog fall (C) Five falls (D) Paghaya Kotlam 14. Penisulas make up most of— (A) Italy and Denmark (B) Poland and France (C) Ireland and Great Britain (D) Austria and the Czech Republic 15. Tornadoes are most likely to occur during— (A) Mid-morning (B) Mid-afternoon (C) Noon (D) Late evening

16. Which US city was struck by the hurricane Katrina ? (A) New York (B) Houston (C) New Orleans (D) Miami (FL) 17. Considering the Assertion (A) and Reasoning (R) select the right answer from the codes given below— Assertion (A) : When the Sun is overhead on a clear sky, one can see blue sky in all directions. Reason (R) : The longer wavelengths of visible light are scattered more effectively than short wavelength. Codes : (A) Both (A) and (R) are correct and (R) explains (A) (B) Both (A) and (R) are correct, but (R) does not explain (A) (C) (A) is correct, but (R) is false (D) (A) is false, but (R) is true 18. Ozone hole is loacted above the— (A) Antarctica (B) Arctic Ocean (C) Equatorial zone (D) Greenland 19. A sudden, violent, gusty wind, which lasts a minute or two and then subsides, usually accompanied by rain or hail, is called— (A) Thunderstorm (B) Hoar (C) Blizzard (D) Squall 20. Considering the Assertion (A) and Reasoning (R) select the right answer from the codes given below— Assertion (A) : The thunderstorms of Bengal and the adjoining Bihar, Odisha and Assam are known as Northwesters. Reason (R) : The squalls come predominantly from north-east. Codes : (A) Both (A) and (R) are correct and (R) explains (A) (B) Both (A) and (R) are correct but (R) does not explain (A) (C) (A) is correct, but (R) is false (D) (A) is false, but (R) is true

12 | UGC-NET Geography-III (S-13) 21. The following question has two set of statements. One labelled as Assertion (A) and the other labelled as Reason (R). Choose your answer from the codes given below— Assertion (A) : The troposphere is the dense lower part of he atmosphere in which all the weather phenomenon like condensation and storms occur. Reason (R) : In troposphere the atmospheric temperature decreases with altitude. Codes : (A) Both (A) and (R) are true and (R) is the correct explanation of (A) (B) Both (A) and (R) are true, but (R) is not the correct explanation of (A) (C) (A) is true, but (R) is false (D) (A) is false, but (R) is true 22. The following question consists of two statements. One labelled as Assertion (A) and the other labelled as (R). Examine the two statements carefully and select the correct answer using the codes given below— Assertion (A) : Jet streams play important role in intensifying weather phenomena such as cyclones, hurricanes and typhoons. Reason (R) : There are severe storms when jet streams interfere with surface wind systems. Codes : (A) Both (A) and (R) are true and (R) is the correct explanation of (A) (B) Both (A) and (R) are true, but (R) is not the correct explanation of (A) (C) (A) is true, but (R) is false (D) (A) is false, but (R) is true 23. Which of the following oceans is spreading due to plate tectonic movement ? (A) Atlantic Ocean (B) Pacific Ocean (C) Indian Ocean (D) Arctic Ocean 24. Match List–I with List–II and choose the correct answer using the codes given below— List–I (Ocean Currents) (a) Oyasiwo (b) Peru (c) Alaska (d) Kurosiwo List–II (Coasts) 1. Western coast of Latin America 2. Kamchatka Peninsula

3. British Columbia 4. Japan Codes : (a) (b) (A) 3 1 (B) 1 2 (C) 4 1 (D) 2 1

(c) 2 3 3 3

(d) 4 4 2 4

25. A reef separated from the main land or inland shore by a deep lagoon is known as— (A) Fringing Reef (B) Atoll Reef (C) Barrier Reef (D) Patch Reef 26. Global change in sea level is caused by— (A) Eustatic change (B) Isostatic change (C) Climatic change (D) Diastrophism 27. The following question has two sets of statements, one labelled as Assertion (A) and the other labelled as (R). Choose your answer from the codes given below— Assertion (A) : Western coast of Europe (UK) and Labrador of (North America) are in the same latitude but Western Coast of Europe (UK) remains unfrozen and Labrador remains frozen for nine months. Reason (R) : Gulf stream (UK) is a warm current and Labrador is a cold current. Codes : (A) Both (A) and (R) are true and (R) is not the correct explanation of (A) (B) Both (A) and (R) are true and (R) is the correct explanation of (A) (C) (A) is true, but (R) is false (D) (A) is false, but (R) is true 28. Which of the following is not true about welfare geography ? (A) It studies the question of who gets what, where and how (B) It is much concerned about spatial distribution of poverty, resources and civic amenities (C) It was part of the reaction against positivism in geography (D) It seeks to change the present economic and political set up

UGC-NET Geography-III (S-13) | 13 29. Who of the following can be considered as an opponent of quantitative revolution ? (A) Stamp (B) Chorley (C) Haggelt (D) Hagerstrand 30. Identify the wrong pair— (A) A. J. Herbert- – Major Natural Regions son (B) H. Robert Mill – Realm of Nature (C) Peter Hagget – Locational Analysis in Human Geography (D) L. D. Stamp – Frontiers in Geographical Teaching 31. The concept of Paradigm was propounded by— (A) Thomas Kuhn (B) E. Soja (C) P. Hall (D) Lewis Mumford 32. Match the following and answer from the codes— List–I (a) A Hundred Years of Geography (b) Geography in the Twentieth Century (c) Man and Nature (d) Influences of geographic Environment List–II 1. Griffith Taylor 2. T. W. Freeman 3. E. C. Semple 4. G. P. Marsh Codes : (a) (b) (c) (d) (A) 1 2 3 4 (B) 3 4 2 1 (C) 4 2 3 1 (D) 2 1 4 3 33. The concept of sustainable development rests on the following principles— (A) Intergeneration transferability (B) Social justice (C) Transfrontier responsibility (D) Intergeneration transferability, social justice, transfrontier responsibility 34. Which of the following is not a thematic map? (A) Map showing population density (B) Map showing distribution of minerals (C) Map showing jute producing areas (D) Topographical map on 1 : 50,000

35. Which one of the correct match ? (A) Geddes (B) Perry (C) Howard (D) Le Corbusier

following pairs is not a – – – –

Le Play School Neighbourhood units Garden city Satellite city

36. The plot of cities of developing countries according to their size and rank on a normal graph paper will assume one of the following shapes. (A) Linear (B) Concave (C) Convex (D) Curvilinear 37. Settlements located at a distance from river banks and other water bodies are called— (A) Wet point settlements (B) Dry point settlements (C) Strong point settlements (D) Nucleated settlements 38. Match the List–I with List–II and select the correct answer using the codes given below— List–I List–II (Value of ‘Rn’ (Pattern of statistic) Distribution) (a) 0·00 – 0·25 1. Random (b) 0·90 – 1·10 2. Highly clustered (c) 1·25 – 1·50 3. Least dispersed (d) 1·95 – 2·15 4. Uniform Codes : (a) (b) (c) (d) (A) 4 2 1 3 (B) 2 1 3 4 (C) 1 2 3 4 (D) 2 4 1 3 39. Which one of the following distances does not fit into Christaller’s scheme of k = 3 hierarchy of central places ? (A) 7 (B) 11 (C) 21 (D) 36 40. Match List–I with List–II and select the correct answer from the codes— List–I List–II (a) Ranchos 1. India (b) Favelas 2. Brazil (c) Kavettits 3. Venezuela (d) Bustee 4. Myanmar

14 | UGC-NET Geography-III (S-13) Codes : (a) (b) (c) (d) (A) 2 3 1 4 (B) 3 2 4 1 (C) 1 2 3 4 (D) 4 2 3 1 41. Given below are the two statements, one labelled as Assertion (A) and other labelled as Reason (R). Select your answer from the codes given below— Assertion (A) : In India, large cities are growing at a very fast rate. Reason (R) : Inspite of the emphasis on their planned development, the small and medium towns have failed to come up to expected norms. Codes : (A) Both (A) and (R) are correct and (R) is the correct explanation of (A) (B) Both (A) and (R) are correct, but (R) is not the correct explanation of (A) (C) (A) is correct, but (R) is false (D) (A) is false, but (R) is correct 42. Given below are the two statements, one labelled as Assertion (A) and the other labelled as Reason (R). Select your answer from the codes given below— Assertion (A) : Annul growth rate of population in developed nations is much lower than in developing nations. Reason (R) : Economic development results in reduction in birth rate. Codes : (A) Both (A) and (R) are correct and (R) explains (A) (B) Both (A) and (R) are correct, but (R) does not explain (A) (C) (A) is true, but (R) is false (D) (A) is false, but (R) is true 43. Which one of the following statements is not correct about migration ? (A) Migration varies inversely with the distnace between source and destination (B) The majority of migrants move in order to improve their economic conditions (C) Governments create migration policies to attract the talents that they lack (D) Women tend to move longer distances than men

44. Match the List–I with List–II and select the correct answer using the categories below— List–I (a) Post neonatal mortality age (b) CMR age (c) Neonatal mortality age (d) IMR age List–II 1. Less than 28 days 2. Less than one year 3. 28 days – 364 days 4. Less than five years Codes : (a) (b) (c) (d) (A) 3 4 2 1 (B) 4 3 1 2 (C) 3 4 1 2 (D) 1 2 3 4 45. Given below are two statements, one labelled as Assertion (A) and the other labelled as Reason (R). Select your answer from the codes given below— Assertion (A) : The distribution of population is highly uneven in the world. Reason (R) : The environmental conditions favourable for human settlement differ from place-to-place. Codes : (A) Both (A) and (R) are corect and (R) is the correct explanation of (A) (B) Both (A) and (R) are correct, but (R) does not explain (A) correctly (C) (A) is true, but (R) is false (D) (A) is false, but (R) is true 46. Which of the following sets of countries does not use coal as essential energy for industries ? (A) Poland and Switzerland (B) Switzerland and Holland (C) Sweden and Italy (D) None of these 47. The greatest share of fisheries comes from— (A) Deep seas (B) Continental shelves (C) Deep ocean trenches (D) Fresh waters

UGC-NET Geography-III (S-13) | 15 48. Which one of the following major sea ports of India does not have natural harbour ? (A) Marmagao (B) Mumbai (C) Cochin (D) Paradeep 49. Match List–I with List–II and select the correct answer using the codes given below— List–I (Cities) (a) Bengaluru (b) Pithampur (c) Mangalore (d) Aurangabad List–II (Industries) 1. Machine tool 2. Aircraft 3. Locomotives and Coaches 4. Automobiles Codes : (a) (b) (c) (d) (A) 2 4 1 3 (B) 2 4 3 1 (C) 4 2 3 1 (D) 4 2 1 3 50. Match the List–I with List–II and select the correct answer using the codes given below— List–I (a) Weber (b) Von Thunen (c) Christaffer (d) Weaver List–II 1. Crop combination 2. Agricultural location 3. Central place theory 4. Theory of Industrial location Codes : (a) (b) (c) (d) (A) 2 4 1 3 (B) 4 2 3 1 (C) 1 2 3 4 (D) 2 3 4 1 51. Given below are the two statements, one labelled as Assertion (A) and other labelled as Reason (R). Select your answer from the codes given below— Assertion (A) : The pattern of consumption of goods and services varies from one country to another. Reason (R) : The population distribution varies from one country to another.

Codes : (A) Both (A) and (R) are correct and (R) explains (A) (B) Both (A) and (R) are corect, but (R) does not explain (A) (C) (A) is true, but (R) is false (D) (A) is false, but (R) is true 52. According to Lynch, the elements which help to perceive urban environment are— (A) Path and node (B) Path, node and landmark (C) Boundaries and districts (D) Path, boundaries, districts, nodes and landmark 53. Which one of the following theories is based on the concept of economic rent ? (A) Agricultural land use (B) Rank-size rule (C) Innovation diffusion (D) Urban land use 54. Green Revolution is a technological package, consistency of— 1. High yielding vareity of seeds 2. Land Reforms 3. Assured Irrigation facility 4. Use of chemical fertilizers Codes : (A) 1, 2 and 3 are correct (B) 2, 3 and 4 are correct (C) 1, 3 and 4 are correct (D) Only 3 is correct 55. Match List–I with List–II and select the correct answer from the codes given below— List–I (Town) List–II (Industry) (a) Kalpakkam 1. Newsprint (b) Jharia 2. Oil Refining (c) Mathura 3. Coal mining (d) Nepanagar 4. Nuclear plant Codes : (a) (b) (c) (d) (A) 4 2 1 3 (B) 4 3 2 1 (C) 1 2 3 4 (D) 3 4 1 2

16 | UGC-NET Geography-III (S-13) 56. Given below are two statements, one labelled as Assertion (A) and other labelled as Reason (R). Select your answer from the codes given below— Assertion (A) : Most transportation networks begin as branching networks. Reason (R) : Because the first priority of the public or private agencies that build them is generally to get all points connected. Codes : (A) Both (A) and (R) are correct and (R) is the correct explanation of (A) (B) Both (A) and (R) are correct, but (R) is not the correct explanation of (A) (C) (A) is true, but (R) is false (D) (A) is false, but (R) is true 57. Which one of the followings is not a general purpose authority ? (A) A District (B) World Health Organisation (WHO) (C) A Department (D) A County 58. Match the List–I with List–II identifying the relation. Select the correct answer from the codes given below— List–I (a) Austric Family (b) Dravidian Family (c) Sino-Tibetan Family (d) Indo-Aryan Family List–II 1. Santhal 2. Bodo Prakrit 3. Karukh Oraon 4. Bodo Codes : (a) (b) (c) (d) (A) 1 2 3 4 (B) 1 3 4 2 (C) 1 4 3 2 (D) 1 4 2 3 59. Who defined urban geogaphy as the study of the city as a system within a system of cities ? (A) H. C. Carter (B) Michael Pacione (C) B. J. L. Berry (D) A. E. Smails

60. Given below are the two statements, one labelled as Assertion (A), and the other labelled as Reason (R). Select the correct answer from the codes given below— Assertion (A) : Regional development planning solves the problem of regional disparities created by sectoral planning. Reason (R) : Sectoral planning concentrates on sectoral growth ignoring regional balance and regional development planning takes care of the growth and development of the delineated regions. Codes : (A) Both (A) and (R) are true and (R) is the correct explanation of (A) (B) Both (A) and (R) are true and but (R) is not the correct explanation of (A) (C) (A) is true, but (R) is false (D) (A) is false, but (R) is true 61. Who gave the theory of Rimland, giving importance to sea powers due to their ability to make a rim around ‘Heartland’, in the book ‘The Geography of Peace’ ? (A) Spykman (B) Truman (C) Haushoffer (D) Mahan 62. Mackinder’s ‘Pivot Area’, which he envisaged to be strategically very important, was— (A) Arabia and its surroundings (B) Central Africa (C) Land between Volga in the west and eastern Siberia in the east. (D) The central regions of ‘new world’ i.e., America’s 63. Read both Assertion (A) and Reasonign (R) and find the correct answer from the codes given below— Assertion (A) : Planning regions generaly tally with administrative divisions. Reason (R) : Plan execution is convenient in uniform administrative regions. Codes : (A) Both (A) and (R) are correct, but (R) does not explain (A) (B) Both (A) and (R) are correct (C) (A) is correct, but (R) is incorrect (D) (A) is incorrect, but (R) is correct

UGC-NET Geography-III (S-13) | 17 64. Bhakra Nangal Project is a joint venture of which one of the following groups of States ? (A) Punjab, Haryana and Rajasthan (B) Haryana, Punjab and Uttar Pradesh (C) Himachal Pradesh, Uttarakhand and Punjab (D) Punjab, Rajasthan and Uttarakhand 65. The ranges of Wester Ghats is locally called as ‘Sahyadris’ in which of the following State? (A) Tamil Nadu (B) Karnataka (C) Kerala (D) Maharashtra 66. Given below are two statements, one labelled as Assertion (A) and the other labelled as Reason (R). Selecte your answer from the codes given below— Assertion (A) : One of the major sources of irrigation in Peninsular India is Tank Irrigation. Reason (R) : Most of the rivers in Peninsular region are seasonal. Codes : (A) Both (A) and (R) are correct and (R) explains (A) (B) Both (A) and (R) are correct, but (R) does not explain (A) (C) (A) is correct, but (R) is false (D) (A) is false, but (R) is true 67. Arrange the following events playing a vital role in evolution of town and country planning in India in a sequential order of their taking place. Use the codes given below— 1. Appointment of Bhore Committee. 2. Enactment of Sanitation Act. 3. Establishment of Improvement Trusts in important towns. 4. Formation of Town and Country Planning Oganisation (TCPO). Codes : (A) 1, 2, 3, 4 (B) 2, 3, 1, 4 (C) 3, 1, 2, 4 (D) 4, 3, 2, 1 68. Which one of the following number of million sheets representing the world is correct ? (A) 54 (B) 136 (C) 152 (D) 504

69. Match List–I with List-II and select the correct answer by using the codes given below— List–I List–II (Topo map (Contour interval scale) in feet) (a) Million 1. 50 (b) One inch 2. 500 (c) Quarter inch 3. 100 (d) Half inch 4. 250 Codes : (a) (b) (c) (d) (A) 2 4 3 1 (B) 4 1 2 3 (C) 2 1 4 3 (D) 1 3 4 2 70. Match List–I with List–II and select the correct answer by using the codes given below— List–I (a) Planimeter (b) Pantograph (c) Barometer (d) Elinometer List–II 1. Area measure of a map 2. Elevation of land surface 3. Map reducteion and enlargement 4. Air pressure of a place Codes : (a) (b) (c) (d) (A) 2 3 4 1 (B) 1 4 2 3 (C) 2 4 1 3 (D) 1 3 4 2 71. The Air Photo Scale of a flat terrain depends upon— 1. Height of the aircraft 2. Focal length of aerial camera Choose the correct answer from the codes given below— Codes : (A) 1 is true, but 2 is false (B) 2 is true, but 1 is false (C) Both 1 and 2 are true (D) Both 1 and 2 are false

18 | UGC-NET Geography-III (S-13) 72. Consider the following statements and choose the correct answer from the codes given below— 1. The contours which represent a convex slope are close together at the foot and are comparatively farther apart at the top. 2. The closeness of contours indicate steepness of slope. Codes : (A) Only 1 is correct (B) Only 2 is correct (C) Both 1 and 2 are correct (D) Neither 1 nor 2 are correct 73. Arrange the following maps in decreasing order of scale and select the correct answer from the codes given below— 1. Atlas map 2. Cadastral map 3. Topographical map 4. Wall maps Codes : (A) 2, 3, 4, 1 (B) 1, 4, 3, 2 (C) 1, 2, 3, 4 (D) 4, 3, 2, 1 74. Analysis of variance (f-ratio test) can only be applied to data measured on— (A) Interval scale (B) Ratio scale (C) Ordinal scale (D) Nominal scale 75. Consider the following statements and choose the correct answer from the codes given below— 1. In a dot map, the value of the dots generally varies. 2. A dot map is usually used to show areal distribution of a phenomena such as rainfall, agricultural production, population, etc. Codes : (A) Only 1 is correct (B) Only 2 is correct (C) Both 1 and 2 are correct (D) Neither 1 nor 2 are correct

Answers with Explanations 1. (C) Concept of Continental Drift theory was initiated in 1858 by A. Snider but developed and popularized by F. B. Taylor (1908) and A. Wegener (1915) which suggests that :

2.

3. 4.

5. 6.

7.

8. 10.

11.

12.

13.

(i) Continents were stable climate changed (ii) Climatic was static, but Continents changed. His emphasis was more on climatic changes. (D) Playa (a level area occupying the centre of an enclosed basin), Pincles (in limestone region, after Lapies in open surface, when water enters, small ridges pincles are formed Swallow hole (in limestone regions) are Geomorphic features associated with erosion. (A) (B) A dip is an angle in a bedding plane which it makes with a horizonal plane, measured in a direction perpendicular to the strike of the rock strata, while strike is a direction taken by a horizontal line on the plane of inclined rock strata. Anticline a arched upfold, syncline a down fold or basin shaped. (C) (C) Correct match is— Endogenetic force—Orogenesis Geosyncline—Mediterranean sea Weathering—Exfoliation Continental drift—Laurasia (C) All are related with Continental drift theory of Wegnear. Palaenteological Evidence —Lystrosaurs fossils Biological Evidence—Glossopteris Jig-saw-Fit Evidence—Coast lines of East S. America and Western Africa Land Bridges—Marsupials—Both coast of (Atlantic ocean) (B) 9. (C) (B) It was Tuzo Wilson who first used the word Plate but the concept of Plate-tectonics was postulated by W. J. Morgan. (A) There is a close relation in volcanic as well as Plate Margins (constructive margin), most of the volcanic are found near conservative Margins or destructive plate margin where two plates convergent infront. (A) Knick point an alternative expression for rejuvenated head, refers to the break of slope in the long profile of a stream which results from a fall of base level (Rejuvenation). (A) Hoggenakka fall is at river Kaveri when it (River) enters to Tamil Nadu.

UGC-NET Geography-III (S-13) | 19 14. (A) Most of the Penisulas are in Italy and Denmark (A Penisula is an area surrounded three or more side by water and is connected to land. 15. (B) Tornado generally occur mid afternoon. 16. (C) 17. (C) It is wrong to say that long wavelength of visible light is effective than the short wavelength visible light. But it is vice-versa i.e. short wavelength (Blue) of visible light is more effective causing to look sky blue during daylight and long wavelength are less effective (Red). 18. (A) Ozone hole is located at Antarctica. 19. (D) Squall → A sudden increase of wind speed for a duration of at least one minute. 20. (A) Norwester are the winds blowing which originate with the collision of Loo (Hot wind) with moist air of causing squall as well as heavy rain. In Bengal they are known as Norwester. 21. (C) In atmosphere’s troposphere, the air temperature decreases rather than atmospheric temperature. 22. (A) Jet stream is a narrow belt of high altitude (about 12,000 m) westerly wind in troposphere, speed varies from 100 km to 184 km in summer and winter. These streams are found in sub tropics (30°–20° N) area, where tropical cyclones originate. These are also associated with Polar-front of mid latitude. 23. (A) With the evidence of Palaeomagnetism and sea floor spreading the opening and closing of ocean basins have been proved for example pacific ocean is narrowing because of American plate (Subduction) on contrary Atlantic ocean is spreading for last 200 million year. 24. (D) 25. (C) Barrier reef is an elongated accumulation of coral lying at low tide level parallel to the coast but separated from it by a wide and deep lagoon or strait. 26. (A) Eustatic change is a change in sea level that occurs everywhere through out the World, not to the movement of land (Isostatic movement), but to the actual fall and rise of the ocean itself.

27. (B)

28. (D)

29. (A) L. D. stamp vehemently opposed the quantative revolution and preferred to term quantative revolution a ‘Civil War’. 30. (D) Frontiers in Geographical Teaching was Written by R. J. Choley and Peter Haggett, not by L. D. Stamp. 31. (A) In ordr to elucidate the process of development of Science, Kuhn prepared a model which he termed as the Paradigm of Science. He defines Paradigm as universally recognised scientific achievements that for a time provide model problems and solution to the community of practitioners. 32. (D) 33. (D) The concept of sustainable rests in intergeneration transferability, social justice and transfrontier responsibility rather than (A), (B) and (C). 34. (D) A thematic map is a map designed to demonstrate particular features or concepts and are opposed to Topographical map. 35. (D) Le Corbusier—was a Swiss born, French–trained architect and a founder of international style. We also planned Chandigarh city. 36. (B) Concave means curving in or hollowed inward, as opposed to convex. 37. (B) Settlements located away from the river basin are known as dry point settlements. 38. (B) 39. (B) 11 is not in the Christaller’s scheme of central place k = 3 hierarchy. 40. (B)

41. (B)

42. (A)

43. (D) It is incorrect to say that women tend to more longer distance from men but it is viceversa. 44. (C) 45. (B) It is correct that the distribution of population is uneven in the world and it is also correct that environmental conditions favourable for human settlement differ from place to place. Because the uneven distribution of population in the world is determined by socio-politico factors also.

20 | UGC-NET Geography-III (S-13) 46. (C) Sweden and Italy do not use coal as a power resource in industry, because they have plenty of water resource. 47. (B) Most of the fishing areas in the world are found near continental shelves. 48. (D) Paradeep is not a natural harbour but Cochin, Marmagao and Mumbai are natural harbour. 49. (A) 50. (B) 51. (B) 52. (D) 53. (A) Theory of agricultural land use (Concept of economic rent) is based upon the above concept. 54. (C) Green Revolution never emphasized upon land reforms but it emphasized above all i.e. 1, 3 and 4 points. 55. (B) 56. (B) 57. (B) WHO is not a general purpose authority. 58. (B) 59. (C) 60. (A) 61. (A) Rimland concept was given by Spykman in 1944. 62. (C) Mackinder in his heartland concept said Land betweeen Volga in west to eastern Siberia in the east is a Pivotal point. Who rules

63. 64. 65. 66. 68. 69. 73.

74. 75.

this area rules the world and also commands the world. (A) (A) Bhakra Nangal Project is a joint venture of Punjab, Haryana and Rajasthan. (D) Western Ghats in Maharashtra are locally known as Sahyadris. (A) 67. (B) (B) 136 No. is the correct which represents the world in million sheets. (C) 70. (B) 71. (C) 72. (C) (A) Correct sequence of decreasing scale in maps is as follows : Cadastral map (large scale), Topographical Map (1/50,000 to 1/1000000 scale), Wall map (small scale) Atlas Map (very small scale). (A) (D) It is wrong, that value of the dot generally varies. But it remains constant and a Dot method is used to show the distribution of Population or any other element. But not agricultural production or Rainfall.

Geography (Paper–II)

UGC-NET/JRF Exam., Solved Paper

December 2013

Geography (Paper-II) Note—This paper contains fifty (50) objective type questions of two (2) marks each. All questions are compulsory. 1. Which of the following processes is responsible for weathering of rocks in the Karst region ? (A) Scree formation (B) Carbonation (C) Oxidation (D) Hydrolysis 2. Match List-I with List-II and select the correct answer from the codes given— List-I (Authors) (a) Woolridge and Morgan (b) Thornbury (c) Van Ricthofen (d) Steers List-II (Books) 1. Principles of Geomorphology 2. Geomorphology 3. Spirit and purpose of Geography 4. Unstable Earth Codes : (a) (b) (c) (d) (A) 1 2 3 4 (B) 2 3 1 4 (C) 3 1 2 4 (D) 4 2 1 3 3. Which one of these glacial features i s believed to have formed in the bed of a subglacial stream ? (A) An Esker (B) A Morraine (C) Drumlin (D) Kame 4. Indian Meteorological Department (IMD) has divided India into how many seismic zones ? (A) 4 (B) 5 (C) 6 (D) 7

5. Bad-land Topography is the product of the combined action of— (A) Wind and Glacier (B) Wind and Water (C) Water and Glacier (D) Water and Temperature 6. Increase in temperature with increase in height is known as— (A) Lapse rate (B) Adiabatic lapse rate (C) Inversion of temperature (D) Normal rate 7. Identify the correct sequence of the given processes regarding rainfall— (A) Unsaturated air, condensation, dew point, precipitation (B) Dew point, condensation, unsaturated air, precipitation (C) Unsaturated air, dew point, condensation, precipitation (D) Dew point, precipitation, condensation, unsaturated air 8. Read out the following conditions— 1. Cloudy sky 2. Cold dry air 3. Strong winds 4. Long winter nights Which of the above conditions promote inversion of temperature ? (A) 1, 2 and 4 (B) 2, 3 and 4 (C) 1 and 4 (D) 2 and 4 9. The climate of Southern Ganga plain in the Thornthwait classification is denoted by— (A) BA’W (B) CA’W (C) AA’W (D) CW’W

4 | UGC-NET Geography-II (D-13) 10. When the wind is deflected due to the rotation of earth it is known as— (A) Planetary wind (B) Geostrophic wind (C) Accidental wind (D) Forced wind 11. Salinity in water bodies increases with— (A) increase in evaporation and decrease in admixture of fresh water (B) decrease in evaporation and increase in admixture of fresh water (C) decrease in evaporation and no change in admixture of fresh water (D) increase in evaporation and increase in admixture of fresh water 12. Given below are two statements, one labelled as Assertion (A) and other labelled as Reason (R). Select your answer from codes given below— Assertion (A) : The salinity in Dead Sea is very low. Reason (R) : Dead Sea is located in tropical dry climatic region, where evaporation is high, precipitation is low and here drainage is internal. Codes : (A) Both (A) and (R) are right (B) Both (A) and (R) are wrong (C) (A) is wrong, but (R) is right (D) (A) is right, but (R) is wrong 13. Which one of the following primary green house gases is not associated with global warming ? (A) Water vapour (B) Carbon dioxide (C) Methane (D) Hydrogen 14. Match List-I with the List-II and select the correct answer from the codes given below— List-I (National Park) (a) Eraviculam (b) Bandipur (c) Hemis (d) Sironi List-II (State) 1. Karnataka 2. Kerala 3. Manipur 4. Jammu & Kashmir Codes : (a) (b) (c) (d) (A) 2 1 4 3 (B) 3 4 2 1 (C) 1 2 4 3 (D) 1 4 3 2

15. Which one of the following is a Taiga Biome ? (A) Sub-Tropical Biome (B) Sub-Arctic Biome (C) Savanna Biome (D) Sub-Sahara Biome 16. Match the following List-I with List-II and select the correct answer from the codes given— List-I (Geographer) (a) Alexander Von Humboldt (b) Ellen Churchill Semple (c) Peter Haggett (d) Jeans Brunhes List-II (Country) 1. France 2. U. K. 3. U. S. A. 4. Germany Codes : (A) (B) (C) (D)

(a) 4 1 4 2

(b) 2 2 3 3

(c) 3 3 2 1

(d) 1 4 1 4

17. Who among the following defined Geography as Discovery of Predictive Patterns during quantitative revolution period ? (A) Haggett (B) Harvey (C) Stewart (D) Bunge 18. In describing the patterns and processes of spatial interaction, geographers are most concerned with— (A) Accessibility and Connectivity (B) Density and Dispersion (C) Diffusion and Pattern (D) Pedestrain Cities 19. Who founded the Humanistic School of Geography ? (A) Wolpert (B) Tuan (C) Kirk (D) Harvey 20. Who amongst the following correlated colour of ocean water and its salinity ? (A) Ibn—Khaldun (B) Al—Masudi (C) Ibn—Sina (D) Al—Maqdisi

UGC-NET Geography-II (D-13) | 5 21. “All history must be treated geographically and all geography must be treated historically.” Who wrote this ? (A) Herodotus (B) Ratzel (C) Blache (D) Davis 22. Which school of thoughts developed the Science of Astronomy ? (A) Greeks (B) Chinese (C) Roman (D) Arabs 23. Demographic transition is a framework that explores the historical sequence of changes in— 1. Fertility 2. Mortality 3. Mortality and migration 4. Age-structure and sex composition Codes : (A) 1 and 4 are correct (B) 3 and 4 are correct (C) only 1 is correct (D) 1 and 2 are correct 24. Name the State which employs the highest number of child labour in the country ? (A) Tamil Nadu (B) Bihar (C) Andhra Pradesh (D) Jharkhand 25. India’s decadal population growth rate has been continuously declining since— (A) 1971-81 (B) 1981-91 (C) 1991-2001 (D) 2001-2011 26. Diego Garcia is an island in which of the following oceans ? (A) Atlantic (B) Pacific (C) Indian (D) Arctic 27. Read Assertion (A) and Reason (R) and find the correct answer from the codes given below— Assertion (A) : Urbanization is a defining phenomenon of this century and the developing countries are at the locus of this transformation. Reason (R) : The urban shift has happened in the last few decades largely due to rapid mega cities growth in the developing countries. Codes : (A) Both (A) and (R) are correct and (R) explains (A)

(B) Both (A) and (R) are correct but (R) does not explain (A) (C) (A) is correct, but (R) is false (D) (A) is false, but (R) is true 28. Match List-I with List-II and select the correct answer using the codes given below— List-I List-II (a) Baku 1. Diamond (b) Kuzbas Basin 2. Iron ore (c) Urals 3. Coal (d) Eastern Siberia 4. Oil Codes : (a) (b) (c) (d) (A) 4 3 2 1 (B) 2 3 4 1 (C) 1 2 3 4 (D) 4 3 1 2 29. Marquette range in U. S. A. is known for— (A) Uranium (B) Copper (C) Gold (D) Iron ore 30. The location of paper industries is highly influenced by which one of the following factors ? (A) Availability and nearness of raw material (B) Nearness of market (C) Availability of capital (D) Transport connectivity 31. Which one of the following is the largest volume of import commodities in India ? (A) Raw materials (B) Iron and Steel (C) Petroleum & Lubricants (D) Pearls and Precious stones 32. Kirkuk, one of the most important oilfields in the world, is located in— (A) Iran (B) Iraq (C) Kuwait (D) Russia 33. To which racial groups the Eskimos belong to ? (A) The Negroids (B) The Mongoloids (C) The Caucasoids (D) The Australoids

6 | UGC-NET Geography-II (D-13) 34. The concept of ‘Cultural landscape’ was promoted by— (A) Ratzel (B) Anne Buttimer (C) Carl Sauer (D) Wilbur Zelinsky 35. Which one of the following Islamic countries is predominantly of Shia sect ? (A) Iraq (B) Iran (C) Pakistan (D) Afghanistan 36. Who, among the following, propounded the concept of compage ? (A) Minshull (B) Blache (C) Whittlesey (D) Isard 37. Organic Theory of the State was propounded by— (A) Mackinder (B) Ratzel (C) Haushofer (D) Isaiah Bawman 38. Quasi Federal form of Government is in— (A) U. S. A. (B) Great Britain (C) Russia (D) India 39. Which of the following was the earliest regional planning exercise in India ? (A) National Capital Region Plan (B) Dandakaranya Area Plan (C) Damodar Valley Project (D) Bhakra-Nangal Project 40. Which of the following is an incorrect pair ? (A) Utilitarian — Functional Planning Planning (B) Comprehensive — Integrated Planning Planning (C) Regional — Spatial Planning Planning (D) Sectoral — Co-ordinated Planning Planning 41. Which one of the following States has the longest coast line ? (A) Tamil Nadu (B) Maharashtra (C) Gujarat (D) Kerala 42. In which of the following States of India the concentration of laterite soils is higher ? (A) Odisha (B) Gujarat (C) Jammu & Kashmir (D) Arunachal Pradesh

43. Given below are two statements, one labelled as Assertion (A) and the other labelled as Reason (R). Select your answer from the codes given below— Assertion (A) : Laterite soils are well developed in Kerala. Reason (R) : Kerala receives high rainfall during monsoon season. Codes : (A) (A) and (R) are correct and (R) explains (A) (B) (A) is correct, but (R) is false (C) (A) is false, but (R) is correct (D) (A) and (R) are correct, but (R) does not explain (A) 44. Given below are two statements, one labelled as Assertion (A) and the other labelled as Reason (R). Select your answer from the codes given below— Assertion (A) : Himalayan rivers are perennial. Reason (R) : The region gets rainfall from the South West monsoon season only. Codes : (A) (A) and (R) are correct and (R) explains (A) (B) (A) and (R) are correct, but (R) does not explain (A) (C) (A) is correct, but (R) is false (D) (A) is false, but (R) is correct 45. The ‘Valley of Kashmir’ lies between which of the following ranges ? (A) Pir-Panjal and Karakoram range (B) Pir-Panjal and Zaskar range (C) Zaskar and Ladakh range (D) Sulaiman and Kirthar range 46. Which one of the following regions has the highest proportion of agricultural land under irrigaton ? (A) Punjab-Haryana Plain (B) Rajasthan-Gujarat Plain (C) West Bengal Delta (D) Uttar Pradesh-Bihar Plain

UGC-NET Geography-II (D-13) | 7 47. Consider the following figures X and Y— A

2 cm B RF 1 : 2000

C

2 cm D RF 1 : 10‚00‚000

Fig. : X Fig. : Y Which of the following statement is correct ? (A) Fig. X is small scale and the distance between A and B is 40 metres (B) Fig. Y is large scale and the distance between C and D is 4 km (C) Fig. X is large scale and Fig. Y is small scale. The distances between A and B and C and D are 40 metres and 20 km respectively (D) Fig. X is large scale and Fig. Y is small scale. The distance between A and B and C and D are equal 48. If the variability of rainfall is to be measured, which of the following techniques will be used ? (A) Mean Deviation (B) Standard Deviation (C) Coefficient of Variation (D) Interquartile Range 49. Which of the following is not a Quantitative Distribution Map ? (A) Choroschematic Map (B) Isopleth Map (C) Dot Map (D) Choropleth Map 50. Occupational structure of population in India at State level is best represented by— (A) Dot Method (B) Isopleth (C) Choropleth (D) Pie diagram

Answers with Hints 1. (B) Carbonation is a process of chemical weathering of rocks by rain water. Thus changes any rock mineral that contain Lime, Soda, Potash or other basic oxides. It is a very significant process in reduction of limestone terrain. Scree is a accumulation of fragmented rock waste below a cliff or a rock face. While oxidation is a process in which oxygen is dissolved in water, reacts with certain rock mineral especially Iron. 2. (C)

3. (B) Sub-glacial pertaining to the environment, beneath a glacier refers to the processes by which a glacier moves across its floor, forms ground morraine. 4. (A) 5. (B) Bad-land Topography is a terrain difficult to traverse. It is used to describe any landscape characterized by deep dissection, ravines, gullies and sharp edged ridges which have been created by feurial erosion of rocks of relatively low resistance. 6. (C) Though there is a decrease in temperature with increasing height (Being air rarified). But under certain conditions there is increase in temp. with increasing height i.e. inversion of temp. 7. (C) 8. (D) For inversion of temperature clear sky and calm air along with cold-dry air with long winter nights are favourable conditions. 9. (B) 10. (A) Planetary Winds are deflected due to the rotation of the earth (Coriolis force). While Geostrophic winds are related to horizental flow of wind, parallel to Isobar. 11. (A) Salinity means water contains dissolved salts. It is owing to the high evaporation, high temperature and no or little admixture of fresh water. 12. (C) Dead Sea, at the border of Israel and Jordan has 238% salinity the maximum among the Inland Seas. 13. (D) Hydrogen is not associated with Global warming. While water vapour fullfills an important role in the Earth’s heat balance mechanism for together with Carbon di-oxide which form a type of Global blanket which prevents excessive heat loss from terrestrial radiation. 14. (A) 15. (B) Taiga biome is in between Temperature grassland and Tundra. 16. (C) 17. (A) Haggett in his book ‘Locational analysis in Human Geog-appealed to adopt the geometrical tradition to explain order, location order and patterns in Human Geog.

8 | UGC-NET Geography-II (D-13) 18. (A) For Geographers Spatial analysis is based on just three fundamental spatial direction— 1. Direction, 2. Distance, 3. Connection (Relative Position) 19. (B) Though Kirk (1951) was the first Geographer to attract a wide audience with his advocacy of humanistic approach. But Tuan (in 1976) was the profounder of Humanistic Geography. 20. (B) Al-Masudi gave to the Atlantic Ocean the name of Dark-Green Sea. He also opined that salt in the seas and oceans comes from the land. 21. (A) Herodotus the father or history was also one of the pioneer geographers and was a strong supporter of idea that “all history must be treated Geographically and all geography must be treated historically.” 22. (D) It is believed that astronomy flourished in Chaldaea, Assyri and Babylonia where the skies were mostly clear. 23. (D) Demographic transition model refers to fertility and mortality. 24. (C) 25. (A) It is since 1971-81 that the growth rate is continuously declining. It was 24·66% in (1971-81) 23·85% (1981-1991), 21·54% (1991-2001), 17·7% (2001 to 2011) 26. (C) Diego Garcia (6°34′5 and 72°24′E) is an Indian Ocean. 27. (A) In developing countries where agricultural sector is reducing and Industrial Sector is increasing urbanization in a focus of thin transformation. Due to over crowdedness especially in Mega cities urban shift has taken place. 28. (A) 29. (D) Marquette range, near the Lake Superior is famous for Iron-ore. 30. (A) 31. (C) 32. (B)

33. (B) Eskimos, Yakuts, Yukaghir are Mongoloid by race. They are short statured people with flat but narrow faces, small Snub noses, yellow brown skin colour, all are the Characteristics of Mongoloids. 34. (C) Sauer, an American Geographer stressed that geographers should proceed generally and trace the development of a natural landscape into a cultural landscape. He emphasized the cultural landscape, its creation involved (a) general geography, (b) regional geography (c) historical geography. 35. (B) 85% of Iran religious affinities include Shia, Iraq (62%) Afghanistan 17% Pakistan 20%. 36. (C) 37. (B) It was Ratzel who profounded the organic theory of state. In his Political Geography he compared the State to an organism. He said like some simple organism state must either grow or die and can never stand as a state. 38. (D) Indian Constitution is not only regarded as federal or unitary in strict sense, it is often defined to be a quasi-federal in nature, as India is a single united nation, is described as, a union of states and is constituted into a Sovereign, Socialist, democratic republic. 39. (C) Damodar Valley Project in 1947 was the first effort of Regional planning. 40. (D) 41. (C) 42. (A) 43. (D) 44. (B) 45. (B) 46. (A) 47. (C) 48. (B) 49. (A) Choroschematic is a type of thematic map in which Quantative aspect is shown. All three Isopleth, Dot and Choropleth are its kind, while choroschematic is a m. Hence all choroschemetic maps are non-Quantative maps. 50. (D) Pie diagram is best suited to show the occupational structure of population in India.

Geography (Paper–III)

UGC-NET/JRF Exam., Solved Paper

December 2013

Geography (Paper-III) Note—This paper contains seventy five (75) objective type questions of two (2) marks each. All questions are compulsory. 1. The concept of geosynclines was given by— (A) James Hall and Dana (B) Hang (C) Holmes (D) Steers 2. The Base Level concept was postulated by— (A) James Hutton (B) J. W. Powell (C) W. M. Davis (D) Walther Penck 3. Which of the following is formed as a result of tectonic forces ? (A) Hanging valley (B) V-shaped valley (C) Rift valley (D) Blind valley 4. Match the List-I with List-II and select the correct answer using the codes given— List-I (a) A plain largely composed of recent alluvium (b) a deep sea plain (c) a plain formed by wind action (d) a level surface lightly covered with thin layer of alluvium List-II 1. Pedi plain 2. Loess plain 3. Flood plain 4. Abyssal plain Codes : (a) (b) (c) (A) 4 3 2 (B) 3 4 1 (C) 4 3 1 (D) 3 4 2

(d) 1 2 2 1

5. “The present is the key to the past.” This statement was made by— (A) W. M. Davis (B) James Hutton (C) Van Ritchthofen (D) A. Penck 6. Which of the following was a part of Lauratia ? (A) Anatolian Plate (B) Chinese Plate (C) Iranian Plate (D) Agean Plate 7. Match List-I with List-II and choose the correct answer using the given codes— List-I (a) Normal fault (c) Lateral fault List-II

(b) Reverse fault (d) Stage fault

1.

Displacement of both the rock blocks in opposite directions 2. Movement of both the rock blocks towards each other 3. Displacement of rock block horizontally along fault plane 4. When the slopes of a series of faults are in same direction Codes : (a) (b) (c) (d) (A) 1 2 3 4 (B) 4 3 2 1 (C) 3 1 2 4 (D) 2 3 1 4 8. When does an escarpment appear ? (A) When a land block moves horizontally (B) When a land block moves vertically (C) When a land block moves due to water logging (D) When a land block moves due to human intervention

4 | UGC-NET Geography-III (D-13) 9. Identify fluvio-glacial deposits from the following— (A) Outwash plain (B) Flood plain (C) Penne plain (D) Pan plain 10. Migration from developed to less developed region is called as— (A) Emigration (B) Immigration (C) Perverse migration (D) Reverse migration 11. “Space is socially or culturally constructed” is the view under— (A) Logical positivism (B) Behaviouralism (C) Post modernism (D) Structuralism 12. A counter clock-wise atmospheric circulation in the northern hemisphere is known as— (A) Pressure gradient (B) Cyclone (C) Anti-cyclone (D) Tornado 13. The author of the book ‘The Unstable Earth’ is— (A) Paschel (B) Gilberk G. K. (C) Mallot C. A. (D) J. A. Steers 14. Which of the following earthquake waves are most destructive ? (A) S-waves (B) P-waves (C) R-waves (D) L-waves 15. Mediterranean climate is characterized by— (A) Dry summer and Humid winter (B) Humid summer and Dry winter (C) Dry summer and Dry winter (D) Humid summer with no winter

18. As per the Koppen’s scheme the Bhwh type of climate is found in— (A) Jammu & Kashmir (B) Rajasthan (C) Gujarat (D) Odisha 19. Cold dry wind experienced particularly in winter along the eastern coast of Atlantic Ocean and in Northern Italy is called— (A) Cyclone (B) Boras (C) Tornado (D) Hurricane 20. The atmosphere gets heated by which one of the following ? (A) Direct rays of the sun (B) Volcanic activity (C) Burning of organic material (D) Radiation from the earth 21. A tropical cyclone was located 500 kms. South-West of Visakhapatnam. The cyclone first moved 250 kms in North-West direction and then suddenly changed the direction to the North-East. Which of the following ports would raise the highest danger signal ? (A) Chennai (B) Haldia (C) Visakhapatanam (D) Tuticorin 22. Which one of the following regions does not receive much rainfall during the South-West monsoon season ? (A) Tamil Nadu Coast (B) Maharashtra (C) Chota Nagpur Plateau (D) Eastern Himalayas

16. Insolation reaches the earth surface in the form of— (A) Short waves (B) Long waves (C) Microwaves (D) Lorenz curve

23. In which type of climate equiplanation process i.e., reduction of relief to a plain takes place ? (A) Arid climate (B) Cold climate (C) Tropical climate (D) Temperate climate

17. The range of visible wave length on electromagnetic spectrum is— (A) 0·4 to 0·7 micrometers (B) 0·7 to 15·0 micrometers (C) 0·3 to 0·9 micrometres (D) 0·3 to 15·0 micrometres

24. Winter rainfall in North-Western part of India is mainly due to— (A) Western disturbance (B) North-East Monsoon (C) North-West Monsoon (D) Depression in the Bay of Bengal

UGC-NET Geography-III (D-13) | 5

Altitude

25. Which one of the following phenomena shown in the sketch given below is correct ?

Temperature

Choose the correct answer from the codes given below— Codes : (A) Normal lapse rate (B) Upper surface inversion (C) Lower surface inversion (D) Environmental lapse rate 26. Given below are two statements, one labelled as Assertion (A) and the other labelled as Reason (R)— Assertion (A) : The eastern coast of India is affected by tropical cyclones more than the western coast. Reason (R) : Tropical cyclone originates only in the Bay of Bengal. In the context of the above two statements, which one of the following is correct ? (A) Both (A) and (R) are correct, but (R) does not explain (A) (B) Both (A) and (R) are correct and (R) explains (A) (C) (A) is correct, but (R) is false (D) (A) is false, but (R) is correct 27. Match List-I and List-II and select the correct answer using the codes given— List-I (Names of tropical cyclones) (a) Hurricanes (b) Typhoons (c) Willy-Willy (d) Taifu List-II (Regions) 1. 2. 3. 4.

Australia Japan U. S. A. China

Codes : (a) (b) (c) (d) (A) 3 4 1 2 (B) 1 2 3 4 (C) 4 3 2 1 (D) 2 1 3 4 28. The temperature that provides favourable ecological conditions for living coral reefs is— (A) less than 10°C (B) 10°C to 15°C (C) 15°C to 20 °C (D) more than 20°C 29. Match List-I with List-II and select the correct answer using the codes given— List-I (a) Coral reef (c) Abyssal plain List-II

(b) Shoal (d) Continental slope

1. Organic Ocean Deposit 2. Deep flat surface 3. Steep descent from the shelf bed 4. Detached elevation with shallow depth Codes : (a) (b) (c) (d) (A) 2 3 1 4 (B) 4 1 3 2 (C) 3 2 4 1 (D) 1 4 2 3 30. Which among the following has postulated the Subsidence Theory related to coral reef ? (A) Dally (B) Agassiz (C) Darwin (D) Davis 31. If a tide occurs at a place at 6 p. m. when will be the next tide occur ? (A) 00·13 a. m. of the next day (B) 06·26 a. m. of the next day (C) 12·39 p. m. of the next day (D) 06·52 p. m. of the next day 32. Match List-I with List-II and select the correct answer using the codes given— List-I (a) (b) (c) (d)

Tide Tsunami Gulf stream Labradone current

6 | UGC-NET Geography-III (D-13) List-II 1.

A warm comparatively fast moving ocean current 2. A cold ocean current 3. The periodic rise and fall of water 4. Rise of sea water in wave form due to seismic activity Codes : (a) (b) (c) (d) (A) 2 1 3 4 (B) 1 2 3 4 (C) 3 4 1 2 (D) 4 3 2 1 33. Which one of the following ecosystem services is not a part of Millennium Ecosystem Assessment (MA) report ? (A) Provisioning (B) Promoting (C) Supportive (D) Regulating 34. Which one of the following places in India the first bird-sanctuary was set up ? (A) Vedanthangal (B) Kundremukha (C) Bannarghata (D) Keoladeo 35. Given below are two statements, one labelled as Assertion (A) and the other labelled as Reason (R). Select your answer from the codes given below— Assertion (A) : Physical factors provide only a partial and deterministic explanation of geographical distribution of population. Reason (R) : Man is by no means passive in his choice of areas for settlement and everywhere demonstrates an ability to exercise some control over his environment. (A) Both (A) and (R) are correct and (R) is the correct explanation of (A) (B) Both (A) and (R) are correct, but (R) is not the correct explanation of (A) (C) (A) is correct, but (R) is false (D) (A) is false, but (R) is correct 36. Which one of the following indicates the principle of transport in Central Place Theory ? (A) K3 (B) K4 (C) K7 (D) K9

37. Assertion (A) : Post-industrial city is largely characterised by horizontal expansion. Reason (R) : Land prices are high in the central city. (A) Both (A) and (R) are correct (B) Both (A) and (R) are correct, but (R) does not explain (A) (C) (A) is correct, but (R) is false (D) (R) is correct, but (A) is false 38. Match List-I with List-II and select the correct answer using the codes given below— List-I List-II (a) Cantonment 1. Kochi (b) Transport 2. Bangalore (c) Mining 3. Mau (d) I. T. 4. Ankleshwar Codes : (a) (b) (c) (d) (A) 1 3 2 4 (B) 3 2 4 1 (C) 3 1 4 2 (D) 3 1 2 4 39. Read Assertion (A) and Reason (R) and find the correct answer from the codes given below— Assertion (A) : Global urban population is not uniformly distributed by location and nearly two-thirds is relatively concentrated within low elevation coastal areas. Reason (R) : Soil fertility in the coastal areas caused concentration of rural settlements. (A) Both (A) and (R) are correct and (R) explains (A) (B) Both (A) and (R) are correct, but (R) does not explain (A) (C) (A) is correct, but (R) is false (D) (A) is false, but (R) is true 40. Which one of the following stages of demographic transition model predicts “ahigh birth, but a-low death rate” ? (A) First stage (B) Second stage (C) Third stage (D) Late-second stage 41. Match the following— List-I (State) (a) Uttarakhand (c) Tamil Nadu

(b) Chhattisgarh (d) Gujarat

UGC-NET Geography-III (D-13) | 7 List-II (Rate of Female Literacy (%) at 2011 census) 1. 69·7 3. 60·2 Codes : (a) (b) (A) 1 2 (B) 2 3 (C) 3 4 (D) 4 3

2. 70·0 4. 73·4 (c) 3 4 1 2

(d) 4 1 2 1

42. Which one of the following is not an assumption or principle of the concentric zone model of urban land case ? (A) Cultural and social heterogeneity of the population (B) Transport is equally easy, rapid and cheap in every direction within the city (C) None of districts is attractive due to differences in terrain (D) There is a concentration of heavy industry 43. Who, amongst the following, defined Geography as a chorological science ? (A) Ptolemy (B) Richthofen (C) Hettner (D) P. E. James 44. Who, amongst the following, has propounded the concept of Time Space Geography ? (A) Hagerstrand (B) Haggett (C) Johnston (D) Harvey 45. Match the following from the codes— List-I (Major Proponent) (a) Gibson (b) Guelke (c) Peet (d) Wright and Kirk List-II (Philosophy of Geography) 1. Idealism 3. Behaviouralism Codes : (a) (b) (c) (A) 1 2 3 (B) 1 2 4 (C) 2 1 4 (D) 2 1 3

2. Realism 4. Radicalism (d) 4 3 3 4

46. Which one of the following ports has been developed on a lagoon ? (A) Chennai (B) Mumbai (C) Kochi (D) Visakhapatnam 47. Truck farming is associated with— (A) Vegetables (B) Milk (C) Cereals (D) Poultry 48. The essential feature of shifting cultivation is— (A) Rotation of crops (B) Rotation of fields (C) Single cropping (D) Use of plenty of fertilizers 49. What is the common practice involved in shifting cultivation ? (A) Use of sophisticated machinery (B) Large scale use of fertilizers (C) Utilization of poor soils through ploughing/fallowing (D) Maximum use of animal power 50. Under the scope of town planning in India is covered— 1. Urban renewal 2. Planning of urban amenities and facilities 3. Building of new towns 4. Building of metropolitan cities Codes : (A) 1 and 3 are correct (B) only 1 is correct (C) 1, 2 and 3 are correct (D) 2, 3 and 4 are correct 51. Match the List-I with List-II and select the correct answer from the codes given below— List-I List-II (a) Rivers (b) Settlements (c) Winds (d) Rocks Codes : (a) (b) (c) (A) 3 4 2 (B) 4 3 1 (C) 2 1 4 (D) 1 2 3

1. 2. 3. 4. (d) 1 2 3 4

Sand stone Gale Levees Satellite

8 | UGC-NET Geography-III (D-13) 52. Match the List-I with List-II and select the correct answer using the codes given below— List-I (a) Falkland current (b) Norwegian current (c) Florida current (d) Okholsk current List-II 1. Warm current 2. Cold current 3. Atlantic Ocean 4. Kamchetka Peninsula Codes : (a) (b) (c) (d) (A) 2 3 1 4 (B) 1 2 4 3 (C) 3 1 2 4 (D) 4 3 1 2 53. Who has, amongst the following geographers, defined Geography of Public Finance ‘who gets what, where, and at what cost’ ? (A) R. J. Chorley (B) David Harvey (C) P. Claval (D) R. J. Bennett 54. Match the List-I with List-II and select the correct answer using the codes given below— List-I (Natural Parks & Wild Life Sanctuaries) (a) Nandan kanan (b) Kaziranga (c) Bandhavgarh (d) Melghat List-II (States) 1. Maharashtra 2. Madhya Pradesh 3. Odisha 4. Assam Codes : (a) (b) (c) (d) (A) 1 2 3 4 (B) 2 3 4 1 (C) 3 4 2 1 (D) 4 1 3 2 55. Which of the following is not a supervised image classification technique ? (A) Parallelopiped classifier (B) Minimum distance to mean classifier (C) Neural network analysis (D) Gaussian maximum likelihood classifier

56. A meteor is— (A) Comet without a tail (B) Detached piece of an asteroid (C) Tiny star (D) Piece of matter which has entered the earth’s atmosphere from outer space 57. Which one of the following tribes does not correctly matched ? (A) Batwa and Congo Basin (B) Ruwala and Central Iran (C) Inuit and Canada (D) Yuklagir and Siberia 58. Which one of the following regions witnessed the growth of mighty civilizations of Roman and Greek ? (A) Temperate region (B) Savanna region (C) Mediterranean region (D) Tundra region 59. Which of the following is not considered a geographic pattern ? (A) Centralized (B) Distributive (C) Linear (D) Random 60. Machu Picchu of Inca civilization is located in— (A) Argentina (B) Brazil (C) Columbia (D) Peru 61. In 1919, Mackinder renamed his ‘Pivot Area’ as ‘Heartland’ in famous book— (A) Foreign Affairs (B) The Round World and the Winning of the Peace (C) The Democratic Ideals and Reality (D) World War and Geography 62. The objectives behind building the planned city of Chandigarh were— 1. To build a city of modern India’s dreams. 2. To settle down refugees coming to India after the partition. 3. To develop an attractive tourist centre in Northwestern India. 4. To provide capital city to Truncated Punjab.

UGC-NET Geography-III (D-13) | 9 Codes : (A) 2 and 4 are correct (B) 2, 3 and 4 are correct (C) 1, 2 and 4 are correct (D) only 4 is correct 63. Which of the following sentences is not a correct one ? (A) Utilitarian planning is a functional planning and it strives to achieve a supreme single specific goal (B) Sectoral planning is a co-ordinated planning with planning in various sectors in an integrated manner (C) Comprehensive planning has also been used with implicit connotation of integrated planning of all elements of a single activity (D) Town planning is an expression of spatial planning of an urban system 64. Several planned cities have been developed in India after Independence. Which one of the following is not a post-independence planned city ? (A) Gandhinagar (B) Chandigarh (C) Jaipur (D) Bhubaneshwar 65. Which one of the following is not a measure of Central tendency ? (A) Arithmetic mean (B) Mean deviation (C) Median (D) Mode 66. Which one of the following Indian Remote Sensing Satellites, IRS-P4 is also known as— (A) Resourcesat (B) Edusat (C) Remotesat (D) Oceansat 67. Which one of the following States has recorded the lowest human development index ? (A) Bihar (B) Himachal Pradesh (C) Punjab (D) Haryana 68. Which one of the following group of States, which includes catchment area of Cauvery river basin is correct ? (A) Tamil Nadu, Kerala, Pondicherry and Andhra Pradesh

(B) Tamil Nadu, Kerala, Andhra Pradesh and Pondicherry (C) Tamil Nadu, Andhra Pradesh, Maharashtra and Kerala (D) Tamil Nadu, Kerala, Karnataka and Pondicherry 69. Which one of the following is a centrographic measure ? (A) Geometric mean (B) Arithmetic mean (C) Mean centre (D) Standard deviation 70. Match List-I with List-II and select the correct answer using the codes given below— List-I (a) Block Mountains (b) Volcanic Mountains (c) Relict Mountains (d) Fold Mountains List-II 1. Aravallies 3. Vindhyachal Codes : (a) (b) (A) 1 2 (B) 2 1 (C) 4 3 (D) 3 4

2. Pennives 4. Mount Popa (c) 3 4 2 1

(d) 4 3 1 2

71. Which one of the following map scales is a large scale ? (A) 1 : 250,000 (B) 1 : 25,000 (C) 1 : 500,000 (D) 1 : 50,000 72. Given below are two statements, one labelled as Assertion (A) and the other labelled as Reason (R)— Assertion (A) : The Raster format data structure provides a greater computational efficiency. Reason (R) : The Raster format data is simple. In the context of the above two statements, which one of the following is correct ? (A) Both (A) and (R) are correct and (R) explains (A)

10 | UGC-NET Geography-III (D-13) (B) Both (A) and (R) are correct, but (R) does not explain (A) (C) (A) is correct, but (R) is false (D) (A) is false, but (R) is correct 73. An original map has the RF-1/50,000 and R.F. of the new map will be 1/250,000. What is the correct proportion of enlargement/ reduction as given below ? (A) Reduction 1/5 (B) Enlargement (5 times) (C) Reduction 1/10 (D) Enlargement (10 times) 74. Which one of the following values of correlation coefficient (r) is not correctly matched degree of relationship ? (A) + 0·99 High (B) + 0·50 Moderate (C) – 0·01 Very low (D) – 0·99 Nil 75. In which of the following year, IRS-1A was launched ? (A) 1982 (B) 1987 (C) 1988 (D) 1990

Answers with Hints 1. (A) The concept of Geosyncline was given by Hall and Dana, but the theory of Geosyncline is due the Hang. 2. (B) The concept of Base-level of erosion was given by J. W. Powell. 3. (C) Hanging valley (Glacier), V shaped valley (River erosion) Blind valley (Krast topography) and Rift velley (tectonic forces). 4. (D) 5. (B) Due to the criticism of Catastrophism, the Uniformitorianism, a new chapter was added by Scottest Geomorphologist James Huttan in 1797. He said, “Present is the key to the past”. 6. (B) 7. (A) 8. (B) Escarpment used as synonym of Cuesta is a asymmetrical ridge produced by differential erosion of gently dipping strata, in which the long gentle slope is generally accordance with the dip of the resistance strata, while the other slope is shorter gentally steeper.

9. (A) Outwash plain or Sandr, Sandar is an Icelandic term, used for outwash plain is formed from glacio-fluvial material carried out from the front of an ice-sheet by melt stream. It is an extensive accumulation of gravel, sand and silt. 10. (C) 11. (C) 12. (B) Cyclone, a system of low atmospheric pressure in which the Barometric gradient is steep, winds circulate, blowing inward in an anticlockwise direction in N. hemisphere and in a clockwise direction in Southern hemisphere. 13. (D) 14. (D) Surface wave or L waves, reaching to landsurface are most destructive than other waves. 15. (A) Mediterranean climate, the summers are dry (temp. 17° to 27° C) while it rains in winter. 16. (A) The radiant energy received from the Sun, transmitted in a form analogous to shortwaves (1/250 to 1/6700) m. m. in length .... is called solar radiation or insolation. While Earth Re-radiates this energy back to atmosphere is known as Long wave terrestrial radiation. 17. (A) 18. (B) 19. (B) Bora, a strong, very dry, northeasterly wind, blows in cold gust from the Balkan mts down to the easterm shore of Adratic sea, most commonly in winter while Hurrican (West indies and Mexico) Tornado (Midwest of U. S. A.) Cyclone (Temperate region as well as tropical areas. 20. (D) Atmosphere is a indirect source of heating. It is the earth which first heated up, there after the earth longwave terrestrial radiation the atmosphere is heated up. 21. (C) It is Visakhapatanam which would raise the highest danger signal. 22. (A) Tamilnadu Coastar Coromandal Coast does not receive rain in S. W. Mansoon season, but gets rain either from retreating monsoon or from N. E. Mansoon. While the Maharashtra (Arabian Sea Branch), Chotanagpur Plateau or Eastern Himalaya (Bay of Bengal branch) get rain from S. W. Monsoon.

UGC-NET Geography-III (D-13) | 11 23. (B) In general all Geomorphological processes which reduce topography to a plain like surface, this process of landscape levelling is under peri-glacial climate (cold climate). 24. (A) It is due to the Western disturbance (Mediterranean sea or Arabian Sea Cyclones) that North-Western part of India receives rainfall in winter. 25. (B) 26. (C) Tropical Cyclones donot originate in Arabian Sea, but they originate in Bay of Bengal as well as Mexico, China Sea etc. 27. (A) 28. (D) Coral a marine polype capable of secreting Calcium Carbonate to built an external Skeleton flourishes in clear, warm, tropical oceans usually in between 30° N and 30° S of equator, but is usually found in Warm Sea (>20°C) outside these limits. 29. (D) 30. (C) Subsidence theory related to card was propounded in 1837, but was reviewed in 1842. In his theory he had a view that there is no about corals survive in narrow warm seas, but reefs formed with them may found in deep sea, where coral can not survive at any cost. 31. (B) Normally at every place ‘Tide’ occurs twice a day (24 hrs.), Because earth takes 24 hrs. to complete a round at any place thus tide may occur after 12 hrs, but in fact it is not so. every day it occurs 26 minutes late. Because Moon along its with revolution of earth, revolves round its own axis. Earth moves from W. to E. Tide centre moves from E to W. But tide centre completes its full round, but moon goes further, Tide centre takes 52 minutes to came infront of moon. Hence Tide occurs 26 minutes later. The next tide will occur at 6·26 A. M. (Next day). 32. (C) 33. (B) 34. (A) The given bird-sanctuaries i.e the Question were set up in given years— 1. Vedanthangal—(Tamilandu)—1936 2. Kundremukha—(Karnataka)—1987 3. Bannarghata—(Karnataka)—1974 4. Keoladeo—(Rajasthan)—1981 35. (B)

36. (B) In Central place theory, principle of Transport is indicated by K4 Market principle is indicated by K3 , Administrative principle by K7 . 37. (A) 38. (C) 39. (B) 40. (B) It is in the second stage (early) high fertility and 11 declining mortality, but in late Second stage slowly decline fertility and sharply decline mortality in first stage fertility or mortality is high in third stage birth rate and death rate decline. 41. (B) 42. (D) The concentric zone model of urban land use is based on following assumptions city with a single centre grows outward Zone I C. B. D., Zone II wholesale market, but light manufacturing industries Zone III low class residence Zone IV Middle class residence Zone V High class residence, Zone VI Commuter zone. 43. (C) Alfred, Hettner a pupil of Richthofen and Ratzel was a physical and regional Geographer. His book Europe was published in 1907. According to him Geography is a chorological Silence or it is the study of regions. 44. (A) 45. (C) 46. (C) Kochi (Karela) has developed Perriyar lagoon. 47. (A) Truck farming or market of ordening is associated with the intensive cultivation of vegetables, Fruits or flowers for commercial purposes. It is done nearby a big city. 48. (B) 49. (C) In Shifting cultivation the forested land is cleared and burned. On that land without manuring, with poor ploughing or following, the deep rooted crops are grown. After 2 or 3 years cultivation, the land is left alone and cultivators go to some other place, but do not comeback at same field. 50. (C) 51. (A) 52. (A) 53. (D) 54. (C) 55. (C) A, B and D are the techniques of superwised image classification of data received through remote sensing (such as IRS satelites). 56. (D) Meteor a body of matter travelling at great speed through space which becomes luminious when it enters to atmosphere at about 150 km above the earth’s surface.

12 | UGC-NET Geography-III (D-13) 57. (B) Ruwala Badawin are found Arab, Aden, Oman etc. not in Iran. 58. (C) In Asia minor, along the coast of the Mediterranean sea Roman and Greek civilization developed. 59. (B) Centralized, linear and Random are the geographic pattern, but distributive is not a geographic pattern. 60. (D) An Inca city 60 km North West of Cusco (Cuszo) Peru. The remains perch 2000 feet (600 meter) above the Vilcanota river on a narrow ridge between the mountain heights Machu-Picchu (old peak) and Huayna Picchu (New peak). 61. (C) In 1904 Mackinder formulated the concept of the geographical Pivet of history which was later on known as Heartland concept (theory) in his famous book “The Democratic Ideas and Realities”. 62. (C) 63. (B) Sectoral Planning is never done an integrated basis among various sectors. 64. (C) Jaipur is not a post independence planned city. While rest three are post independence planned Cities.

65. (B) Mean Median and Mode are the measures of central tendency, but Mean deviation is not used as a central tendency. 66. (D) IRS-P4 is also known as Ocean Sat (launched on 26th May, 1999 from ASLVC2). 67. (A) As per H. D. I. Report 2011 the given states in Question have following H. D. I.— Bihar—0·367 Haryana—0·552 Punjab—0·605 Himanchal Pradesh—0·652 68. (D) 69. (C) 70. (D) 71. (B) RF 1 : 25,000 is a large scale map while RF 1 : 250,000, 1 : 500,000 are very small scale map and RF 1 : 50,000 is comparatively a small scale map in comparision of RF 1 : 25,000. 72. (C) 73. (A) RF 1/250,000 is 5 times reduction of RF 1/50,000 (i.e. Shall be 1/5 of RF 1 : 50,000). 74. (D) –0·99 will be high Negative correlation rather than nil correlation. 75. (C) IRS 1A was launched on 17th March, 1988.

Geography (Paper-II) UGC-NET/JRF-Exam., 2014 Solved Paper

June 2014

Geography (Paper II) Note—This paper contains Fifty (50) objective type questions of two (2) marks each. All questions are compulsory. 1. The landscapes produced by a single dominant geomorphic process are called as— (A) Simple Landscapes (B) Compound Landscapes (C) Monocyclic Landscapes (D) Multicyclic Landscapes 2. Flat topped sand ridges with long dimensions extending parallel to the prevailing winds, but locking the collapsing fronts are known as— (A) Whalebacks (B) Seifs (C) Barchans (D) Sand drifts 3. The river valleys whose courses are controlled by factors which are not determinable are termed as— (A) Consequent (B) Subsequent (C) Insequent (D) Resequent 4. Match List-I with List-II and select the correct answer from the codes given below— List-I (Books) (a) Principles of Geomorphology (b) The study of Landforms (c) Unstable Earth (d) Morphology of the Earth List-II (Authors) 1. L.C. King 2. W.D. Thornbury 3. R.J. Small 4. J. A. Steers Codes : (a) (b) (c) (d) (A) 2 3 4 1 (B) 1 2 3 4 (C) 4 3 2 1 (D) 3 1 4 2

5. The slope replacement model was propounded by— (A) Wood (B) Davis (C) Penck (D) Strahler 6. Who among the following observed that summer haeting of Tibetan Plateau was the most important factor in the monsoonal circulation over India ? (A) Fohn (B) Koteswaram (C) Frost (D) Halley 7. According to Koppen’s Scheme of climatic classification ‘E’ (Polar or Mountain) type climatic is found in— (A) Nagaland (B) Arunachal Pradesh (C) Sikkim (D) Jammu & Kashmir 8. Which one of the following is not a factor that affects wind ? (A) Pressure gradient (B) Friction (C) Magnetism (D) Carioles effect 9. Which one of the locations listed below should have highest annual temperature range ? (A) Equatorial Island (B) Tropical coast (C) Polar ice cap (D) Mid-latitude continental centre 10. Which term is used to describe the conversion of a solid directly to a gas, without passing through the liquid State ? (A) Evaporation (B) Condensation (C) Sublimation (D) Freezing

4 |

UGC-NET/JRF Geography-II (J-14)

11. The main source of carbonmonoxides, as air pollutants is— (A) Thermal decomposition of fats (B) Dye-making (C) Blast furnaces (D) Gasoline motor exhausts 12. The vertical movement of cold water from deeper oceanic layers to replace warmer surface is called as— (A) Turbidity (B) Emergence (C) Upwelling (D) Halocline 13. Which one of the following types of sea floor sediments includes calcareous and siliceous oozes ? (A) Fluvial (B) Terrigenous (C) Biogenous (D) Hydrogenous 14. Vast numbers of herbivorus animals and carnivorous such as lions, Jackals and hyenas are found in— (A) Equatorial rainforest (B) Taiga (C) Campos of Brazil (D) Savannas of Africa 15. Match List-I with List-II and select the correct answer from the codes given below— List-I (a) Green Plants (b) Animal Parasites (c) Plant Parasites (d) Fungi List-II 1. Decomposer 2. Primary consumer 3. Secondary Consumer 4. Producer Codes : (a) (b) (c) (d) (A) 4 3 2 1 (B) 2 1 4 3 (C) 3 2 1 4 (D) 1 2 4 3 16. Nomothetic approach in Geography emphasizes upon— (A) Inductive study (B) Law making study (C) Ideographic study (D) Empirical study

17. Which one of the following pairs does not match correctly ? Author Book (A) Ibn Khaldun — Muqaddimah (B) Al-Balakhi — Kitabul Ashkal (C) Al-Masudi — Routes and Realms (D) Al-Biruni — Kitab-al-Hind 18. Match List-I with List-II and select the correct answer from the codes given below— List-I (Geographers) (a) F. Ratzel (b) C. Ritter (c) R. Harthorne (d) O. Schluter List-II (Concepts Developed) 1. Landscape Science 2. Areal differentiation 3. Teleological view 4. Social Darwinism Codes : (a) (b) (c) (d) (A) 4 3 2 1 (B) 3 4 1 2 (C) 2 4 3 1 (D) 4 1 2 3 19. Which one of the following is in correct sequence in the development of German Geographical thought ? (A) Troll, Peschel, Hettner, Ratzel (B) Peschel, Ratzel, Hettner, Troll (C) Peschel, Troll, Hettner, Ratzel (D) Ratzel, Peschel, Troll, Hettner 20. Who coined the term ‘Cosmography’ and divided it into uranography and Geography ? (A) O. Peschel (B) C. Ritter (C) B. Vareneous (D) A. Humboldt 21. The book entitled ‘Perspective on Nature of Geography (1959)’ was authored by— (A) Ellen Churchill Semple (B) Rudolf Kjellen (C) Richard Hartshorne (D) Isaih Bowman 22. Net population change is determined by— (A) Mortality (B) Migration (C) Fertility (D) Both (A) and (C)

UGC-NET/JRF Geography-II (J-14) | 5 23. Rural house is an index of the— (A) Environment (B) Racial Element (C) Fashion and Style (D) Modern Architectural Design 24. The stages of inception in the dynamics of urban growth refers to— (A) The factor which gives rise to a city in a particular place (B) The process through which separate functional districts are carved out (C) The outward, often radial, movement of activities from the centre (D) The pattern of forcing activties out of an area 25. The Chief means of transport in Tundras is— (A) Camel (B) Donkeys (C) Sledge (D) Horse 26. The development of Chennai Bangaluru corridor is based on— (A) Automobile and I.T. Industries (B) Cotton-textile and garment industries (C) Pharmaceutical and Chemical Industries (D) Residential and Retail Development 27. Of late, the major focus in urban industrial development is on— (A) Nodal Development (B) Cluster Development (C) Port-City Oriented Development (D) Both (A) and (C) 28. High level managerial and executive administrative position fall in the category of— (A) Primary activities (B) Secondary activities (C) Tertiary activities (D) Quarternary activities 29. Which of the following statement are correct? 1. Lignite and low grade bituminous coal were formed during quarternary period. 2. Dolomite is used as refractory material and flux in iron and steel industry. 3. Solar energy, wind, hydel power and biogas are renewable resources.

Select the correct answer using codes given below— Codes : (A) 1, 2 and 3 (B) 1 and 2 (C) 2 and 3 (D) 1 and 3 30. What are the three bases for spatial interaction as described in Edward Ullman’s model ? (A) Complementarity, Transferability and intervening opportunity (B) Complementarity, Commodity specific relationship and surplus-deficit relationship (C) Complimentarity, convenience and residential neighbourhood (D) Human behaviour, convenience and transferability 31. Japan is one of the leading industrial countries in the world because it has— 1. Developed hydel power. 2. Large deposits of metallic minerals. 3. High level of technological capability. 4. Insular location. Which of these statements are correct ? (A) 1, 2, 3 and 4 (B) 1, 2 and 3 (C) 1 and 3 (D) 2 and 4 32. Given below are two statements one labelled as Assertion (A) and the other labelled as Reason (R)— Assertion (A) : With the development of H.Y.V. rubber trees and later synthetic rubber, the rubber of the Amazon basin has become a little more than memory. Reason (R) : Resources are created and destroyed by human culture. In the context of the above two statements, which of the following is correct ? Codes : (A) (A) is correct, but (R) is wrong (B) (A) is wrong, but (R) is correct (C) Both (A) and (R) are correct and (R) explains (A) (D) Both (A) and (R) are wrong 33. Which one of the following culture realms is characterised by (I) heritage of ancient civilization and traditions of classical culture

6 |

UGC-NET/JRF Geography-II (J-14) (II) rise of ancient city states; and (III) importance of agriclture and horticulture including viticulture ? (A) East European (B) Indian (C) Mediterranean (D) Chinese

34. The paradoxical situation of co-existence of the patrilineal society and the dominating position of women characterize— (A) Khasi Tribe (B) Bhil Tribe (C) Gond Tribe (D) Tharu Tribe 35. Geography of public policy concerned with Geographical study of— 1. Creation and implementation of public policies. 2. Implementation and monitoring of public policies. 3. Creation and evaluation of public policies. 4. Evaluation of public policies. (A) 1 and 2 are correct (B) 2 and 3 are correct (C) 2 and 4 are correct (D) 1 and 3 are correct 36. Which one of the following languages belong to the Semito-Hamitic linguistic family ? (A) Gondi (B) Hebrew (C) Spanish (D) Dutch 37. The statement that “Who rules East Europe, commands the Heartland who rules the Heartland, commands the world Island…” was made by— (A) Mackinder (B) Spykman (C) Alfred Mahan (D) Whittlesey 38. The entire planning technique in India had been based on the model prepared by— (A) P. C. Mahalanobis (B) Fieldman (C) Domar (D) V.M. Dandekar 39. Regional development is a— (A) Single functional Approach (B) Multi-functional Approach (C) Place Prosperity (D) Rationalisation of population distribution

40. According to Hufschmidt, bringout the chronological sequence in the main phases of development of Regional planning in the last 100 years of its history— I. Cultural Regionalism Orientation. II. Metropolitan Orientation. III. Naturl Resource Orientation. IV. Regional Science Orientation. Codes : (A) III, II, IV, I (B) I, III, IV, II (C) IV, I, III, II (D) III, I, IV, II 41. Who has defined that “there is only one region—the surface of the earth on which mankind finds its home” ? (A) Minshull (B) Vidal de la Blasche (C) Fenneman (D) Herbertson 42. The Agricultural region is a— (A) Homogenous region (B) Functional region (C) Nodal region (D) Natural region 43. Which one of the following sequences of the Himalayan ranges from South to North is correct ? (A) Karakorm–Ladakh–Zaskar–Pir Panjal (B) Ladakh–Zaskar—Pir Panjal–Karakoram (C) Zaskar–Pir Panjal–Karakoram–Ladakh (D) Pir Panjal–Zaskar–Ladakh–Karakoram 44. The oldest and the richest coal field of India with respect to quantity of production and quality is— (A) Bokaro (B) Jharia (C) Raniganj (D) Makum-Nazira 45. The highest and the lowest population density difference during 2001 and 2011 was recorded respectively in— (A) Haryana and Arunachal Pradesh (B) Uttar Pradesh and Tamil Nadu (C) West Bengal and Nagaland (D) Bihar and Nagaland 46. The most important source of irrigation of Great Indian Desert (Thar) is— (A) Canal (B) Well (C) Tube-well (D) All of these

UGC-NET/JRF Geography-II (J-14) | 7 47. The longest west flowing Peninsular river is— (A) Godavari (B) Krishna (C) Narmada (D) Tapti 48. Match List-I with List-II and select the correct answer from the codes given below— List-I (Spectral Bands) (a) Visible region (b) Infrared (c) Microwave (d) Short radio waves List-II (Wavelength) 1. 102 -106 micrometre 2. 1 mm–1·0 metre 3. 0·7–10 3 micrometre 4. 0·4–0·7 micrometre Codes : (a) (b) (c) (d) (A) 4 2 3 1 (B) 3 4 1 2 (C) 4 3 1 2 (D) 3 4 2 1 49. Flow maps are used to show the distribution of which type of following data ? (A) Line (B) Area (C) Point (D) Volume 50. Cumulative frequency distribution is shown by— (A) Histogram (B) Frequency Curve (C) Ogive (D) Pie diagram

Answers with Hints 1. (C) Mono-cycle landscape is a topography that has been created by a single-cycle of erosion, while polycylic landscape or tract of a country is that Geomorphic features which have been developed under several cycles or parts cycles of erosion. 2. (B) Sief is an Arabic term adopted to describe a knief-edge-ridge of sand or longitudinal dune. Its axis lies paralled with the direction of wind and may extent to many kms. 3. (A) Consequent, a stream in which the direction of flow is controlled by or dependent upon the original slope of land. 4. (A) 5. (C)

6. (B) Koteshwaram (1958) in his article entitled “The Easterly set stream in the tropics” emphasized that Tibetean plateau situated at the height of 5 km from M.S.L. heated with Sun’s radiation and acts as a heater, by which a clockwise air circulation develops from East to West at the height over peninsulor India and descends Southward after cooling in South west direction proceeds towards North India. Due to lack of this, heavy rain occurs. 7. (D) As per Koppen’s climatic classification Jammu & Kashmir comes under ‘E’ climate, while Sikkim, Arunachal Pradesh etc. in Dfc climate. 8. (C) Magnetism is a factor that does not affects wind. 9. (D) Mid Latitude continental centre will have highest range of temperature Equatorial being throughout hot troprical coast with moderate climate and polar icecap, with throughout snow, donot have high range of temperature. 10. (C) Sublimation is a process where by a solid converted into gas with no liquid state intervening. 11. (D) The main cause of Carbon monoxide as a air pollutants is Gasoline motor-exhausts. 12. (C) Upwelling transfer of colder water from lower layers upwards to the higher layers of a waterbody by means of a current, such transfers are common in the zones of divergence of occean currents, thereby replacing the warmer surface layers by upwelling the colder water from below. 13. (C) Biogenic sediment is a sediment that is created by living organism either animal or plant e.g. coral limstone. 14. (D) Vast number of Herbivorous animals are Carnivorous such as Lions, Jackals or Hyenas are found in Savannas of Africa. 15. (A) 16. (B) According to some scholar. Geography is an idiographic Science for others it is a nomethetic Science. Which deals with general laws. Systematic Geog. or General Geog, comes under this category. Humboldt, Sauer and their followers laid stress on systematic Geography and the formulation of general law. 17. (C) 18. (A)

8 |

UGC-NET/JRF Geography-II (J-14)

19. (B) Correct sequence of the German Geographical thought is as follows Peschel (18261875) Ralzel (1844-1904) Heltner (18591941), Troll (1899-1968). 20. (D) While dealing with the subject matter of Geography Humboldt coined the term ‘Cosmography’ and divided it into Uranography and Geography. 21. (C) Perspective on Nature of Geog. (1959) was written by Richard Hartshone. 22. (B) 23. (A) Rural house is an index of environment because material used in them is according to environment. 24. (A) 25. (C) Sledge is a means of transport used by Tundras People. 26. (A) 27. (D) 28. (D) High level managerial executive and administrated position comes under the quarternary activities. 29. (C) 30. (A) Edward Ullman’s model of Spatial interaction is based on complementary, transferability and intervening opportunities. 31. (C) Japan is one of the leading industrial countries due to development of Hydel power as well as high level of technological ability. Though Japan is poor in mineral resources.

32. (B) 33. (C) 34. (D) 35. (B) 36. (B) 37. (A) 38. (A) 39. (B) Regional development is a multifunctional approach. 40. (D) 41. (A) 42. (C) 43. (D) Correct sequence of Himalayan Ranges from South to North is as follows—Pirpanjal —Zaskar—Ladakh—Karakoram. 44. (C) With respect to the quantity and quality Raniganj is the oldest and the richest coal field in India. 45. (D) During 2001-2011 census the highest and lowest population density difference is the Bihar—1106 – 881 = 225 persons, Nagaland 120–119 = 1. 46. (A) The most suited method of irrigation is Great Indian Desert (Thar) in Canal. Though wells and tubewells can be used, but it is not possible because water-level in this desert will be at very low level. 47. (C) Narmada and Tapti are the two rivers which flow towards the west. Narmada is 1079 km long while Tapti is 724 km long. 48. (C) 49. (D) Flow maps are used to show the distribution of volume on the map. 50. (C) Ogive is used to show the comulative frequency distributer, rather than Histiongram, frequency curve and Pie-diagram.

Geography (Paper-III) UGC-NET/JRF-Exam., 2014 Solved Paper

June 2014

Geography (Paper III) Note : This paper contains seventy five (75) objective type questions of two (2) marks each. All questions are compulsory. 1. Lateral Planation Theory of pediment formation was proposed by— (A) Lawson (B) Davis (C) Gilbert (D) McGee 2. Match List-I with List-II and select the correct answer from the codes given below— List-I (Books) (a) Geomorphology (b) Essays in Geomorphology (c) Physical Geography (d) An outline of Geomorphology List-II (Authors) 1. Wooldridge and Morgen 2. Strahler 3. Lobeck 4. Dury Codes : (a) (b) (c) (d) (A) 3 4 2 1 (B) 1 4 2 3 (C) 3 2 4 1 (D) 4 3 1 2 3. Which of the following is strongly presumptive faultline scarps ? (A) Superimposed drainage across a fault (B) Poor correlation between rock resistence and topographic forms (C) Frequent severe earthquakes (D) Actual Fault Plane identified along a scarp 4. Paternoster lakes are features typical of— (A) Lava Plains (B) Piedmonts (C) Deserts (D) Glacial Troughs

5. Match List-I with List-II and select the correct answer from the codes given below— List-I (Theories of Cavern formation) (a) Two cycle theory (b) Water table theory (c) Static waterzone theory (d) Invasion Theory List-II (Scholar) 1. Davis 3. Malott Codes : (a) (b) (A) 1 4 (B) 1 2 (C) 4 3 (D) 2 1

2. Garner 4. Swinnerton (c) 2 3 1 4

(d) 3 4 2 3

6. Which one of the following processes is typical to glacial erosion ? (A) Plucking (B) Hydrolic action (C) Deflation (D) Corrosion 7. Given below are two statements, one labelled as assertion (A) and the other labelled as Reason (R). Select your answer from the codes given below— Assertion (A) : Deep chemical decay of rocks is one of the outstanding features of humid tropical regions. Reason (R) : Intensity of chemical weathering depends to a large degree upon an abundance of water and high air temperatures. Codes : (A) Both (A) and (R) are true and (R) is correct explanation of (A) (B) Both (A) and (R) are true, but (R) is not correct explanation of (A)

UGC-NET/JRF Geography-III (J-14) | 3 (C) (A) is true, but (R) is false (D) (R) is true, but (A) is false 8. Crickmay, while disagreeing with Davis, supplanted peneplain with the term— (A) Pediplain (B) Panplane (C) Etch plain (D) Structural plain 9. What is the correct Sequence of the cloud types in a warm front away from its trailing edge ? (A) Altostratus – Nimbostratus – Cirrus– Cirrostratus (B) Cirrostratus–Nimbostratus–Altostratus – Cirrus (C) Cirrus – Nimbostratus – Altostratus – Cirrostratus (D) Nimbostratus – Alto Stratus – Cirrostratus – cirrus 10. Westerlies are examples of— (A) Macro atmospheric motions (B) Tornados (C) Microscale atmospheric motions (D) Gusts 11. Match the following List-I with List-II and select the correct answer from the code given below— List-I (a) Temparature falls with height (b) Reflects radio waves back to earth (c) Contain most of the ozone (d) Fall in temperature stops List-II 1. Ionosphere 2. Stratosphere 3. Tropopause 4. Troposphere Codes : (a) (b) (c) (d) (A) 4 2 1 3 (B) 3 2 4 1 (C) 4 1 2 3 (D) 4 3 1 2 12. If the elevation of the sun declines, the albedo will— (A) Decline (B) Increase (C) Remain the same (D) No change

13. Seasonal contrast in pressure between land and sea gives rise to— (A) Monsoons (B) Westerly winds (C) Land and Sea breezes (D) Trade winds 14. V. Bjerknes Model is related with— (A) Origin of anticyclones (B) Origin of Mid Latitude Cyclones (C) Origin of Tropical Cyclones (D) Origin of monsoon 15. Which one of the following is not correct ? (A) Cold front is marked with triangular spikes (B) Warm front is marked with Semicircles (C) A fast advancing warm front can cause violent lifting (D) The tornados develop along squall line 16. Some heat is received by the atmosphere even after Sunset from— (A) Invisible Solar radiation (B) Latent heat (C) Radioactive processes (D) Terrestrial radiation 17. Given below are two statements, one labelled as Assertion (A) and the other labelled as Reason (R). Select your answer from the codes given below— Assertion (A) : The growth of grass and other vegetative cover is limited to selvas Reason (R) : The separate crowns of trees coalesce and form a thick canopy above the forest floor. Codes : (A) Both (A) and (R) are true and (R) is correct explanation of (A) (B) Both (A) and (R) are true, but (R) is not correct explanation of (A) (C) (A) is true, but (R) is false (D) (R) is true, but (A) is false 18. ‘Montreal Protocol’ is related to— (A) Global Warming (B) Ozone Depletion (C) Air Pollution (D) Water Pollution

4 | UGC-NET/JRF Geography-III (J-14) 19. The objective of Kyoto Protocol was to reduce green house gas emission by— (A) An average of 10 per cent below 1995 level by the year 2010 (B) An average of 8 per cent below 1998 level by the year 2012 (C) An average of 20 per cent below 1997 level by the year 2009 (D) An average of 5 per cent below 1990 level by the year 2012 20. Xerophytes can tolerate and stand against— (A) High temperature (B) Severe Cold (C) Humidity (D) The drought Conditions 21. Consider the following statements— 1. Ozone is found mostly in the stratosphere. 2. Ozone layer lies 55–75 km above the surface of the earth. 3. Ozone absorbs ultra-violet rays from the Sun. 4. Ozone layer has no significance for life on the earth. Which of the above statements are correct ? (A) 1 and 3 (B) 2 and 4 (C) 2 and 3 (D) 1 and 4 22. Given below are two statements, one labelled as Assertion (A) and the other labelled as Reason (R). Select your answer from the codes given below— Assertion (A) : Oceanic salinity is low in the equatorial region. Reason (R) : Equatorial region is characterized by heavy rainfall, cloudiness and humidity. Codes : (A) Both (A) and (R) are true and (R) is the correct explanation of (A) (B) Both (A) and (R) are true and (R) is not correct explanation of (A) (C) (A) is true, but (R) is false (D) (A) is false, but (R) is true 23. The statement—“Systematic geography must move into theoretical spheres and Regional Geography into a search for generic and not unique studies” was made by—

(A) (B) (C) (D)

Richard Hartshorne Issiah Bowman William Bunge Peter Haggett

24. In the development of Quantitative Revolution, a new field of study called ‘Social Physics’ stating that “dimensions of society are analogous to the physical dimensions and include numbers of people, distance and time” was introduced by— (A) J. Q. Stewart (B) Edward Ullman (C) W. L. Garrison (D) William Warntz 25. Given below are two statements, one laballed as Assertion (A) and the other labelled as Reason (R)— Assertion (A) : The major Thrust of Quantitative Revolution is to bring law making, model building and Theorization in Geography. Reason (R) : After the second world war, young geographers discarded the empirical, descriptive and gazetteer type of Geography by using the mathematical language rather than the language of literature. Select your answer from the codes given below— Codes : (A) (A) is correct, but (R) is wrong (B) (A) is wrong, but (R) is correct (C) Both (A) and (R) are correct and (R) is the correct explanation of (A) (D) Both (A) and (R) are wrong 26. Match List-I with List-II and select the correct answer using the codes given below— List-I (a) T. Haggerstrand (b) William Bunge (c) Gilbert White (d) Walter Isard List-II 1. Theoretical Geography 2. Geographic Dissemination of Innovations 3. Methods of Regional Analysis 4. Human Responses to Floods

UGC-NET/JRF Geography-III (J-14) | 5 Codes : (a) (A) 2 (B) 2 (C) 4 (D) 3

(b) 1 3 2 4

(c) 4 4 3 1

(d) 3 1 1 2

27. Match List-I and List-II and select the correct answer using the codes given below— List-I (a) Otto Schluter (b) Von Richthofen (c) Oscar Peschel (d) Friedrich Ratzel List-II 1. Political geography 2. Das Ausland 3. Major work on China 4. Visible content of the landscape Codes : (a) (b) (c) (d) (A) 4 1 2 3 (B) 4 3 2 1 (C) 3 4 1 2 (D) 2 1 4 3 28. Which one of the following statements is not true about Humanism in Geography ? (A) Humanism does not treat humans as machines (B) Humanistic geography gives importance to human awareness, human consciousness and human creativity (C) Humanists accept the reduction of space and place to geometrical concepts of surface. (D) Humanism developed as criticism against positivism and quantitative revolution 29. Match List-I with List-II and select the correct answer using the codes given below— List-I (Philosophical approaches) (a) Idealistic (b) Phenomenological (c) Radical (d) Welfare List-II (Advocates) 1. Relph 2. Harris and Guelke 3. Smith 4. Bunge

Codes : (a) (A) 4 (B) 2 (C) 3 (D) 2

(b) 2 1 4 3

(c) 1 4 2 1

(d) 3 3 1 4

30. Who among the following was a strong supporter of Ritter’s teleological ideas ? (A) Guyot (B) Peschel (C) Gerland (D) Ratzel 31. A process of population deconcentration away from large urban settlements to nonmetropolitan areas is termed as— (A) Urban dispersal (B) Urban sprawl (C) Counter-urbanization (D) Sub-urbanization 32. In the acceleration stage of urbanization the urban population constitutes— (A) More than 70% of the total population of the state (B) 25% to 70% (C) 10% to 25% (D) Less than 10% 33. The concept of primate city was advanced by— (A) Jefferson (B) Zift (C) Mumford (D) Sjoberg 34. Given below are two statements, one laballed as Assertion (A) and the other labelled as Reason (R). Select your answer from the codes given below— Assertion (A) : Losch’s model is less restrictive than Christoller’s. Reason (R) : Losch treated each function as having a separate range, threshold and hexagonal hinterland. Codes : (A) Both (A) and (R) are true and (R) is the correct explanations of (A) (B) Both (A) and (R) are true, but (R) is not the correct explanation of (A) (C) (A) is true, but (R) is false (D) (A) is false, but (R) is true

6 | UGC-NET/JRF Geography-III (J-14) 35. Which of the following conditions is characterised by the second stage of Demographic Transition ? (A) Low birth rate, High death rate (B) High birth rate, High death rate (C) High birth rate, declining death rate (D) Low birth rate, Low death rate 36. Match following List-I and List-II and select the correct answer from the codes given below— List-I (Books) (a) The Nature of Geography (b) The Urban Question (c) Social Justice and the City (d) Locational Analysis in Human Geography List-II (Authors) 1. D. Harvey 2. P. Haggett 3. R. Hartshorne 4. M. Castells Codes : (a) (b) (c) (d) (A) 4 1 2 3 (B) 3 4 1 2 (C) 1 2 3 4 (D) 2 3 4 1 37. ‘Chain migration’ is based on— (A) Job opportunities (B) Kinship (C) Proximity to place of earlier Residence (D) Combination of (A) and (C) 38. Which one of the following pairs is not correctly matched ? Scholar Concept (A) E. W. Burgess – Concept of megalopolis (B) W. J. Reily – Law of retail gravitation (C) G. K. Ziff – Rank-size rule (D) Patrick Geddes – Concept of conurbation 39. Consider the following statements and select the correct answer from the given codes— 1. Most of the fishing grounds occur in areas where the continental shelf is wide.

2.

Fishing activity is well developed in warm tropical waters. 3. Mixing of warm and cold currents brings plant nutrients for fish. 4. Inland fisheries are more significant than other types in India. Codes : (A) 1 and 2 are correct (B) 1 and 3 are correct (C) 2, 3 and 4 are correct (D) 1, 2 and 3 are correct 40. Given below are two statements, one labelled as Assertion (A) and the other labelled as Reason (R). Assertion (A) : There are regional disparities in the spatial organisation of economy. Reason (R) : Interactions among inherent factors in economic activities affect the texture of spatial organisations. In the context of the above two statements, which one of the following is correct ? Codes : (A) Both (A) and (R) are true and (R) is the correct explanation of (A) (B) Both (A) and (R) are true, but (R) is not the correct explanation of (A) (C) (A) is true, but (R) is false (D) (A) is false, but (R) is true 41. Which one of the following crop practices refers to crop rotation ? (A) Cultivating more than one crop in the same field in different seasons (B) Cultivating the same crop in the same field in different seasons (C) Cultivating many crops in the same field in one season (D) Cultivating one crop in one season followed by a different crop in the next season in the same field 42. When the raw material used in an industry is gross and ubiquitous, the location of the firm will be— (A) Near the source of raw material (B) Anywhere between the source of raw material market (C) Near the market (D) Near the capital city

UGC-NET/JRF Geography-III (J-14) | 7 43. In Great Britain a little pulp is produced, but paper making is important, while in Sweden pulp manufacture is far more important than paper making because– (A) Great Britain is technologically advanced as compared to Sweden. (B) Pulp making is raw material oriented, paper making is more market oriented (C) Sweden is a poor country (D) Great Britain has plenty of forest resources for paper making

Select the correct connectivity matrix for the above network— (A)

b 1 0 1 1 1

c 2 1 0 0 0

d 1 2 1 1 1

e 1 2 1 1 1

a b c d e

a 0 1 1 1 0

b 1 0 1 0 0

c 1 1 0 1 1

d 1 0 1 0 0

e 0 0 1 0 0

a b c d e

a 0 2 0 0 2

b 1 3 1 0 3

c 3 1 0 0 1

d 2 3 1 1 0

e 2 0 1 1 0

a b c d e

a 0 2 2 1 1

b 1 4 3 2 1

c 3 1 1 0 0

d 1 4 0 1 1

e 0 0 2 1 1

(B)

44. Match List-I with List-II and select the correct answer from the codes given below— List-I (Location) (a) Detroit (c) Akron List-II (Industry)

a b c d e

a 0 1 1 0 1

(b) Yokohama (d) Glasgow (C)

1. Iron and Steel 3. Ship building Codes : (a) (b) (c) (A) 2 1 3 (B) 4 3 2 (C) 3 2 1 (D) 1 2 3

2. Synthetic rubber 4. Automobile (d) 4 1 4 4

45. Break-of-bulk point is a place— (A) Where large sized and bulky goods are crushed into (B) Where goods are transferred from one mode of transport to another (C) Where packaging of the bulky goods is carried out at large scale (D) The place meant for the transport of bulky products 46. Consider the following road network diagram of an area : a

b

c d e

(D)

47. Which one of the following theories emphasizes the policy oriented concepts such as propulsive firms, leading industries and agglomeration ? (A) Central Place Theory (B) Theory of Industrial location (C) Growth pole theory (D) Theory of spatial organization 48. The techniques usually employed for actual delineation of functional regions are— (A) Flow and gravitational analysis (B) Simple correlation analysis (C) Composite index method analysis (D) Ranking coefficient method analysis

8 | UGC-NET/JRF Geography-III (J-14) 49. The gravity principle in identification of nodal regions states that the interaction between two geographic points is directly related to— (A) Their masses (B) Their distance (C) Their mode of transport (D) Their size of settlements 50. Match the following— List-I (Regional Planners) (a) V. Nath (b) Bhat and Rao (c) Sen Gupta & Sdasyuk (d) C. S. Chandrasekhara List-II (Number of major planning regions identified for India) 1. 13 2. 19 3. 11 4. 15 Codes : (a) (b) (c) (d) (A) 4 3 2 1 (B) 4 2 3 1 (C) 3 4 1 2 (D) 2 3 4 1 51. Given below are two statements; one labelled as Assertion (A) and other labelled as Reason (R). Assertion (A) : A planning region should be large enough to contain a range of resources, conditions and attributes so as to serve a desired degree of economic viability. Reason (R) : Its resource position should be such that a satisfactory level of product combination for consumption and for exchange would be feasible. Select your answer from the codes given below— Codes : (A) Both (A) and (R) are true and (R) is the correct explanation (B) Both (A) and (R) are true, but (R) is not the correct explanation (C) (A) is true, but (R) is false (D) (A) is false, but (R) is true

52. Preparation of Sub-plans on regional basis was taken up in— (A) Fourth plan (B) Fifth plan (C) Sixth plan (D) Seventh plan 53. Who among the following presented a framework of economic regions of different orders following the soviet concept of economic regions and production specialisation ? (A) P. Sen Gupta (B) V. Nath (C) S. P. Chatterjee (D) Bhat and Prakasa Rao 54. Which one of the perienced the lowest during 2001–11 ? (A) Tamil Nadu (C) Kerala

following states exgrowth rate of literates (B) Nagaland (D) Goa

55. Which one of the following places in India does not get the vertical rays of the Sun ? (A) Srinagar (J & K) (B) Mumbai (C) Chennai (D) Thiruvananthapuram 56. Which one of the following atomic power stations is located in Tamil Nadu ? (A) Tarapur (B) Kota (C) Kalpakkam (D) Narora 57. Which one of the following matches is notcorrect ? (A) Himadri – Greater Himalya (B) Shiwalik – Outer Himalaya (C) Western Ghat – Peninsular India (D) Gondwanaland – Lesser Himalaya 58. Which one of the following major seaports of India does not have natural harbour ? (A) Mumbai (B) Cochin (C) Marmagao (D) Paradeep 59. The member of agro-climatic regions in India as per Planning Commission is— (A) Twelve (B) Thirteen (C) Fourteen (D) Fifteen 60. About three-fourth of manganese ore reserves of India are found in— (A) Odisha, Madhya Pradesh, Maharastra, Goa

UGC-NET/JRF Geography-III (J-14) | 9 (B) Karnataka, Odisha, Madhya Pradesh, Maharastra (C) Karnataka, Madhya Pradesh, Goa, Andhra Pradesh (D) Madhya Pradesh, Maharastra, Andhra Pradesh, Jharkhand 61. Match List-I with List-II and select the correct answer from the codes given below— List-I (Statistic) (a) Standard Distance (b) Nearest Neighbour (c) Correlation (d) Eigen Value List-II (Analysis) 1. Principal Component 2. Scatter diagram 3. Settlement pattern 4. Centrographic measure Codes : (a) (b) (c) (d) (A) 3 4 1 2 (B) 4 3 2 1 (C) 1 4 3 2 (D) 2 3 4 1 62. Which of the following is a GIS operation ? (A) Image displaying (B) Contrast stretching (C) Map over laying (D) Map designing 63. Which of the following diagram is used to show proportion of different land uses in an area ? (A) Pie (B) Wind rose (C) Line (D) Flow-Chart

66. Match List-I with List-II and select a correct answer using the codes given below— List-I (Instruments) (a) GPS base (b) Pantograph (c) Planimeter (d) Stereoscope List-II (Functions) 1. Distance and area 2. 3-D Vision 3. Global location 4. Reduction and enlargement Codes : (a) (b) (c) (d) (A) 3 1 4 2 (B) 2 4 3 1 (C) 3 4 1 2 (D) 1 2 3 4 67. The most appropriate sampling Technique to represent the heterogenous population of a region is— (A) Cluster (B) Random stratified (C) Systematic (D) Purposive 68. Which one of the following characteristic features of Indian federalism does not have a geographical base ? (A) Vast area size (B) Physical and cultural diversities (C) Regional inequalities in socio-economic development (D) Division of powers and functions between Union and states

64. Mean, median and mode coincide if the distribution of values is— (A) Negativety skewed (B) Normal (C) Positivety skewed (D) Poisson

69. Which of the following countries is known as a Unique assemblage of human races in the world ? (A) Union of South Africa (B) U. S. A. (C) Indonesia (D) India

65. The contour interval followed in SOI Toposheet with 1/50,000 scale is— (A) 20 metres (B) 50 metres (C) 100 metres (D) 150 metres

70. ‘Geostrategic model’, dividing the world into ten regions, was propounded by— (A) P. J. Taylor (B) John Short (C) John Agnew (D) Saul Cohen

10 | UGC-NET/JRF Geography-III (J-14) 71. Match items in List-I with items in List-II and select the correct answer using the codes given below— List-I (Name of Administrative Unit) (a) District (b) Department (c) Country (d) Kreise List-II (Name of the Country) 1. France 2. India 3. Germany 4. Great Britain Codes : (a) (b) (c) (d) (A) 1 2 3 4 (B) 4 2 3 1 (C) 2 1 4 3 (D) 3 2 1 4 72. For measuring the success of changes in electroral boundaries, we use the concept of— (A) Representation (B) Malapportionment (C) Legitimate participation (D) Non-legitimate participation 73. Welfare approach in geographic studies emphasises spatial variation in— (A) Socio-Economic development (B) Public-private partnership in development (C) Infrastructural development (D) Quality of life 74. The ‘Cultural Turn’ in geography has been responsible for— (A) Bringing social and cultural geography closer to each other (B) Social geography taking over cultural geography (C) Creating void between social and cultural geography (D) Reducing the significance of Social geography 75. Match List-I with List-II and select the correct answer from the codes given below— List-I (Industry) List-II (Location) (a) Aluminium 1. Mysore (b) Copper smelting 2. Renukoot (c) Zink smelting 3. Vishakhapatnam (d) Silk 4. Khetri

Codes : (a) (A) 2 (B) 1 (C) 4 (D) 3

(b) 4 2 3 2

(c) 3 4 1 4

(d) 1 3 2 1

Answers with Hints 1. (C) Gilbert in 1877 in Henry Mountains of Utah described the formation as hills of Planation cut across the upturned edges in tilted bed. 2. (A) Correct match of the list-I and list-II is as follows : Geomorphology → Lobeck, Essays in Geomorphology → Dury, Physical Geography → Strahler, An outline of Geomorphology → Wooldridge and Morgen. 3. (D) 4. (D) Paternoster lakes is a series of lakes in a Glaciated valley, impounded behind moraine or ice rock basins closed at their lower and by rock bars. 5. (A) The correct sequence of Cavern formation theory along with their scholar is as follows : Two cycle theory → Davis, Water table theory → Swinnerton, Static wate zone theory → Gardner, Invasion theory → Malott. 6. (A) Plucking is a process of block removal or Quarrying carried out by glacier ice whereby the well jointed blocks are mechanically pulled from the rock floor, generally the down stream side of a rocky protuberance. 7. (C) Chemical weathering is the processes which lead to the decomposition or breaking of solid rocks by means of chemical reaction. It comprise carbonation, hydrolysis, oxidation, reduction etc. Though in this process water plays a deceive role, but high temp. does not play an important role. 8. (B) Crickmay in his discussion used the world panplane in place of peneplain. 9. (D) In warm front, the arrival of front is indicated by the formation of cirrus clouds across a clean sky followed in succession by Cirro-stratus → alto-stratus and finally Nimbostratus (at tail). 10. (A)

UGC-NET/JRF Geography-III (J-14) | 11 11. (C) Correct match of List-I with List-II is : Temperature falls with height →Troposphere Reflects radio waves to Earth→Inosphere Contain most of the ozone →Stratosphere Fall in temperature stops→Tropopause 12. (B) Albedo means lose of energy without heating the earth : with the elevation of the sun decline, the albedo will increase. 13. (A) Due to seasonal constrast in presure between land and sea Monsoon winds originate. 14. (B) V. Bjerknes model is related with Polar front theory i.e., the origin of Mid-latitude cyclons. 15. (C) 16. (B) It is due to latent heat (condensation) atmosphere heated even after Sunset. 17. (D) In Selvas (Equatorial forest) in Amezan basin, the grass and other vegetable covers are not found, but the separate crowns of trees coalesce and form a thick canopy above the forest floor is common. 18. (B) Montreal Protocol is related to Ozone depletion. 19. (D) The object of Kyoto Protocol was to reduce the emission of Green house gases at an average of 5% below 1990 level by year 2012. 20. (D) The Xerophytes vegetation can tolerate the drought condition. 21. (A) Ozone is found mostly in stratosphere at the height of 20 to 25 km, absorbs the Ultraviolet rays from the Sun. 22. (A) Due to heavy rainfall, cloudiness and humidity the occanic salinity is low in the equatorial region. 23. (C) 24. (A) 25. (C) 26. (A) Correct Match of List-I and List-II is as follows : Geographic—Dissemination of Innovations— T. Haggerstrand Theoretical Geography—William Bunge Human Responses towards the Floods— Gilbert White Methods of Regional Analysis—Walter Isard 27. (B) Correct Match of List-I and List-II is as follows : Friedrich Ratzel→Political Geography

28. 29.

30. 32. 33. 34.

35.

36.

37. 38.

Oscar Peschel→Das Ausland Von-Richthofen→Major work on China Otto-Schluter→Visible content of Landscape (C) (B) Correct Match of List-I Philosophical approaches and List-II Advocates is as follows : Idealistic →Harris and Guelke Phenomenological→Relph Redical →Bunge Welfare→Smith (A) 31. (B) (B) 25% to 70% population accelerated in the stage of urbanization. (A) Concept of primate city was advanced by M. Jefferson. (A) Losch’s model was much less restrictive than Christoller’s. Rather than bunch all functions into seven ‘orders’ he treated each as having a separate range, threshold and hexagonal hinterland. Where feasible, functions were clustered into settlements but all central places with certain functions in them need not as in Christoller’s scheme also contain all the functions with smaller ranges and thresholds. This produced a much wider range of settlements in terms of size and complexity of business profiles : whereas Christoller’s theory produced a stepped hierarchical urban system, Losch’s was consistent with a more continuous distribution of urban sizes. (C) In the second stage of Demographic Transition High birth rate and declining death rate, couses rapid population growth. (B) Correct Match of List-I Books and List-II Authors is as follows : The Nature of Geography→R. Hartshorne The Urban Question →M. Castells Social Justice and the city→D. Harvey Locational Analysis in Human Geography → P. Hagget. (D) (A) Concentric zone theory was profounded by E. W. Burgess, Not the concept of Megalopolis which was Given by Gean Gaterman.

12 | UGC-NET/JRF Geography-III (J-14) 39. (C) It is not necessary that fishing ground occurs only in the areas where the continental shelf is wide. 40. (A) 41. (D) 42. (B) 43. (B) Great Britain has the advantage of trade for paper with other countries, which Sweden does not have. 44. (B) 45. (B) Break of Bulk point means where goods are transferred from one mode of transport to another mode. 46. (B) 47. (C) 48. (D) 49. (B) 50. (A) V. Nath gave 15 main and 48 sub-regions based at the homogeneity in physical factors, and agricultural land use and cropping pattern. ● L. S. Bhatt and V.L.S. Prakasa Rao proposed a regional framework for resource development and gave 11 major regions. ● P. Sengupta and Sdasyuk gave 3 macro regions and 19 sub-regions based on population resources. ● C. S. Chandrashekhara gave 13 macro and 35 meso planning regions based on criteria of physical economic and ecological factors to demarcate the macro planning regions. 51. (A) 52. (B) 53. (A) 54. (C) Following is the growth of Literacy rate in the states in between 2001–11 : States

2001

2011

Growth

Kerala Tamil Nadu Goa Nagaland

94·2 82·4 88·4 71·2

96·1 86·8 92·6 82·8

1·9 4·4 4·2 11·6

55. (A) Sun rays never fall vertically away from 1 1 23 ° N. Lat. to 23 ° S. Lat. Sri Nagar is 2 2 1 situated away from 23 ° N, hence the SUN 2 rays will never fall vertically at this place. 56. (C) 57. (D) Gondwanaland is related to Pengea while lesser Himalaya is related to Miocene period (Tertiary period).

58. (D) Paradeep (Odisha) is not a natural harbour, while other threes are natural harbours. 59. (D) 60. (B) Karnataka (22%), Odisha (45%), Madhya Pradesh (13%) and Maharashtra (8%) contributes 3/4th of manganese ore reserves in India. 61. (D) 62. (C) 63. (A) 64. (B) 65. (A) 66. (C) 67. (B) 68. (D) 69. (D) In India Drawadian Mangol, Aryan, Negreto and Caucasoid races are found. 70. (D) Today geographers now view the world in terms of spatial patterns that are not containable within national boundaries. They see the world as an interdependent system and the nation-state as part of a world that is shared area. Saul Cohen wrote Geography and Politics in a World Divided. Cohen divides the world into two geostrategic regions viz., trade dependent maritime regions and Eurasian Continental Realm, which is interior in direction along with 10 geopolitical regions. 71. (C) The correct match of the administrative units and country is as follows : District—India Department—France Country—Great Britain Kreise—Germany 72. (B) Malapportionment occurs “when geographical units have shares of legislative seats that are not equal to their share of population. Malapportionment as a formal and often deliberate pathology of electoral system” in democratic regions. 73. (D) 74. (A) ‘Cultural Turn’ in Geography is responsible to forming social and cultural Geography together. 75. (A) Correct match of Industries and their location is as follows : Aluminium—Renukoot Copper smelting—Khetri Zink smelting—Vishakhapatnam Silk—Mysore

Geography (Paper–II)

UGC-NET/JRF Exam. (December 2014) Solved Paper

December 2014

Geography (Paper–II) Note—This paper contains fifty (50) objective type questions of two (2) marks each. All questions are compulsory. 61. The term ‘active layer’ is associated with— (A) Fluvial deposition (B) Periglacial processes (C) Plate tectonic (D) Processess of soil formation 62. Match List-I with List-II and select the correct answer from the codes given below— List-I (Concept theory) (a) Base, level (b) Parallel retreat (c) Cycle of erosion (d) Stream competency and capacity List-II (Put forwarded by) 1. G. K. Gilbert 2. W. Penck 3. J. W. Powell 4. W. M. Davis Codes : (a) (b) (c) (d) (A) 3 2 4 1 (B) 2 1 3 4 (C) 3 2 1 4 (D) 2 4 1 3 63. Hydration process belongs to— (A) Physical and Biological processes (B) Physical and Chemical processes (C) Biological and Chemical process (D) Chemical processes 64. The basaltic ocean crust has an average density of— (A) 2·65 g/cm3 (B) 2·75 g/cm3 3 (C) 2·90 g/cm (D) 3·0 g/cm3

65. What is the major wind related phenomenon responsible for summer drought in Mediterranean type of region ? (A) Shifting of trade winds polewards in summers (B) Shifting of trade winds equatorwards in summers (C) Blowing of Westerlies from the continents in summers (D) Blowing South-Westerly 66. Which one of the following is not a factor to affect the atmospheric pressure ? (A) Altitude (B) Presence of water vapour (C) Temperature (D) Rainfall 67. Find out the correct sequence of planetary winds found from equator to pole— (A) Westerlies, Trade, Polar wind (B) Trade, Westerlies, Polar wind (C) Trade, Polar wind, Westerlies (D) Polar wind, Westerlies, Trade 68. The rain-fed and rain-shadow areas are the characteristics of the— (A) Cyclonic ranifall (B) Convectional rainfall (C) Orographic rainfall (D) Thermal rainfall 69. In the List-I given the names of local wind and List-II given the names of the area. Match correctly List-I and List-II. Select the correct answer from the codes given below— List-I (Local Wind) List-II (Area) (a) Loo 1. The Rockies (b) Khamsin 2. The Alps (c) Chinoock 3. Rajasthan Desert (d) Fohn 4. Egypt

4 | Geography-II (U.G.C.) D-14 Codes : (a) (A) 1 (B) 3 (C) 2 (D) 4

(b) 2 4 4 3

(c) 3 1 3 2

(d) 4 2 1 1

10. Find combination of two given areas where rainfall is characterised by Tropical monsoon rain— I. The Sahyadri II. The Ganga Delta III. Kashmir Valley IV. Ladakh Region (A) I and III (B) II and IV (C) I and II (D) III and IV 11. Given below are two statements, one labelled as Assertion (A) and the other labelled as Reason (R). Select your answer from the codes given below— Assertion (A) : Deforestation result in the increase of concentration of carbon dioxide in the atmosphere. Reason (R) : Forests utilise carbon dioxide during the process of photosynthesis for manufacturing of food. Codes : (A) Both (A) and (R) are true and (R) is the correct explanation of (A) (B) Both (A) and (R) are true and (R) is not the correct explanation of (A) (C) (A) is true, but (R) is false (D) (A) is false, but (R) is true 12. Match List-I with List-II and select the correct answer from the codes given below— List-I List-II (a) Gour 1. Lithosphere (b) Graben 2. Littoral zone (c) Gabro 3. Aeolian area (d) Groin 4. Orogenic belt Codes : (a) (b) (c) (d) (A) 3 4 1 2 (B) 3 4 2 1 (C) 2 4 3 1 (D) 3 2 1 4

13. The ‘wave-cut platform’ is found in— (A) California coast (B) Hokkaido coast (C) Madagascar coast (D) Malabar coast 14. The average depth of ‘equatorial-counter’ current is— (A) 50 m (B) 100 m (C) 150 m (D) 200 m 15. The Gulf Stream and the Kuroshio Current move forcefully northward as a result of— (A) Western intensification (B) Eastern intensification (C) Northern intensification (D) Southern intensification 16. ‘The Principles of Human Geography’ was authored by— (A) E. Huntington (B) H. H. Borrow (C) F. Ratzel (D) E. C. Semple 17. The term ‘Cosmography’ was coined by— (A) Mackinder Halford J. (B) Humbolt (C) Kant Emmanuel (D) Ptolemy 18. Match the following List-I with List-II and select the correct answer from the codes given— List-I (a) Unity in Diversity (b) Social Determinism (c) Terrestrial Whole (d) Interaction List-II 1. Vidal-de-la Blache 2. Jean Brunhes 3. Carl Ritter 4. Frieduch Ratzel Codes : (a) (b) (c) (d) (A) 3 2 4 1 (B) 4 2 1 3 (C) 3 4 1 2 (D) 2 1 4 3 19. The concept of ‘Spread’ and ‘Backwash’ effects was introduced by— (A) A. O. Hirschman (B) Gunnar Myrdal (C) Gunder Frank (D) Sameer Amin

Geography-II (U.G.C.) D-14 | 5 20. The philosophical view advocating that man is responsible for making his own nature is known as— (A) Positivism (B) Functionalism (C) Existentialism (D) Pragmatism 21. Match the following List-I with List-II and select the correct answer from the codes given below— List-I (Authors) List-II (Settlements) (a) Chirstaller 1. Market Centres (b) Galphin 2. Industrial Nodes (c) Losch 3. Central Places (d) Weber 4. Rurban Centres Codes : (a) (b) (c) (d) (A) 3 4 1 2 (B) 4 3 2 1 (C) 1 2 3 4 (D) 2 1 4 3 22. Match the following List-I with List-II and select the correct answer from the codes given below— List-I (State) (a) Arunachal Pradesh (b) Jammu & Kashmir (c) West Bengal (d) Kerala List-II (Tribes) 1. Adiyam Kammar, Kondkappu 2. Asur, Birhor, Lepcha, Korwa 3. Dafla, Khampti, Singpho 4. Garra, Gaddi Codes : (a) (b) (c) (d) (A) 4 3 1 2 (B) 4 1 3 2 (C) 3 4 2 1 (D) 3 2 4 1 23. Which State in India has recorded the lowest growth rate during 2001–11 Census ? (A) Kerala (B) West Bengal (C) Rajasthan (D) Bihar 24. Read Assertion (A) and Reason (R) and find the correct answer from the codes given below—

Assertion (A) : The very high population density of more than 400 persons/km2 extends from upper Indus-Satlaj-Ganga Brahmaputra Valley and Delta. Reason (R) : This phenomena is attributed to rich alluvial soils, flat land, rice and wheat cultivation, irrigation facilities and favourable climatic conditions. Codes : (A) Both (A) and (R) are true and (R) is the correct explanation of (A) (B) Both (A) and (R) are true, but (R) is not the correct explanation of (A) (C) (A) is true, but (R) is false (D) (A) is false, but (R) is true 25. The decadal growth of population (1991– 2001) according to the 2001 Census is— (A) 23·86% (B) 21·34% (C) 24·66% (D) 24·80% 26. Among the following natural resources which resource is providing more livelihood to the people ? (A) Water (B) Land (C) Forest (D) Mineral 27. Which one of the following factors is more crucial for agricultural development ? (A) Tractors (B) Fertilizers (C) Irrigation (D) Gentle slope 28. Given below are two statements, one labelled as Assertion (A) and the other labelled as Reason (R). Select your answer from the codes given below— Assertion (A) : China has the very fast growing economy of the world. Reason (R) : China has rich mineral resources and huge diversified labour force. Codes : (A) (A) is true, but (R) is false (B) (A) is false, but (R) is true (C) Both (A) and (R) are true and (R) is not correct explanation (D) Both (A) and (R) are true and (R) is correct explanation 29. What is the ideal minimum temperature for growth of sugarcane ? (A) 10°C (B) 15°C (C) 20°C (D) 25°C

6 | Geography-II (U.G.C.) D-14 30. Who developed the technique to measure the agricultural efficiency ? (A) S. S. Bhatia (B) R. L. Singh (C) S. M. Rafiullah (D) L. D. Stamp 31. Given below are two statements, one labelled as Assertion (A) and the other labelled as Reason (R). Select your answer from the codes given below— Assertion (A) : Panchayati Raj Act empowered local bodies participation in planning process. Reason (R) : Panchayati Raj institutions are having three tier systems in planning and executing development projects within their jurisdiction. Codes : (A) (A) is true, but (R) is false (B) (A) is false, but (R) is true (C) Both (A) and (R) are true and (R) is correct explanation of (A) (D) Both (A) and (R) are true, but (R) is not the correct explanation of (A) 32. Who Popounded the Heartland Theory used in political geography ? (A) Alfred Mackinder (B) O. H. K. Spate (C) Arthur Geddes (D) J. A. Steers 33. What do you mean by ‘Lebensraum’ in political Geography ? (A) Dead space (B) Living space (C) Administrative unit (D) Religious organisation 34. Which one of the following factors affecting more at present to the voting behaviour in India ? (A) Development (B) Caste system (C) Religion base (D) Old base of political parties 35. Which of the following names of shifting cultivation in Odisha is correct ? (A) Podu (B) Bewar (C) Pendo (D) Jhum

36. India has been divided into 13 Macro Planning regions. Which of the following authors have delineated planning regions of India ? (A) P. Sen Gupta and Sadasyuk Gailina (B) R. P. Mishra, K. V. Sundaram and V.L.S. Prakasa Rao (C) L. S. Bhatta (D) O.H.K. Spate and Learmonth 37. Which of the following geographers defined regions as : “A georgraphic area unified culturally, unified at first economically and later by consensus of thought, education. recreation etc. which distinguishes it from other areas. ?” (A) Vidal de al Blache (B) A. J. Herbertson (C) R. E. Dickinson (D) K. Young 38. ‘The Eardkunde’ was published in the year 1817. Which one of the following authored the book ? (A) O. Peschel (B) Carl Ritter (C) W. M. Davis (D) G. Taylor 39. Regional Imbalances of Economic development largely affected by which of the following factors ? (A) Regional disparity in resources (B) Lesser resource utilization (C) Lack of education (D) Lesser demand of the people 40. Match the following List-I with List-II and select the correct answer from the codes given below— List-I (Book) (a) Geographia (b) Cosmos (c) Nations and Nationalities (d) Authropogeography List-II (Geographer) 1. Alexander von Humboldt 2. Reclus 3. Ratzel 4. Strabo

Geography-II (U.G.C.) D-14 | 7 Codes : (a) (A) 2 (B) 1 (C) 4 (D) 3

(b) 3 2 1 2

(c) 1 3 2 4

(d) 4 4 3 1

41. The Pir Panjal range lies in— (A) Outer Himalayas (B) Trans Himalayas (C) Greater Himalayas (D) Middle Himalayas 42. Match List-I with List-II and select the correct answer using the codes given below— List-I (Major Rivers) List-II (Tributaries) (a) Vaigai 1. Tista (b) Cauvery 2. Yamuna (c) Brahmpautra 3. Saruli (d) Ganga 4. Amaravati Codes : (a) (b) (c) (d) (A) 1 3 4 2 (B) 3 4 2 1 (C) 3 4 1 2 (D) 1 3 2 4 43. Which one of the following statements indicates the chief characteristics of the Policentric city ? (A) Circumferential urban beltway (B) Replacing sub-urban to urban (C) All roads lead to downtown (D) Many downtowns 44. Which one of the following number of states is correct for ‘Empowered Action-Group States’ of India ? (A) 5 (B) 12 (C) 8 (D) 16 45. Find the sequence of higher to lower rainfall from the following areas of India— I. Ganga Delta II. Sahyadri III. Eastern U.P. IV. Northern Gujarat (A) I, II, III, IV (B) II, I, III, IV (C) III, IV, II, I (D) IV, III, II, I 46. Which one of the following statistical techniques involves a large number of variables to be summarised into smaller dimensions ?

(A) Chi-square (B) Correlation (C) Factor Analysis (D) Regression 47. Which one of the following averages is not a measure of central tendency ? (A) Harmonic mean (B) Geographic mean centre (C) Geometric mean (D) Arithmetic mean 48. Match List-I with List-II and select the correct answer using the codes given below— List-I List-II (Satellites) (Counteries) (a) IRS 1. India (b) SPOT 2. Europe (c) ENCISAT 3. U.S.A. (d) IKONOS 4. France Codes : (a) (b) (c) (d) (A) 1 4 3 2 (B) 1 3 4 2 (C) 2 1 3 4 (D) 4 3 1 2 49. Which one of the following diagrams is most suitable for depicting the distribution of urban land-use types of spatially spread urban centres ? (A) Histogram (B) Sphere (C) Bar (D) Pie 1 is enlarged to 50,000 5-times, which one of the following scales is correct for the enlarged map ? 1 1 (A) (B) 250,000 100,000 1 1 (C) (D) 10,000 2,500

50. A map with the scale of

Answers with Hints 1. (B) Active layer is the top layer of soil in a Perma trost zone-subject to seasonal freezing and thawing and which during the melt season becomes very mobile. 2. (A) 3. (B) Hydertaion is one of the major processes of mechanical weathering, invalving the

8 | Geography-II (U.G.C.) D-14

4.

5.

6.

7. 8. 9. 10.

11. 13. 14. 15.

16. 17.

18. 19.

20.

21.

addition of water to a mineral, causing it to expand and therety initiage stress with in the rock, once mineral have exprienced hyderation they become more suspectible to the effect of chemical weathering especially those of carbonation and oxidiation. (C) Basaltic ocean crust has an average density 2·90/g/cm3 (As it is composed of Sima). (A) During the summer, the shifting of wind belts take place towards the Nanu i . e. polewards, that causes summers to dry in Mediterranean Regions. (D) Altitude, temperature and watervapour affects the atmospheric pressure, but rainfall does not effects. (B) (C) Rain-fed and Rain-shadow areas are associated with orographic rainfall. (B) (C) Tropical Monsoon rain is associated with Sahyadri Ganga Delta rather than Kashmir and Ladakh Region which have very seanty rain from monsoon winds. (B) 12. (A) (A) Wave-cut platform is a characteristic of California-Coast. (B) Average depth of the equatorial counter current is 100 meter. (A) It is due to Western intersification that Gulf stream and Kuroshio current more forcefully northward. (A) ‘The Principles of Human Geography’ was written by E. Huntington. (B) While dealing with the subject matter of Geography Humbolt coined the term Cosmography and divided it into Uranography and Geography. (C) (B) The idea of Backwash originated in the international trade theory in a book by Gunnar Myrdal. (C) Existentialism is a philosophical view that man is responsible for making his own nature, it lays stress an personal freedom, personal decision and personal commitment. (A) 22. (C)

23. (A) The deeded growth rate in the given states during 2001-11 was as follows— Karela—4·92% Rajasthan—21·38% West Bengal—13·78% Bihar—20·60% 24. (A) 25. (B) The decadal growth rate during 1991– 2001 was 21·34% (finally 21·5%). 26. (B) Land with productive value, provided more livelihood to the people. 27. (C) It is Irrigation which is more crucial for agriculture. 28. (D) 29. (C) 30. (D) L. D. Stamp in his landuse survey of Great Britain developed the techniuqe of measurement of agriculture efficiency. 31. (C) 32. (A) It was Alfred Mackinder who profounded the Heartland theory. 33. (B) In 1897, Ratzel wrote Political Geography in which he compared state to an organism. He emphasized the fect that a state, like some simple organism, must either grow or die and can never standstill. It was his Philosophy of Lebensraum living space-which created the controversy of superior and interior. 34. (B) At present the voting behaviour of people in India is affected by caste system. 35. (A) 36. (B) 37. (D) 38. (B) Eardkunde, a monumental work of Carl Ritter Science, Eardkunde is a comprehensive common word which stands for science of earth in relation to nature and history. 39. (A) 40. (C) 41. (D) Pir Panjal Range lies in Middle Himalayas. 42. (C) 43. (D) Policentric city is the organisation of a region around several political, social and financial centres (Down towns). 44. (C) Bihar, Chhattisgarh, Jharkhand, Madhya Pradesh, Odisha, Rajastha, Uttaranchal and U.P. are the eight Empowered Action Group States. 45. (B) 46. (C) 47. (B) 48. (B) 49. (B) 50. (C)

Geography (Paper–III)

UGC-NET/JRF Exam. (December 2014) Solved Paper

December 2014

Geography (Paper–III) Directions—This paper contains Seventy five (75) obective type questions of two (2) marks each. All questions are compulsory. 61. The most fundamental principle of Earth Science ‘Uniformitarianism’ for the first time presented in the book entitled— (A) Principles of Geology (B) Geomorphology from the Earth (C) Unstable Earth (D) Theory of the Earth 62. Which of the following phrased the concept “Landscape is function of process, structure and stage” ? (A) W.M. Davis (B) L. C. King (C) A.N. Strahler (D) Thornbury 63. What are the evidences in favour of the Continental Drift Theory ? I. Zig-Saw fit II. Trans current faults III. Fossils in different areas IV. Convection in mantle Codes : (A) I and IV (B) II and IV (C) I and III (D) I and II 64. The theory of sea-floor spreading was proposed by— (A) Holmes and Davis (B) Airry and Pratts (C) Hess and Dietz (D) Wenger and Ratzel 65. Match the List-I with List-II and select the correct answer from the codes given below— List-I (Agents of Erosion) (a) Running water (b) Moving ice (c) Blowing wind (d) Dashing wave

List-II (Erosional Features) I. Hanging valley II. Incised meander III. Head and Bay IV. Barchan Codes : (a) (b) (c) (d) (A) 1 2 3 4 (B) 2 1 4 3 (C) 1 2 4 3 (D) 2 1 3 4 66. The net results of water inputs and water outputs in Earth’s hydrosphere is in a— (A) Steady-state equilibrium (B) Dynamic equilibrium (C) Quasi-steady-state equilibrium (D) Quasi-dynamic equilibrium 67. Which of the following condition is not essential for a limestone landscape to develop into karst topography ? (A) The limestone formation must contain 100% calcium carbonate for solution processes (B) Complex patterns of joints in the otherwise impermeable limestone (C) There must be an aerated zone between the ground surface and the water table (D) Good vegetation cover 68. Identify the correct sequence of terminology used for denoting different parts along a sloping ground from top to bottom— I. Debris slope II. Waning slope III. Waxing slope IV. Free face (A) II, III, I, IV (B) III, II, IV, I (C) III, IV, II, I (D) III, IV, I, II

4 | Geography-III (U.G.C.) D-14 69. Which one of the following places on the earth is the hottest ? (A) Quetta (B) At Aziza (C) Jacobabad (D) Jaisalmer 10. Match the following List-I with List-II and select the correct answer from the codes given— List-I (a) Westerlies (b) Doldrum (c) Horse latitudes (d) Monsoon List-II 1. Between trade wind and westerlies 2. Seasonal wind 3. Roaring forties 4. Equatorial belt Codes : (a) (b) (c) (d) (A) 1 2 3 4 (B) 3 4 1 2 (C) 2 3 4 1 (D) 4 2 1 3 11. Find the sequence of the onset of South-West monsoon of the following areas from the earlier to later dates— I. Kerala II. Mumbai III. Andaman-Nicobar IV. Odisha (A) I, II, III, IV (B) III, I, II, IV (C) II, III, IV, I (D) IV, II, I, III 12. Find the incorrect statement from the following— (A) Equatorial regions receive heavy convectional rain (B) High rainfall totals are linked to westerlies in both hemispheres (C) There is an increase of precipitation at an altitude above 2 kms (D) On the eastern side of the ocean in the sub-tropical region are dry regions 13. Given below are two statements, one labelled as Assertion (A) and the other labelled as Reason (R). Select your answer from the codes given below— Assertion (A) : Trans-humane type of population movement is noticed in Kashmir valley.

Reason (R) : Rainfall in Kashmir valley during winter is due to Westerly disturbances. Codes : (A) Both (A) and (R) are correct and (R) is the reason of (A) (B) Both (A) and (R) are correct and (R) is not reason of (A) (C) (A) is correct, but (R) is incorrect (D) (A) is not correct, but (R) is correct 14. Which of the two following cyclones did occur in India in 2014 ? I. Hudhud II. Phailin III. Nilopher IV. Lehar (A) I and II (B) I and III (C) II and III (D) III and IV 15. Match the following List-I with List-II and select the correct answer from the codes given— List-I List-II (a) North America 1. The Steppes (b) South America 2. The Velds (c) Africa 3. The Prairies (d) Europe 4. The Pampas Codes : (a) (b) (c) (d) (A) 3 4 1 2 (B) 1 2 3 4 (C) 2 3 4 1 (D) 3 4 2 1 16. Which of the following is a cold current ? (A) Brazil current (B) Gulf stream (C) Benguela current (D) Agualhas current 17. The term ‘brine’ is applicable to— (A) Arabian Sea (B) Baltic Sea (C) China Sea (D) Sargasso Sea 18. Given below are two statements, one labelled as Assertion (A) and the other labelled as Reason (R). Select your answer from the codes given below— Assertion (A) : The Sahara desert enjoys a dry climate. Reason (R) : The eastern side of ocean in sub-tropics along with cold oceanic current.

Geography-III (U.G.C.) D-14 | 5 Codes : (A) Both (A) and (R) are true and (R) is the correct explanation of (A) (B) Both (A) and (R) are true, but (R) is not correct explanation of (A) (C) (A) is true, but (R) is false (D) (A) is false, but (R) is true 19. The Challenger Expedition of 1874 was headed by the country— (A) United States of America (B) United Kingdom (C) U.S.S.R. (D) India 20. Which of the following group of statements are correct ? I. Tropical grassland located in Brazil are known as Llanos and Campose. II. Savanna is a type of equatorial rainy forest in Africa. III. Temperate grassland in Argentina is known as Pampas. IV. Selva is a coniferous of Europe and Asia. (A) I and II (B) II and III (C) III and IV (D) I and III 21. Match the following List-I with List-II and select the correct answer from the codes given— List-I (Region) List-II (Animals) (a) Tundra 1. Kangaroo (b) Savanna 2. Penguin (c) Australia 3. Zebra (d) Antarctica 4. Reinder Codes : (a) (b) (c) (d) (A) 1 2 3 4 (B) 4 3 1 2 (C) 3 4 2 1 (D) 2 1 4 3 22. Which one of the following marine ecosy stems has the highest primary production ? (A) Open seas (B) Tidal estuaries (C) Continental shelves (D) Algae beds and reefs

23. Hekistothermal plants are found in— (A) Mediterranean climate (B) Hot desert (C) Equatorial climate (D) Cold desert 24. Who wrote the book ‘Civilization and Climate’ ? (A) Jafferson (B) Semple (C) Trewartha (D) Huntington 25. The term ‘anthropogeographic’ was coined by Ratzel and was used by him for the— (A) Geography of man in terms of individuals and races (B) Geography of man and his works in relation to earth’s surface (C) Organic theory of society and state (D) Study of harmonious reciprocal relation of man and nature 26. Find the sequence of the following contributions by geographers— I. The origin of species II. Cosmos III. Anthropogeographic IV. Erd Knude (A) I, II, III, IV (B) III, IV, II, I (C) II, I, IV, III (D) IV, III, I, II 27. Who among the following is the protagonist of Neo-Determinism ? (A) Blache (B) Griffith Taylor (C) Bruhnes (D) Saur 28. Match the following List-I with List-II and select the correct answer from the codes given— List-I List-II (Modern name of (Name of River in River) Rigveda) (a) Sutlej 1. Lauhitya (b) Brahmaputra 2. Sindhu (c) Indus 3. Vitasta (d) Jhelum 4. Sutudri Codes : (a) (b) (c) (d) (A) 4 2 3 1 (B) 4 1 2 3 (C) 2 3 4 1 (D) 4 2 1 3

6 | Geography-III (U.G.C.) D-14 29. Who was the author of Cosmos ? (A) Ritter (B) Humbolt (C) Ratzel (D) Bruhnes 30. Which of the following region does have the highest population density as per 2011 Census ? (A) Punjab Plain (B) Upper Ganga Plain (C) Ganga Delta (D) Brahmputra Valley 31. Match the following List-I with List-II and select the correct answer from the codes given— List-I List-II (Ackerman-Population (Name of the Resource Region) Countries) (a) U.S. type 1. India (b) Europe type 2. Malaysia (c) Brazilian type 3. Australia (d) Egyptian type 4. South Korea Codes : (a) (b) (c) (d) (A) 1 2 3 4 (B) 3 4 2 1 (C) 2 1 4 3 (D) 4 3 2 1 32. Given below are two statements, one labelled as Assertion (A) and the other labelled as Reason (R). Select your answer from the codes given below— Assertion (A) : Amazon basin has very low density of population. Reason (R) : The Amazon basin is endowed with hot and humid climate that led to luxuriant selvas. Codes : (A) (A) is correct, but (R) is false (B) (A) is false, but (R) is true (C) Both (A) and (R) are correct and (R) explains the (A) (D) Both (A) and (R) are correct, but (R) does not explain (A) 33. Which one of the following is the correct sequence of stages of urban development as postulated by Lewis Mumford ? I. Polis II. Ecopolis III. Megalopolis IV. Metropolis

(A) (B) (C) (D)

II, I, IV, III I, II, III, IV III, IV, II, I IV, III, I, II

34. What is the trend of Urbanization in India ? (A) Urban population is rising slowly (B) Urban population is rising rapidly (C) Growth rate of urban population is lower than total (D) Urban population is falling 35. Which one of the following densely populated areas of the world does not rely basically on agriculture ? (A) The Nile Valley of Egypt (B) The Plains of Mainland China (C) The Island of Java (D) The North Eastren U.S.A. 36. Both Ulanbator in Mongolia and Lagos in Nigeria are example of— (A) Primate cities (B) World cities (C) Enclaves (D) Edge cities 37. Which of the following is an example of a quinary-sector of economic activity ? (A) Working at a cash register McDonald’s (B) Serving as a researcher for human cloning (C) Serving on the U.S. President’s Cabinet (D) Converting crude oil into gasoline 38. In which country of the world petroleum is treated as ‘source of power not the economy ? (A) United States of America (B) Saudi Arabia (C) Iraq (D) Kuwait 39. With one of the following types of agriculture of India is classified in the world agriculture system by D. Whittellesey ? (A) Nomadic Herding (B) Livestock Ranching (C) Intensive Subsistence Agriculture (D) Commercial Agriculture

Geography-III (U.G.C.) D-14 | 7 40. On the map of Mexico given below at which point ‘maquiladra’ is most likely to be found?

(B)

400 0 400 800 kms.

(A) Point A

(D) Point D

41. Consider the following costs and revenue lines of four firms located in a region— e

sts Co

enu

Rev

a 0 1 1 0 0 0

b 1 1 0 0 1 0

c 1 0 0 1 1 0

d 0 0 1 1 1 0

e 0 1 1 0 0 1

f 0 0 0 1 1 0

a b c d e f

a 0 1 0 0 0 0

b 1 0 0 0 1 0

c 1 0 0 1 1 0

d 0 0 1 0 0 0

e 0 1 1 1 0 1

f 0 0 0 0 1 0

a b c d e f

a 0 1 1 0 0 0

b 1 0 0 0 1 0

c 0 1 0 1 1 0

d 0 1 1 0 1 0

e 0 0 0 1 0 1

f 0 0 0 0 1 0

(C)

(B) Point B

(C) Point C

a b c d e f

(D) a b Location

c

d

Select the best location of maximizing the profit of a firm. (A) b (B) a (C) d (D) c 42. All of the following have been considered growing industrial countries except— (A) India (B) Indonesia (C) Brazil (D) China 43. Consider the following road network diagram of an area— b a c

e

d

Select the correct connectivity above network— (A) a b c d a 0 1 1 0 b 1 01 0 0 c 1 0 0 1 d 0 0 1 1 e 0 1 1 1 f 0 0 0 0

f

matrix for the e 0 1 1 0 0 1

f 0 0 0 1 1 0

44. Given below are two statements, one labelled as Assertion (A) and the other labelled as Reason (R). Select your answer from the codes given below— Assertion (A) : Air transport is a very useful means of transport in mountainous and hilly regions. Reason (R) : It is very difficult to construct roads and railways in mountainous regions. Codes : (A) Both (A) and (R) are true and (R) is the correct explains of (A) (B) Both (A) and (R) are true, but (R) is not the correct explains of (A) (C) (A) is true, but (R) is false (D) (A) is false, but (R) is true 45. Which of the following significantly weaken the strength of Mackinder’s geopolitical heartland theory ? (A) Ascendance of the United States international influence after World War II (B) Existence of a pivot area

8 | Geography-III (U.G.C.) D-14 (C) Growth of Soviet power in Eastern Europe (D) Rise of Nazi Germany 46. The relationship among power structures, the environment and economic inequalities is termed as— (A) Political ecology (B) Gerrymandering (C) Ecoterrorism (D) Cultural diffusion 47. A primary differentiation between a state and a nation is that a State is a— (A) Political abstract, whereas a nation is a human group (B) Mutable concept, whereas a nation is permanent (C) Fixed geographical item, whereas a nation is not liked to a territory (D) Product of history, whereas a nation is a product of people 48. The following question, consist of two statements, one labelled as Assertion (A) and the other labelled as a Reason (R). Examine the two statements and select the correct answer using the codes given below— Assertion (A) : There is hardly any pure race even in the remotest and isolated areas of the world. Reason (R) : All human being comprise a single species, they manifest differences in certain physical traits, and being from one species all members are mutually fertile. Codes : (A) Both (A) and (R) are true and (R) is the correct explains of (A) (B) Both (A) and (R) are true, but (R) is not the correct explains of (A) (C) (A) is true, but (R) is false (D) (A) is false, but (R) is true 49. Match List-I with List-II and select the correct answer using the codes given below the list— List-I List-II (a) Caucasoid 1. European (b) Negroid 2. African (c) Mongoloid 3. Oriental (d) Indic 4. Hindu

Codes : (a) (A) 1 (B) 4 (C) 3 (D) 1

(b) 2 3 2 4

(c) 3 2 4 3

(d) 4 1 1 2

50. Given below are two statements, one labelled as Assertion (A) and the other labelled as Reason (R). Select the correct answer using the codes given below— Assertion (A) : Central Asia has a number of group such as Mongols, Kazakhs, Uzbeks and Kirghiz. Reason (R) : Most of these people are herders and great horsemen. Codes : (A) Both (A) and (R) are true and (R) is the correct explanation of (A) (B) Both (A) and (R) are true, but (R) is not the correct explanation of (A) (C) (A) is true, but (R) is false (D) (A) is false, but (R) is true 51. Which Arab geographer served in the Court of Muhammad Bin Tughlaq ? (A) Al-Beruni (B) Ibn-e-Khaldun (C) Ibn-Batuta (D) Al-Idrisi 52. Match the following List-I with List-II and select the correct answer from the codes given— List-I (Planners) (a) Howard (b) Perry (c) Geddes (d) Corbuser List-II (Contributions) 1. Neighbourhood idea 2. Conurbation 3. Radiant city 4. Garden city Codes : (a) (b) (c) (d) (A) 4 3 2 1 (B) 3 4 1 2 (C) 4 1 2 3 (D) 3 2 1 4

Geography-III (U.G.C.) D-14 | 9 53. Who did bring spatial dimension to the concept of growth ? (A) Perraux (B) Myrdal (C) Bouldville (D) Losch 54. Match the following List-I with List-II and select the correct answer from the codes given— List-I (Planning Programmes) (a) Metropolitan Planning (b) Tribal Area Planning (c) J.N.U.R.M. (d) River Basin Planning List-II (Associated Features) 1. Command area 2. Upgradation of urban infrastructure 3. Social and cultural sensibility 4. Green belt Codes : (a) (b) (c) (d) (A) 1 2 3 4 (B) 4 3 1 2 (C) 3 4 2 1 (D) 4 3 2 1 55. Which one of the following suggested new method to measure regional imbalances in development and allocation of funds to different states of India ? (A) A. Mitra (B) Ashok Dutta (C) Raghu Rajan (D) A. Kundu 56. The Community Development Programme was initiated primarily to— (A) bring in development among people by raising the literacy level (B) increase the control of the masses over the locally available resources (C) bring out overall development of the villages through self-help (D) bring out development of urban facilities 57. Match the following List-I with List-II and select the correct answer from the codes given— List-I (Mountain Range) (a) The Western Ghats (b) The Eastern Ghats (c) The Nilgiris (d) The Aravalis

List-II (Highest Peak) 1. Doddabetta 2. Gurusikhar 3. Annamudai 4. Deomali Codes : (a) (b) (c) (A) 1 2 3 (B) 4 2 1 (C) 2 3 4 (D) 3 4 1

(d) 4 3 1 2

58. Given below are two statements, one labelled as Assertion (A) and the other labelled as Reason (R). Select the correct answer from the codes given below— Assertion (A) : Bangaluru has grown into the hub of information technology and ITES. Reason (R) : City is provided with world class infrastructure, power supply and security. Codes : (A) Both (A) and (R) are correct and (R) is the correct explanation of (A) (B) Both (A) and (R) are correct, but (R) is not the correct explanation of (A) (C) (A) is correct, but (R) is false (D) (A) is false, but (R) is correct 59. Given below are two statements, one labelled as Assertion (A) and the other labelled as Reason (R). Select the correct answer from the codes given below— Assertion (A) : The Golden Quadrilateral Express Highway is the visionary project to facilitate India joining the group of Developed Countries of the World. Reason (R) : GQEH greatly reduced time distance in flow of transport and commodities and boosted urban growth and non-primary activities. Codes : (A) Both (A) and (R) are correct and (R) is the correct explanation of (A) (B) Both (A) and (R) are correct, but (R) is not the correct explanation of (A) (C) (A) is correct, but (R) is false (D) (A) is false, but (R) is correct

10 | Geography-III (U.G.C.) D-14 60. Match the following List-I with List-II and select the correct answer from the codes given— List-I (Linguist Mosaic Family) (a) The Austric (b) The Dravidian (c) The Sino-Tibetan (d) The Indo-Aryan List-II (Language) 1. Lepcha 2. Sanskrit 3. Santhali 4. Malayalam Codes : (a) (b) (c) (d) (A) 1 2 3 4 (B) 2 1 4 3 (C) 3 4 1 2 (D) 4 3 2 1 61. Given below are two statements, one labelled as Assertion (A) and the other labelled as Reason (R). Select the correct answer from the codes given below— Assertion (A) : Indus and Ganges are the antecedent rivers. Reason (R) : These rivers pass through the very narrow and deep gorges in the Himalayan moutain region. Codes : (A) Both (A) and (R) are correct and (R) is the correct explanation of (A) (B) Both (A) and (R) are correct, but (R) is not the correct explanation of (A) (C) (A) is correct, but (R) is false (D) (A) is false, but (R) is correct 62. Match the following List-I with List-II and select the correct answer from the codes given— List-I (Rivers) List-II (Origin) (a) Chenab 1. Great Himalaya (b) Beas 2. Kullu Hills (c) Ganga 3. Gangotri (d) Brahmaputra 4. Kailas Range Codes : (a) (b) (c) (d) (A) 1 2 3 4 (B) 1 3 2 4 (C) 4 3 1 2 (D) 4 3 1 2

63. Main cause of difference between shifting cultivation and contour farming in India, is— (A) Environmental effect (B) Technological development (C) Availability of land (D) Population density 64. Match the following List-I with List-II and select the correct answer from the codes given— List-I (Plateau) List-II (States) (a) Bhander 1. Chhatisgarh (b) Ranchi 2. Madhya Pradesh (c) Bastar 3. Jharkhand (d) Shillong 4. Meghalaya Codes : (a) (b) (c) (d) (A) 1 2 3 4 (B) 1 3 4 2 (C) 2 3 1 4 (D) 2 1 3 4 65. Which one of the North-Eastern states has the highest concentration of tribal population ? (A) Meghalaya (B) Assam (C) Mizoram (D) Nagaland 66. Arrange the cities from the higher to lower temperature— I. Jaipur II. Mubai III. Bengaluru IV. Jaisaler (A) IV, I, II, III (B) I, II, III, IV (C) II, III, I, IV (D) III, II, IV, I 67. Given below are two statements, one labelled as Assertion (A) and the other labelled as Reason (R). Select the correct answer from the codes given below— Assertion (A) : Women status in the NorthEastern States of India is encouraging. Reason (R) : Participation of women in economic activities in these States is quite good. Codes : (A) Both (A) and (R) are correct and (R) is the correct explanation of (A) (B) Both (A) and (R) are correct, but (R) is not the correct explanation of (A) (C) (A) is correct, but (R) is false (D) (A) is false, but (R) is correct

Geography-III (U.G.C.) D-14 | 11 68. Which of the following thematic maps is combined with complex themes ? Select the correct combination of complex-thematic map from the codes given below— 1. Soil spread 2. Forest cover 3. Crop combination 4. Rainfall potential Codes : (A) 1 and 2 (B) 2 and 3 (C) 3 and 4 (D) 2 and 4 69. Which one of the following number of Parallels with one degree interval can be drawn between the poles on a globe ? (A) 180 (B) 178 (C) 179 (D) 181 70. Which one of the following symbols does not represents a point data ? (A) Spot heights (B) Bench marks (C) Contours (D) Triangulation stations 71. The image of which satellite data product is used as substitute of cadastral map— (A) Landstate TM (B) SPOT Xs (C) INSAT 1B (D) IKONOS 72. Which one of the following Digital numbers is correctly matched with its byte value ? Bytes Digital Value (A) 6 — 128 (B) 8 — 512 (C) 7 — 256 (D) 5 — 32 73. Which one of the following locations is correct with reference to Ground control points ? (A) Satellite control station (B) Co-ordinates of places i.e., latitudes and longitudes (C) Places with established co-ordinates and height above MSL (D) Height of places above MSL 74. Which one of the following is not an outcome of GIS modelling ? (A) Binary model

(B) Index model (C) Regression model (D) Iconic model 75. Which one of the following circumstances Pearson’s coefficient of Correlation is used ? (A) when the data set is positively skewed (B) when the data set is negatively skewed (C) when the data is normally distributed (D) when the data is abnormally distributed

Answers with Hints 1. (D) Uniformitarianism, the fundamental principle of modern Geology was profounded by Hutton (1785). It was first presented in the book theory of Earth by Whel-Well in 1832. 2. (A) The phase Landscape is function of process structure and function was given by W.M. Davis in his Cycle of erosion. 3. (C) The Zig-Sawfit as well as fossils in different areas are the two evidences in the favour of continental Drift Theory. 4. (C) The theory of sea-floor-spreading was proposed by Hess and Dietz in 1960. 5. (B) 6. (A) The net result of water inputs and water out puts in Earth’s hydrosphere is in a steady state equilibrium. 7. (D) Good vegetation cover is not an essential feature in limestone landscape development in karst topography. 8. (D) 9. (B) 10. (B) 11. (A) 12. (C) The statement that there is an increasement of precipitation at the atitude about 2 km is wrong. 13. (C) In winter season the Kashmir valley experiences snowfall rather than rain hence A is correct and R is wrong. 14. (B) Hudhud (Odisha coast) and Nilopher (Gujrat) are the two cyclones that occur in India in 2014. 15. (D) 16. (C) Benguela current is a cold current which flows towards the North along the western coast of South Africa. 17. (D) The word ‘brine’ means salty water can be used for Sargasso sea. 18. (B) In sub tropics zone the eastern side the oceans have cold currents (cannaries current).

12 | Geography-III (U.G.C.) D-14 19. (B) Challenger expedition of 1874 was headed by united kingdom. 20. (D) 21. (B) 22. (D) 23. (D) Hekisto thermal plants are those plants which can exists in regions with very low temperature i.e. and desert climate, which has mean low temperature of warmest month 10°C. 24. (D) The book civilization and climate was written by Huntington. 25. (B) The main focus an ‘Anthropogeographic’ of Ratzel was ‘the effects of different physical features and location on the style of life of mean. 26. (C) The correct sequence is II, I, IV, III— Cosmos—1845 The Origin of Species—1859 Erd Knude—1861–62 Anthropogeographic—1872 27. (B) Griffith Taylor was the proof against of Neo-determinism. 28. (B) 29. (B) Humbolt was the author of Cosmos which was published in 1845. 30. (C) Ganga Delta (West Bengal 1028, Bihar 1106) has the highest density of Population. 31. (B) 32. (C) It is due to the luxuriant vegetation along with hot & humid climate, the region (Amazon basin) is sparsely populated. 33. (A) 34. (B) In urban population is rising rapidly. Now it is at present more than 30%.

35. (D) The North Eastern U.S.A. is industrially developed. 36. (A) 37. (C) Quarternary sector of Economic activity includes the education, technologies etc. Thus Serving on the U.S. Presidents Cabinet comes under this category. 38. (A) 39. (C) In India, due to large population and limited land resource intensive subsistence type of agriculture is done. 40. (A) 41. (C) 42. (B) 43. (A) 44. (A) Both A and R are correct, because Air transport is very useful means of transport in mountains and hilly regions, as it is very difficult to construct reads or Railways there. 45. (A) Mackinder’s heartland theory of commanding waved was weakened due to the emergency of USA as a supreme power after the world war II. 46. (A) 47. (A) 48. (A) 49. (A) 50. (B) 51. (C) 52. (C) 53. (C) 54. (D) 55. (C) 56. (C) 57. (D) 58. (A) 59. (A) 60. (C) 61. (D) 62. (A) 63. (B) 64. (C) 65. (B) The concentration of tribal population in the given states as per 2011 census is as follows— Meghalaya—25,25,861 Assam—38,84,331 Mizoram—10,36,315 Nagaland—17,10,973 66. (A) 67. (A) 68. (C) 69. (B) 70. (C) 71. (D) 72. (D) 73. (C) 74. (D) 75. (C)

Geography (Paper–II)

UGC-NET/JRF Exam. (June 2015) Solved Paper

June 2015

Geography (Paper–II) Note—This paper contains fifty (50) objective type questions of two (2) marks each. All questions are compulsory. 1. Match the List-I with List-II and select the correct answer from the given codes— List-I (Concept) (a) Peneplain (b) Uniformitarianism (c) Tetrahedral hypothesis (d) Convectional Current Theory List-II (Proposer) 1. Holmes 2. W.M. Davis 3. Hutton 4. Lothian Green Codes : (a) (b) (c) (d) (A) 2 3 4 1 (B) 3 1 2 4 (C) 1 2 4 3 (D) 4 2 3 1 2.

Ria is an example of— (A) Deposited land features (B) Submerged upland shorelines (C) Emerged upland shorelines (D) Eroded land forms

5.

Who postulated the concept of Base level ? (A) Walther Penck (B) W.M. Davis (C) J.W. Powell (D) James Hutton

6.

According to single-cell circulation model proposed by George Hadley the most important factor causing the global atmospheric circulation is— (A) The rotation of the earth (B) The jet stream (C) Subsidence at the equator (D) Temperature contrast between equator and pole

7.

The westerlies and trade winds are example of ‘……… winds.’ (A) mesoscale (B) toposcale (C) macroscale (D) microscale

8.

When water evaporates the energy that was used to evaporate the water— (A) heats up the surrounding air as evaporation takes place (B) will never be available again (C) is stored as latent heat in the evaporated water molecule (D) is destroyed

9.

3.

Who postulated the ‘Progressive Wave Theory of Tides’ ? (A) C. Darwin (B) William Whewill (C) R.A. Haris (D) Daly

Meteorologists group tropical cyclones into three main categories according to— (A) barometric pressure in the eye of storm (B) precipitation (C) diameter of the storm (D) wind velocity

4.

Who wrote the book ‘Illustration of the Huttonian Theory of the Earth’ ? (A) James Hutton (B) John Playfair (C) Vidal de la Blache (D) W.M. Davis

10. A given volume of air holds 20 grams of water vapor. At that temperature, the air would saturated with 80 grams of water vapor. What is the relative humidity ? (A) 40% (B) 100% (C) 25% (D) 80%

4 | Geography-II (U.G.C.) J-15 11. Surface circulation of ocean water is driven by— (A) Resisting force (B) Tidal force (C) Gravitational force (D) Blowing wind 12. For coastal sand dune formation, the ideal grain size diameter is— (A) 0·15 mm (B) 0·50 mm (C) 0·25 mm (D) 0·75 mm 13. Coastal area greatly affected by storm surge is— (A) Andhra coast (B) West Bengal coast (C) Chennai coast (D) Kerala coast 14. How many essential nutrients are required by plants to fulfil their growth requirements ? (A) 24 (B) 22 (C) 20 (D) 18 15. Mesophytes are predominated in— (A) Tropical region (B) Sub-polar region (C) Temperate region (D) Polar region 16. Who is regarded as the father of Human Geography ? (A) Ratzel (B) Blache (C) Trewartha (D) Vallaux 17. Match List-I with List-II and select the correct answer from the codes given below— List-I (Approaches) (a) Neo-Determinism (b) Welfare (c) Behavioral (d) Modern Humanist List-II (Contributors) 1. D.M. Smith 2. Guman Olsson 3. Griffith Taylor 4. William Kirk Codes : (a) (b) (c) (d) (A) 1 2 3 4 (B) 3 1 4 2 (C) 4 3 2 1 (D) 2 4 1 3

18. Who is the pioneer in Commerical Geography ? (A) H.J. Mackinder (B) G.G. Chisholm (C) Patrick Geddes (D) A.J. Herbertson 19. Which one of the following philosophers did propagate the principle of Unity in Diversity ? (A) Carl Ritter (B) Immanuel Kant (C) Alexander Von Humboldt (D) O.H.K. Spate 20. To which school of Geography does E.C. Semple belong ? (A) American School of Geography (B) Russian School of Geography (C) British School of Geography (D) French School of Geography 21. Which one of the following states recorded the lowest decadal growth rate of population according to the 2011 census ? (A) Arunachal Pradesh (B) Madhya Pradesh (C) Punjab (D) Nagaland 22. According to 2011 census, which state recorded the highest child sex-ratio (0-6 years) ? (A) Manipur (B) Mizoram (C) Meghalaya (D) Arunachal Pradesh 23. Out of total population of 121 crore, what was the level of urbanisation in 2011 census ? (A) 33·16% (B) 32·16% (C) 30·16% (D) 31·16% 24. In which one of the cities first city improvement trust was set up ? (A) Bombay (B) Calcutta (C) Hyderabad (D) Lucknow 25. Horwood and Boyce described which term epigrammatically as “Devoted to people, paper work and parcels” ? (A) Frame (B) Core (C) Periphery (D) Fringe 26. Consider the following statements— 1. Nearly 30% of the petroleum production in the world is from the off shore areas.

Geography-II (U.G.C.) J-15 | 5 2.

The exclusive economic zone in the ocean extends upto 100 nautical miles from the coast. 3. Only less than 1% of marine area is declared as protected area. 4. The largest marine protected area is the Great Barrier Reef. Which of these statements are correct ? (A) 1 and 2 (B) 1, 3 and 4 (C) 1 and 4 (D) 2 and 3 27. Which one of the following groups of countries represents the flow of food grains to food deficit regions of the world ? (A) North America, Argentina, Indian Sub continent and China to Western Europe, Middle East and Japan (B) Australia, South, Africa, India to Middle East, Western Europe and North Africa (C) North America, Argentina, South Africa and Australia to Japan, Russia and Western Europe (D) North America, Australia, Russia to China, Japan and Middle East 28. Which one of the following groups of countries requires import of iron to sustain their demand ? (A) Western Europe, Brazil and Canada (B) Japan, Western Europe and U.S.A. (C) India, China and South Africa (D) U.S.A., Western Europe and Mexico 29. Which one of the following is the busiest oceanic trade route ? (A) Panama (B) North Atlantic route (C) Suez Canal (D) Cape route 30. Balance of Trade of India is always negative with the Middle Eastern Countries due to the import of— (A) Food Material (B) Cotton Textile (C) Coal and Iron (D) Petroleum 31. Which one of the following authors has classified Indian Races ? (A) R.L. Singh (B) Sir Herbert Risley (C) Huntigton (D) Trewartha

32. Heart land theory is postulated by— (A) P.E. James (B) H.J. Mackinder (C) G.G. Chisholm (D) H.R. Mill 33. Given below are two statements, one labelled as Assertion (A) and other labelled as Reason (R). Select your answer from the codes given below— Assertion (A) : Cuba and U.S.A. are nearer to each other in geographical space, but they are not friendly neighbours. Reason (R) : There is strong differences in political philosophy. Codes : (A) Both (A) and (R) are true and (R) is the correct explanation of (A) (B) Both (A) and (R) are true, but (R) is not the correct explanation of (A) (C) (A) is true, but (R) is false (D) (A) is false, but (R) is true 34. Which one of the following authors has defined the phrase “Culture is that complex who which includes knowledge, belief, art, moral, law, customs acquired by a man as member society” ? (A) E.B. Taylor (B) R.H. Lowie (C) Kroeber (D) Goldenweiser 35. Indian people elect their representatives to House of People by— (A) Single preference ballot (B) People’s choice in Gram Sabha (C) Nomination by a panel of experts (D) Election through state legislature 36. Whose theory did propagate that ‘an area is poor because it is poor’ ? (A) Myrdal (B) Berry (C) Hirschman (D) Boudeville 37. In which one of the following concept the term ‘Triple Bottom Line’ (TBL) reflecting the importance of environmental, social and economic factors in decision making is defined ? (A) Inclusivity (B) Sustainability (C) Poly-centricity (D) Mono-centric 38. Which one of the following Regions is characterised by communication networks flowing upwards and downwards through a

6 | Geography-II (U.G.C.) J-15 succession of service centres of varying significance ? (A) Heterogenous (B) Homogenous (C) Nodal (D) Administrative

(C) Arithmetic mean is always a positive number (D) Arithmetic mean is not useful for any further statistical analysis of the data

39. Which one of the following planning regions develops along transportation line or irrigations channels ? (A) Transitional Regions (B) River Valley Regions (C) Axial development Regions (D) City Regions

47. Which one of the following central tendencies is the appropriate method for the study of dispersion ? (A) Arithmetic mean (B) Median (C) Mode (D) Geometric mean

40. Who has designed the plan of Chandigarh City ? (A) Ernst May (B) Le Corbusier (C) Soria Y Matte (D) Charles Edward Jeanneret

48. Which one of the following cartographic techniques is suitable for measuring spatial association among different attributes of the regional economy ? (A) Bar diagram (B) Pie diagram (C) Choropleth (D) Isopleth

41. In which part of India Dandakaranaya is situated ? (A) North Eastern (B) South Eastern (C) Central (D) Northern 42. Teesta river is a tributary of— (A) Subansiri (B) Meghna (C) Ganga (D) Brahmapurtra 43. Which one of the following states has the maximum area under teak forest of India ? (A) Uttar Pradesh (B) Bihar (C) Karnataka (D) Madhya Pradesh 44. In which one of the following rocks almost all workable oil deposits of India is located ? (A) Ancient granites (B) Ancient sedimentaries (C) Metamorphic formations (D) Tertiary sedimentaries 45. Which one of the following coastal areas of India was affected by ‘Hudhud’ cyclone ? (A) Chennai coast (B) Kerala coast (C) Andhra Pradesh coast (D) Bengal coast 46. Which one of the following is correct ? (A) Arithmetic mean is a numerical value (B) Arithmetic mean is not affected by the variability in the data sets

49. Match List-I with List-II and select the correct answer from the codes given below— List-I (Method) (a) Isopleth (b) Dot maps (c) Scatter diagrams (d) Star diagrams List-II (Uses) 1. Natural groupings 2. Distribution of population 3. Wind direction 4. Changes which are relatively gradual Codes : (a)

(b)

(c)

(d)

(A)

4

2

3

1

(B)

2

4

3

1

(C)

4

2

1

3

(D)

2

4

1

3

50. Which one of the following is shown with the help of Isobaths ? (A) Salinity of sea water (B) Depth of ocean water (C) Altitude of oceanic ridge (D) Amplitude of wave

Geography-II (U.G.C.) J-15 | 7

Answers with Hints 1. (A) 2. (B) ‘Ria’ is a spanish term widely adopted to describe a submerged coastal valley of estuarly resulting from a rise of sea level. 3. (B) Progressive Wave Theory of Tides was postulated by William Whewill. 4. (B) ‘Illustration of the Huttonian Theory of the Earth’ was written by John Playfair in 1802. 5. (C) It was J.W. Powell who under the exploration of Colorado river of the west said that the erosion done by river reaches to its last stage. Thereafter no erosion takes place and this last stage of erosion is known as Base level. 6. (D) Single Cell Circulation Model given by George Hadley is based on temperature contrast between equator and pole ward in which warm air moves towards pole wards or cold air moves toward equator in the same horizontal plane. 7. (C) Planetary winds like westerlies and trade winds synoptic : Cyclones and anti cyclones, Hurricanes are macroscale winds while thunder storm tornadoes mesoscole winds, gust or dust devils are microscale winds. 8. (C) Latent heat is an amount of energy emitted or absorbed when a body changes it state or phase without any change of temperature with in that body. It is very important in atmospheric, processes where water can exists as a vapour, it is emitted during the process of condensation. 9. (D) Tropical cyclones are classified in accordance with the world meteorologists on the bases of wind speed near the centre. A.H. × 100 10. (C) R.H. = H.C. R.H. = Relative Humidity A.H. = Absolute Humidity H.C. = Humidity Capacity at that temperature 20 × 100 Hence, = = 25% 80

11. (D) It is due to the blowing winds (Prevaiting winds) that the surface circulation of ocean water takes place. 12. (C) 13. (B) Generally West Bengal coast is affected by storm surge. 14. (D) 15. (C) Mesophytes are generally predominate in neither hot or cold areas i.e., it predominates at temperate regions i.e., temperate meadows. 16. (A) Ratzel is known as the founder or father of Human Geography. He was essentially a scholar of classical languages. 17. (B)

18. (B)

19. (A) The fundamental principle evolved by Ritter was Unity in diversity. According to him, there is a fundemental unity in the biotic and abiotic components of habitat in which man sculptures his cultural environment. 20. (A) 21. (D) Decadal growth rate in given states is as follows : 1. Nagaland : 0·6%; 2. Punjab : 13·9%; 3. Madhya Pradsh : 20·3%; 4. Arunachal Pradesh : 26·0 (Final data). 22. (D) Following is the child sex ratio as per census 2011 (final data) in the gives states : 1. Arunachal Pradesh : 972; 2. Mizoram : 970; 3. Meghalaya : 970; 4. Manipur : 936. 23. (D) As per 2011 census final data the Urban ratio is 31·16% and Rural ratio is 68·84%. 24. (A) Bombay improvement trust, first set up in 1898 in response to the Mumbai plauge epidemic. 25. (B)

26. (B)

27. (C)

28. (B)

29. (B) North Atlantic route joining the west European and east American trade centres is the busiest oceanic route. 30. (D) Trade balance of India with Middle Eastern Countries in negative due to import of petroleum. 31. (B) Herbert Risley classified the tribes of Bengal.

8 | Geography-II (U.G.C.) J-15 32. (B) The heart land concept was postulaed by H.J. Mackinder who in 1913 used this word in Geography and World Politics. 33. (A) 34. (A) 35. (A) 36. (A) 37. (B) The term ‘Triple Bottom Live’ or ‘3 BL’ is an acounting framework with three parts Social, Environmental (Ecology) and finance. These three divisions are also called three PS people, planet and profit or the three pillar of sustainability. 38. (C) Communication network flowing upwards and downwards through the succession of service centres of varying significance is characterised by nodal region in which an area organised round a not or a focal point

and is defined by interactions and connections of service centres. 39. (C) The axial region develops along transport lines or irrigation channels. 40. (B) The planner of Chandigarh city was Le Corbusier. 41. (C) Dandakaranaya region lies in Central India (Chhattisgarh). 42. (D) 43. (D) 44. (B) Oil deposits are found in Ancient sedimentary rocks. 45. (C) 46. (A) 47. (B) 48. (A) 49. (C) 50. (B)

Geography (Paper–III)

UGC-NET/JRF Exam. (June 2015) Solved Paper

June 2015

Geography (Paper–III) Note—This paper contains seventy five (75) objective type questions of two (2) marks each. All questions are compulsory. 1. Which of the following is formed due to tectonic forces ? (A) Rift valley (B) hanging valley (C) Super imposed valley (D) Antecedent valley 2.

3.

4.

Drowned glaciated valleys in high latitudes regions are known as— (A) Ocean trenches (B) Submarine ridges (C) Fiords (D) Submarine canyons Match List-I with List-II and select the correct answer from the codes given : List-I List-II (a) Basalt 1. Geneis (b) Granite 2. Quartzite (c) Sandstone 3. Schist (d) Shale 4. Slate Codes : (a) (b) (c) (d) (A) 2 1 4 3 (B) 1 2 3 4 (C) 4 2 1 3 (D) 3 1 2 4 Consider the following statements regarding kants view on the origin of the earth : 1. Kant introduced the Newtonian law of gravitation in his theory. 2. Kant developed his theory accepting the principle of conservation of angular momentum.

3.

Though Laplace put forward the nebular hypothesis of origin of the earth, Kant is regarded by many as the real propounder of the nebular hypothesis. Which of the statements are correct ? (A) 1 and 2 (B) 2 and 3 (C) 1 and 3 (D) 1, 2 and 3 5.

Excessive folding results in— (A) Reverse fault (B) Geosyncline (C) Nappe formation (D) Block disintegration

6.

“All coral reefs begun as fringe reefs around an inland”. This was indicated in— (A) Day’s glacial control theory (B) Daly’s subsidence theory (C) Darwin’s glacial control theory (D) Darwin’s subsidence theory

7.

Guttenberg discontinuity is found between the— (A) Upper core and lower core (B) Mantle and the core (C) Crust and the mantle (D) Upper mantle and lower mantle

8.

Who postulated the concept of Sea floor spreading ? (A) W.J. Morgan (B) T.J. Wilson (C) Le Pichon (D) Harry Hess

9.

Mauna Loa, in Hawaii is famous for— (A) Botanical garden (B) Monitoring sea level rise since 1950 (C) Monitoring rainfall (D) Continuous monitoring of atmospheric CO2 since 1957

4 | Geography-III (U.G.C.) J-15 10. Match the List-I and List-II select the correct answer from the codes given below : List-I (a) Isohel (b) Isohaline (c) Isotach (d) Isoneph List-II 1. line joining places having same average percentage of cloudiness. 2. line joining the places having same duration of sunshine. 3. line joining the places having equal salinity in the ocean. 4. line joining places having equal wind speed. Codes : (a) (b) (c) (d) (A) 2 3 1 4 (B) 1 2 3 4 (C) 2 3 4 1 (D) 4 3 1 2 11. The only truely continuous pressure belt on the earth is— (A) Southern hemisphere subtropical high (B) Northern hemisphere subtropical high (C) Equatorial low (D) Southern hemisphere sub polar low 12. Which two global winds originate from the subtropical highs ? (A) Polar easterlies and westerlies (B) Trade winds and polar easterlies (C) Trade winds and westerlies (D) Chinook and Foehn 13. An aircraft is flying at an altitude of 10 km. At that altitude the temperature is –40°C. What is the ambient temperature on the ground— (A) 24°C (B) 25°C (C) 30°C (D) 20°C 14. Which one of the following is not a necessary condition for condensation ? (A) Saturation (B) Surface (C) High altitude (D) Water vapour 15. Given below are two statements. One labelled as Assertion (A) and other labelled as Reason (R). Select your answer from the codes given below :

Assertion (A) : Land surface is heated more quickly and to a greater extent than the water surface when subject to an equal amount of insolation. Reason (R) : The specific heat of land is more than that of water. Codes : (A) Both (A) and (R) are true and (R) is the correct explanation of (A) (B) Both (A) and (R) are true but (R) is not a correct explanation of (A) (C) (A) is true but (R) is false (D) (A) is false but (R) is true 16. Which of the following does not enhance the instability of air ? (A) radiation cooling of earth’s surface after sun set (B) radiation cooling from cloud top (C) heating an air mass from below as it passes over a warm surface (D) intense solar heating in the lowermost atmosphere 17. The speed of a ocean current is about— (A) 2% of the speed of prevailing wind (B) 4% of the speed of prevailing wind (C) 6% of the speed of prevailing wind (D) 8% of the speed of prevailing wind 18. The ocean water was iceberg free during the era— (A) Archaean (B) Paleozoic (C) Cenozoic (D) Mesozoic 19. The salinity of sea ice ranges from— (A) 0–3% (B) 3–10% (C) 11–17% (D) 28–35% 20. Match List-I with List-II and select the correct answer from the codes given below— List-I (River) (a) Ganga (b) Mekong (c) Brahmputra (d) Yangtze List-II (Sediment transported to ocean in 104 tons/y) 1. 726 2. 500 3. 1600 4. 1000

Geography-III (U.G.C.) J-15 | 5 Codes : (a) (A) 3 (B) 2 (C) 3 (D) 4

(b) 2 1 4 3

(c) 4 3 1 1

(d) 1 4 2 2

21. The place where an organism or community of organisms lives is called— (A) Environment (B) Atmosphere (C) Habitat (D) Society 22. Photosynthesis is the process by which— (A) Radiant energy converted into chemical energy (B) Radiant energy converted into thermal energy (C) Radiant energy converted into bioenergy (D) Radiant energy converted into geothermal energy 23. ‘Vadose water’ remain in between— (A) Crust and mantle (B) Mantle and core (C) Ground surface and water table (D) Ground surface and tropopause 24. Which one of the following is not a macronutrients to plants ? (A) Carbon (B) Iron (C) Nitrogen (D) Oxygen 25. The concept of the laws of ‘primate city’ was given by— (A) E. Huntington (B) E. C. Semple (C) M. Jefferson (D) W. Christaller 26. To which school does Eratosthenes, the father of geography belongs to— (A) Roman (B) Arab (C) German (D) Greek 27. Von Thunen’s Agricultural Locational theory is based on— (A) Empirical Approach (B) Normative Approach (C) Deductive Approach (D) Behavioral Approach 28. Who was greatly influenced by Le Play’s triology of Place-Work-Folk and adopted

alternate triology of Environment-FunctionOrganism ? (A) Roxby (B) Geddes (C) Herbertsen (D) Chisholm 29. Which Indian geographer has written the book ‘Geography of Purans’ ? (A) S.M. Ali (B) P.P. Karan (C) M. Shafi (D) B. Dubey 30. Match List-I with List-II and select the correct answer from the codes given below : List-I (Modern Indian Geographers) (a) R. L. Singh (b) G. S. Gosal (c) C. D. Despande (d) V. L. S. Prakash Rao List-II (Field of Specialisation) 1. Urban geography 2. Settlement geography 3. Regional development and planning 4. Population geography Codes : (a) (b) (c) (d) (A) 1 2 3 4 (B) 3 1 4 2 (C) 2 4 1 3 (D) 4 3 2 1 31. Who among the following scholars is not related with behavioural geography ? (A) W. K. Kirk (B) Peter Gould (C) Yi Fu Twan (D) Gilbert White 32. Which one of the following states in India has the lowest population density in the latest census ? (A) Arunachal Pradesh (B) Jammu and Kashmir (C) Sikkim (D) Mizoram 33. Which of the following stages of demographic transition indicate the state of under development ? (A) High stationary stage (B) Early expanding stage (C) Late expanding stage (D) Low stationary stage

6 | Geography-III (U.G.C.) J-15 34. Which one of the following districts in India has recorded the highest sex ratio in 2011 ? (A) Mahe (Pondicherry) (B) Almora (Uttarakhand) (C) Alwar (Rajasthan) (D) Thanjavur (Tamil Nadu) 35. Which one of the following density is calculated using following formula : NK ED = , where N is the number of SK inhabitants, K is per capita quantity of requirement, S is area in square kilometers, and K′ is the quantity of resources produced per km2 . (A) Arithmetic (B) Agricultural (C) Physiological (D) Economic 36. Which one of the following measures of central tendency is used to indicate the ‘centre of population ? (A) Median (B) Mode (C) Mean (D) Harmonic mean 37. Which of the concept relates population size to the land area with a view to assess pressure of population upon the resources of the area ? (A) Population Growth Rate (B) Population Density (C) Agriculture Density (D) Physiological Density 38. Which one of the following religions has maximum territorial region of dominance in the World ? (A) Budhism (B) Islam (C) Christianity (D) Hinduism 39. Which one of the following authors has given the concept of. ‘Demand cone’ as shown in the diagram ? F Price Q

a R b P O

S Q d Quantity T (a)

P (b)

F

(A) Pareto (C) Christaller

(B) August Losch (D) Weber

40. Which of the following groups of crops can be grown in the area having temperature 20° – 27°C and annual rainfall more than 150 cm ? (A) Barley, Jute, Tea (B) Rubber, Rice, Jute (C) Tea, Coffee, Maize (D) Rice, Wheat, Maize 41. Match List-I with List-II and select the correct answer from the codes given below : List-I (Authors) (a) Weber (b) Smith (c) Hotelling (d) Hoover List-II (Theory) 1. Minimum production cost 2. Maximum profit 3. Minimum transport cost 4. Market competition Codes : (a) (b) (c) (d) (A) 3 2 4 1 (B) 1 2 3 4 (C) 3 4 1 2 (D) 1 2 4 3 42. Given below are two statements, one labelled as Assertion (A) and other labelled as Reason (R) select your answer from the codes given below : Assertion (A) : All Japanese steel production occurs of coastal locations. Reason (R) : The Japanese steel capacity has a coastal exposure and direct access to deepwater ports. Codes : (A) Both (A) and (R) are the true and (R) is the correct explanation of (A) (B) Both (A) and (R) are true, but (R) as not the correct explanation of (A) (C) (A) is true, but (R) is false (D) (A) is false, but (R) is true

Geography-III (U.G.C.) J-15 | 7 43. Consider the following map of an area— 2 5

4

School Hospital Railway station Post office

3 7 6

8 0

(A)

(B)

(C)

1 0 1 1 1 1 1 1

1

1

0

0

5 1 1 0 1 6 1 1 1 0 7 0 0 1 0 8 1 1 0 1 44. Which one of the following is correct sequence of urban land uses in respect of ab, bc and cd as per given graph ?

500 km

Select the correct matrix for the above map— 1 2 3 4 5 6 7 8

4

1 1 1 1 1 1 1 1

1 1 1 0 0 1 0 1

1 0 0 0 1 1 1 0

1 2 3 4 5 6 7 8

1 0 1 1 1 1 0 1

1 1 0 1 1 1 1 1

1 1 1 0 0 1 1 1

1 0 1 0 1 1 0 0

1 2 3 4 5 6 7 8

1 0 1 1 1 1 0 1

1 1 1 1 1 1 0 1

1 1 0 0 0 1 1 1

1 0 1 0 1 1 0 0

1 2 3

1 0 1

1 1 1

1 1 0

1 0 1

Commerical Economic rent

1

(D)

0

a

Residential Industrial 4

b

8

10 12 14 Distance (km.)

c

d

(A) Industrial, Residential and Commercial (B) Commercial, Residential and Industrial (C) Commercial, Industrial and Residential (D) Residential, Commercial and Industrial 45. Whittelessy classified agricultural systems of the world. How many agricultural system were identified by him ? (A) 10 (B) 19 (C) 14 (D) 13 46. Which one of the following group of countries is leading total coal import countries of the world in 2013 according to the world coal association ? (A) Bangladesh, South Africa and France, India (B) Poland, South Africa, Japan and France (C) China, Japan, India, South Korea (D) Japan, South Korea, France, Canada 47. Match List-I with List-II and select the correct answer from codes given below : List-I (Risely’s classification of Indian Race) (a) Indo-Aryans (b) Dravidians (c) Mongoloids (d) Mongolo-Dravidians

8 | Geography-III (U.G.C.) J-15 List-II (Representatives of the respective races) 1. The Bhils of Rajasthan 2. The Brahmins of Odisha 3. The Jats of Haryana 4. The Bhutias of Utarakhand Codes : (a) (b) (c) (d) (A) 1 2 3 4 (B) 4 3 2 1 (C) 2 4 1 3 (D) 3 1 4 2 48. Given below are two statements, one labelled as Assertion (A) and other labelled as Reason (R), select your answer from the codes given below : Assertion (A) : Indian ocean has become the military bases by many super-powers. Reason (R) : There is strong geo-political benefits in Indian ocean and adjacent countries. Codes : (A) Both (A) and (R) are true and (R) is the correct explanation of (A) (B) Both (A) and (R) are true, but (R) is not the correct explanation of (A) (C) (A) is true, but (R) is false (D) (A) is false, but (R) is true 49. Match List-I with List-II and select the correct answer from the codes given below : List-I (State) List-II (Tribes) (a) Gujarat 1. The gonds (b) Madhya Pradesh 2. The Mikirs (c) Assam 3. The Khonds (d) Odisha 4. The Bhils Codes : (a) (b) (c) (d) (A) 1 2 3 4 (B) 3 4 1 2 (C) 4 1 2 3 (D) 2 3 4 1

50. Match List-I with List-II and select the correct answer from the codes given below : List-I (Countries) (a) U.S.A. (b) China (c) India (d) U.K. List-II (Party System Government) 1. One party 2. Two party 3. Three party 4. Multi party Codes : (a) (b) (c) (d) (A) 1 2 3 4 (B) 2 1 4 3 (C) 3 4 2 1 (D) 4 3 1 2 51. Which of the following statements is the base of the concept of caste region ? (A) Numerical strength of a caste (B) Social ranking of a caste in a region (C) Proportion of a caste in population (D) Share of a caste in land ownership 52. Which one of the following is the propounder of the shown model ? MIGRANTS Intervening obstacles DESTINATION ORIGIN −Negative factors +Positve factors

(A) (B) (C) (D)

Zelinsky’s Mobility Model Rely’s Model of Migration Gosal Model of Migration Lee’s Model of Migration

53. Which one of the following regions is defined as “A contiguous geographical area characterised by cultural homogeneity …”. (A) Population Region (B) Geographical Region (C) Cultural Realm (D) Settlement zone 54. Which constitutional Amendment Act has 12th schedule devolving functions to municipal bodies ? (A) 73rd (B) 72nd (C) 74th (D) 75th

Geography-III (U.G.C.) J-15 | 9 55. Which of the urban structure models labelled three areas : (1) the city of death, (2) the city of need and (3) the city of superfluity ? (A) Exploitive model of urban structure (B) Multiple nuclei (C) Sector model (D) Social area analysis model 56. To study regional imbalances and regional disparities in India, which one of the following methods was applied ? (A) Unweighted Ranks (B) Composite Index (C) Principal Component Analysis (D) Input-Output Analysis 57. Which one of the following factors does not determine the kinds and amount of economic activity in the region ? (A) Cost of Transportation (B) Inequality of Income (C) Technology of Production (D) Demand Side of the Equation 58. The main thrust of regional planning is— (A) To remove areal disparity (B) To initiate economic growth (C) To have industrial expansion (D) To increase per capita income 59. Regional growth model by Douglass C. North emphasizes the role of ‘……… factors’ is regional growth. (A) Hexagenous (B) Endogenous (C) Indogenous (D) Exogenous 60. Match List-I with List-II and select the correct answer from the codes given below : List-I (Concept) (a) Urban growth (b) Town group (c) Urban (d) Urbanisation List-II (Definitions) 1. relatively large and densely settled populations engaged primarily in nonagricultural economic pursuits. 2. net population increase of towns and cities. 3. proportionate increase of the urban population in relation to total population in a given country.

4.

a group of towns which adjoined one another so closely as to form a single inhabited urban locality. Codes : (a) (b) (c) (d) (A) 4 1 3 2 (B) 2 4 1 3 (C) 3 2 4 1 (D) 1 3 2 4 61. Where is Govind Sagar Lake located ? (A) Ranjit Sagar Dam (B) Hirakud Dam (C) Kosi Dam (D) Bhakra Nangal Dam 62. The maximum % share of irrigated area of the gross cultivated area is found in— (A) Jammu-Kashmir (B) Uttar Pradesh (C) Punjab (D) Haryana 63. Kakrapara irrigation project is on the river— (A) Narmada (B) Godavari (C) Tapti (D) Mahanadi 64. The river Indus Originates from— (A) Mansarowar Lake (B) Kailash range (C) Loktak lake (D) Tibet 65. Given below are two statements, one labelled as Assertion (A) and other labelled as Reason (R) select your answer from the codes given below : Assertaion (A) : Punjab, Haryana and Uttar Pradesh are major wheat producing states of India. Reason (R) : The well drained fertile soil, 10 to 15°C temperature at the time of sowing and about 75 cm rainfall are necessary for good wheat production. Codes : (A) Both (A) and (R) are true and (R) is the correct explanation of (A) (B) Both (A) and (R) are true, but (R) is not the correct explanation of (A) (C) (A) is true, but (R) is false (D) (A) is false, but (R) is true

10 | Geography-III (U.G.C.) J-15 66. Which of the canal has transformed agricultural activities of Sri Ganga Nagar, Bikaner and Jaisalmer districts ? (A) Kanhar Canal (B) Ram Ganga Canal (C) Sharada Sahayak Canal (D) Indira Gandhi Canal

73. Which one of the following terms shows variance of a particular distribution ? — — Σ(X – X ) (A) Σ(X – X ) (B) N —2 — ⎤ 1/2 ⎡⎢ Σ(X – X ) Σ(X – X )2⎥ ⎥⎦ (C) (D) ⎢⎣ N N

67. Report published by UNDP which compares countries based on education levels of people, their health status and per capita income is— (A) Human Education Report (B) Human Population Report (C) Human Development Report (D) Human Quality Report

74. Assume their are four families in a country, The average per capita income of these families is 5,000. If the income of three families is 3,000; 4,000 and 7,000 respectively, what is the income of fourth family ? (A) 7500 (B) 2000 (C) 3000 (D) 6000

68. Spatial data is stored in computer by using— (A) Plotter digitizer and keyboard (B) Keyboard and plotter (C) Scanner, digitizer and keyboard (D) Digitizer and scanner 69. LANDSAT, SPOT and IRS are examples of— (A) Sun-synchronous satellites (B) Geostationary satellites (C) Radars (D) Natural satellites 70. Which one of the following statistical method is best suited for groundwater exploration by using GIS tools ? (A) Standard deviation method (B) Principal component analysis method (C) Trial and error method (D) Index-overlay method 71. Which one of the following countries first launched Operational Remote Sensing satellite ? (A) USA (B) UK (C) USSR (D) Germany 72. Raster data start from— (A) Top-Right corner of the displayed window (B) Top-Left corner of the displayed window (C) Bottom-Right corner of the displayed window (D) Bottom-Let corner of the displayed window

75. Match List-I with List-II and select the correct answer from the codes given below— List-I (Landsat 4 and 5 TM) (a) IFOV (b) Revisit (c) Altitude (d) Swath width List-II (System Characteristics) 1. 185 km 2. 705 km 3. 16 days 4. 30 × 30 m for 1-5, 7 bands Codes : (a) (b) (c) (d) (A) 1 2 3 4 (B) 2 3 1 4 (C) 4 3 2 1 (D) 4 3 1 2

Answers with Hints 1. (A) Rift valley is formed due to tectonic forces while hanging valley is the result of Glaciation and Superimposed Valley and Antecedent Valley are the result of running water. 2. (C) Fiord, is a long narrow inlet of the sea bounded by steep mountain slope which are of great height and extend to considerable depth below sea level. It is formed by the submergence of glacially over deepened valley. 3. (D)

Geography-III (U.G.C.) J-15 | 11 4. (D) Kant in his Hypothesis of Earth origin was of the view premordial matters collided with one another due to gravitation force, resenting rotation due to principle of conservation of angular momentum 5. (C) Excessive folding is the result of Nappe formation (Nappe is a large scale tectonic over told in the earth crustal rocks which has moved forward as a recumbent fold. 6. (D) Charles Darwains (1837) in his theory of origin of coral reefs said All corals are formed as a fringe reef till there is no difference in the level of sea and laud. 7. (B) Guttenberg discontinuity occurs at a depth of 2900 km and separates mantle from core 8. (D) The concept of sea floor spreading was postulated by Harry Hess in 1960. 9. (D) Mauna Loa is a famous volcane for continuous monitoring of atmospheric CO2 since 1957 (As an active volcane). 10. (C) 11. (C) Equatorial low pressure beet is a truely continuous pressure belf because here the sun rays fall vertically through out the flor, causing the area to be hot. 12. (C) Sub tropical high pressure belt lies 30°– 35° N and South of the equator from this belt trade winds below towards equator and westerlies below towards the sub polar low pressure belt. 13. (B) 14. (C) Condensation is a processes by which vapour changes into the liquid or solid form. For this high altitude is not a necessary condition. 15. (C) 16. (A) 17. (A) Speed of the ocean current’s about 2% of the speed of prevailing winds. 18. (D) 19. (B) 20. (C) 21. (C) A habitat is the place where a group of organisms live. 22. (A) 23. (C) The term vadose is derived from the latine vadosus, meaning shallow, vadose. Zone extend from the surface to the water table, i.e. lowest water table if there is more than one. 24. (B) 25. (C) The concept of Primate city was given by M. Jefferson.

26. (D) 27. (B) 28. (B) It was Patricls Geddes who was greatly influenced by the Herbert Spanears theory along with the Le-play theory. He established the spencer’s theory on Le-plays theory of Place-work-folk and alternate it us Environment (Place). Work (Function) folk (Organism). 29. (A) Geography of Puranas was written by S.M. Ali. 30. (C) 31. (C) 32. (A) Density of population as per 2011 census of India in given states is as follows : 1. Arunachal Pradesh — 17 persons km2 2. Jammu Kashmir — 96 persons km2 3. Sikkim — 86 persons km2 4. Mizoram — 52 persons km2 33. (B) 34. (A) Sex Ratio in the given states is as follow : 1. Mahe — 1184 women per 1000 men 2. Almora — 1142 women per 1000 men 3. Alwar — 895 women per 1000 men 4. Thanjavur — 1035 women per 1000 men 35. (D) 36. (C) 37. (B) 38. (C) Christianity dominates the maximum territorial regions. 39. (B) The concept of ‘demand cone’ was given by August Loseh in his locational theory of Demand. 40. (B) Rubber, rice and jute are three crops that need 20° to 27°C Temperature and more than 150 cms rainfall. 41. (A) 42. (A) There is very little Iron production in Japan. But Japan is second hour steel producing country in the world and Japan has got a coastal exposure and direct access to the deep water Sea. 43. (C) 44. (B) 45. (D) Whittelessy’s classification is widely accepted one and is based on 1. Crop and livestock association, 2. Labour and Capital intensity, 3. Productivity, consumption pattern of production, 5. Method and technique used. His scheme outlives thirteen types of agricultural practices.

12 | Geography-III (U.G.C.) J-15 46. (C) As per world coal association report of 2013. The countries that import coals were China (327 mt), Japan (196 mt), India (180 mt), South Korea (126 mt). 47. (D) 48. (A) 49. (C) 50. (B) 51. (C) The bases of Caste region is the proportion of the caste in a region. 52. (D) 53. (C) A cultural region (Realm) refers to an area over which the cultural traits of human group may be identified. 54. (C) 55. (A) Explosive model of Urban structure lebelled with these three areas— 1. The city of death. 2. The city of need and the city of superfluity. 56. (C) 57. (B) 58. (A) 59. (D) 60. (B)

61. (D) Govind Sagar an artificial lake is situated at Bhakra Nangal Dam. 62. (C) 63. (C) Kakrapara irrigation project is at River Tapti. 64. (A) River Indus Originates from Mansarover lake in Tibet at the height of 5182 meters. 65. (A) 66. (D) It is Indira Gandhi canal formerly known as Rajasthan canal has transformed the agriculture activities of Sri Ganganagar, Bikaner, Jaisalmer. 67. (C) It is HDI that is based on Literacy role life expectancy per capita income, released by UNDP. 68. (D) 69. (A) 70. (D) 71. (A) 72. (B) 73. (C) 74. (D) 75. (C)